Ibps RRB Clerk Prelims Bundle PDF

You might also like

Download as pdf or txt
Download as pdf or txt
You are on page 1of 722

IBPS RRB Clerk Prelims – Ultra Practice Bundle PDF

IBPS RRB Clerk Prelims – Reasoning Ability

S.no Topic Page number


1. Seating- Linear
Questions 6
Solutions 14
2. Seating- Circle
Questions 27
Solutions 35
3. Seating- Square

Page 1 of 722

Subscribe the Xpress Video Course & Mock Test Package for Bank & Insurance Exams
If there are any suggestions/ errors in our PDFs Feel Free to contact us via this email: admin@exampundit.in
IBPS RRB Clerk Prelims – Ultra Practice Bundle PDF
Questions 53
Solutions 61
4. Seating- Rectangle
Questions 78
Solutions 86
5. Seating- Triangle
Questions 94
Solutions 104
6. Certain Number - Linear
Questions 132
Solutions 140
7. Blood Relation
Questions 156
Solutions 163
8. Number Sequence
Questions 170
Solutions 178
9. Alphabet Series
Questions 187
Solutions 195
10. Coding Decoding
Questions 204
Solutions 212
11. Inequality
Questions 215
Solutions 224
12. Direction
Questions 232

Page 2 of 722

Subscribe the Xpress Video Course & Mock Test Package for Bank & Insurance Exams
If there are any suggestions/ errors in our PDFs Feel Free to contact us via this email: admin@exampundit.in
IBPS RRB Clerk Prelims – Ultra Practice Bundle PDF
Solutions 241
13. Alpha Numeric Series
Questions 246
Solutions 255
14. Data Sufficiency
Questions 264
Solutions 273
15. Puzzle- Days
Questions 301
Solutions 310
16. Seating- Parallel Row
Questions 321
Solutions 330
17. Puzzle- Floor with Flat
Questions 357
Solutions 368
18. Certain Number- Circle
Questions 400
Solutions 407
19. Puzzle- Floor
Questions 425
Solutions 433
20. Puzzle- Box/Stack
Questions 445
Solutions 453
21. Puzzle- Month
Questions 463
Solutions 472

Page 3 of 722

Subscribe the Xpress Video Course & Mock Test Package for Bank & Insurance Exams
If there are any suggestions/ errors in our PDFs Feel Free to contact us via this email: admin@exampundit.in
IBPS RRB Clerk Prelims – Ultra Practice Bundle PDF
22. Puzzle- Month and Date
Questions 482
Solutions 492
23. Puzzle- Random
Questions 503
Solutions 512
24. Syllogism
Questions 525
Solutions 537
25. Conditional Coding
Questions 546
Solutions 555

IBPS RRB Clerk Prelims – Quantitative Aptitude


S.no Topic Page number
1. Approximation
Questions 560
Solutions 566

2. Simplification
Questions 571
Solutions 577

3. Missing Number
Questions 581
Solutions 586

4. Wrong Number
Questions 592
Solutions 597

Page 4 of 722

Subscribe the Xpress Video Course & Mock Test Package for Bank & Insurance Exams
If there are any suggestions/ errors in our PDFs Feel Free to contact us via this email: admin@exampundit.in
IBPS RRB Clerk Prelims – Ultra Practice Bundle PDF
5. Quadratic Equation
Questions 603
Solutions 610

6. DI – Table
Questions 624
Solutions 636

7. DI- Bar
Questions 642
Solutions 656

8. DI- Line
Questions 662
Solutions 675

9. DI- Pie
Questions 680
Solutions 699

10. Caselet DI
Questions 705
Solutions 715

Page 5 of 722

Subscribe the Xpress Video Course & Mock Test Package for Bank & Insurance Exams
If there are any suggestions/ errors in our PDFs Feel Free to contact us via this email: admin@exampundit.in
IBPS RRB Clerk Prelims – Ultra Practice Bundle PDF
1). Linear Seating Arrangement
Directions (1-5): Study the information carefully and 3. Which of the following groups of friends is
answer the questions given below. sitting to the right of E?
There are eleven people A, B, C, D, E, F, G, H, I, J and K (a) GK
sitting in a straight line facing South direction but not (b) AE
necessarily in the same order. Two People sits between G (c) DC
and E. I sit five places away from J. I and J are not (d) CH
immediate neighbor of G. C and D are not immediate (e) None of these
neighbours. Three persons sit between Both H and G and 4. Who is sitting in the middle of the row?
H and D. F sits to the left of H. Four persons sits between (a) G
K and F. E sits to the right of H. B sits at the right end of (b) E
the row. H sits five places away from B. (c) C
1. Which of the following people are sitting fifth to the (d) J
Left of K? (e) H
(a) C 5. How many person sits between I and D?
(b) H (a) 1
(c) D (b) 2
(d) F (c) 3
(e) None of these (d) 4
2. Which of the following statements is true in the (e) Cannot be determined
context of the above sitting arrangement? Directions (6-10): Study the information carefully and
(a) There are three students sitting between D and C answer the questions given below:
(b) K is between I and J. Eight persons E, F, G, H, I, K, J and L are sitting in the
(c) C is sitting between F and H. linear line row facing east. N sits 2nd to the right of H who
(d) K and I are immediate neighbours of G. is immediate to K. Either F or G sit at any of the extreme
(e) None of these ends. H and L are immediate neighbours to each other.
Number of person between H and I is one more than the
Page 6 of 722

Subscribe the Xpress Video Course & Mock Test Package for Bank & Insurance Exams
If there are any suggestions/ errors in our PDFs Feel Free to contact us via this email: admin@exampundit.in
IBPS RRB Clerk Prelims – Ultra Practice Bundle PDF
number of person between H and L. L sits 2nd to left of J. 10. Which of the following is true?
H is third from any end of the row. (a) No one sits to the left of K
6. How many persons sitting between I and J? (b) H sits second to the right of N
(a) Four (c) G is sitting extreme end of the line.
(b) Five (d) I is sitting extreme end of the line.
(c) Three (e) Both (c) and (d) are true
(d) Two Directions (11-15): Study the information carefully
(e) Six and answer the questions given below:
7. Who sits second to the left of G? Ten persons are sitting in a row facing north. Less than
(a) N two persons sit to the left of J. Two persons sit between J
(b) J and E. A is an immediate neighbour of E. As many
(c) Either (a) or (b) persons sits to the right of A sits to the left of H. less than
(d) K five persons sit between J and H. I sit immediate right of
(e) I H. W and O are immediate neighbour. As many persons
8. Four of the following five are alike in a certain way sits to the left of N sits to the right of W. S sits fifth to the
and hence form a group, which of the following left of K.
does not belong to the group? 11. Find the position of K from right end of the row?
(a) I, K (a) Fourth to the right
(b) H, L (b) Fourth to the left
(c) N, J (c) Second to the right
(d) G, N (d) Second to the left
(e) G, F (e) Third to the right
9. How many persons sit to the left of J? 12. Which two persons are sitting at the corner?
(a) Five (a) N, S
(b) None (b) A, H
(c) Three (c) J, O
(d) Two (d) A, H
(e) One (e) S, E
Page 7 of 722

Subscribe the Xpress Video Course & Mock Test Package for Bank & Insurance Exams
If there are any suggestions/ errors in our PDFs Feel Free to contact us via this email: admin@exampundit.in
IBPS RRB Clerk Prelims – Ultra Practice Bundle PDF
13. How many person sits between N and W? immediate neighbour of S. Both V and P facing same
(a) 1 direction to each other. X sits fourth to the left of Y, both
(b) 6 are facing opposite direction. Persons who sit at extreme
(c) 3 ends faces opposite directions. R does not face south. U
(d) 4 and X facing same direction.
(e) 5 16. Who is sitting to the immediate right of V?
14. Who is sitting third to the left of S? (a) T
(a) J (b) U
(b) N (c) Y
(c) No One (d) P
(d) A (e) Q
(e) E 17. If all are made to sit alphabetically from right to
15. What is the position of W with respect to E? left, then position of how many will remain same?
(a) Fourth to the right (a) One
(b) Sixth to the left (b) Two
(c) Second to the right (c) Three
(d) CND (d) More than three
(e) None of these (e) None
Directions (16-20): Study the information carefully 18. What if the position of U with respect to S?
and answer the questions given below (a) Third to the right
Ten persons are sitting in a row some are facing north (b) Immediate right
while some are facing south. P sits fourth to the left of Q (c) Fifth to the left
and sits one of the extreme ends of the row. More than (d) Second to the left
three persons are sitting between R and Q. S sits third to (e) Third to the left
the right of T and is an immediate neighbour of R, both 19. How many people are facing north direction?
are facing same direction. U sits fourth to the right of the (a) One
one who sits immediate left of S.V sits fourth to the left of (b) Two
the one who sits second to the right of W who is not an (c) Four
Page 8 of 722

Subscribe the Xpress Video Course & Mock Test Package for Bank & Insurance Exams
If there are any suggestions/ errors in our PDFs Feel Free to contact us via this email: admin@exampundit.in
IBPS RRB Clerk Prelims – Ultra Practice Bundle PDF
(d) More than four 22. I sits to the immediate right of?
(e) Three (a) J
20. In which of the following pair all person facing (b) L
south direction? (c) Both J and L
(a) WQU (d) M
(b) YVS (e) No one
(c) TSR 23. How many persons facing south direction?
(d) PXU (a) 5
(e) XUQ (b) 4
Directions (21-25): Study the information carefully (c) 2
and answer the questions given below (d) 3
Eight persons I, J, K, L, M, N, O and P are seating in a (e) 1
row, some are facing north and some are facing south but 24. Which of the following statement is false?
not necessarily in the same order. I sit third to the right of (a) I sits immediate left of J
K and I do not sit at any end of the row. N sits two places (b) P and O faces the opposite direction
away from P. No one sits to the right of N. As many (c) M do not sits at the end of the line
persons sit to the right of I as sit to the left of J who does (d) L sits third to the right of P
not faces south. P doesn’t face north. M sits second to the (e) There are three persons between K and J
right of the one who sits third to the left of O. M is not an 25. Which of the following statement is true?
immediate neighbor of O. M and L face opposite direction (a) L and I faces the same direction
with respect to each other. L faces same direction as P (b) M is the immediate neighbour of I and J
faces. (c) J sits third to the left of N
21. How many persons sit between O and P? (d) K faces south direction
(a) Three (e) O sits second to the right of I
(b) Four Directions (26-30): Study the following information to
(c) Five answer the given questions
(d) Two Twelve people are sitting in two parallel rows containing
(e) Six six people each in such a way that there is an equal
Page 9 of 722

Subscribe the Xpress Video Course & Mock Test Package for Bank & Insurance Exams
If there are any suggestions/ errors in our PDFs Feel Free to contact us via this email: admin@exampundit.in
IBPS RRB Clerk Prelims – Ultra Practice Bundle PDF
distance between adjacent persons. In row 1 – G, H, I, J, 28. Which of the following is true with respect to the
K and L are seated (but not necessarily in the same order) given information?
and all of them are facing south. In row 2 – C, D, R, T, V (a) L faces one of the immediate neighbours of D
and W are seated (but not necessarily in the same order) (b) L sits exactly between G and H.
and all of them are facing North. Therefore, in the given (c) None of the given options is true
seating arrangement each member seated in a row faces (d) G is an immediate neighbour of K
another member of the other row. T sits fourth to the right (e) T faces J.
of C. The one facing C sits third to the right of K. Only 29. Which of the following is true regarding H?
one person sits between K and G. The one facing L sits (a) H sits second to the right of L.
second to the Left of D. L does not sit at any of the extreme (b) C is an immediate neighbour of the person who
ends of the line. Only two people sit between D and W. faces H
The one facing D sits second to the right of J. H is not an (c) Both G and K are immediate neighbours of H
immediate neighbour of L. R is not immediate neighbour (d) Only one person sits between H and I
of T. (e) None of the given options is true
26. Which of the following groups of people represents 30. Who amongst the following sits second to the right
the people sitting at extreme ends of both the rows? of the person who faces V?
(a) I, J, W, T (a) H
(b) C, R, L, G (b) L
(c) D, H, J, T (c) I
(d) C, V, L, K (d) G
(e) I, W, G , R (e) J
27. Who amongst the following faces, K? Direction (31-35): Study the following information
(a) W carefully and answer the given questions:
(b) C Fourteen persons are sitting in two parallel rows such that
(c) R seven persons are sitting in each row. H, I, J, K, L, M and
(d) D N are sitting in row-2 facing south while G, F, E, D, C, B
(e) V and A are sitting in row-1 facing north. Persons sitting in
Row-1 were facing the persons sitting in Row-2. N sits
Page 10 of 722

Subscribe the Xpress Video Course & Mock Test Package for Bank & Insurance Exams
If there are any suggestions/ errors in our PDFs Feel Free to contact us via this email: admin@exampundit.in
IBPS RRB Clerk Prelims – Ultra Practice Bundle PDF
third to the right of H and neither of them sits at an (e) Only two persons sit between I and H
extreme end of the row. The one faces H sits immediate 34. What is the position of B with respect to D?
left to C. Only one person sits between C and F. The one (a) Second to the left
who faces F sits third to the left of L. D sits to the (b) Third to the right
immediate right of A. D neither faces N nor L. K is an (c) Immediate right
immediate neighbour of the one who faces D. The one (d) Immediate left
who faces J sits fifth to the right of G. I sits fourth to the (e) Second to the right
right of M. B sits at one of position to the left of E. 35. What is the position of G with respect to F?
31. Four of the following are alike in a certain way so (a) Third to the left
form a group which of the following does not (b) Second to the left
belong to that group? (c) Forth to the left
(a) L (d) Third to the right
(b) M (e) Fifth to the right
(c) A Direction (36-40): Study the following information
(d) G carefully and answer the given questions:
(e) E Nine persons A, B, C, D, E, G, H and I are sitting in the
32. How many person sits between F and C? three parallel rows, but not necessarily in the same order.
(a) Two All are facing south. Three persons are sitting in each row.
(b) One Row 1 is south of Row 2 and Row 2 is south of Row 3.
(c) None Ahead and behind means immediately as per arrow
(d) Three direction on same line. H sits second to the right of C. G
(e) More than three is ahead H. Only one person’s sitting between E and A. B
33. Which of the following is true regarding I? sits ahead D. Both D and B are not the immediate
(a) No one sits to the right of I neighbors of E. F sits ahead of one who immediate right
(b) I sits third to the right of L of I. F sits right of E.
(c) M is an immediate neighbour of I. 36. Which of the following person sits in row 3?
(d) F is an immediate neighbour of the one who faces (a) G
I (b)
Page 11 of 722

Subscribe the Xpress Video Course & Mock Test Package for Bank & Insurance Exams
If there are any suggestions/ errors in our PDFs Feel Free to contact us via this email: admin@exampundit.in
IBPS RRB Clerk Prelims – Ultra Practice Bundle PDF
(c) I (d) A
(d) F (e) F
(e) D Direction (41-45): Study the following information
(f) E carefully and answer the given questions:
37. Who is immediate left of A? Twelve persons sit in two parallel rows. Seven seats are in
(a) E each row. There is one vacant seat in each row. In row 2,
(b) G A, B, C, D, E and H are sitting and facing north while in
(c) B row 1, N, I, J, U, O and T are sitting and facing south.
(d) I Seats of Row-1 face the seats of Row-2. J sits at one of the
(e) F extreme end. Only one person sits between J and the
38. How many persons ahead of B? vacant seat. Two persons sit between D and the vacant
(a) 3 seat. The one who faces E sits second to the left of T. J
(b) 2 does not face C. The one who faces A sits third to the right
(c) 1 of N, who does not sit at any of the extreme end. The one
(d) 4 who faces vacant seat sit second to the left of D. As many
(e) 5 persons sit to the right of H sit to the right of I. Three
39. Who among the following is immediate neighbour persons sit between E and C. Only one person sits between
of D? O and the one faces B.
(a) H 41. How many person sits between vacant seat of row
(b) C 1 and I.
(c) B (a) 0
(d) Both (a) and (b) (b) 3
(e) G (c) 2
(d) 4
40. Who is immediate right of G? (e) 5
(a) No one 42. How many persons sit between E and the one who
(b) B faces I.
(c) I (a) 4
Page 12 of 722

Subscribe the Xpress Video Course & Mock Test Package for Bank & Insurance Exams
If there are any suggestions/ errors in our PDFs Feel Free to contact us via this email: admin@exampundit.in
IBPS RRB Clerk Prelims – Ultra Practice Bundle PDF
(b) 3 Ten persons F, G, H, I, J, U, V, W, X and Y are sitting in
(c) 2 a parallel row such that all are facing south. Persons from
(d) 1 U to Y are in 2nd row while rest is in 1st row. The persons
(e) 5 in 2nd row are facing back of persons in 1st row. X sits third
43. Which of the following statement is true regarding to the right of W and sits opposite to J. F sits second to the
C? right of I. U sits to the immediate right of Y. F does not sit
(a) C faces I opposite to V. G sits opposite to neither V nor X. There is
(b) C and D are immediate neighbor. at most one person between H and G. H and F are not
(c) Vacant seat faces C neighbours.
(d) J sits 3rd left to the one who faces C 46. Who among the following person diagonally sits?
(e) none of these (a) U, G
44. Which of the following faces D? (b) W, I
(a) O (c) X, H
(b) I (d) V, F
(c) J (e) Y, G
(d) T 47. Who among the following sits extreme end of the
(e) none of these row?
45. Four of the following belongs to a group based on (a) F
their seating arrangement find the one that does (b) I
not belong to that group? (c) U
(a) J (d) J
(b) U (e) W
(c) E 48. Who sits second to the left of the one who is
(d) N opposite to U?
(e) A (a) F
Direction (46-50): Study the following information (b) I
carefully and answer the given questions: (c) G
(d) H
Page 13 of 722

Subscribe the Xpress Video Course & Mock Test Package for Bank & Insurance Exams
If there are any suggestions/ errors in our PDFs Feel Free to contact us via this email: admin@exampundit.in
IBPS RRB Clerk Prelims – Ultra Practice Bundle PDF
(e) J 50. Who sits second to the left of W?
49. How many persons sit between X and the one who (a) Y
faces I? (b) U
(a) 2 (c) X
(b) 1 (d) V
(c) 3 (e) No one
(d) None
(e) None of these

1). Linear Seating Arrangement - Solutions with Explanation

SOLUTION (1-5):
All the person are faces South
1. B sits at the right end of the row.
2. H sits five places away from B.
3. Three persons sit between Both H and G and H and D.

8. F sits to the left of H.


9. Four persons sit between K and F.
10. C and D are not immediate neighbours.

4. Two People sits between G and E.


5. I sit five places away from J.
6. I and J are not immediate neighbor of G. Final Arrangement

7. E sits to the right of H

Page 14 of 722

Subscribe the Xpress Video Course & Mock Test Package for Bank & Insurance Exams
If there are any suggestions/ errors in our PDFs Feel Free to contact us via this email: admin@exampundit.in
IBPS RRB Clerk Prelims – Ultra Practice Bundle PDF

1.d
2.c
3.a
4.e
5.e
SOLUTION (6-10):
All the person are faces East
1. H is third from any end of the row 5. Number of person between H and I is one more than
2. N sits 2nd to the right of H who is immediate to K the number of person between H and L.
3. H and L are immediate neighbours to each other 6. Either F or G sits at any of the extreme ends.

4. L sits 2nd to left of J. Final Arrangement

Page 15 of 722

Subscribe the Xpress Video Course & Mock Test Package for Bank & Insurance Exams
If there are any suggestions/ errors in our PDFs Feel Free to contact us via this email: admin@exampundit.in
IBPS RRB Clerk Prelims – Ultra Practice Bundle PDF
4. As many persons sits to the right of A sits to the left
of H.
5. Less than five persons sit between J and H.

6.a
7.c
8.d (not immediate neighbour)
6. I sit immediate right of H.
9.a
7. S sits fifth to the left of K.
10.d
8. W and O is immediate neighbour.
SOLUTION (11-15):
9. As many persons sits to the left of N sits to the right
All the person are faces North
of W
1. Less than two persons sit to the left of J.
2. Two persons sit between J and E.

3. A is an immediate neighbour of E. Final Arrangement

Page 16 of 722

Subscribe the Xpress Video Course & Mock Test Package for Bank & Insurance Exams
If there are any suggestions/ errors in our PDFs Feel Free to contact us via this email: admin@exampundit.in
IBPS RRB Clerk Prelims – Ultra Practice Bundle PDF
6. Persons who sit at extreme ends faces opposite
directions. (means if one end faces North the other end
faces South)

11.e 7. R does not face south. (means R faces north )

12.c
13.b
14.c
15.e
SOLUTION (16-20):
Some are facing north some facing south
1. P sits fourth to the left of Q and sits one of the extreme 8. X sits fourth to the left of Y, both are facing opposite
ends of the row. (here Q faces either north or south direction. (Means if X faces north then Y faces south)
direction) 9. U and X facing same direction. .(means they both
2. More than three persons are sitting between R and Q. faces either north or south)
3. S sits third to the right of T and is an immediate
neighbour of R. (here T faces either north or south
direction)

10. V sits fourth to the left of the one who sits second to
the right of W who is not an immediate neighbour of
4. U sits fourth to the right of the one who sits immediate S.
left of S (here S faces either north or south direction) 11. Both V and P facing same direction to each other.
5. R and S faces same direction. (means they both faces
either north or south)

Page 17 of 722

Subscribe the Xpress Video Course & Mock Test Package for Bank & Insurance Exams
If there are any suggestions/ errors in our PDFs Feel Free to contact us via this email: admin@exampundit.in
IBPS RRB Clerk Prelims – Ultra Practice Bundle PDF

4. M sits second to the right of the one who sits third to


Final Arrangement the left of O.
5. M is not an immediate neighbor of O.
6. I sit third to the right of K and I do not sit at any end
of the row

16.c
17.a
18.c
19.d
20.d
SOLUTION (21-25):
Some are facing north some facing south
1. N sits two places away from P.
2. No one sits to the right of N.
3. P doesn’t face north.

Page 18 of 722

Subscribe the Xpress Video Course & Mock Test Package for Bank & Insurance Exams
If there are any suggestions/ errors in our PDFs Feel Free to contact us via this email: admin@exampundit.in
IBPS RRB Clerk Prelims – Ultra Practice Bundle PDF
7. As many persons sit to the right of I as sit to the left of 22.e
J who does not faces south. 23.d
24.b
25.c
SOLUTION (26-30):
1. T sits fourth to the right of C
2. The one facing C sits third to the right of K.

8. M and L face opposite direction with respect to each


other.
9. L faces same direction as P faces.

3. Only one person sits between K and G


4. The one facing L sits second to the Left of D.
Final Arrangement 5. L does not sit at any of the extreme ends of the line.

21.a
Page 19 of 722

Subscribe the Xpress Video Course & Mock Test Package for Bank & Insurance Exams
If there are any suggestions/ errors in our PDFs Feel Free to contact us via this email: admin@exampundit.in
IBPS RRB Clerk Prelims – Ultra Practice Bundle PDF

8. H is not an immediate neighbour of L.


9. R is not immediate neighbour of T.

6. Only two people sit between D and W.


7. The one facing D sits second to the right of J.
Final Arrangement

Page 20 of 722

Subscribe the Xpress Video Course & Mock Test Package for Bank & Insurance Exams
If there are any suggestions/ errors in our PDFs Feel Free to contact us via this email: admin@exampundit.in
IBPS RRB Clerk Prelims – Ultra Practice Bundle PDF

26.a 5. D sits to the immediate right of A.


27.e 6. D neither faces N nor L.
28.e 7. K is an immediate neighbour of the one who faces D.
29.c
30.d
SOLUTION (31-35):
1. N sits third to the right of H and neither of them sits at
an extreme end of the row.
2. The one faces H sits immediate left to C.
3. Only one person sits between C and F.
4. The one who faces F sits third to the left of L.

8. The one who faces J sits fifth to the right of G.


9. B sits at one of position to the left of E

Page 21 of 722

Subscribe the Xpress Video Course & Mock Test Package for Bank & Insurance Exams
If there are any suggestions/ errors in our PDFs Feel Free to contact us via this email: admin@exampundit.in
IBPS RRB Clerk Prelims – Ultra Practice Bundle PDF

31.c
32.b
33.d
34.e
35.a
SOLUTION (36-40):
1. H sits second to the right of C.
10. I sit fourth to the right of M.
2. G is ahead H.

Final Arrangement

3. Only one person’s sitting between E and A.


4. F sits right of E.
5. Both D and B are not the immediate neighbors of E.

Page 22 of 722

Subscribe the Xpress Video Course & Mock Test Package for Bank & Insurance Exams
If there are any suggestions/ errors in our PDFs Feel Free to contact us via this email: admin@exampundit.in
IBPS RRB Clerk Prelims – Ultra Practice Bundle PDF

6. B sits ahead D.
36.d
7. F sits ahead of one who immediate right of I.
37.e
38.a
39.d
40.a

SOLUTION (41-45):
1. J sits at one of the extreme ends.
2. Only one person sits between J and the vacant seat.
3. The one who faces vacant seat sit second to the left of
D
4. Two persons sit between D and the vacant seat.
Final Arrangement

Page 23 of 722

Subscribe the Xpress Video Course & Mock Test Package for Bank & Insurance Exams
If there are any suggestions/ errors in our PDFs Feel Free to contact us via this email: admin@exampundit.in
IBPS RRB Clerk Prelims – Ultra Practice Bundle PDF

5. The one who faces E sits second to the left of T


6. Three persons sit between E and C.
7. J does not face C

10. As many persons sit to the right of H sit to the right of


I.

8. The one who faces A sits third to the right of N, who


does not sit at any of the extreme end.
9. Only one person sits between O and the one faces B.

Page 24 of 722

Subscribe the Xpress Video Course & Mock Test Package for Bank & Insurance Exams
If there are any suggestions/ errors in our PDFs Feel Free to contact us via this email: admin@exampundit.in
IBPS RRB Clerk Prelims – Ultra Practice Bundle PDF

Final Arrangement

2. U sits to the immediate right of Y.


3. F does not sit opposite to V.
41.a 4. F sits second to the right of I.
42.a
43.c
44.d
45.d
SOLUTION (46-50):
1. X sits third to the right of W and sits opposite to J.

Page 25 of 722

Subscribe the Xpress Video Course & Mock Test Package for Bank & Insurance Exams
If there are any suggestions/ errors in our PDFs Feel Free to contact us via this email: admin@exampundit.in
IBPS RRB Clerk Prelims – Ultra Practice Bundle PDF

5. G sits opposite to neither V nor X.


6. There is at most one person between H and G.
Final Arrangement

7. H and F are not neighbours.

46.c
47.d

Page 26 of 722

Subscribe the Xpress Video Course & Mock Test Package for Bank & Insurance Exams
If there are any suggestions/ errors in our PDFs Feel Free to contact us via this email: admin@exampundit.in
IBPS RRB Clerk Prelims – Ultra Practice Bundle PDF
48.b 50.e
49.a

2). Circle Seating Arrangement


Directions (1-5): Study the information carefully and (e) D
answer the questions given below. 3. Who is to the immediate left of D?
There are eleven people B, C, D, E, F, G, H, I, J, K and L (a) H
sitting in a circle facing outside the centre but not (b) J
necessarily in the same order. H sits three places away (c) K
from F. Four persons are sitting between I and K, who is (d) F
not sitting near to H. D and H are not immediate (e) B
neighbours. Both G and E sit four places away from J. G 4. Who is second to the Right of E?
sits to the right of J. L sits four places away from G. F sits (a) L
to the right of G. K sits fourth to the right of C. J sits five (b) K
placesfrom C. F sits second to the left of J. (c) F
1. Which of the following combinations represents (d) H
the first and the second to the left of C respectively? (e) D
(a) EL 5. In which of the following combinations the third
(b) EF person is second to the left of the second person?
(c) IG (a) GBD
(d) GB (b) DHJ
(e) KF (c) FLK
2. Who is third to the right of L? (d) LFE
(a) I (e) None of these
(b) J Directions (6-10): Study the information carefully and
(c) K answer the questions given below
(d) H

Page 27 of 722

Subscribe the Xpress Video Course & Mock Test Package for Bank & Insurance Exams
If there are any suggestions/ errors in our PDFs Feel Free to contact us via this email: admin@exampundit.in
IBPS RRB Clerk Prelims – Ultra Practice Bundle PDF
Nine persons are sitting in a circle only two are facing 9. Who among the following sits at immediate left of
outside the circle and they both are not sitting together. G S?
is immediate left of L. Only 2 persons sit between H and (a) G
I who does not sit adjacent to M. L sits 5 places to the left (b) L
of M and both of them facing opposite direction to each (c) I
other. N who is not facing centre and not immediate (d) N
neighbour of I. J sits fourth to the right of S who faces (e) H
inside. S is not near to G and K. K sits second to G. 10. How many person(s) sits between H and L?
6. How many person(s) sits between M and G when (a) 3
counted to the left G? (b) 4
(a) 1 (c) 5
(b) 2 (d) 2
(c) 3 (e) Either a or b
(d) 4 Directions (11-15): Study the information carefully
(e) 5 and answer the questions given below
7. Who among the following is third to the right of I? Ten seats in a circle such that nine persons are sitting
(a) H around a circular table facing outside the centre. One of
(b) M the seats is vacant. L sits fourth to the right of H. Only one
(c) L person sit between L and N. Vacant seat is immediate left
(d) G to N. O sits third to the left of P. T sits immediate left of
(e) J O. P and L are not immediate neighbours. O and N are not
8. Which of the following combinations correctly immediate neighbours. Only one person sits between Q
show the neighbor(s) of L? and R. R sits second to the right of S.
(a) J, G 11. What is the position of T with respect to S?
(b) I, N (a) Third to the left
(c) G, K (b) Immediate left
(d) N, S (c) Second to the left
(e) M. K (d) Third to the right
Page 28 of 722

Subscribe the Xpress Video Course & Mock Test Package for Bank & Insurance Exams
If there are any suggestions/ errors in our PDFs Feel Free to contact us via this email: admin@exampundit.in
IBPS RRB Clerk Prelims – Ultra Practice Bundle PDF
(e) None of these (e) None of these
12. Which of the following statement is correct? Directions (16-20): Study the information carefully
(a) H sits immediate right of S. and answer the questions given below
(b) Only three persons sit between N and L. Ten persons are sitting in a circle facing outside the centre.
(c) Q sits second to the left of N. D sit third to the right of L. B is an immediate neighbor of
(d) L sits to the immediate left of R. D. Three persons sits between B and G. Less than three
(e) None of these persons sit between L and G when counted to the left of
13. How many seats between the one who sits to the L. F sits immediate left of G. Three persons sit between M
immediate left of H and the one who sits to the and P. M is immediate right of V. R sits fifth to the right
immediate right of R? of H who is not immediate neighbor of L.
(a) One 16. What is the position of F with respect to L?
(b) Two (a) Second to the left
(c) Four (b) Immediate left
(d) No one (c) Third to the right
(e) None of these (d) Third to the left
14. Which of the following person are immediate (e) None of these
neighbor of vacant seats? 17. Which of the following pairs sits opposite to each
(a) R-P other?
(b) N-Q (a) R, F
(c) P-S (b) P, V
(d) H-O (c) D, P
(e) T-L (d) L, V
15. What is the position of P with respect to the one (e) V, G
who sits to the immediate right of T? 18. Which of the following statement is true?
(a) Second to the right (a) R sits opposite to F
(b) Second to the left (b) D sits at immediate right of V
(c) Third to the right (c) Only one person sits between P and B
(d) Third to the left (d) P sits second to the left of F
Page 29 of 722

Subscribe the Xpress Video Course & Mock Test Package for Bank & Insurance Exams
If there are any suggestions/ errors in our PDFs Feel Free to contact us via this email: admin@exampundit.in
IBPS RRB Clerk Prelims – Ultra Practice Bundle PDF
(e) All the above statements are false the inside of the circle. T is not opposite to U and R. Q
19. What is the position of the one who sits the faces to the outside of the centre.
immediate right of L with respect to the one who 21. Which of the following person sits to the immediate
sits second to the left of M? left of S?
(a) Second to the left (a) W
(b) Second to the right (b) V
(c) Third to the left (c) Q
(d) Third to the right (d) T
(e) None of these (e) None of these
20. If we change the direction of H then which of the 22. How many persons sit between R and P?
following person sit third to the right of H? (a) One
(a) P (b) Two
(b) G (c) Three
(c) D (d) Four
(d) V (e) None of these
(e) B 23. If V is related to P, U is related to Q, in the same
Directions (21-25): Study the information carefully way S is related to which of the following?
and answer the questions given below (a) Q
Eight persons i.e. P, Q, R, S, T, U, V and W are sitting (b) T
around a circle at an equal distance. Three persons on the (c) V or R
circle are facing opposite side and other five are sitting (d) W
facing to the center of the circle. Q is sitting third to the (e) None of these
left of U. Two are sitting between U and T. S is not near 24. Which of the following statement is true?
to W. Two persons are sitting between W and T. R is (a) U sits second to the left of V
sitting third to the right of S. S sits second to the right of (b) Three persons sit between Q and R
P. Among P, R and S one is facing inside the center of the (c) More than two persons sit between V and S when
circle. R is sitting second to the left of V, who is facing counted to the right of V
(d) P sits third to the right of W
Page 30 of 722

Subscribe the Xpress Video Course & Mock Test Package for Bank & Insurance Exams
If there are any suggestions/ errors in our PDFs Feel Free to contact us via this email: admin@exampundit.in
IBPS RRB Clerk Prelims – Ultra Practice Bundle PDF
(e) Immediate neighbours of T facing same direction (a) Fourth to the left
to each other. (b) Third to the left
25. Which of the following sits third to the right of V? (c) Second to the left
(a) S (d) Fifth to the right
(b) T (e) None of these
(c) P 28. How many persons sit between W and T, when
(d) R counted from the left of former?
(e) None of these (a) 1
Directions (26-30): Study the information carefully (b) 2
and answer the questions given below (c) 3
Eight persons are sitting around a circular table. Some of (d) 4
them are facing inside the center while some are facing (e) 5
outside the center. S sits second to the right of P. T sits 29. Who sits third to the left of the one who is
third to the left of S. U sits opposite to Q and Q is not an immediate right of V?
immediate neighbor of S. S is facing same direction of P. (a) R
V sits Immediate to the right of W who is not immediate (b) U
neighbor of Q. R neither sits opposite to T nor P. Both V (c) S
and W are facing opposite direction. W is facing outside (d) P
the center of the circle. Both T and Q are facing opposite (e) Q
direction as V. While both R and U are facing same 30. How many persons sit between U and R?
direction to each other but opposite to S. (a) 2
26. How many persons faces outside the centre? (b) 3
(a) 1 (c) 4
(b) 2 (d) 5
(c) 3 (e) Either a or c
(d) 4 Directions (31-35): Study the information carefully
(e) 5 and answer the questions given below
27. What is the position of Q with respect to P?
Page 31 of 722

Subscribe the Xpress Video Course & Mock Test Package for Bank & Insurance Exams
If there are any suggestions/ errors in our PDFs Feel Free to contact us via this email: admin@exampundit.in
IBPS RRB Clerk Prelims – Ultra Practice Bundle PDF
Twelve people N, O, X, V, Z, W, H, I, J, K, L and M are (e) None of these
sitting in the circle. All of them facing inside the centre. N 34. How many persons are there exactly between J and
sits third to the left of Z. Three persons sit between Z and H?
H. Three people sit between M and I. M is not immediate (a) One
neighbour of Z. H sits second to the right of L. I faces N. (b) Two
Only one Person sits between W and O. Neither W nor O (c) Three
faces H and Z. X does not face H. J sits third to the left of (d) None
X who does not faces W. K is not an immediate neighbour (e) None of these
of X. Neither W nor O is opposite to J 35. Who among the following is third to the right of X?
31. Who among the following sits 4th to the right of J? (a) H
(a) I (b) L
(b) V (c) M
(c) X (d) O
(d) L (e) N
(e) M Directions (36-40): Study the information carefully
32. Who among the following is immediate neighbour and answer the questions given below
of O? Seven people C, D, E, F, G, H and I sitting in a Circle with
(a) Z, W equal distance between them, but not necessarily in the
(b) K, Z same order. Some of them are facing centre and some are
(c) N, H facing outside the centre. Only two people sit between I
(d) M, L and D. C sits second to the right of D. The immediate
(e) W, J neighbours of C face opposite directions. Only one person
33. In which of the following pairs second person sits sits between C and E. H sits third to the right of E. Both
second to the right of the first person? the immediate neighbours of E face the same direction as
(a) M,O C. C faces the same direction as that of E. G faces inside.
(b) K, H D sits to the immediate left of G.
(c) Z, J 36. Who amongst the following sits on the immediate
(d) N, M right of H?
Page 32 of 722

Subscribe the Xpress Video Course & Mock Test Package for Bank & Insurance Exams
If there are any suggestions/ errors in our PDFs Feel Free to contact us via this email: admin@exampundit.in
IBPS RRB Clerk Prelims – Ultra Practice Bundle PDF
(a) C 40. How many people are seated between G and C
(b) I (counting clockwise from G)?
(c) D (a) Two
(d) G (b) Four
(e) None of these (c) None
37. Who amongst the following sits third to the right of (d) One
G? (e) Three
(a) D Directions (41-45): Study the information carefully
(b) E and answer the questions given below
(c) F Eight Persons G, H, I, J, T, B, E and X are sitting in a
(d) I circular table. Two persons are facing outside the center
(e) None of these and others are facing inside the center. G sits 2nd to the
38. How many persons faces outside the centre? left of B who sits immediate right of T. There are three
(a) 1 persons sit between X and E who is not an immediate
(b) 2 neighbor of G. H sits 3rd to the left of the person who sits
(c) 3 immediate right of B. E and J are immediate neighbors of
(d) 4 I. G and X are facing opposite direction as T are facing.
(e) 5
39. If all the people are made to sit in an alphabetical 41. What is position of E with respect to G?
order, in clockwise direction, starting from C, the (a) Third to the right
position of whom amongst the following remains (b) E and G are opposite to each other
the same (excluding C)? (c) Second to the right
(a) 1 (d) Third to the left
(b) 2 (e) Fifth to the right
(c) 3 42. Who sits adjacent to both B and G?
(d) 4 (a) T
(e) None of these (b) X
(c) H
Page 33 of 722

Subscribe the Xpress Video Course & Mock Test Package for Bank & Insurance Exams
If there are any suggestions/ errors in our PDFs Feel Free to contact us via this email: admin@exampundit.in
IBPS RRB Clerk Prelims – Ultra Practice Bundle PDF
(d) J One of the seats is vacant. M is fourth to the right of K.
(e) I There are two people between vacant seat and M. L sits
43. Which of the following pairs of the person sits immediate right of vacant seats. R seats immediate right
between H and E when counted to the left of E? of M. There are two persons between N and J. P sits
(a) X, G immediate left of Q. There are two people between P and
(b) I, J N when counted to the left of N.
(c) T, G 46. Who sits second to the left of Vacant Seat?
(d) B, T (a) J
(e) T, X (b) N
44. Who among the following sit between X and J when (c) S
counted in anti-clockwise direction from X? (d) Q
(a) No one sits between X and J as X and J are (e) None of these
immediate neighbours of each other 47. Who sits adjacent to both K and L?
(b) G (a) N
(c) T (b) L
(d) H (c) S
(e) E (d) No one
45. If I is made to face the opposite direction, who (e) None of these
would sit to his immediate right? 48. How many persons sits between N and J when
(a) H counted to the left of J?
(b) J (a) None
(c) E (b) One
(d) B (c) Two
(e) T (d) Three
Directions (46-50): Study the information carefully (e) Four
and answer the questions given below 49. Which of the following pairs are immediate
Ten seats in a circle such that nine J, K, L. M, N, P, Q and neighbor of each other?
S persons are sitting around a circular table facing centre. (a) S, K
Page 34 of 722

Subscribe the Xpress Video Course & Mock Test Package for Bank & Insurance Exams
If there are any suggestions/ errors in our PDFs Feel Free to contact us via this email: admin@exampundit.in
IBPS RRB Clerk Prelims – Ultra Practice Bundle PDF
(b) Q, R (a) P
(c) R, J (b) R
(d) P, R (c) M
(e) None of these (d) J
50. Who sits fourth to the right of Q? (e) Vacant

2). Circle Seating Arrangement - Solutions with Explanation


SOLUTION (1-5): 14. K sits fourth to the right of C
11. Both G and E sit four places away from J. 15. Four persons are sitting between I and K.
12. G sits to the right of J.
13. J sits five places from C.

16. H sits three places away from F.

Page 35 of 722

Subscribe the Xpress Video Course & Mock Test Package for Bank & Insurance Exams
If there are any suggestions/ errors in our PDFs Feel Free to contact us via this email: admin@exampundit.in
IBPS RRB Clerk Prelims – Ultra Practice Bundle PDF
17. K is not sitting near to H. SOLUTION (6-10):
18. L sits four places away from G 1. L sits 5 places to the left of M and both of them
19. D and H are not immediate facing opposite direction to each other.
2. G is immediate left of L

Final Arrangement

1.c
2.b
3. K sits second to G.
3.e
4. J sits fourth to the right of S who faces inside.
4.c
5. S is not near to G and K
5.d

Page 36 of 722

Subscribe the Xpress Video Course & Mock Test Package for Bank & Insurance Exams
If there are any suggestions/ errors in our PDFs Feel Free to contact us via this email: admin@exampundit.in
IBPS RRB Clerk Prelims – Ultra Practice Bundle PDF

6. Only 2 persons sit between H and I who does not


sit adjacent to M.

Page 37 of 722

Subscribe the Xpress Video Course & Mock Test Package for Bank & Insurance Exams
If there are any suggestions/ errors in our PDFs Feel Free to contact us via this email: admin@exampundit.in
IBPS RRB Clerk Prelims – Ultra Practice Bundle PDF

7. N who is not facing centre and not immediate


neighbour of I.

Page 38 of 722

Subscribe the Xpress Video Course & Mock Test Package for Bank & Insurance Exams
If there are any suggestions/ errors in our PDFs Feel Free to contact us via this email: admin@exampundit.in
IBPS RRB Clerk Prelims – Ultra Practice Bundle PDF
9.c
10.e
SOLUTION (11-15):
1. Only one person sit between L and N.
2. Vacant seat is left to N.
3. L sits fourth to the right of H.

Final Arrangement

4. O sits third to the left of P.


5. T sits immediate left of O.
6. P and L are not immediate neighbours.
6.c
7. O and N are not immediate neighbours.
7.a
8.c
Page 39 of 722

Subscribe the Xpress Video Course & Mock Test Package for Bank & Insurance Exams
If there are any suggestions/ errors in our PDFs Feel Free to contact us via this email: admin@exampundit.in
IBPS RRB Clerk Prelims – Ultra Practice Bundle PDF

Final Arrangement

8. Only one person sits between Q and R.


9. R sits second to the right of S.

11.a
Page 40 of 722

Subscribe the Xpress Video Course & Mock Test Package for Bank & Insurance Exams
If there are any suggestions/ errors in our PDFs Feel Free to contact us via this email: admin@exampundit.in
IBPS RRB Clerk Prelims – Ultra Practice Bundle PDF
12.c 6. R sits fifth to the right of H who is not immediate
13.c neighbor of L.
14.b
15.c
SOLUTION (16-20):
1. D sits third to the right of L.
2. B is an immediate neighbor of D.
3. Three persons sit between B and G.
4. Less than three persons sit between L and G when
counted to the left of L.

5. F sits immediate left of G.


7. Three persons sit between M and P.

Page 41 of 722

Subscribe the Xpress Video Course & Mock Test Package for Bank & Insurance Exams
If there are any suggestions/ errors in our PDFs Feel Free to contact us via this email: admin@exampundit.in
IBPS RRB Clerk Prelims – Ultra Practice Bundle PDF
8. M is immediate right of V.

Final arrangement

16.d
17.b
18.e
19.a
20.c
SOLUTION (21-25):
1. Q is sitting third to the left of U.
2. Two are sitting between U and T.
Page 42 of 722

Subscribe the Xpress Video Course & Mock Test Package for Bank & Insurance Exams
If there are any suggestions/ errors in our PDFs Feel Free to contact us via this email: admin@exampundit.in
IBPS RRB Clerk Prelims – Ultra Practice Bundle PDF
3. S is not near to W. 9. R is sitting second to the left of V, who is facing the
4. Two persons are sitting between W and T inside of the circle.
5. Q faces to the outside of the centre. 10. T is not opposite to U and R.

6. R is sitting third to the right of S.


7. S sits second to the right of P.
8. Among P, R and S one is facing inside the center of
the circle.

Page 43 of 722

Subscribe the Xpress Video Course & Mock Test Package for Bank & Insurance Exams
If there are any suggestions/ errors in our PDFs Feel Free to contact us via this email: admin@exampundit.in
IBPS RRB Clerk Prelims – Ultra Practice Bundle PDF
22.e (R and P are immediate neighbour)
23.c(Two person sits to the left S and V or R)
24.b
25.a
SOLUTION (26-30):
1. S sits second to the right of P.
2. T sits third to the left of S.
3. U sits opposite to Q and Q is not an immediate
neighbor of S.
4. S is facing same direction of P.

Final Arrangement

5. R neither sits opposite to T nor P.


21.c

Page 44 of 722

Subscribe the Xpress Video Course & Mock Test Package for Bank & Insurance Exams
If there are any suggestions/ errors in our PDFs Feel Free to contact us via this email: admin@exampundit.in
IBPS RRB Clerk Prelims – Ultra Practice Bundle PDF
6. Both R and U are facing same direction to each other 10. Both T and Q are facing opposite direction as V.
but opposite to S.
7. V sits Immediate to the right of Wwho is not
immediate neighbor of Q.
8. Both V and W are facing opposite direction.
9. W is facing outside the centre of the circle

Final Arrangement

26.e
27.b
28.c
29.d
30.e
SOLUTION (31-35):

Page 45 of 722

Subscribe the Xpress Video Course & Mock Test Package for Bank & Insurance Exams
If there are any suggestions/ errors in our PDFs Feel Free to contact us via this email: admin@exampundit.in
IBPS RRB Clerk Prelims – Ultra Practice Bundle PDF
1. N sit third to the left of Z.
2. Three persons sit between Z and H

7. Only one Person sits between W and O.


8. Neither W nor O faces H and Z

3. Three people sit between M and I


4. M is not immediate neighbour of Z
5. I face N
6. H sits second to the right of L

Page 46 of 722

Subscribe the Xpress Video Course & Mock Test Package for Bank & Insurance Exams
If there are any suggestions/ errors in our PDFs Feel Free to contact us via this email: admin@exampundit.in
IBPS RRB Clerk Prelims – Ultra Practice Bundle PDF

9. X does not face H.


Final Arrangement
10. J sits third to the left of X who does not face W.
11. Neither W nor O is opposite to J

31.d
32.b
33.c
34.e (five)
35.a
SOLUTION (36-40):
1. G faces inside.
2. D sits to the immediate left of G.
12. K is not an immediate neighbour of X. 3. C sits second to the right of D.
Page 47 of 722

Subscribe the Xpress Video Course & Mock Test Package for Bank & Insurance Exams
If there are any suggestions/ errors in our PDFs Feel Free to contact us via this email: admin@exampundit.in
IBPS RRB Clerk Prelims – Ultra Practice Bundle PDF
4. Only one person sits between C and E

5. Only two people sit between I and D.


6. H sits third to the right of E

Page 48 of 722

Subscribe the Xpress Video Course & Mock Test Package for Bank & Insurance Exams
If there are any suggestions/ errors in our PDFs Feel Free to contact us via this email: admin@exampundit.in
IBPS RRB Clerk Prelims – Ultra Practice Bundle PDF

36.a
7. C faces the same direction as that of E.
37.d
8. The immediate neighbours of C face opposite
38.b
directions.
39.c
9. Both the immediate neighbours of E face the same
40.a
direction as C
SOLUTION (41-45):
1. G sits 2nd to the left of B
2. H sits 3rd to the left of the person who sits
immediate right of B

Final Arrangement

Page 49 of 722

Subscribe the Xpress Video Course & Mock Test Package for Bank & Insurance Exams
If there are any suggestions/ errors in our PDFs Feel Free to contact us via this email: admin@exampundit.in
IBPS RRB Clerk Prelims – Ultra Practice Bundle PDF

5. There are three persons sit between X and E who


is not an immediate neighbor of G
3. E and J are immediate neighbors of I. 6. G and X are facing opposite direction as T are
4. B sits immediate right of T facing.

Page 50 of 722

Subscribe the Xpress Video Course & Mock Test Package for Bank & Insurance Exams
If there are any suggestions/ errors in our PDFs Feel Free to contact us via this email: admin@exampundit.in
IBPS RRB Clerk Prelims – Ultra Practice Bundle PDF
2. R seats immediate right of M.

3. There are two people between vacant seat and M.


4. L sits immediate right of vacant seats.
Final Arrangement

41.d
42.a
43.b
44.d
45.b
SOLUTION (46-50): 5. P sits immediate left of Q.
All the person facing Inside 6. There are two persons between N and J
1. M is fourth to the right of K.
Page 51 of 722

Subscribe the Xpress Video Course & Mock Test Package for Bank & Insurance Exams
If there are any suggestions/ errors in our PDFs Feel Free to contact us via this email: admin@exampundit.in
IBPS RRB Clerk Prelims – Ultra Practice Bundle PDF
7. There are two people between P and N when Final Arrangement
counted to the left of N.

46.b
47.d
48.c
49.d
50.e

3). Square Seating Arrangement


Directions (1-5): Study the following information S sits third to the right of R. Only two persons sit between
carefully to answer the given question: W and Q. V and W are immediate neighbors of S. P sits
There are eight friends P, Q, R, S, T, U, V and W seating third right of V. T does not facing outside the centre. U is
around a square table but not necessary in same order. not the immediate neighbor of R. W does not sit opposite
They are seating in a way that person who seated at corner to R.
facing outside the centre and person who seated at middle 1. Who sits third to the right of R?
of the side facing inside from the centre. (a) S
Page 52 of 722

Subscribe the Xpress Video Course & Mock Test Package for Bank & Insurance Exams
If there are any suggestions/ errors in our PDFs Feel Free to contact us via this email: admin@exampundit.in
IBPS RRB Clerk Prelims – Ultra Practice Bundle PDF
(b) T (b) V
(c) W (c) S
(d) P (d) U
(e) None of these (e) None of these
2. How many persons sit between P and W when Directions (6-10): Study the following information
counted to the left of P? carefully to answer the given question:
(a) None Eight friends namely –A, B, C, D, E, F, G and H are seated
(b) One around a square table with two person seats on each of the
(c) Two sides. All of them are facing outside the centre. G sits
(d) Three fourth to the right of H.F is sitting to the immediate right
(e) None of these of B. G is immediate neighbour of A and D. C is sitting
3. Which of the following person sits third to the left opposite to A and to the immediate left of E.
of Q? 6. Who sits fourth to right of B?
(a) U (a) H
(b) P (b) E
(c) R (c) C
(d) T (d) D
(e) None of these (e) None of these.
4. If we change the direction of S who sits second to 7. Who sits immediate left of H?
the right of S? (a) D
(a) P (b) B
(b) Q (c) E
(c) R (d) C
(d) T (e) None of these
(e) U 8. How many persons sit between B and E?
5. Who sits opposite to the one who sits immediate left (a) Four
of T? (b) One
(a) W (c) Three
Page 53 of 722

Subscribe the Xpress Video Course & Mock Test Package for Bank & Insurance Exams
If there are any suggestions/ errors in our PDFs Feel Free to contact us via this email: admin@exampundit.in
IBPS RRB Clerk Prelims – Ultra Practice Bundle PDF
(d) Five Apple laptop does sit immediate right of the one who has
(e) Either (b) or (d) HP laptop. C and the one who has Lenovo laptop is
9. Who sits to the immediate left of the person who immediate neighbour. E is an immediate neighbor of the
sits second to the right of A? one who has Sony Laptop and does not sits at corner of
(a) G the table. One person sits between Apple laptopand G. H
(b) D does not have Apple laptop.
(c) H 11. Which of the following person have apple laptop?
(d) E (a) E
(e) None of these (b) H
10. Which of the following pairs are immediate (c) C
neighbour? (d) D
(a) F,E (e) None of these
(b) F,D 12. How many persons sit between H and the one who
(c) F,C has HP laptop?
(d) C,D (a) One
(e) None of these (b) Two
Directions (11-15): Study the following information (c) Three
carefully to answer the given question: (d) Four
Eight persons A, B, C, D, E, F, G and H are sitting around (e) None of these
a square table facing centre but not necessarily in the same 13. Which of the following does not sits at corner of the
order. Four of them sits at corner and while others are table?
sitting at middle of the sides. Each one of them has (a) H
different Laptop such as Lenovo, Sony, Apple and HP. (b) A
Not more than two persons have same mobiles. (c) B
B sits second to the right of A. Two persons sit between B (d) C
and D. D and A are not immediate neighbours. Person (e) D
sitting on opposite sides have same mobiles. B has HP 14. A have which of the following laptop?
Laptop and sits second to the left of C. The one who has (a) Lenovo
Page 54 of 722

Subscribe the Xpress Video Course & Mock Test Package for Bank & Insurance Exams
If there are any suggestions/ errors in our PDFs Feel Free to contact us via this email: admin@exampundit.in
IBPS RRB Clerk Prelims – Ultra Practice Bundle PDF
(b) Sony one who likes Pink does not sits at corner. J does not like
(c) HP green colour. One who like Green colour sits immediate
(d) Apple left of one who like Yellow colour.
(e) None of these 16. Which of the following person likes Orange?
15. Which of the following person sits fifth to the left (a) V
of C? (b) S
(a) A (c) D
(b) C (d) Cannot be determined
(c) D (e) None of these
(d) G 17. Which of the following colour does M likes?
(e) None of these (a) Blue
Directions (16-20): Study the following information (b) Green
carefully to answer the given question: (c) Pink
Eight persons R, A, V, G, D, J, M and S sitting in square (d) Black
table. Four of them sits at corner and while others are (e) None of these
sitting at middle of the sides. The person who sits at corner 18. How many persons sit between D and the one who
facing towards centre and other faces outside. They all likes Yellow, when counted from right of D?
like different colours such as Pink, Green, Purple, Red, (a) None
Black, Blue, Yellow and Orange not necessarily in the (b) One
same order. R likes Blue colour. D sits immediate left of (c) Two
R. Only two persons were sitting between D and one who (d) Three
like Black colour. G sits immediate left of one who likes (e) None of these
black colour. Only three persons were sitting between A 19. Which of the following person sits opposite to each
and M. One who like orange colour sits third to the right other?
of M and that is not J. J sits third to the left of one who (a) M, D
like Black colour. Only one person was sitting between G (b) J, S
and V. One who like Pink coloursits immediate right of (c) G, R
the one who likes Red colour. D does not like Pink and the (d) V, J
Page 55 of 722

Subscribe the Xpress Video Course & Mock Test Package for Bank & Insurance Exams
If there are any suggestions/ errors in our PDFs Feel Free to contact us via this email: admin@exampundit.in
IBPS RRB Clerk Prelims – Ultra Practice Bundle PDF
(e) S, A (b) Q
20. If J and A interchange their place, then who sits (c) R
third to the left of J? (d) S
(a) S (e) T
(b) G 22. The age difference between V and P is how many
(c) The one who likes Red years?
(d) M (a) 12
(e) Both (c) and (d) (b) 13
Directions (21-25): Study the following information (c) 14
carefully to answer the given question: (d) 11
Eight persons P, Q, R, S, T, U, V and Ware sitting around (e) Either b or d
a square table in such a way that four of them sit at four 23. Which of the following combination is true?
corners, who are facing towards centre, and rest four who (a) T – 18years
sits at the middle of the side, are facing outside. They have (b) W- 27years
different ages in years i.e. 18, 5, 25, 49, 27, 16, 36 and 17. (c) S- 49years
P sits at the middle of the side and has age less than (d) V- 27years
20years. Only two persons sits between P and T. S’s age (e) U- 16years
is 25years sits second to the right of V. Difference 24. Sum of the age of immediate neighbor of R is how
between the ages of P’s and the one who sits immediate many years?
left of P is one. V does not face inside. The one whose age (a) 63
is 36years sits second to the left of W. Only one person (b) 65
sits between U and Q whose age is 49years. The one (c) 62
whose age is an odd number greater than 20years sits third (d) 61
to the right of S. R’s ages is even number. W sits third to (e) 60
the left of V. U’s age is a prime number. 25. Which of the following person sit opposite to U?
21. Who sits third to the left of the one whose age is (a) V
17years? (b) Q
(a) P (c) S
Page 56 of 722

Subscribe the Xpress Video Course & Mock Test Package for Bank & Insurance Exams
If there are any suggestions/ errors in our PDFs Feel Free to contact us via this email: admin@exampundit.in
IBPS RRB Clerk Prelims – Ultra Practice Bundle PDF
(d) T 28. How many persons are sitting between W and R,
(e) U when counted from left of R?
Directions (26-30): Study the following information (a) 2
carefully to answer the given question: (b) 3
Twelve persons M, N, O, P, Q, R, S, T, U, V, W and X are (c) 4
seated around a square table facing the outside the centre. (d) 5
There was one person on each of the corners and two (e) 6
persons on each of the sides. S was 3rd to the right of Q 29. Which of the following odd out of rest?
who was seated at one of the corners. P was 2nd to the (a)T, N
right of T who is opposite to N. N and V was seated at a (b) W, M
gap of 3 persons. V is opposite to S's neighbor. O was to (c) Q, P
the immediate left of R. P was seated opposite to U's (d) V, O
neighbor. W was 3rd to the right of X. W was seated (e) S, R
opposite to M. 30. Who sits third to the right of P?
26. Which among the following person sitting middle (a) Q
of the table? (b) R
(a) S (c) V
(b) R (d) N
(c) P (e) M
(d) T Directions (31-35): Study the following information
(e) Q carefully to answer the given question:
27. What is the position of M with respect to X? 12 friends O, P, Q, R, S, T, U, V, W, X, Y and Z sitting
(a) Second to the right. around a square table facing the centre of the table. There
(b) Fourth to the right was one person on each of the corners and two persons on
(c) Third to the right. each of the sides Q sits opposite to the person who sits
(d) Third to the left. adjacent to V. 2 people sit between O & P. P sits 2nd to
(e) None of these. the right of U who does not sits at corner of the table. S is
not adjacent to P. Neither P nor V is adjacent to Z. U sits
Page 57 of 722

Subscribe the Xpress Video Course & Mock Test Package for Bank & Insurance Exams
If there are any suggestions/ errors in our PDFs Feel Free to contact us via this email: admin@exampundit.in
IBPS RRB Clerk Prelims – Ultra Practice Bundle PDF
opposite to W who is to the immediate right of R. X is to (e) None of these.
the second to right of S and sits opposite to T.Y is not 35. If O and X interchange the positions then who faces
immediate neighbour of T. Y doesn’t sit at any corner of O?
the table (a) P
31. Who sits opposite to S? (b) Q
(a) Q (c) R
(b) R (d) S
(c) T (e) T
(d) U Directions (36-40): Study the following information
(e) P carefully to answer the given question:
32. Which of the following sits fifth to the right of U? J, W, O, V, P, U, H and G are sitting around a square table
(a) Q in such a way that four of them sit at four corners of the
(b) R square while four sit in the middle of each of the four
(c) S sides. The ones who sit at the four corners face the outside
(d) O while those who sit in the middle of the sides face inside.
(e) P Four of them are males and rests are females. Two males
33. Which of the following statement is true? sit in the middle of the sides and two at the corners. J sits
(a) Only two persons sit between V and U when second to the right of H. H sits in the corner of the table.
counted clockwise direction from V. O sits fourth to the left of her husband and her husband is
(b) Z and T are opposite to each other. not an immediate neighbour of J. W sits third to left of his
(c) R sits immediate right of S. wife. W does not sit at any of the sides. V sits adjacent to
(d) X and Q are immediate neighbour both W and G. G is the wife of J. P is a female. P and U
(e) All are true are not sitting opposite to each other.
34. What is the position of W with respect to T? 36. Who is husband of O?
(a) W sits fourth to the right of T. (a) U
(b) W sits fourth to the left of T. (b) G
(c) W sits third to the right of T. (c) V
(d) Both (a) and (b) (d) W
Page 58 of 722

Subscribe the Xpress Video Course & Mock Test Package for Bank & Insurance Exams
If there are any suggestions/ errors in our PDFs Feel Free to contact us via this email: admin@exampundit.in
IBPS RRB Clerk Prelims – Ultra Practice Bundle PDF
(e) None of these 8 friends P, Q, R, S, T, U, V and W sits around a square
37. What is the position of W with respect to his wife? table. 4 of them sits at four corners & 4 of them sits at the
(a) Immediate right centre of each side. Not more than 2 people facing the
(b) Second to the right same direction sits together. P sits 3rd to the right of S who
(c) Second to the left faces outside the centre. One person sits between P & R.
(d) Immediate left Immediate neighbors of W face the opposite direction. Q
(e) None of these sits at middle. T & U are immediate neighbours& both
38. Which of the following person are immediate faces opposite direction. W faces outside the table & is not
neighbor of O? an immediate neighbor of S. Two people are sitting
(a) W, J between R and V, who is facing outside the table. S & V
(b) W, G don't have three people in between them. V is not
(c) U, H immediate neighbor of either T or U also not opposite to
(d) G, O T.
(e) None of these 41. Who is to the immediate right of V?
39. How many person sit between G and J? (a) Q
(a) 1 (b) R
(b) 2 (c) S
(c) 3 (d) T
(d) 4 (e) None of these
(e) None of these 42. Who is to the third to the right of Q ?
40. Who sits third to the right of G? (a) S
(a) J (b) U
(b) P (c) P
(c) V (d) T
(d) W (e) None of these
(e) None of these 43. Who is to the immediate left of U?
Directions (41-45): Study the following information (a) P
carefully to answer the given question: (b) W
Page 59 of 722

Subscribe the Xpress Video Course & Mock Test Package for Bank & Insurance Exams
If there are any suggestions/ errors in our PDFs Feel Free to contact us via this email: admin@exampundit.in
IBPS RRB Clerk Prelims – Ultra Practice Bundle PDF
(c) T (a) E
(d) R (b) A
(e) Data inadequate (c) D
44. Who sits opposite to T? (d) F
(a) W (e) B
(b) V 47. Who sits second to the left of B ?
(c) S (a) B
(d) Q (b) D
(e) None of these (c) H
45. If we change the direction of V then who is third to (d) F
the left of V? (e) A
(a) T 48. Who is second to the right of E?
(b) S (a) G
(c) W (b) A
(d) V (c) C
(e) R (d) E
Directions (46-50): Study the following information (e) H
carefully to answer the given question: 49. How many persons sit between B and C, when
Eight persons are sitting in two square tables, 1 table is counted from left of B?
inner side of another table. 4 persons are sitting on each (a) One
table. The persons who sit in inner side of the table sit in (b) Four
the middle side and faces outside the center and the (c) No one
persons who sit on outer side of the square sit in the middle (d) Two
and faces inside. E sits second to the right of C who is not (e) Three
an immediate neighbor of G, who sits immediate left of F. 50. What is the position of E with respect to B?
H does not sit with F. B is to immediate right of C. G and (a) Third to the right
A faces each other. D does not sit at outer table. (b) Second to the right
46. Who sits to the immediate left of H? (c) Third to the left
Page 60 of 722

Subscribe the Xpress Video Course & Mock Test Package for Bank & Insurance Exams
If there are any suggestions/ errors in our PDFs Feel Free to contact us via this email: admin@exampundit.in
IBPS RRB Clerk Prelims – Ultra Practice Bundle PDF
(d) Second to the left (e) None of these

3). Square Seating Arrangement - Solution with Explanation

SOLUTION (1-5):
20. S sits third to the right of R
21. V and W are immediate neighbors of S.
22. W does not sit opposite to R.

25. T does not facing outside the centre.


26. U is not the immediate neighbor of R.

23. Only two persons sit between W and Q.


24. P sits third right of V

Page 61 of 722

Subscribe the Xpress Video Course & Mock Test Package for Bank & Insurance Exams
If there are any suggestions/ errors in our PDFs Feel Free to contact us via this email: admin@exampundit.in
IBPS RRB Clerk Prelims – Ultra Practice Bundle PDF

Case-1 gets eliminated since T is not facing outside the


centre.

Final Arrangement

1.a
2.c
3.c
4.b
5. d
SOLUTION (6-10):
1. G sits fourth to the right of H
2. A is immediate neighbour of G and D.

Page 62 of 722

Subscribe the Xpress Video Course & Mock Test Package for Bank & Insurance Exams
If there are any suggestions/ errors in our PDFs Feel Free to contact us via this email: admin@exampundit.in
IBPS RRB Clerk Prelims – Ultra Practice Bundle PDF

3. C is sitting opposite to A and to the immediate left of


E.

4. F is sitting to the immediate right of B.

Final Arrangements

Page 63 of 722

Subscribe the Xpress Video Course & Mock Test Package for Bank & Insurance Exams
If there are any suggestions/ errors in our PDFs Feel Free to contact us via this email: admin@exampundit.in
IBPS RRB Clerk Prelims – Ultra Practice Bundle PDF

6.d
7.c 6. The one who has Apple laptop does sit immediate
8.e right of the one who has HP laptop
9.a 7. C and the one who has Lenovo laptop is immediate
10.d neighbour.
SOLUTION (11-15): 8. Person sitting on opposite sides have same mobiles
1. B sits second to the right of A.
2. Two persons sit between B and D.
3. D and A are not immediate neighbours.
4. B has HP Laptop and sits second to the left of C.
5. Person sitting on opposite sides have same mobiles (It
means HP laptop are opposite)

9. One person sits between Apple laptop and G.

Page 64 of 722

Subscribe the Xpress Video Course & Mock Test Package for Bank & Insurance Exams
If there are any suggestions/ errors in our PDFs Feel Free to contact us via this email: admin@exampundit.in
IBPS RRB Clerk Prelims – Ultra Practice Bundle PDF
10. E is an immediate neighbor of the one who has Sony Final Arrangements
Laptop and does not sits at corner of the table.

11.a
12.c
13.e
14.b
15.e
SOLUTION (16-20):
1. R likes Blue colour.
2. D sits immediate left of R.
3. Only two persons were sitting between D and one
who like Black colour.
4. J sits third to the left of one who like Black colour
11. H does not have Apple laptop

Page 65 of 722

Subscribe the Xpress Video Course & Mock Test Package for Bank & Insurance Exams
If there are any suggestions/ errors in our PDFs Feel Free to contact us via this email: admin@exampundit.in
IBPS RRB Clerk Prelims – Ultra Practice Bundle PDF
9. J does not like green colour.
10. One who like Green colour sits immediate left of one
who like Yellow colour.

5. G sits immediate left of one who likes black colour.


6. Only one person was sitting between G and V.
7. Only three persons were sitting between A and M.
8. One who like orange colour sits third to the right of
M and that is not J

11. One who like Pink colour sits immediate right of the
one who likes Red colour.
12. D does not like Pink and the one who likes Pink does
not sits at corner.

13.
Page 66 of 722

Subscribe the Xpress Video Course & Mock Test Package for Bank & Insurance Exams
If there are any suggestions/ errors in our PDFs Feel Free to contact us via this email: admin@exampundit.in
IBPS RRB Clerk Prelims – Ultra Practice Bundle PDF

5. P sits at the middle of the side


Final Arrangements 6. Only two persons sits between P and T.
7. The one whose age is an odd number greater than
20years sits third to the right of S.

16.a
17.b
18.e
19.d
20.b
SOLUTION (21-25):
1. V does not face inside
2. S’s age is 25years sits second to the right of V.
3. W sits third to the left of V.
4. The one whose age is 36years sits second to the left
of W.

Page 67 of 722

Subscribe the Xpress Video Course & Mock Test Package for Bank & Insurance Exams
If there are any suggestions/ errors in our PDFs Feel Free to contact us via this email: admin@exampundit.in
IBPS RRB Clerk Prelims – Ultra Practice Bundle PDF
8. Difference between the ages of P’s and the one who
sits immediate left of P is one.
9. Only one person sits between U and Q whose age is
49 years.
10. R’s ages is even number (here only 18 year is left)
11. U’s age is a prime number.

21.c
22.e
23.b
24.a
25.d
SOLUTION (26-30):
1. S was 3rd to the right of Q who was seated at one of
the corners
2. V is opposite to S's neighbour.

Final Arrangements
Page 68 of 722

Subscribe the Xpress Video Course & Mock Test Package for Bank & Insurance Exams
If there are any suggestions/ errors in our PDFs Feel Free to contact us via this email: admin@exampundit.in
IBPS RRB Clerk Prelims – Ultra Practice Bundle PDF

3. N and V was seated at a gap of 3 persons.


4. P was 2nd to the right of T who is opposite to N.

8. Wwas 3rd to the right of X.

5. P was seated opposite to U's neighbor.


6. W was seated opposite to M.
7. O was to the immediate left of R

Final Arrangement
Page 69 of 722

Subscribe the Xpress Video Course & Mock Test Package for Bank & Insurance Exams
If there are any suggestions/ errors in our PDFs Feel Free to contact us via this email: admin@exampundit.in
IBPS RRB Clerk Prelims – Ultra Practice Bundle PDF

4. X is to the second to right of S and sits opposite to T.


26.d
27.e(4th )
28.b
29.d
30.b
SOLUTION (31-35):
1. P sits 2nd to the right of U who does not sits at corner
of the table.
2. S is not adjacent to P.
3. U sits opposite to W who is to the immediate right of
R.

Page 70 of 722

Subscribe the Xpress Video Course & Mock Test Package for Bank & Insurance Exams
If there are any suggestions/ errors in our PDFs Feel Free to contact us via this email: admin@exampundit.in
IBPS RRB Clerk Prelims – Ultra Practice Bundle PDF
5. Q sits opposite to the person who sits adjacent to V.

Final Arrangements

6. Neither P nor V is adjacent to Z


31.e
7. Y is not immediate neighbour of T.
32.d
8. Y doesn’t sit at any corner of the table
33.d
With the condition (8) case-2 arrangement gets
34.a
eliminated
35.e

Page 71 of 722

Subscribe the Xpress Video Course & Mock Test Package for Bank & Insurance Exams
If there are any suggestions/ errors in our PDFs Feel Free to contact us via this email: admin@exampundit.in
IBPS RRB Clerk Prelims – Ultra Practice Bundle PDF
SOLUTION (36-40):
1. J sits second to the right of H.
2. H sits in the corner.
3. W sits third to left of his wife.
4. W does not sit at any of the sides.

7. O sits fourth to the left of her husband and her


husband is not an immediate neighbour of J.
8. Two males sit in the middle of the sides and two at
the corners.

5. V sits adjacent to both W and G


6. G is the wife of J.

9. P is a female.
10. P and U are not sitting opposite to each other

Page 72 of 722

Subscribe the Xpress Video Course & Mock Test Package for Bank & Insurance Exams
If there are any suggestions/ errors in our PDFs Feel Free to contact us via this email: admin@exampundit.in
IBPS RRB Clerk Prelims – Ultra Practice Bundle PDF
Case-2 gets eliminated since P and U are not sitting 1. P sits 3rd to the right of S who faces towards the
opposite to each other. outside the centre.
2. One person sits between P & R.

Final Arrangement

36.a
37.e (3rd left)
38.a
39.c
40.b
SOLUTION (41-45):

Page 73 of 722

Subscribe the Xpress Video Course & Mock Test Package for Bank & Insurance Exams
If there are any suggestions/ errors in our PDFs Feel Free to contact us via this email: admin@exampundit.in
IBPS RRB Clerk Prelims – Ultra Practice Bundle PDF

3. Two people are sitting between R and V, who is


facing outside the table.
4. E sits second to the right of C who is not an immediate
neighbor of G.
5. S & V don't have three people in between them.
6. Not more than 2 people facing the same direction sits
together

7. T & U are immediate neighbours.


8. V is not immediate neighbor of either T or U also not
opposite to T.
9. W faces outside the table & is not an immediate
neighbor of S.

Page 74 of 722

Subscribe the Xpress Video Course & Mock Test Package for Bank & Insurance Exams
If there are any suggestions/ errors in our PDFs Feel Free to contact us via this email: admin@exampundit.in
IBPS RRB Clerk Prelims – Ultra Practice Bundle PDF

10. Immediate neighbors of W face the opposite


direction.
11. T and U both faces opposite direction
12. Not more than 2 people facing the same direction sits
together.

Final Arrangement

Page 75 of 722

Subscribe the Xpress Video Course & Mock Test Package for Bank & Insurance Exams
If there are any suggestions/ errors in our PDFs Feel Free to contact us via this email: admin@exampundit.in
IBPS RRB Clerk Prelims – Ultra Practice Bundle PDF
41.e(P)
42.b
43.c
44.e(P)
45.e
SOLUTION (46-50):
1. E sits second to the right of C who is not an immediate
neighbor of G.
2. B is to immediate right of C

6. D does not sit at outer table.

3. G and A faces each other.


4. G sits immediate left of F.
5. H does not sit with F

Final Arrangement
Page 76 of 722

Subscribe the Xpress Video Course & Mock Test Package for Bank & Insurance Exams
If there are any suggestions/ errors in our PDFs Feel Free to contact us via this email: admin@exampundit.in
IBPS RRB Clerk Prelims – Ultra Practice Bundle PDF
46.c
47.e
48.c
49.c
50.e(Immediate neighbour)

4). Rectangle Based Sitting Arrangement

Directions (1-5): Answer the questions based on the e. None of these


information given below. 2) Who among the following are the immediate
Eight persons A, B, C, D, E, F, G and H are sitting around neighbors of D?
a rectangular table but not in the same order such that four a. G and B
are sitting at the corner side of the table and face away b. G and C
from the center and other are sitting in the middle of the c. H and F
table and face towards the center. d. A and E
G sits at the corner of the table. Immediate neighbor of G e. None of these
sits second to the left of A. C is not an immediate neighbor 3) Four of the following five are related to each other
of G and A. C sits opposite to E, who is not an immediate in some way and thus formed a group. Choose the one
neighbor of G. B sits to the immediate right of E. H sits which does not belong to that group.
second to the left of D. a. E
1) How many persons sit between D and E when b. A
counted from the right of D? c. D
a. One d. H
b. Two e. B
c. Three 4) Who sits second to the left of F?
d. Four a. D
Page 77 of 722

Subscribe the Xpress Video Course & Mock Test Package for Bank & Insurance Exams
If there are any suggestions/ errors in our PDFs Feel Free to contact us via this email: admin@exampundit.in
IBPS RRB Clerk Prelims – Ultra Practice Bundle PDF
b. E e. G
c. H 7) Who sits opposite to H?
d. C a. G
e. None of these b. F
5) Who among the following sits third to the right of c. C
B? d. A
a. E e. None of the above
b. A 8) Find the odd one out.
c. F a. H
d. H b. C
e. None of these c. F
Direction (6-10): Answer the questions based on the d. B
information given below. e. G
Eight persons A, B, C, D, E, F, G and H are sitting around 9) Who sits to the immediate left of G?
the rectangular table. Four persons sit on corners while a. B
four sits on the sides. Persons at the corners face towards b. F
the centre while persons at sides face away from the c. A
centre. d. C
A sits opposite to E, who sits second to the right of D. H e. H
sits immediate right of B, who sits at one of the sides. G 10) How many persons sit between H and E when
is not adjacent to D. F does not sit at any side. C does not counted from the left of E?
sit immediate right of D.A and B are not immediate a. Three
neighbours. b. Five
6) Who sits second to the right of A? c. Four
a. F d. One
b. E e. Two
c. D Directions (11-15): Answer the questions based on the
d. B information given below.
Page 78 of 722

Subscribe the Xpress Video Course & Mock Test Package for Bank & Insurance Exams
If there are any suggestions/ errors in our PDFs Feel Free to contact us via this email: admin@exampundit.in
IBPS RRB Clerk Prelims – Ultra Practice Bundle PDF
Eight persons P, Q, R, S, T, U, V and W are sitting around c. 1
the rectangular table facing towards the centre. Two d. 3
persons sit on each side. Each of them has different e. None of these
number of houses from 1 to 8. 14) Who among the following has 7 houses?
S sits opposite to V, who sits to the immediate left of one, a. W
who has 8 houses. P has lesser houses than that of Q. T b. U
sits to the immediate left of R, who has 6 houses. T and R c. R
sit on the same side. S has double the number of houses of d. T
Q. P sits adjacent to one, who has 4 houses. Q does not sit e. None of these
adjacent to V. W has odd number of houses. The one, who 15) Who sits opposite to W?
has 1 house, sits opposite to one, who has 7 houses. Person a. The one, who has 6 houses
with 3 houses sits to the immediate left of V. b. The one, who has 3 houses
11) Who sits third to the left of R? c. Q
a. P d. T
b. Q e. None of these
c. The one, who has 4 houses Direction (16-20): Answer the questions based on the
d. The one, who has 8 houses information given below.
e. None of these Eight persons P, Q, R, S, A, B, C and D are sitting in a
12) Who sits opposite to R? rectangular table facing center such that two persons are
a. U sitting at each side of the rectangular and none of the
b. V persons is sitting at the corners of the rectangular.
c. P B sits to the immediate right of S. Three persons sit
d. R between B and A. R is an immediate neighbor of A but
e. None of these does not sit adjacent to Q. D sits third to the right of R. Q
13) What is the difference in the number of houses W and P are not immediate neighbors of each other. Q and C
and R? do not sit on the same sides of the table. P and C are not
a. 4 immediate neighbors of each other.
b. 2 16) Who sits 2nd to the right of Q?
Page 79 of 722

Subscribe the Xpress Video Course & Mock Test Package for Bank & Insurance Exams
If there are any suggestions/ errors in our PDFs Feel Free to contact us via this email: admin@exampundit.in
IBPS RRB Clerk Prelims – Ultra Practice Bundle PDF
a. A c. D and S
b. R d. A and P
c. S e. B and Q
d. B Direction (21-25): Answer the questions based on the
e. None of these information given below.
17) Who sits on the immediate right of A? Eight persons are sitting around a rectangular table. Four
a. Q persons are sitting at middle of the sides of the rectangular
b. R and are facing towards inside. Remaining four are sitting
c. P at corners and they face outside.
d. B Two persons are sitting between A and F. C, who is an
e. None of these immediate neighbor of A, sits opposite to D. E sits 3rd to
18) Who sits on the same side on which C sits? the right of G. H sits immediate right of E. B sits opposite
a. S to H, who does not face the towards the centre.
b. A 21) Who sits to the immediate right of C?
c. Q a. E
d. R b. A
e. None of these c. H
19) Who sits opposite to B on the table? d. F
a. P e. None of the above
b. Q 22) Who sits opposite to F?
c. A a. H
d. D b. C
e. None of these c. B
20) Four of the following five are related to each other d. E
in some way and thus formed a group. Choose the one e. None of the above
which does not belong to that group. 23) ___ sits 2nd to the left of G.
a. R and C a. B
b. P and Q b. H
Page 80 of 722

Subscribe the Xpress Video Course & Mock Test Package for Bank & Insurance Exams
If there are any suggestions/ errors in our PDFs Feel Free to contact us via this email: admin@exampundit.in
IBPS RRB Clerk Prelims – Ultra Practice Bundle PDF
c. A Anil. Ram faces towards the center. Veer is not an
d. D immediate neighbor of Anuj.
e. None of these 26) Who sits third to the left of Ram?
24) What is the position of H with respect to F? a. Kapil
a. 3rd to the right b. Anuj
b. Immediate left c. Kumar
c. 3rd to the left d. Veer
d. 4th to the right e. None of these
e. None of the above 27) Who sits diagonally opposite to Veer in the table?
25) Find the odd one out. a. Anuj
a. H b. Kumar
b. B c. Ram
c. A d. Amit
d. G e. None of these
e. C 28) Who sits to the immediate right of Anil?
Direction (26-30): Answer the questions based on the a. Kumar
information given below. b. Veer
Eight friends Ram, Anuj, Shiv, Amit, Anil, Kumar, Kapil c. Ram
and Veer are sitting around a rectangular table in such a d. Amit
way four of them sit at the four corners of the rectangular e. None of these
while four sit in the middle of each of four sides. Those 29) Four of the following five are related to each other
who sit at the four corners face away from the center while in some way and thus formed a group. Choose the one
those who sit in the middle of the sides face towards the which does not belong to that group.
center of the rectangular table. a. Ram
Kapil sits third to the right of Shiv, who faces towards the b. Anil
center. Amit sits third to the left of Anil, who does not sit c. Amit
in the middle of the sides. Only one person sits between d. Anuj
Amit and Anuj. Anuj is not an immediate neighbor of e. Shiv
Page 81 of 722

Subscribe the Xpress Video Course & Mock Test Package for Bank & Insurance Exams
If there are any suggestions/ errors in our PDFs Feel Free to contact us via this email: admin@exampundit.in
IBPS RRB Clerk Prelims – Ultra Practice Bundle PDF
30) Who among the following sits second to the right 32) Who sits 4th to the left of U?
of Amit? a. R
a. Kapil b. V
b. Veer c. Either (a) or (b)
c. Anuj d. T
d. Kumar e. None of these
e. None of these 33) What is the position of W with respect to Q?
Directions (31-35): Answer the questions based on the a. 2nd to the right
information given below: b. Immediate left
Eight persons, P, Q, R, S, T, U, V and W are sitting at the c. 3rd to the right
four corners and four sides of a rectangular table. Number d. 2nd to the left
of persons facing away from the centre is not equal to the e. None of these
number of persons facing the centre. 34) Counting from the left of T, how many persons sit
Q is sitting 2nd to the right of P, who is facing away from between T and U?
the centre. Q is sitting adjacent to U, who is not sitting in a. Five
the middle of any sides. S is sitting 2nd to the right of U b. Two
but not adjacent to P. Both V and R are sitting c. Three
immediately left of each other. W is not sitting at the d. One
corner of the table. Immediate neighbors of W are facing e. None of these
the opposite direction of W. T and S are facing the 35) ______sits to the immediate right of R.
opposite directions. W is not sitting opposite to Q. T is not a. V
facing outside. S and Q are not immediate neighbours. b. S
31) Who sits 2rd to the right of P? c. P
a. V d. Cannot be determined
b. Q e. None of these
c. S Directions (36-40): Answer the questions based on the
d. U information given below.
e. None of these
Page 82 of 722

Subscribe the Xpress Video Course & Mock Test Package for Bank & Insurance Exams
If there are any suggestions/ errors in our PDFs Feel Free to contact us via this email: admin@exampundit.in
IBPS RRB Clerk Prelims – Ultra Practice Bundle PDF
Eight persons, Payal, Pihu, Priya, Priyank, Param, Preeti, e. None of the above
Pratik and Praveen are sitting at four corners and four 39) _______ is sitting adjacent to Pihu.
sides of a rectangular table. Persons sitting at corners are a. Priyank
facing the centre, while others are facing away from the b. Preeti
centre. c. Either (a) or (b)
Pratik is sitting 2nd to the right of Pihu. Payal is sitting 3rd d. Both (a) and (b)
to the right of Pratik. Preeti is sitting 2nd to the left of e. None of the above
Payal. One person is sitting between Preeti and Priyank. 40) What is the position of Praveen with respect to
Neither Praveen nor Priya is facing the centre. Param is Priyank?
not sitting adjacent to Priya. a. 3rd to the left
36) Find the odd one out. b. 2nd to the right
a. Param c. 3rd to the right
b. Preeti d. 4th to the left
c. Priya e. None of the above
d. Priyank Directions (41-45): Answer the questions based on the
e. Payal information given below.
37) _____ is sitting opposite to Param. Eight persons A, B, C, D, E, F, G and H sit around the
a. Preeti rectangular table such that four persons sit on the corners
b. Praveen while four sit on the sides. Persons at the sides face
c. Payal towards the centre while persons at the corners face away
d. Priyank from the centre.
e. None of the above D does not sit adjacent to C or G. C sits third to the right
38) How many persons are sitting between Pratik and of G. H faces towards the centre and sits opposite to B. D
Praveen, when counted from the left of Pratik? and A are immediate right of each other. F is not
a. 3 immediate left of G. E is not adjacent to C.
b. 5 41) Who sits opposite to G?
c. 1 a. B
d. 4 b. D
Page 83 of 722

Subscribe the Xpress Video Course & Mock Test Package for Bank & Insurance Exams
If there are any suggestions/ errors in our PDFs Feel Free to contact us via this email: admin@exampundit.in
IBPS RRB Clerk Prelims – Ultra Practice Bundle PDF
c. F Directions (46-50): Answer the questions based on the
d. A information given below.
e. None of the above Eight persons P, Q, R, S, T, U, V and W are sitting at the
42) Who sits to the immediate left of D? four corners and four sides of a rectangular table. The ones
a. H sitting at the corner of the table face away from the center
b. B and the one sitting in the middle of the sides of the table
c. G face towards the center. The names of the two persons
d. Either (a) or (b) sitting opposite to each other are not the succeeding or
e. F preceding letters of each other according to alphabetical
43) Find the odd one out. series.
a. F P sits to the immediate left to V, who sits at the corner of
b. E the table. Only one person sit between P and W. R sits
c. C immediate left of Q. P sits opposite to one of the
d. D immediate neighbors of R. T does not sit at any corner. U
e. A does not sit opposite to P.
44) Who sits to the immediate right of G? 46) Who sits opposite to U?
a. E a. Q
b. F b. R
c. B c. W
d. A d. S
e. D e. None of the above
45) Who sits opposite to E? 47) Who sits second to the left of R?
a. A a. V
b. G b. W
c. C c. P
d. F d. S
e. None of the above e. None of the above
48) Who sits third to the right of U?
Page 84 of 722

Subscribe the Xpress Video Course & Mock Test Package for Bank & Insurance Exams
If there are any suggestions/ errors in our PDFs Feel Free to contact us via this email: admin@exampundit.in
IBPS RRB Clerk Prelims – Ultra Practice Bundle PDF
a. W c. P
b. T d. U
c. S e. T
d. Q 50) Which of the following persons are immediate
e. None of the above neighbors of S?
49) Four of the following five are related to each other a. U and Q
in some way and thus formed a group. Choose the one b. P and T
which does not belong to that group. c. T and W
a. Q d. Q and V
b. W e. None of the above

4). Rectangle Based Sitting Arrangement - Detailed Explanation with


Solutions
1. b
2. b
3. a (E sitting at corner and faces outwards)
4. d
5. c
Solution 1-5
1. G sits at the corner of the table. Immediate neighbor of
G sits second to the left of A.
2. C is not an immediate neighbor of G and A. C sits Case II:
opposite to E who is not an immediate neighbor of G.
Case I:

Page 85 of 722

Subscribe the Xpress Video Course & Mock Test Package for Bank & Insurance Exams
If there are any suggestions/ errors in our PDFs Feel Free to contact us via this email: admin@exampundit.in
IBPS RRB Clerk Prelims – Ultra Practice Bundle PDF
Case 1:

3. B sits to the immediate right of E so, case II is invalid.


H sits second to the left of D.
The final arrangement is as follows:
Case 2:

Answers
6. c
7. b
8. d (B is sitting at middle and faces outwards) 2. H sits immediate right of B, who sits at one of the sides.
9. a 3. G is not adjacent to D.
10. c 4. F does not sit at any side.
Solution 6-10 5. C does not sit immediate right of D.
1. A sits opposite to E, who sits second to the right of D. So, case 2 is rejected as F cannot sit in the arrangement in
So, E can sit at the side or corner in order to make two case 2.
cases.
Page 86 of 722

Subscribe the Xpress Video Course & Mock Test Package for Bank & Insurance Exams
If there are any suggestions/ errors in our PDFs Feel Free to contact us via this email: admin@exampundit.in
IBPS RRB Clerk Prelims – Ultra Practice Bundle PDF

Answers
11. c Case 2:
12. a
13. d
14. d
15. c
Solution 11-15
1. S sits opposite to V, who sits immediate left of one, who
has 8 houses.
It means V and the one, who has 8 houses, can sit on same
side or on the side adjacent to each other.
2. The one, who has 1 house, sits opposite to one, who has
Case 1:
7 houses.
3. S has double number of houses than that of Q.
4. P has lesser houses than that of Q.
5. P sits adjacent to one, who has 4 houses.
6. Q does not sit adjacent to V.
7. W has odd number of houses.
8. Person with 3 houses sits immediate left of V.
Page 87 of 722

Subscribe the Xpress Video Course & Mock Test Package for Bank & Insurance Exams
If there are any suggestions/ errors in our PDFs Feel Free to contact us via this email: admin@exampundit.in
IBPS RRB Clerk Prelims – Ultra Practice Bundle PDF
So P, Q and S must have 1, 2 and 4 houses respectively.
So, case 2 is rejected. W must have 3 houses.

Case II:

Answers
16. b
17. c
18. d
19. a
20. b (P and Q are not neighbors)
Solution 16-20
1. B sits to the immediate right of S. 4. Q and P are not immediate neighbors of each other.
2. Three persons sit between B and A. 5. Q and C do not sit on the same sides of the table. P and
3. R is an immediate neighbor of A. D sits third to the right C are not an immediate neighbor of each other.
of R. So, we can’t fix the positions of Q and P and C in case I
Case I: so, case I is invalid.
The final arrangement is as follows:

Page 88 of 722

Subscribe the Xpress Video Course & Mock Test Package for Bank & Insurance Exams
If there are any suggestions/ errors in our PDFs Feel Free to contact us via this email: admin@exampundit.in
IBPS RRB Clerk Prelims – Ultra Practice Bundle PDF

3. Two persons are sitting between A and F.


Answers 4. C who is an immediate neighbor of A, sits opposite to
21. b D.
22. d So, C faces D which means C is sitting immediate left of
23. a B.
24. c The final arrangement is as follows:
25. e (C is sitting at middle and faces inwards)
Solution 21-25
1.E sits 3rd to the right of G which means either G sits at
the middle or corner of the table.
2. H sits immediate right of E. B sits opposite to H, who
is sitting at the corner.
It means: H does not face towards the centre of the table
which means H sits at the corner of the table so if H sits at
the corner which means G also sits at the corner

Answers
26. a
27. b
28. c

Page 89 of 722

Subscribe the Xpress Video Course & Mock Test Package for Bank & Insurance Exams
If there are any suggestions/ errors in our PDFs Feel Free to contact us via this email: admin@exampundit.in
IBPS RRB Clerk Prelims – Ultra Practice Bundle PDF
29. b (Anil sitting at corner and faces outwards) 5. Veer is not an immediate neighbor of Anuj which is not
30. c possible in case II, so this case is invalid.
Solution 26-30 The final arrangement is as follows:
1. Kapil sits third to the right of Shiv, who faces towards
the center.
2. Amit sits third to the left of Anil, who does not sit in the
middle of the sides.
Case I:

Answers
31. b
32. c
Case II: 33. a
34. d
35. d (either P or S)
Solution 31-35
1. Q is sitting 2nd to the right of P, who is facing away from
the centre.
2. Q is sitting adjacent to U, who is not sitting in the
middle of any sides, i.e. P must be sitting in the middle of
3. Only one person sits between Amit and Anuj. Anuj is the side.
not an immediate neighbor of Anil. 3. S is sitting 2nd to the right of U but not adjacent to P, i.e.
4. Ram faces towards the center. U must be facing away from the centre.

Page 90 of 722

Subscribe the Xpress Video Course & Mock Test Package for Bank & Insurance Exams
If there are any suggestions/ errors in our PDFs Feel Free to contact us via this email: admin@exampundit.in
IBPS RRB Clerk Prelims – Ultra Practice Bundle PDF
4. W is not sitting at the corner of the table and sitting 38. b
opposite to Q. T is not facing outside. 39. d
5. Both V and R are sitting immediately left of each other. 40. a
6. Immediate neighbors of W are facing the opposite Solution 36-40
direction of W. 1. Pratik is sitting 2nd to the right of Pihu.
Case 1: When U is sitting adjacent to P. Both T and S are 2. Payal is sitting 3rd to the right of Pratik.
facing the opposite direction, so this case is not possible. 3. Preeti is sitting 2nd to the left of Payal.
4. One person is sitting between Preeti and Priyank.
5. Param is not sitting adjacent to Priya.
It means Payal is sitting opposite to Priyank and Preeti is
sitting adjacent to Pihu.
Case 1: When Pihu is sitting at the corner. Neither Praveen
nor Priya is facing the centre, so this case is not possible.

Case 2: When U is sitting 3rd to the right of P.


The final arrangement is as follows:

Case 2: When Pihu is sitting in the middle of the side.


The final arrangement is as follows:

Answers
36. c (Priya sits at middle and faces outwards)
37. a

Page 91 of 722

Subscribe the Xpress Video Course & Mock Test Package for Bank & Insurance Exams
If there are any suggestions/ errors in our PDFs Feel Free to contact us via this email: admin@exampundit.in
IBPS RRB Clerk Prelims – Ultra Practice Bundle PDF
Case 2:

3. D and A are immediate right of each other.


Answers
4. D does not sit adjacent to C or G.
41. d
5. F is not immediate left of G.
42. d
6. E is not adjacent to C.
43. e (A sits at middle and faces inwards)
So, case 2 is rejetced as E cannot sit adjacnet to C.
44. b
45. c
Solution 41-45
1. C sits third to the right of G.
2. H faces towards the centre and sits opposite to B.
So, H and B must be sitting at the sides.
Case 1:

Answers
46. b
47. a
48. d
49. d (U sits at corner and faces outwards)
50. c

Page 92 of 722

Subscribe the Xpress Video Course & Mock Test Package for Bank & Insurance Exams
If there are any suggestions/ errors in our PDFs Feel Free to contact us via this email: admin@exampundit.in
IBPS RRB Clerk Prelims – Ultra Practice Bundle PDF
Solution 46-50 4. T does not sit at any corner. U does not sit opposite to
1. P sits immediate left to V, who sits at the corner of the P.
table. So, T must be sitting opposite to P. Also, S can’t sit
2. Only one person sit between P and W. R sits immediate opposite to R which means U is sitting opposite to R.
left of Q. The final arrangement is as follows:
3. P sits opposite to one of the immediate neighbors of R.
So, W either sits to the right or left of P but R sits
immediate left of Q and one of the immediate neighbors
of R sits opposite to P which is not possible when W sits
to the left of P. So, W must be sitting to the right of P.

5). Triangular Seating


Directions (1-5): Study the following information Gaurav sits second to the right of Dolly. Two persons sit
carefully and answer the questions given below: between Gaurav and Vikky. Kinjal sits to the immediate
Six persons i.e. Gaurav, Kinjal, Dolly, Naina, Vikky and left of Vikky. Two persons sit between Paras and Kinjal.
Paras are sitting around a triangular table. Three of them Naina does not sit at the middle side of the table.
are sitting at the corner of the table and the other three are
sitting at the middle of the table. All of them are facing 1. Which among the following pairs sit immediate left
inside the table. and immediate right of each other’s respectively?

Page 93 of 722

Subscribe the Xpress Video Course & Mock Test Package for Bank & Insurance Exams
If there are any suggestions/ errors in our PDFs Feel Free to contact us via this email: admin@exampundit.in
IBPS RRB Clerk Prelims – Ultra Practice Bundle PDF
A) Gaurav, Paras 5. Who sits exactly between Naina and Paras?
B) Vikky, Dolly A) Person sits second to the right of Dolly
C) Naina, Kinjal B) Person sits third to the left of Kinjal
D) Both A and B C) Person sits fifth to the right of Paras
E) Both B and C D) Both A and B
E) Both A and C
2.How many persons sit between Vikky and the person
sits to the immediate right of Gaurav? Directions (6-10): Study the following information
A) 1 carefully and answer the questions given below:
B) 2 Five persons i.e. Mishti, Samir, Pulkit, Akshat and Kiran
C) 3 are sitting around a six seated (three seats at the corner and
D) 4 three seats at the middle) triangular table. One seat of the
E) Both A and C table is Vacant. All of them are facing inside the table.
Akshat and Kiran are immediate neighbours of each other.
3. Who among the following sits to the immediate left Mishti faces Kiran. Pulkit and Samir are sitting opposite
of the person who sits third to the left of Paras? to each other. One seat is there between Samir’s seat and
A) Person sits second to the right of Vikky Mishti’s seat. Vacant place is in the middle of the table.
B) Person sits in front of Dolly Kiran doesn’t sit second to the left of Pulkit.
C) Person sits to the immediate left of Gaurav
D) Both A and B 6. Who among the following sits exactly between
E) Both A and C Mishti and Samir?
4. __ Sits fifth to the left of Dolly and sits __ to the left A) Person sits fourth to the left of Mishti
of Kinjal. B) Person sits third to the right of Samir
A) Gaurav, Fourth C) No one
B) Paras, Fifth D) Either A or C
C) Gaurav, Fifth E) Either B or C
D) Paras, Third
E) None of These 7. __ sit to the immediate left of Kiran.
Page 94 of 722

Subscribe the Xpress Video Course & Mock Test Package for Bank & Insurance Exams
If there are any suggestions/ errors in our PDFs Feel Free to contact us via this email: admin@exampundit.in
IBPS RRB Clerk Prelims – Ultra Practice Bundle PDF
A) Person sits fifth to the left of Pulkit E) Both A and C
B) Person sits fifth to the right of Kiran
C) Person sits infront of Vacant seat Directions (11-15): Study the following information
D) Both A and B carefully and answer the questions given below:
E) All A, B and C Six persons i.e. K, L, M, N, O and P are sitting around a
triangular table. Two persons are sitting on each side of
8. How many persons sit between Akshat and Mishti the table. All of them are facing inside the table.
when counted from the right of Akshat? Two persons sit between M and O. N sits second to the
A) 4 right of O. One person sits between K and L. L is not the
B) 3 neighbour of N. P sits to the immediate right of M on the
C) 2 same side.
D) 1
E) 0 11. Which among the following pairs sit exactly
between O and N?
9. Which among the following pairs are immediate A) M
neighbours of each other? B) K
A) Person sits to the immediate left of Kiran and Pulkit C) P
B) Akshat and the person sits third to the left of Samir D) Both A and B
C) Person sits third to the left of Mishti and Pulkit E) Both B and C
D) Both A and B
E) Both A and C 12. Which of the following statements is/are true?
A) P sits second to the right of O
10. Who among the following sits second to the right of B) K sits fourth to the left of L
Akshat? C) P sits second to the left of N
A) Person sits to the immediate right of Kiran D) Both A and B
B) Person sits to the immediate left of Mishti E) Both A and C
C) Person sits second to the right of Pulkit
D) Both A and B
Page 95 of 722

Subscribe the Xpress Video Course & Mock Test Package for Bank & Insurance Exams
If there are any suggestions/ errors in our PDFs Feel Free to contact us via this email: admin@exampundit.in
IBPS RRB Clerk Prelims – Ultra Practice Bundle PDF
13.Who among the following sits fourth to the right of sitting on each side of the table. All of them are facing
M? inside the table.
A) Person sits exactly between N and O Tina sits second to the left of Prem. One person sits
B) Person sits third to the right of L between Tina and Anshu. Nidhi and Dhruv are immediate
C) Person sits third to the left of P neighbours of Anshu. Dhruv sits fourth to the right of
D) Both A and C Amit. Prem and Nidhi do not sit on the same side of the
E) Both A and B table.

14. Which of the following statements is/are not true? 16. Which among the following pair sits to the
A) L sits exactly between M and K immediate left and immediate right of each other
B) N sits to the immediate right of K respectively?
C) P sits exactly between N and L A) Tina, Dhruv
D) K sits second to the right of L B) Prem, Amit
E) All are True C) Nidhi, Anshu
D) Dhruv, Anshu
15. Who among the following pairs sit on the same E) None of These
side?
A) N, K 17. Who sits exactly between Amit and Nidhi?
B) P, O A) Person sits second to the left of Tina
C) M, L B) Person sits immediate right of Amit
D) K, O C) Person sits third to the left of Anshu
E) None of These D) Both A and C
E) Both B and C
Directions (16-20): Study the following information
carefully and answer the questions given below: 18. __ sits second to the right of ___
Six persons i.e. Prem, Dhruv, Nidhi, Anshu, Tina and A) Tina, Anshu
Amit are sitting around a triangular table. Two persons are B) Amit, Nidhi
C) Amit, Dhruv
Page 96 of 722

Subscribe the Xpress Video Course & Mock Test Package for Bank & Insurance Exams
If there are any suggestions/ errors in our PDFs Feel Free to contact us via this email: admin@exampundit.in
IBPS RRB Clerk Prelims – Ultra Practice Bundle PDF
D) Both A and C E sits third to the right of D. D does not sit at the corner of
E) Both A and B the table. B sits second to the left of D. C sits to the
immediate right of E. F sits between B and D. I sit third to
19. Who sits to the immediate right of Prem? the right of A. G sits at the middle side of the table but not
A) Person sits to the immediate right of Anshu with D.
B) Person sits to the immediate left of Anshu
C) Person sits second to the right of Dhruv 21. Who among the following sits fourth to the right of
D) Person sits second to the left of Amit E?
E) None of These A) Person sits second to the left of D
B) G
20. If all the persons sit in the alphabetical order C) B
starting from Amit in clockwise direction then the D) Both A and C
position of how many persons remain unchanged E) None of These
excluding Amit?
A) 4 22. Which of the following statements is/are true?
B) 3 A) H sits second to the right of F
C) 0 B) Three persons sit between G and A, from left of G
D) 2 C) B sits sixth to the right of H
E) 1 D) Both A and B
E) All A, B and C
Directions (21-25): Study the following information
carefully and answer the questions given below: 23. How many persons are between D and B when
Nine Persons i.e. A, B, C, D, E, F, G, H and I are sitting counted from the right of D?
around a triangular table such that three of them are sitting A) 8
at the corner of the table and two persons are sitting on B) 7
each side of the table. All of them are facing inside the C) 6
table. D) 5
E) None of these
Page 97 of 722

Subscribe the Xpress Video Course & Mock Test Package for Bank & Insurance Exams
If there are any suggestions/ errors in our PDFs Feel Free to contact us via this email: admin@exampundit.in
IBPS RRB Clerk Prelims – Ultra Practice Bundle PDF
24. _sits second to the left of C,and sits second to the 26. Who among the following sits third to the right of
right of __ Navya?
A) G and F A) Person sits second to the left of the vacant seat
B) A and the person sit second to the right of B B) Person sits second to the right of the vacant seat
C) Person sits second to the right of D and G C) Person sits to the immediate left of Arya
D) Both A and B D) Both A and B
E) None of These E) Both A and C

25. Who among the following sits exactly between I 27. How many persons are sitting between Komal and
and A? Janvi when counted from the left of Komal?
A) Person sits fourth to the right of C A) 4
B) Person sits third to the left of E B) 3
C) Person sits second to the left of G C) 2
D) Both A and B D) 5
E) Both A and C E) 1

Directions (26-30): Study the following information 28. Which among the following pairs sit to the
carefully and answer the questions given below: immediate left and to the immediate right of each other
Six persons i.e. Navya, Komal, Priya, Sanjhi, Arya and respectively?
Janvi are sitting around a nine seated (Three seats at A) Arya and Komal
corner and two seats on each side) triangular table. All of B) Priya and Janvi
them are facing inside the table. Three seats are vacant. C) Sanjhi and Navya
Navya sits to the immediate left of Sanjhi but on different D) Navya and Janvi
side of the table. Komal sits second to the right of Sanjhi. E) All of the Above
Priya sits three places away from Komal. Janvi sits second
to the left of Navya and third to the right of Arya. Arya 29. Which of the following statements is/are not true
sits at the corner of the table. regarding Janvi?
A) Janvi is an immediate neighbour of Priya
Page 98 of 722

Subscribe the Xpress Video Course & Mock Test Package for Bank & Insurance Exams
If there are any suggestions/ errors in our PDFs Feel Free to contact us via this email: admin@exampundit.in
IBPS RRB Clerk Prelims – Ultra Practice Bundle PDF
B) Komal sits third to the left of Janvi 31. Who among the following sits third to the left of
C) Janvi sits fourth to the left of Komal Nishi?
D) Both B and C A) Person sits to the immediate left of Payal
E) Both A and B B) Person sits second to the left of Anita
C) Person sits exactly between Payal and Nisha
30. If Sanjhi and Priya exchange their position then D) Both A and B
who among the following sits fifth to the right of E) None of These
Sanjhi?
A) Nil 32. Who among the following sits exactly between
B) Person sits immediate right of Komal Preeti and Mahima?
C) Person sits third to the right of Navya A) Person sits third to the left of Preeti
D) Both A and C B) Person sits fourth to the right of Nisha
E) None of These C) Person sits to the immediate left of the vacant seat
D) All A, B and C
Directions (31-35): Study the following information E) Both A and C
carefully and answer the questions given below:
Seven persons i.e. Maggie, Mahima, Payal, Nisha, Nishi, 33. Who among the following sits at the same side of
Anita and Preeti are sitting around a triangular table. the table?
Three seats at the corner of the table and two seats on each A) Preeti, Maggie
side of the table. All of them are facing inside the table. B) Anita and Nishi
Two seats are vacant but not adjacent. C) Mahima and the vacant seat
Payal who sits at the corner of the table is an immediate D) All of the Above
neighbour of Anita. Nishi sits second to the right of Payal. E) None of the Above
Nisha sits four places away from Nishi when counting
from the right of Nishi. Preeti sits to the immediate left of 34. How many persons are sitting between Maggie and
Nisha. Mahima sits second to the left of Anita. One of the Mahima?
corner seats of the table is vacant near nishi Maggie sits A) 5
immediate left of preeti. B) 4
Page 99 of 722

Subscribe the Xpress Video Course & Mock Test Package for Bank & Insurance Exams
If there are any suggestions/ errors in our PDFs Feel Free to contact us via this email: admin@exampundit.in
IBPS RRB Clerk Prelims – Ultra Practice Bundle PDF
C) 3 C) Third, Second
D) 2 D) Third, Fourth
E) Either C or D E) None of These

35. Which of the following statements is/are correct 37. Which among the following pairs sit adjacent to
regarding Anita? each other?
A) Anita sits third to the right of Nisha A) R and the person sit immediate left of P
B) Anita sits fourth to the right of Preeti B) W and the person sit immediate left of T
C) One person sits between Anita and Payal C) S and the person sit second to the right of S
D) All are True D) Both A and B
E) None is True E) Both A and C

Directions (36-40): Study the following information 38. How many persons are sitting between X and W
carefully and answer the questions given below: when counted from the right of X?
Nine persons i.e. P, Q, R, S, T, U, V, W and X are sitting A) 5
around a triangular table. Three persons sit on each side of B) 4
the table. All of them are facing inside the table. C) 3
One person sits between P and V. Both P and V are sitting D) 2
on the same side of the table. One person sits between V E) 1
and X. Q sits to the immediate right of X. R sits two places
away from P. U sits exactly between R and Q. W is not an 39. Who among the following sits fifth to the left of Q?
immediate neighbour of both R and X. Three persons sit A) T
between S and T when counted from the right of T. B) W
C) P
36. Q sits __ to the right of V who sits__ to the left of D) R
X. E) None of These
A) Second, Third
B) Fourth, Third 40. Which of the following statements is/are true?
Page 100 of 722

Subscribe the Xpress Video Course & Mock Test Package for Bank & Insurance Exams
If there are any suggestions/ errors in our PDFs Feel Free to contact us via this email: admin@exampundit.in
IBPS RRB Clerk Prelims – Ultra Practice Bundle PDF
A) P and V are sitting on the same side A) Jaya sits second to the right of the vacant seat
B) U sits fourth to the left of W B) One person sits between Jaya and Riya
C) R sits exactly between T and U C) Nimo sits second to the right of Jaya
D) All are True D) Both A and C
E) None is True E) All are True

Directions (41-45): Study the following information 43. Which of the following pair are immediate
carefully and answer the questions given below: neighbors?
Six persons i.e. Seema, Nimo, Ritu, Riya, Jaya and Pari A) Pari and Vacant seat
are sitting around a triangular table. There are three seats B) Jaya and Nimo
on each side of the table. All of them are facing outside C) Pari and Nimo
the table. Three seats are vacant but they are not adjacent. D) Both A and B
One person sits between Sima and Pari but not on the same E) Both A and C
side of the table. Nimo sits to the immediate right of Pari.
Riya sits two places away from Sima. Ritu sits alone on 44. Who among the following sits exactly between Jaya
one of the sides of the table. Jaya and Riya are not an and Nimo?
immediate neighbours of each other. Jaya does not sit A) Person sits third to the left of Ritu
adjacent to vacant. B) Person sits third to the right of Sima
C) Person sits fifth to the left of Riya
41. Who sits second to the right of Nimo? D) Both A and C
A) Person sits second to the right of Riya E) Both A and B
B) Person sits third to the right of Pari
C) Person sits exactly between Riya and Jaya 45. If Jaya and Riya exchange their positions then who
D) Both A and B among the following sits fifth to the right of Jaya?
E) Both A and C A) Nimo
B) Pari
42. Which of the following statements is/are true C) Ritu
regarding Jaya? D) Riya
Page 101 of 722

Subscribe the Xpress Video Course & Mock Test Package for Bank & Insurance Exams
If there are any suggestions/ errors in our PDFs Feel Free to contact us via this email: admin@exampundit.in
IBPS RRB Clerk Prelims – Ultra Practice Bundle PDF
E) None of These B) 4
C) 2
Directions (46-50): Study the following information D) 3
carefully and answer the questions given below: E) 0
Nine persons i.e. A, B, C, D, E, F, G, H and I are sitting
around a triangular table. All of them are facing outside 48. Who among the following sits fourth to the left of
the table. Three persons are sitting on each side of the I?
table. A) Person sits exactly between H and F
A and H are sitting to the immediate left and immediate B) Person sits third to the right of G
right of each other but on different sides of the table. E sits C) Person sits fifth to the left of B
second to the right o f H. G sits fourth to the left of H. B D) Both A and B
and D are immediate neighbours of G, B sits second to the E) All of the Above
left of D. Two persons sit between B and F who is not an
immediate neighbour of E. C sits second to the left of I. 49. How many persons sit between G and H when
counted from the left of G?
46. Who among the following pairs sit together on the A) 5
same side of the table? B) 4
A) I, Band E C) 6
B) A, F and D D) 3
C) B, G and H E) None of These
D) F, D and G
E) None of These 50. Which of the following statements is/are true?
A) B sits exactly between F and C
47. If all the persons sit in the alphabetical order B) B sits third to the left of A
starting from A in clockwise direction then the position C) A sit third to the left of E
of how many persons remain unchanged excluding A? D) Both A and C
A) 5 E) Both A and B

Page 102 of 722

Subscribe the Xpress Video Course & Mock Test Package for Bank & Insurance Exams
If there are any suggestions/ errors in our PDFs Feel Free to contact us via this email: admin@exampundit.in
IBPS RRB Clerk Prelims – Ultra Practice Bundle PDF

5). Triangular Seating - Solutions with Detailed Explanation


Answers (1-5):
1) E
2) E
3) B
4) D
5) A

Solutions (1-5):
STEP II: Two persons sit between Gaurav and Vikky.
STEP I: Gaurav sits second to the right of Dolly.
As per this statement, the arrangement will look like this:
As per this statement, there will be two cases getting
framed and they are as follows: CASE I:

CASE I:

CASEII:

CASE II:
Page 103 of 722

Subscribe the Xpress Video Course & Mock Test Package for Bank & Insurance Exams
If there are any suggestions/ errors in our PDFs Feel Free to contact us via this email: admin@exampundit.in
IBPS RRB Clerk Prelims – Ultra Practice Bundle PDF

STEP III: Kinjal sits to the immediate left of Vikky.

STEP IV: Paras faces Kinjal.


As per this statement, the arrangement will look like this:

As per this statement, the arrangement will look like this:


CASE I:

CASE I:

CASE II:

CASE II:

Page 104 of 722

Subscribe the Xpress Video Course & Mock Test Package for Bank & Insurance Exams
If there are any suggestions/ errors in our PDFs Feel Free to contact us via this email: admin@exampundit.in
IBPS RRB Clerk Prelims – Ultra Practice Bundle PDF

STEP V: Naina does not sit at the middle side of the table. Answers (6-10):

As per this statement, CASE II will get eliminated and we 6) D


will continue with CASE I and the final arrangement will 7) E
look like this: 8) C
9) D
CASE I: 10) A

Solutions (6-10):

STEP I: Akshat and Kiran are immediate neighbours of


each other. Mishti faces Kiran.

As per this statement, there will be two cases getting


framed and they are as follows:

CASE II:
CASE I:

Page 105 of 722

Subscribe the Xpress Video Course & Mock Test Package for Bank & Insurance Exams
If there are any suggestions/ errors in our PDFs Feel Free to contact us via this email: admin@exampundit.in
IBPS RRB Clerk Prelims – Ultra Practice Bundle PDF

Case-1 and Case-1-A gets eliminated since Vacant place


is in the middle of the table.
CASE II:
Case-2-A gets eliminated since One seat is there between
Samir’s seat and Mishti’s seat.

STEP II: Pulkit and Samir are sitting opposite to each


other. One seat is there between Samir’s seat and Final arrangement
Mishti’s seat. Vacant place is in the middle of the table.
Kiran doesn’t sit second to the left of Pulkit.

Page 106 of 722

Subscribe the Xpress Video Course & Mock Test Package for Bank & Insurance Exams
If there are any suggestions/ errors in our PDFs Feel Free to contact us via this email: admin@exampundit.in
IBPS RRB Clerk Prelims – Ultra Practice Bundle PDF

Answers (11-15):
CASE II:

11) E
12) B
13) D
14) C
15) A

Solutions (11-15):

STEP I: Two persons sit between M and O.

As per this statement, two cases will get framed and they
STEP II: N sits second to the right of O.
will look like this:

As per this statement, the arrangement will look like this:


CASE I:

CASE I:

Page 107 of 722

Subscribe the Xpress Video Course & Mock Test Package for Bank & Insurance Exams
If there are any suggestions/ errors in our PDFs Feel Free to contact us via this email: admin@exampundit.in
IBPS RRB Clerk Prelims – Ultra Practice Bundle PDF
CASE I:

CASE II:

CASE II:

STEP III: P sits to the immediate right of M on the same


side.

As per this statement, CASE II will get eliminated and we STEP IV: One person sits between K and L.
will continue with CASE I and the arrangement will look
like this: As per this statement, the arrangement will look like this:
Page 108 of 722

Subscribe the Xpress Video Course & Mock Test Package for Bank & Insurance Exams
If there are any suggestions/ errors in our PDFs Feel Free to contact us via this email: admin@exampundit.in
IBPS RRB Clerk Prelims – Ultra Practice Bundle PDF
CASE I: 17) B
18) D
19) B
20) E

Solutions (16-20):

STEP I: Tina sits second to the left of Prem.

As per this statement, there will be two cases getting


framed and they are as follows:

CASE I:
STEP V: L is not the neighbour of N.

As per this statement, the final arrangement will look like


this:

CASE II:

Answers (16-20):
16) C
Page 109 of 722

Subscribe the Xpress Video Course & Mock Test Package for Bank & Insurance Exams
If there are any suggestions/ errors in our PDFs Feel Free to contact us via this email: admin@exampundit.in
IBPS RRB Clerk Prelims – Ultra Practice Bundle PDF

STEP II: One person sits between Tina and Anshu. STEP III: Nidhi and Dhruv are immediate neighbours of
Anshu.
As per this statement, the arrangement will look like this:
As per this statement, the arrangement will look like this:
CASE I:
CASE I:

CASE II: CASE II:

Page 110 of 722

Subscribe the Xpress Video Course & Mock Test Package for Bank & Insurance Exams
If there are any suggestions/ errors in our PDFs Feel Free to contact us via this email: admin@exampundit.in
IBPS RRB Clerk Prelims – Ultra Practice Bundle PDF

STEP IV: Dhruv sits fourth to the right of Amit. STEP V: Prem and Nidhi do not sit on the same side of
the table

As per this statement, the arrangement will look like this:


As per this statement, CASE I will get eliminated and we

CASE I: will continue with CASE II and the final arrangement will
look like this:

CASE I:

CASE II:

Page 111 of 722

Subscribe the Xpress Video Course & Mock Test Package for Bank & Insurance Exams
If there are any suggestions/ errors in our PDFs Feel Free to contact us via this email: admin@exampundit.in
IBPS RRB Clerk Prelims – Ultra Practice Bundle PDF
CASE II: As per these statements, there will be two cases getting
framed and they will look like this:

CASE I:

Answers (21-25):
CASE II:

21) D
22) E
23) C
24) B
25) A

Solutions (21-25):

STEP I: E sits third to the right of D. D does not sit at the


corner of the table.
STEP II: B sits second to the left of D.

Page 112 of 722

Subscribe the Xpress Video Course & Mock Test Package for Bank & Insurance Exams
If there are any suggestions/ errors in our PDFs Feel Free to contact us via this email: admin@exampundit.in
IBPS RRB Clerk Prelims – Ultra Practice Bundle PDF
As per this statement, the arrangement will look like this: STEP III: C sits to the immediate right of E.

CASE I: As per this statement, the arrangement will look like this:

CASE I:

CASE II:
CASE II:

Page 113 of 722

Subscribe the Xpress Video Course & Mock Test Package for Bank & Insurance Exams
If there are any suggestions/ errors in our PDFs Feel Free to contact us via this email: admin@exampundit.in
IBPS RRB Clerk Prelims – Ultra Practice Bundle PDF
STEP IV: F sits between B and D. STEP V: I sit third to the right of A.

As per this statement, the arrangement will look like this: As per this statement, the arrangement will look like this:

CASE I: CASE I:

CASE II:
CASE II:

Page 114 of 722

Subscribe the Xpress Video Course & Mock Test Package for Bank & Insurance Exams
If there are any suggestions/ errors in our PDFs Feel Free to contact us via this email: admin@exampundit.in
IBPS RRB Clerk Prelims – Ultra Practice Bundle PDF
STEP VI: G sits at the middle side of the table but not with Answers (26-30):
D. 26) E
As per this statement, CASE II will get cancelled and we 27) C
will continue with CASE I and the final arrangement will 28) B
look like this: 29) D
30) A
CASE I: Solutions (26-30):

STEP I: Navya sits to the immediate left of Sanjhi but on


different side of the table.

As per this statement, there will be two cases getting


framed and the arrangement will look like this:

CASE I:

CASE II:

CASE II:
Page 115 of 722

Subscribe the Xpress Video Course & Mock Test Package for Bank & Insurance Exams
If there are any suggestions/ errors in our PDFs Feel Free to contact us via this email: admin@exampundit.in
IBPS RRB Clerk Prelims – Ultra Practice Bundle PDF

STEP II: Komal sits second to the right of Sanjhi.

As per this statement, the arrangement will look like this: STEP III: Priya sits three places away from Komal.

CASE I: As per this statement, the arrangement will look like this:
CASE I:

CASE II: CASE II:

Page 116 of 722

Subscribe the Xpress Video Course & Mock Test Package for Bank & Insurance Exams
If there are any suggestions/ errors in our PDFs Feel Free to contact us via this email: admin@exampundit.in
IBPS RRB Clerk Prelims – Ultra Practice Bundle PDF
CASE II:

STEP IV: Janvi sits second to the left of Navya and third
to the right of Arya.
STEP V: Arya sits at the corner of the table.

As per this statement, the arrangement will look like this:


As per this statement, CASE II will get cancelled and we

CASE I: will continue with CASE I and the final arrangement will
look like this:

CASE I:

Page 117 of 722

Subscribe the Xpress Video Course & Mock Test Package for Bank & Insurance Exams
If there are any suggestions/ errors in our PDFs Feel Free to contact us via this email: admin@exampundit.in
IBPS RRB Clerk Prelims – Ultra Practice Bundle PDF
CASE II: CASE I:

CASE II:
Answers (31-35):

31) D
32) E
33) D
34) E
35) E

Solutions (31-35):
STEP I: Payal who sits at the corner of the table is an
immediate neighbour of Anita.

As per this statement, there will be two cases getting STEP II: Nishi sits second to the right of Payal.
framed and they will look like this:
Page 118 of 722

Subscribe the Xpress Video Course & Mock Test Package for Bank & Insurance Exams
If there are any suggestions/ errors in our PDFs Feel Free to contact us via this email: admin@exampundit.in
IBPS RRB Clerk Prelims – Ultra Practice Bundle PDF
As per this statement, the arrangement will look like this: As per this statement, the arrangement will look like this:
CASE I:
CASE I:

CASE II: CASE II:

STEP III: Nisha sits four places away from Nishi when
STEP IV: Preeti sits to the immediate left of Nisha.
counting from the right of Nishi.

Page 119 of 722

Subscribe the Xpress Video Course & Mock Test Package for Bank & Insurance Exams
If there are any suggestions/ errors in our PDFs Feel Free to contact us via this email: admin@exampundit.in
IBPS RRB Clerk Prelims – Ultra Practice Bundle PDF
As per this statement, the arrangement will look like this: As per this statement, CASE II will get eliminated and we
will continue with CASE I and the arrangement will look
CASE I: like this:

CASE I:

CASE II:
CASE II:

STEP V: Mahima sits second to the left of Anita.

Page 120 of 722

Subscribe the Xpress Video Course & Mock Test Package for Bank & Insurance Exams
If there are any suggestions/ errors in our PDFs Feel Free to contact us via this email: admin@exampundit.in
IBPS RRB Clerk Prelims – Ultra Practice Bundle PDF
STEP VI: One of the corner seats of the table is vacant. STEP I: One person sits between P and V. Both P and V
Two seats are vacant but not adjacent. are sitting on the same side of the table.

As per these statements, the final arrangement will look As per this statement, there will be two cases getting
like this: framed and they will look like this:

CASE I: CASE I:

Answers (36-40): CASE II:

36) C
37) A
38) A
39) C
40) D

Solutions (36-40):

Page 121 of 722

Subscribe the Xpress Video Course & Mock Test Package for Bank & Insurance Exams
If there are any suggestions/ errors in our PDFs Feel Free to contact us via this email: admin@exampundit.in
IBPS RRB Clerk Prelims – Ultra Practice Bundle PDF
STEP II: One person sits between V and X. STEP III: Q sits to the immediate right of X.

As per this statement, the arrangement will look like this: As per this statement, the arrangement will look like this:

CASE I: CASE I:

CASE II:
CASE II:

Page 122 of 722

Subscribe the Xpress Video Course & Mock Test Package for Bank & Insurance Exams
If there are any suggestions/ errors in our PDFs Feel Free to contact us via this email: admin@exampundit.in
IBPS RRB Clerk Prelims – Ultra Practice Bundle PDF
STEP IV: R sits two places away from P. STEP V: U sits exactly between R and Q.

As per this statement, the arrangement will look like this: As per this statement, the arrangement will look like this:

CASE I: CASE I:

CASE II: CASE II:

Page 123 of 722

Subscribe the Xpress Video Course & Mock Test Package for Bank & Insurance Exams
If there are any suggestions/ errors in our PDFs Feel Free to contact us via this email: admin@exampundit.in
IBPS RRB Clerk Prelims – Ultra Practice Bundle PDF
STEP VI: W is not an immediate neighbour of both R and STEP VII: Three persons sit between S and T when
X. counted from the right of T.

As per this statement, the arrangement will look like this: As per this statement, CASE II will get eliminated and we
will continue with CASE I and the final arrangement will
CASE I: look like this:

CASE I:

CASE II:

CASE II:

Page 124 of 722

Subscribe the Xpress Video Course & Mock Test Package for Bank & Insurance Exams
If there are any suggestions/ errors in our PDFs Feel Free to contact us via this email: admin@exampundit.in
IBPS RRB Clerk Prelims – Ultra Practice Bundle PDF

CASE II:
Answers (41-45):

41) B
42) E
43) C
44) D
45) A

Solutions (41-45):
STEP I: One person sits between Sima and Pari but not
on the same side of the table.
STEP II: Nimo sits to the immediate right of Pari.

As per this statement, there will be two cases getting


As per this statement, the arrangement will look like this:
framed and they will look like this:

CASE I:
CASE I:

Page 125 of 722

Subscribe the Xpress Video Course & Mock Test Package for Bank & Insurance Exams
If there are any suggestions/ errors in our PDFs Feel Free to contact us via this email: admin@exampundit.in
IBPS RRB Clerk Prelims – Ultra Practice Bundle PDF

CASE II: CASE II:

STEP III: Riya sits two places away from Sima.

STEP IV: Ritu sits alone on one of the sides of the table.
As per this statement, the arrangement will look like this:
CASE I:
Page 126 of 722

Subscribe the Xpress Video Course & Mock Test Package for Bank & Insurance Exams
If there are any suggestions/ errors in our PDFs Feel Free to contact us via this email: admin@exampundit.in
IBPS RRB Clerk Prelims – Ultra Practice Bundle PDF
As per this statement, CASE I will get eliminated and we STEP V: Jaya and Riya are not an immediate neighbours
will continue with CASE II and the arrangement will look of each other. Three seats are vacant but they are not
like this: adjacent.
CASE I:
As per these statements, the final arrangement will look
like this:

CASE II:

CASE II:

Answers (46-50):

46) B
47) C
48) E
49) B

Page 127 of 722

Subscribe the Xpress Video Course & Mock Test Package for Bank & Insurance Exams
If there are any suggestions/ errors in our PDFs Feel Free to contact us via this email: admin@exampundit.in
IBPS RRB Clerk Prelims – Ultra Practice Bundle PDF
50) D

Solutions (46-50):

STEP I: A and H are sitting to the immediate left and


immediate right of each other but on different sides of the
table.

As per this statement, there will be getting two cases


framed and they will look like this:

CASE I: STEP II: E sits second to the right of H.

As per this statement, the arrangement will look like this:

CASE I:

CASE II:

Page 128 of 722

Subscribe the Xpress Video Course & Mock Test Package for Bank & Insurance Exams
If there are any suggestions/ errors in our PDFs Feel Free to contact us via this email: admin@exampundit.in
IBPS RRB Clerk Prelims – Ultra Practice Bundle PDF
CASE II: CASE II:

STEP III: G sits fourth to the left of H. STEP IV: B and D are immediate neighbours of G and B
sits to the left of D.
As per this statement, the arrangement will look like this:

As per this statement, the arrangement will look like this:


CASE I:

CASE I:

Page 129 of 722

Subscribe the Xpress Video Course & Mock Test Package for Bank & Insurance Exams
If there are any suggestions/ errors in our PDFs Feel Free to contact us via this email: admin@exampundit.in
IBPS RRB Clerk Prelims – Ultra Practice Bundle PDF
CASE II: CASE II:

STEP V: Two persons sit between B and F who is not an


immediate neighbour of E.
STEP VI: C sits second to the left of I.
As per this statement, the arrangement will look like this:
As per this statement, CASE I will get eliminated and we
CASE I: will continue with CASE I and the final arrangement will
look like this:

CASE I:

Page 130 of 722

Subscribe the Xpress Video Course & Mock Test Package for Bank & Insurance Exams
If there are any suggestions/ errors in our PDFs Feel Free to contact us via this email: admin@exampundit.in
IBPS RRB Clerk Prelims – Ultra Practice Bundle PDF

CASE II:

6). Certain Number of Persons Linear


Directions (1-5): Study the following information 1. Which among the following pairs sit immediate left
carefully and answer the questions given below: and immediate right of each other’s?
A certain number of persons are sitting in a row facing A) J,X
north. B) Z,K
J sits third to the right of K. Two persons sit between K C) W,M
and M. Only one person sits between M and L. W is an D) Both A and B
immediate neighbour of both M and L. X sits third to the E) Both B and C
right of Y who sits to the immediate left of J. Z is the only 2. If all the persons sit in the alphabetical order from
known neighbour of K and sits second to the right of M. left to right then position of how many persons remain
Number of person sits to the left of L is one less than the unchanged? (Applicable for known person’s seat)
number of persons sits to the right of X. Less than fourteen A) 3
persons sit in a row. L sits at one of the extreme ends of B) 2
the row. C) 0

Page 131 of 722

Subscribe the Xpress Video Course & Mock Test Package for Bank & Insurance Exams
If there are any suggestions/ errors in our PDFs Feel Free to contact us via this email: admin@exampundit.in
IBPS RRB Clerk Prelims – Ultra Practice Bundle PDF
D) 1 sits exactly in between Hina and Alex. Kulfi sits fifth to
E) 4 the left of Kajol. Sona sits third to the right of Dean. Mary
3. Who among the following sits to the immediate left and Sona sit at the extreme left end and the extreme right
of the person who sits third to the left of Z? end of the row respectively. Mona sits third to the right of
A) Person sits second from the left end of the row. Mary. There is only one person sitting to the immediate
B) Person sits fifth to the left of K left of Kulfi.
C) Person sits fourth to the left of K 6. Who among the following sits exactly middle of the
D) Both A and B row?
E) Both A and C A) Person sits third to the left of Alex
4. __ Sits fifth to the left of Y sits __ to the left of X. B) Person sits third to the right of Mona
A) W, Eighth C) Person sits third to the right of Hina
B) M, Ninth D) Both A and B
C) L, Tenth E) Both B and C
D) M, Eighth 7. Number of persons sit to the left of Hina is equal to
E) None of these the number of persons sit to the right of__.
5. Who sits exactly between Z and X? A) Person sits fifth from the right end of the row.
A) Person sits second to the right of K B) Person sits fifth to the right of Mona
B) Person sits second to the left of J C) Person sits fifth to the left of Sona
C) Person sits fourth to the right of M D) Both A and B
D) Both A and B E) All A, B and C
E) Both A and C 8. How many persons are sitting in the row?
Directions (6-10): Study the following information A) 15
carefully and answer the questions given below: B) 14
A certain number of persons are sitting in a row facing C) 13
north. D) 12
Hina sits fifth from the left end of the row. Three persons E) 11
sit between Hina and Alex. Dean sits to the immediate 9. Which among the following pairs sit at the end of the
right of Alex who does not sit at the end of the row. Kajol row?
Page 132 of 722

Subscribe the Xpress Video Course & Mock Test Package for Bank & Insurance Exams
If there are any suggestions/ errors in our PDFs Feel Free to contact us via this email: admin@exampundit.in
IBPS RRB Clerk Prelims – Ultra Practice Bundle PDF
A) Person sits to the immediate left of Kulfi and Sona A) Sana
B) Mary and the person sit third to the right of Dean B) Alia
C) Person sits third to the left of Hina and Sona C) Rohit
D) Both A and B D) Angel
E) Both A and C E) Pihu
10. If a new person, Mihir sits exactly between Alex 12. Which of the following statements is/are true
and Kajol then who among the following sits exactly regarding Rohit?
between Mihir and Kulfi? A) Rohit sits second to the right of Sana
A) Person sits to the immediate right of Mona B) Rohit sits fourth to the left of Akhil
B) Person sits to the immediate left of Kajol C) Rohit sits second from the left end
C) Person sits second to the right of Kulfi D) Both A and B
D) Both A and B E) Both A and C
E) Both A and C 13.Who among the following sits fourth to the right of
Sana?
Directions (11-15): Study the following information A) Person sits exactly between Pihu and Angel
carefully and answer the questions given below: B) Person sits third to the right of Akhil
A certain number of persons are sitting in a row facing C) Person sits third from the right end of the row
north. D) Both A and C
Pihu sits to the immediate right Akhil. Sana sits second to E) Both A and B
the left of Akhil. Angel sits five places away from Sana. 14. Which of the following statements is/are not true?
Angel does not sit to the left of Alia. Two persons are A) Sana sits exactly between Rohit and Akhil
sitting between Alia and Sana. Rohit sits to the immediate B) Angel sits second from the right end
right of Alia. Puja who sits third from the right end sits C) Rohit sits exactly between Alia and Sana
exactly between Pihu and Angel. Number of persons sit to D) Alia sits second from the left end of the row
the right of Angel is equal to the number of persons sit to E) All are true
the left of Alia. 15. Who sits third to the right of Alia?
11. Which among the following sit exactly between A) Person sits second to the left of Akhil
Akhil and Puja? B) None
Page 133 of 722

Subscribe the Xpress Video Course & Mock Test Package for Bank & Insurance Exams
If there are any suggestions/ errors in our PDFs Feel Free to contact us via this email: admin@exampundit.in
IBPS RRB Clerk Prelims – Ultra Practice Bundle PDF
C) Puja 18. Number of persons sit to the left of D is equal to the
D) Person sits third to the right of Rohit number of persons sit to the right of___
E) None of these A) Person sits second to the right of C
B) Person sits third to the right of D
Directions (16-20): Study the following information C) Person sits second to the left of H
carefully and answer the questions given below: D) Both A and C
A certain number of persons are sitting in a row facing E) Both A and B
north. 19. How many persons are sitting in the row?
Two persons sit between G and H. B sits fourth to the right A) 14
of G. D sits six places away from H towards left. Two B) 13
persons sit between A and D. A sit second from the left C) 12
end and B sits second from the right end of the row. C and D) 11
E are immediate neighbours of G. C sits to the left of E. F E) 10
sits third to the left of C and second to the right of A. 20. If all the persons sit in the alphabetical order (left
16. Which among the following pair sits to the to right) then the position of how many persons remain
immediate left and immediate right of each other unchanged? (Applicable for known person’s seat)
respectively? A) 4
A) G, C B) 3
B) D, F C) 0
C) H, B D) 2
D) E, G E) 1
E) None of these
17. Who sits exactly between C and B? Directions (21-25): Study the following information
A) Person sits to the immediate right of G carefully and answer the questions given below:
B) No one A certain number of persons are sitting in a row facing
C) Person sits between G and E north.
D) Both A and C Three persons sit between P and R. Q sits second to the
E) H left of R. One person sits between R and S. U sits sixth to
Page 134 of 722

Subscribe the Xpress Video Course & Mock Test Package for Bank & Insurance Exams
If there are any suggestions/ errors in our PDFs Feel Free to contact us via this email: admin@exampundit.in
IBPS RRB Clerk Prelims – Ultra Practice Bundle PDF
the right of R. U does not sit at the end of the row. One E) None of these
person sits between S and T who sits fourth from the right 25. Who among the following sits exactly between Q
end. and S?
21. Who among the following sits fourth to the right of A) Person sits fourth to the right of P
R? B) Person sits third to the left of T
A) Person sits second to the left of U C) Person sits second to the left of Q
B) S D) Both A and B
C) T E) Both A and C
D) Both A and C
E) None of these Directions (26-30): Study the following information
22. Which of the following statements is/are true? carefully and answer the questions given below:
A) R sits second to the right of Q A certain number of persons are sitting in a row facing
B) Three persons sit between P and R south.
C) T sits sixth to the right of Q Four persons sit between Tanu and Bebo. Kitu sits second
D) Both A and B to the right of Tanu. One person sits between Mini and
E) All A, B and C Kitu. Riya sit second to the right of Bebo. One person sits
23. How many persons are there in the row? between Riya and Megha. Kali sits third to the left of Riya.
A) 14 Pari who sits exactly between Mini and Kitu, sits third
B) 13 from the right end. Number of persons sit to the right of
C) 12 Kitu is two more than the number of persons sit to the left
D) 11 of Kali.
E) None of these 26. Who among the following sits third to the right of
24. _sits second to the left of U, sits second to the right Riya?
of __ (same order) A) Person sits second to the left of Kitu
A) T and S B) Person sits second to the right of Megha
B) T and the person sit fourth to the right of R C) Person sits exactly between Pari and Riya
C) Person sits second to the right of S and T D) Both A and B
D) Both A and B E) Both A and C
Page 135 of 722

Subscribe the Xpress Video Course & Mock Test Package for Bank & Insurance Exams
If there are any suggestions/ errors in our PDFs Feel Free to contact us via this email: admin@exampundit.in
IBPS RRB Clerk Prelims – Ultra Practice Bundle PDF
27. How many persons are there in the row? Directions (31-35): Study the following information
A) 15 carefully and answer the questions given below:
B) 14 A certain number of persons are sitting in a row facing
C) 13 south.
D) 12 O sits second to the left of N. Two persons sit between N
E) 11 and P. M sits second to the right of N. Two persons sit
28. Which among the following pairs sit to the between K and M. K sits to the right of M. Q sits second
immediate left and to the immediate right of each other to the left of P. L and M are immediate neighbours of each
respectively? other. L sits third to the right of N. K and Q are sitting at
A) Tanu and Megha ends.
B) Kali and Bebo 31. Who among the following sits third to the left of N?
C) Mini and Pari A) Person sits to the immediate left of O
D) Pari and Kitu B) Person sits second to the left of Q
E) All of the above C) Person sits exactly between O and Q
29. Which of the following statements is/are not true D) Both A and B
regarding Megha? E) None of these
A) Megha sits exactly middle of the row 32. Who among the following sits exactly between M
B) Kitu sits third to the left of Megha and O?
C) Megha sits fourth to the left of Bebo A) Person sits third to the left of L
D) Both B and C B) Person sits fourth to the left of K
E) Both A and B C) Person sits exactly middle of the row
30. If Mini and Riya exchange their position then who D) All A, B and C
among the following sits fifth to the right of Riya? E) Both A and C
A) Person sits immediate left of Tanu 33. Who among the following sits at the end of the row?
B) Person sits second to the left of Mini A) Person sits second to the right of L
C) Person sits third to the right of Bebo B) Person sits second to the left of P
D) Both A and C C) K and Q
E) None of these D) All of the above
Page 136 of 722

Subscribe the Xpress Video Course & Mock Test Package for Bank & Insurance Exams
If there are any suggestions/ errors in our PDFs Feel Free to contact us via this email: admin@exampundit.in
IBPS RRB Clerk Prelims – Ultra Practice Bundle PDF
E) None of the above B) Foruth, Third
34. How many persons are sitting between M and P? C) Third, Second
A) 5 D) Third, Fourth
B) 4 E) None of these
C) 3 37. Which among the following pairs sit adjacent to
D) 0 each other?
E) None of these A) Raju and the person sit immediate left of Mihir
35. Which of the following statements is/are correct B) Ketan and the person sits immediate left of Kirti
regarding O? C) Kirti and the person sits second to the right of Ketan
A) O sits third from the right end D) Both A and B
B) O sits fourth to the right of M E) Both A and C
C) One person sits between O and P 38. How many persons are there in the row?
D) All are true A) 12
E) None is true B) 13
C) 14
Directions (36-40): Study the following information D) 15
carefully and answer the questions given below: E) 16
A certain number of persons are sitting in a row facing 39. Who among the following sits fifth to the left of
south. Kirti?
Ajay and Preet sit at the end of the row. Two persons sit A) Nimo
between Sonu and Ajay. Ketan sits second to the left of B) Raju
Sonu. Kirti sits third to the right of Ketan. Three persons C) Mihir
sit between Preet and Mihir. Nimo sits fourth from the end D) Ketan
of the row. Raju sits to the immediate left of Nimo. There E) None of these
is a gap of one place between Ketan and Mihir. 40. Which of the following statements is/are true
36. Kirti sits__ to the right of Ketan who sits__ to the regarding Preet?
left of Sonu. A) Preet sits at the left end of the row
A) Second, Third B) Preet sits second to the left of Raju
Page 137 of 722

Subscribe the Xpress Video Course & Mock Test Package for Bank & Insurance Exams
If there are any suggestions/ errors in our PDFs Feel Free to contact us via this email: admin@exampundit.in
IBPS RRB Clerk Prelims – Ultra Practice Bundle PDF
C) Preet sits fourth to the left of Mihir E) None of these
D) All are true 43. _ sits to the immediate right of Dhun, sits third to
E) None is true the right of___
A) Person sits second to the left of Jivi and Mili
Directions (41-45): Study the following information B) Person sits third to the right of Iksa and Mili
carefully and answer the questions given below: C) Ekta and the person sits second to the left of Dhun
A certain number of persons are sitting in a row facing D) Both A and C
south. E) Both A and B
Ekta and Dhun are immediate neighbours of each other. 44. Who among the following sits second to the right of
Mili sits second to the left of Dhun. Mili is not an the one who sits fourth to the left of Alka?
immediate neighbour of Ekta. Jivi sits second to the right A) Person sits second to the left of Alka
of Ekta. Iksa is an immediate neighbour of Jivi. Gini sits B) Person sits fifth from the left end
second to the right of Iksa. One person sits between Alka C) Person sits third to the left of Jivi
and Gini. Gini sits fourth from the right end of the row. D) All A, B and C
Not more than two known persons sit adjacent together. E) Both A and B
41. Who among the following sits third to the right of 45. How many persons sits between the person sits
Ekta? third to the left of Iksa and Mili?
A) Person sits exactly between Alka and Jivi A) 8
B) Person sits second to the left of Gini B) 2
C) Person sits to the immediate left of Jivi C) 6
D) Both A and B D) 4
E) None of these E) None of these
42. If Jivi and Dhun exchange their position then who
sits third to the right of Jivi? Directions (46-50): Study the following information
A) Iksa carefully and answer the questions given below:
B) Ekta A certain number of persons are sitting in a row facing
C) Dhun south.
D) Gini
Page 138 of 722

Subscribe the Xpress Video Course & Mock Test Package for Bank & Insurance Exams
If there are any suggestions/ errors in our PDFs Feel Free to contact us via this email: admin@exampundit.in
IBPS RRB Clerk Prelims – Ultra Practice Bundle PDF
Three persons sit between R and T. S sits exactly between A) Person sits exactly between R and T
R and T. One person sits between R and Q. P sits to the B) Person sits third to the right of U
immediate right of Q. U sits to the immediate left of T who C) Person sits fifth from the left end
sits third from the left end. Number of persons sits to the D) Both A and B
left of U is equal to the number of persons sits to the right E) All of the above
of P. 49. How many persons sit between Q and T?
46. How many persons are there in the row? A) 5
A) 13 B) 4
B) 11 C) 6
C) 15 D) 3
D) 12 E) None of these
E) 14 50. If a new person, M sits between R and S then which
47. What is the position of R with respect to U? statement is true regarding M?
A) 5th to the left A) M sits exactly middle of the row
B) 4th to the right B) M sits third to the left of T
C) 5th to the right C) M sits third to the left of Q
D) 4th to the left D) Both A and C
E) None of these E) Both A and B
48. Who among the following sits fourth to the left of
Q?

6). Certain Number of Persons Linear - Detailed Explanation with


Answers
Answers (1-5): As per this statement, the arrangement will look like this:
Solutions:
STEP I: J sits third to the right of K.

Page 139 of 722

Subscribe the Xpress Video Course & Mock Test Package for Bank & Insurance Exams
If there are any suggestions/ errors in our PDFs Feel Free to contact us via this email: admin@exampundit.in
IBPS RRB Clerk Prelims – Ultra Practice Bundle PDF

STEP IV: W is an immediate neighbour of both M and L.


STEP II: Two persons sit between K and M.

As per this statement, the arrangement will look like this:


As per this statement, the arrangement will look like this:

CASE I:

STEP III: Only one person sits between M and L.

CASE II:
As per this statement, the arrangement will look like this:

CASE I:

STEP V: Z is the only known neighbour of K and sits


second to the right of M
As per this statement, CASE I will get eliminated and we
will continue with CASE I and it will look like this:
CASE II:

CASE I:

Page 140 of 722

Subscribe the Xpress Video Course & Mock Test Package for Bank & Insurance Exams
If there are any suggestions/ errors in our PDFs Feel Free to contact us via this email: admin@exampundit.in
IBPS RRB Clerk Prelims – Ultra Practice Bundle PDF
1) E
2) C
3) B
4) D
5) A
CASE II:
Answers (6-10):
STEP I: Hina sits fifth from the left end of the row.

As per this statement, the arrangement will look like this:

STEP VI: X sits third to the right of Y who sits to the


immediate left of J.

As per this statement, the arrangement will look like this:

STEP II: Three persons sit between Hina and Alex.

As per this statement, there will be two cases getting

STEP VII: Number of persons sits to the left of L is one framed and they will look like this:

less than the number of persons sits to the right of X. L


sits at one of the extreme ends of the row. Less than CASE I:

fourteen persons sit in a row.

As per this statement, the final arrangement will look like


this:

CASE II:

Page 141 of 722

Subscribe the Xpress Video Course & Mock Test Package for Bank & Insurance Exams
If there are any suggestions/ errors in our PDFs Feel Free to contact us via this email: admin@exampundit.in
IBPS RRB Clerk Prelims – Ultra Practice Bundle PDF
CASE II:

STEP III: Dean sits to the immediate right of Alex who


does not sit at the end of the row.
STEP V: Kulfi sits fifth to the left of Kajol.
CASE I:
As per this statement, the arrangement will look like this:

CASE I:

CASE II:

CASE II:

STEP IV: Kajol sits exactly in between Hina and Alex.

As per this statement, the arrangement will look like this:


STEP VI: Sona sits third to the right of Dean. Mary and
CASE I: Sona sit at the extreme left end and the extreme right end
of the row respectively. Mona sits third to the right of
Mary. There is only one person sitting to the immediate
left of Kulfi.

Page 142 of 722

Subscribe the Xpress Video Course & Mock Test Package for Bank & Insurance Exams
If there are any suggestions/ errors in our PDFs Feel Free to contact us via this email: admin@exampundit.in
IBPS RRB Clerk Prelims – Ultra Practice Bundle PDF
As per these statements, CASE I will get eliminated and STEP II: Sana sits second to the left of Akhil.
we will continue with CASE II and it will look like this:
As per this statement, the arrangement will look like this:
CASE I:

STEP III: Angel sits five places away from Sana.


CASE II:

As per this statement, there will be two cases getting


framed and they are as follows:

CASE I:
6) B
7) D
8) C
9) D
10) A
CASE II:

Solutions (11-15):

STEP I: Pihu sits to the immediate right Akhil.


As per this statement, the arrangement will look like this:

STEP IV: Angel does not sit to the left of Alia. Two
persons are sitting between Alia and Sana.

Page 143 of 722

Subscribe the Xpress Video Course & Mock Test Package for Bank & Insurance Exams
If there are any suggestions/ errors in our PDFs Feel Free to contact us via this email: admin@exampundit.in
IBPS RRB Clerk Prelims – Ultra Practice Bundle PDF
As per these statements, CASE I will get eliminated and STEP VII: Number of persons sit to the right of Angel is
we will continue with CASE II and it will look like this: equal to the number of persons sit to the left of Alia.
As per this statement, the final arrangement will look like
CASE I: this:

CASE II:
11) E
12) B
13) D
14) C
15) A
STEP V: Rohit sits to the immediate right of Alia. Solutions (16-20):

As per this statement, the arrangement will look like this: STEP I: Two persons sit between G and H.

As per this statement, there will be two cases being framed


and they will look like this:

STEP VI: Puja who sits third from the right end sits CASE I:
exactly between Pihu and Angel.

CASE II:

Page 144 of 722

Subscribe the Xpress Video Course & Mock Test Package for Bank & Insurance Exams
If there are any suggestions/ errors in our PDFs Feel Free to contact us via this email: admin@exampundit.in
IBPS RRB Clerk Prelims – Ultra Practice Bundle PDF
STEP IV:Two persons sit between A and D. A sit second
from the left end and B sits second from the right end of
the row.

STEP II: B sits fourth to the right of G. CASE I:

CASE I:

CASE II:

CASE II:

STEP V: C and E are immediate neighbours of G. C sits


to the left of E. F sits third to the left of C and second to
the right of A.
STEP III: D sits six places away from H towards left.
CASE I: As per these statements, CASE II will get eliminated and
we will continue with CASE I and the final arrangement
will look like this:
CASE I:
CASE II:

CASE II:

16) C
17) B

Page 145 of 722

Subscribe the Xpress Video Course & Mock Test Package for Bank & Insurance Exams
If there are any suggestions/ errors in our PDFs Feel Free to contact us via this email: admin@exampundit.in
IBPS RRB Clerk Prelims – Ultra Practice Bundle PDF
18) D As per this statement, the arrangement will look like this:
19) B
20) E CASE I:
Solutions (21-25):
STEP I: Three persons sit between P and R.
As per this statement, there will be two cases getting
framed and the arrangement will look like this:
CASE I:

CASE II:

CASE II:

STEP IV: U sits sixth to the right of R.


STEP II: Q sits second to the left of R. As per this statement, the arrangement will look like this:
As per this statement, the arrangement will look like this: CASE I:
CASE I:

CASE II:

CASE II:

STEP V: U does not sit at the end of the row. One person
sits between S and T who sits fourth from the right end.

STEP III: One person sits between R and S.


Page 146 of 722

Subscribe the Xpress Video Course & Mock Test Package for Bank & Insurance Exams
If there are any suggestions/ errors in our PDFs Feel Free to contact us via this email: admin@exampundit.in
IBPS RRB Clerk Prelims – Ultra Practice Bundle PDF
As per these statements, CASE II will get eliminated and
we will continue with CASE I and the final arrangement
will look like this:
CASE I:
STEP II: Kitu sits second to the right of Tanu.
As per this statement, the arrangement will look like this:
CASE I:

CASE II:

CASE II:
21) D
22) E
23) C
24)D
STEP III: One person sits between Mini and Kitu. Pari
25) A
who sits exactly between Mini and Kitu, sits third from
the left end.
Solutions (26-30):
As per these statements, the arrangement will look like
STEP I: Four persons sit between Tanu and Bebo.
this:
As per this statement, there will be two cases getting
CASE I:
framed and they will look like this:
CASE I:

CASE II:

CASE II:

Page 147 of 722

Subscribe the Xpress Video Course & Mock Test Package for Bank & Insurance Exams
If there are any suggestions/ errors in our PDFs Feel Free to contact us via this email: admin@exampundit.in
IBPS RRB Clerk Prelims – Ultra Practice Bundle PDF
STEP IV: Riya sits second to the right of Bebo. As per this statement, the arrangement will look like this:
As per this statement, CASE II will get eliminated and we CASE I:
will continue with CASE I and the arrangement will look
like this:

CASE I:
STEP VII: Number of persons sit to the right of Kitu is
two more than the number of persons sit to the left of Kali.

As per this statement, the final arrangement will look like


this:
CASE II:
CASE I:

STEP V: One person sits between Riya and Megha.


26) E
As per this statement, the arrangement will look like this: 27) C
28) B
CASE I: 29) C
30) E (No one)

Solutions (31-35):
STEP I: O sits second to the left of N.
As per this statement, the arrangement will look like this:
STEP VI: Kali sits third to the left of Riya.

Page 148 of 722

Subscribe the Xpress Video Course & Mock Test Package for Bank & Insurance Exams
If there are any suggestions/ errors in our PDFs Feel Free to contact us via this email: admin@exampundit.in
IBPS RRB Clerk Prelims – Ultra Practice Bundle PDF
As per these statements, the arrangement will look like
this:
CASE I:
STEP II: Two persons sit between N and P.
As per this statement, there will be two cases getting
framed and they will look like this:
CASE I:
CASE II:

CASE II:

STEP V: Q sits second to the left of P.


As per this statement, the arrangement will look like this:
CASE I:
STEP III: M sits second to the right of N.
As per this statement, the arrangement will look like this:
CASE I:

CASE II:

CASE II:

STEP IV: Two persons sit between K and M. K sits to the STEP VI: L and M are immediate neighbours of each
right of M. other. L sits third to the right of N.

Page 149 of 722

Subscribe the Xpress Video Course & Mock Test Package for Bank & Insurance Exams
If there are any suggestions/ errors in our PDFs Feel Free to contact us via this email: admin@exampundit.in
IBPS RRB Clerk Prelims – Ultra Practice Bundle PDF
As per these statements, CASE II will get eliminated and
we will continue with CASE I and the final arrangement
will look like this:
CASE I: STEP II: Two persons sit between Sonu and Ajay.
As per this statement, the arrangement will look like this:
CASE I:

CASE II:

CASE II:

31) A
32) E
33) D
34) B STEP III: Ketan sits second to the left of Sonu.
35) E As per this statement, the arrangement will look like this:
Solutions (36-40): CASE I:
STEP I:Ajay and Preet sit at the end of the row.
As per this statement, there will be two cases getting
framed and they will look like this:
CASE I: CASE II:

CASE II: STEP IV: Kirti sits third to the right of Ketan.
As per this statement, the arrangement will look like this:
CASE I:
Page 150 of 722

Subscribe the Xpress Video Course & Mock Test Package for Bank & Insurance Exams
If there are any suggestions/ errors in our PDFs Feel Free to contact us via this email: admin@exampundit.in
IBPS RRB Clerk Prelims – Ultra Practice Bundle PDF

36) C
CASE II: 37) E
38) A
39) C
40) D
STEP V: Three persons sit between Preet and Mihir. Solutions (41-45):
There is a gap of one place between Ketan and Mihir. STEP I: Ekta and Dhun are immediate neighbours of each
As per these statements, CASE II will get eliminated and other.
we will continue with CASE I and the arrangement will As per these statements, there will be two cases getting
look like this: framed and they are as follows:
CASE I: CASE I:

CASE II:

CASE II:

STEP VI: Nimo sits fourth from the end of the row. Raju
sits to the immediate left of Nimo.
As per these statements, the final arrangement will look STEP II: Mili sits second to the left of Dhun.
like this: As per this statement, the arrangement will look like this:
CASE I: CASE I:

Page 151 of 722

Subscribe the Xpress Video Course & Mock Test Package for Bank & Insurance Exams
If there are any suggestions/ errors in our PDFs Feel Free to contact us via this email: admin@exampundit.in
IBPS RRB Clerk Prelims – Ultra Practice Bundle PDF
CASE I:

CASE I (A):
CASE II:

STEP VI: Gini sits second to the right of Iksa.


STEP III: Mili is not an immediate neighbour of Ekta.
As per this statement, the arrangement will look like this:
As per this statement, CASE II will get eliminated and we
CASE I:
will continue with CASE I and the arrangement will look
like this:
CASE I:

CASE I (A):

STEP IV: Jivi sits second to the right of Ekta.


As per this statement, the arrangement will look like this:
CASE I: STEP VII: One person sits between Alka and Gini.
As per this statement, the arrangement will look like this:
CASE I:

STEP V: Iksa is an immediate neighbour of Jivi.


As per this statement, two cases will be framed and they
CASE I (A):
will look like this:

Page 152 of 722

Subscribe the Xpress Video Course & Mock Test Package for Bank & Insurance Exams
If there are any suggestions/ errors in our PDFs Feel Free to contact us via this email: admin@exampundit.in
IBPS RRB Clerk Prelims – Ultra Practice Bundle PDF
As per these statements, there will be two cases getting
framed and they will look like this:
CASE I:
STEP VIII: Gini sits fourth from the left end of the row.
Not more than two persons sit adjacent together.

As per these statements, CASE I (A) will get eliminated


CASE II:
and we will continue with CASE I and the final
arrangement will look like this:
CASE I:

STEP II: One person sits between R and Q.


As per this statement, the arrangement will look like this:
CASE I:
CASE I (A):

CASE II:

41)B
42) C
43) D
STEP III: P sits to the immediate right of Q.
44) A
As per this statement, the arrangement will look like this:
45) B
CASE I:

Solutions (46-50):
STEP I: Three persons sit between R and T. S sits exactly
between R and T.

Page 153 of 722

Subscribe the Xpress Video Course & Mock Test Package for Bank & Insurance Exams
If there are any suggestions/ errors in our PDFs Feel Free to contact us via this email: admin@exampundit.in
IBPS RRB Clerk Prelims – Ultra Practice Bundle PDF
CASE II: CASE II:

STEP IV: U sits to the immediate left of T who sits third STEP V: Number of persons sits to the left of U is equal
from the right end. to the number of persons sits to the right of P.
As per this statement, CASE II will get eliminated and we As per this statement, the final arrangement will look like
will continue with CASE I and the arrangement will look this:
like this: CASE I:
CASE I:

46) B 49) A
47) C 50) D
48) E

Page 154 of 722

Subscribe the Xpress Video Course & Mock Test Package for Bank & Insurance Exams
If there are any suggestions/ errors in our PDFs Feel Free to contact us via this email: admin@exampundit.in
IBPS RRB Clerk Prelims – Ultra Practice Bundle PDF

7). Blood Relation Question


Direction (1-3): Read the following information Direction (4-6): Read the following information
carefully to answer the question that follows: carefully to answer the question that follows:
There are seven members P, Q, R, S, T, U and V in a There are six members P, Q, R, S, T and U in the family
family. of three generations. There is no single parent in the
R is father-in-law of U, who is grandmother of V. P has family. U is aunt of T. R is mother-in-law of S’s husband.
two sons. T is father of S. The gender of S and Q is same. Q has only one child. S and Q are not married to each
R is married to P. other.
1) Who is mother of Q? 4) How is U related to S?
a. R a. Daughter
b. U b. Sister-in-law
c. P c. Sister
d. S d. Mother
e. None of these e. Daughter-in-law
2) How is V related to T? 5) Who is the father of S?
a. Grandson a. Can’t be determined
b. Grandfather b. P
c. Mother c. Q
d. Granddaughter d. T
e. Cannot be determined e. None of these.
3) How is Q related to U? 6) How R is related to T?
a. Brother a. Daughter
b. Brother-in-law b. Father
c. Sister-in-law c. Grandmother
d. Father d. Sister
e. None of these e. None of these.

Page 155 of 722

Subscribe the Xpress Video Course & Mock Test Package for Bank & Insurance Exams
If there are any suggestions/ errors in our PDFs Feel Free to contact us via this email: admin@exampundit.in
IBPS RRB Clerk Prelims – Ultra Practice Bundle PDF
Direction (7-9): Read the following information of C. D is the daughter of A. F is the brother of A. A and
carefully and answer the question that follow: C are married couple.
A, B, C, D, E, F, and H are family members related to each 10) Who is the brother in law of C?
other. A is the father of D who is the brother of F. E is the a. A
husband of H. F is the sister of E. B is the son of D and b. B
husband of C. c. D
7) How is E related to B? d. E
a. Brother-in-law e. F
b. Father-in-law 11) How many females are there in the family?
c. Brother a. 1
d. Uncle b. 2
e. Father c. 3
8) How is H related to F? d. 4
a. Sister-in-law e. None of these
b. Sister 12) How is F related to B?
c. Mother a. Maternal Uncle
d. Mother-in-law b. Brother
e. Aunty c. Daughter
9) Who is the grandson of A? d. Paternal Uncle
a. D e. Aunt
b. F Direction (13-15): Read the following information
c. E carefully to answer the question that follows:
d. C There are five members P, Q, R, S and T in the family of
e. B three generations. No single parent has a child. R is father-
Direction (10-12): These questions are based on the in-law of T. S is son of Q, who is not married to R. P’s and
following information. Q’s gender is same.
A family consist of six members A, B, C, D, E and F. B is 13) Who is mother of Q?
the son of C but C is not the mother of B. E is the brother a. R
Page 156 of 722

Subscribe the Xpress Video Course & Mock Test Package for Bank & Insurance Exams
If there are any suggestions/ errors in our PDFs Feel Free to contact us via this email: admin@exampundit.in
IBPS RRB Clerk Prelims – Ultra Practice Bundle PDF
b. P e. Cannot be determined
c. T 17) Who is S’s father-in-law?
d. Cannot be determined a. Q
e. None of these b. P
14) How is S related to T? c. T
a. Daughter d. R
b. Son-in-law e. Cannot be determined
c. Son Direction (18-20): Read the following information
d. Father carefully to answer the question that follows:
e. None of these There are seven members P, Q, R, S, U, V and W in a
15) How is P related to S? family in which married couples are in 1st and 2nd
a. Granddaughter generation. There is no single parent in the family.
b. Son-in-law U is the only daughter of Q, who has three children. W is
c. Grandmother the only sister in law of U. P is the grandfather of S, whose
d. Grandfather father is V’s brother.
e. None of these 18) How is S related to U?
Directions (16-17): Study the information given below a. Son
carefully and answer the questions that follow. b. Nephew
At a coffee shop, there are 5 members present – P, Q, R, c. Niece
S and T. R is the daughter of Q and her son is T. There are d. Can’t be determined.
two couples present at the gathering. T’s father is sitting e. None of these
next to P who is a male. R’s parents are present in the 19) How is W related to P?
gathering. T is not married. a. Daughter
16) Who is S in this gathering? b. Niece
a. Q's husband c. Daughter in law
b. R’s husband d. Can’t be determined
c. T’s son e. None of these
d. Q’s son 20) How V related to Q?
Page 157 of 722

Subscribe the Xpress Video Course & Mock Test Package for Bank & Insurance Exams
If there are any suggestions/ errors in our PDFs Feel Free to contact us via this email: admin@exampundit.in
IBPS RRB Clerk Prelims – Ultra Practice Bundle PDF
a. Brother c. Son-in-law
b. Son d. Sister-in-law
c. Daughter e. None of these
d. Father Directions (24-26): Answer the questions based on the
e. Sister information given below.
Directions (21-23): Answer the questions based on the There are six members A, B, C, D, E and F in the family
information given below. of three generations. There is equal number of males and
There are seven members P, Q, R, S, T, U and V in the females. No single parent has a child. F is father of A, who
family of three generations. There is couple in each is married to D. B is grandmother of C. B has two children.
generation. Q is daughter-in-law of P, who has two E and D have same gender.
children. U is the son of R and does not have any sibling. 24) How is E related to D if A is a female?
U is married to V. S is not married. There is no single a. Sister-in-law
parent in the family. b. Mother
21) How is T related to R if P is a female? c. Father
a. Father d. Brother-in-law
b. Mother e. None of these
c. Son-in-law 25) How is B related to D?
d. Sister-in-law a. Mother-in-law
e. None of these b. Mother
22) How is U related to S? c. Sibling
a. Uncle d. Nephew
b. Sister e. None of these
c. Niece 26) How F related to E?
d. Nephew a. Father
e. None of these b. Mother
23) How R is related to V? c. Sibling
a. Father d. Nephew
b. Mother e. None of these
Page 158 of 722

Subscribe the Xpress Video Course & Mock Test Package for Bank & Insurance Exams
If there are any suggestions/ errors in our PDFs Feel Free to contact us via this email: admin@exampundit.in
IBPS RRB Clerk Prelims – Ultra Practice Bundle PDF
Direction (27-30): These questions are based on the a. A
following information. b. C
A, B, C, D, E, F, G, H, I and J are 10 family members c. B
living together. There are three couples living together d. D
with two children each. e. G
C and D are brothers while C is brother-in-law of E. B is Direction (31-32): Study the information given below
mother-in-law of F. G and H are sons of E and F carefully and answer the questions that follow.
respectively. I and J are grand-children of A. J is not H’s There are eight members in a family – A, B, C, D, E, F, G
sibling and I is not j’s sibling. E has one daughter. and H. A is grandmother of G who is the cousin of H. A
27) Who is B in the house? is married to B. B has one son and one daughter. C who is
a. A’s wife the son of A is married to E. F is the wife of D. H is the
b. C’s mother son of D and nephew of C. E has one son.
c. E’s mother-in-law 31) How is H related to B?
d. G’s grand-mother a. Grandson
e. All of these b. Son
28) What is the relation between I and H? c. Brother-in-law
a. Brothers d. Son-in-law
b. Sisters e. Cannot be determined
c. Either brothers or brother-sister 32) How is F related to B?
d. Neither brothers nor brother-sister a. Son
e. Can’t be determined b. Daughter
29) Who is J’s father? c. Mother
a. A d. Sister
b. C e. Daughter-in-law
c. D Directions (33-35): Study the following information
d. G and answer the questions given below.
e. None of these There are eight members in a family namely A, B, C, D,
30) Who is E’s father-in-law? E, F, G and H are going to fair in their village. In this
Page 159 of 722

Subscribe the Xpress Video Course & Mock Test Package for Bank & Insurance Exams
If there are any suggestions/ errors in our PDFs Feel Free to contact us via this email: admin@exampundit.in
IBPS RRB Clerk Prelims – Ultra Practice Bundle PDF
family there is a married couple. B is son of C. G, C’s 36) How is J related to D?
sister, has two daughters. H is maternal uncle of G’s a. Father
daughters. A is father of D, who is sister to B. b. Mother
33) How is E related to D? c. Grandson
a. Niece d. Granddaughter
b. Nephew e. Cousin sister
c. Cousin 37) How is F related to G?
d. Sister-in-law a. Son
e. None of these b. Daughter
34) Which of the following is a married couple? c. Grandson
a. GB d. Granddaughter
b. AC e. Can’t be determined
c. HC 38) How is H related to J?
d. Can’t be determined a. Sister
e. None of these b. Brother
35) How is H related to D? c. Aunt
a. Paternal Uncle d. Mother
b. Niece. e. Father
c. Maternal Uncle. Directions (39-41): Answer the questions based on the
d. Data Inadequate. information given below.
e. None of these. There are eight members, A through H in a family. There
Directions (36-38): Study the following information is no single parent in the family.
carefully and answer the questions given below. F’s mother’s only brother’s father is B. E is the only
A, B, C, D, E, F, G H, I and J are the ten people in a family. brother-in-law of H. D’s mother is the only daughter-in-
B is the brother of H, who is the only daughter of A. J is law of B. Number of males is more than the number of
the only daughter of I, who is married to the only son of females. G is the only daughter of A. E has a sibling.
D. C is the son-in-law of D, who is the father of B. G is 39) How is E related to F?
the paternal Grandmother of E, who is the sister of F. a. Father
Page 160 of 722

Subscribe the Xpress Video Course & Mock Test Package for Bank & Insurance Exams
If there are any suggestions/ errors in our PDFs Feel Free to contact us via this email: admin@exampundit.in
IBPS RRB Clerk Prelims – Ultra Practice Bundle PDF
b. Uncle e. Sangeeta and Meeta
c. Brother 43) Who is Sohan’s wife?
d. Either (a) or (b) a. Sita
e. None of these b. Geeta
40) How is F related to B? c. Meeta
a. Grandson d. Sangeeta
b. Granddaughter e. Either Meeta or Sita
c. Son 44) Which of the following is not a correct pair of wife
d. Daughter and husband?
e. None of these a. Sangeeta and Mohan
41) How is G related to H? b. Meeta and Sohan
a. Sister c. Sita and Shyam
b. Wife d. Geeta and Ram
c. Aunt e. Shyam and Meeta
d. Cannot be determined Directions (45-47): Answer the questions based on the
e. None of these information given below.
Directions (42-44): These questions are based on the There are seven members, A, B, C, D, E, F and G in a
following information. family. There is no single parent in the family. G’s father’s
In a family, there are eight members. Four men Ram, only sister’s only daughter in law is F. A is the only
Shyam, Mohan and Sohan and four women Sita, Geeta, brother-in-law of D, who is a male member. E is the only
Sangeeta and Meeta. Sangeeta has two married sons and sister-in-law of B.
one married daughter. Ram is Mohan’s son-in-law. Geeta 45) Who is the husband of B?
is Shyam’s sister. Sita is not Sohan’s wife. a. A
42) Who are the two daughter-in-laws of Mohan? b. D
a. Sita and Geeta c. C
b. Sita and Meeta d. Cannot be determined
c. Geeta and Meeta e. None of these
d. Sangeeta and Sita 46) If C is the son of B then how is B related to G?
Page 161 of 722

Subscribe the Xpress Video Course & Mock Test Package for Bank & Insurance Exams
If there are any suggestions/ errors in our PDFs Feel Free to contact us via this email: admin@exampundit.in
IBPS RRB Clerk Prelims – Ultra Practice Bundle PDF
a. Son 48) How is W related to V?
b. Daughter a. Mother
c. Aunt b. Grandmother
d. Niece c. Sister
e. None of these d. Aunt
47) How C related to G? e. None of these
a. Son 49) How is Q related to V?
b. Daughter a. Grandfather
c. Nephew b. Father
d. Niece c. Uncle
e. None of these d. Aunt
Direction (48-50): Read the information carefully and e. None of these
answer the following questions. 50) How is R related to P?
There are eight members P, Q, R, S, T, U, V, W in a a. Daughter in law
family. U is brother of T who is sister in law of R. S is b. Daughter
grandson of P who is father of Q. V is only daughter of R c. Granddaughter
who is wife of U. Q is the son of W who is mother of U. d. Son
S is the only son of U. e. None of these

7). Blood Relation Questions - Solutions with Explanation

1. c 1. R is father-in-law of U, who is grandmother of V.


2. e 2. P has two sons.
3. b 3. T is father of S.
Solution 1-3 4. P and U are married.
From the given information, 5. T he genders of S and Q is same. R is married.
Based on given data, we can draw family tree
Page 162 of 722

Subscribe the Xpress Video Course & Mock Test Package for Bank & Insurance Exams
If there are any suggestions/ errors in our PDFs Feel Free to contact us via this email: admin@exampundit.in
IBPS RRB Clerk Prelims – Ultra Practice Bundle PDF
8. a
9. e
Solution 7-9
1) A is the father of D who is the brother of F (means D is
the brother of F)
2) E is the husband of H.
3) F is the sister of E.
4) B is the son of D and husband of C.

Answers
4. b
5. c
6. c
Solution 4-6
From the given information,
1. U is aunt of T. R is mother-in-law of S’s husband.
2. Q has only one child. Answers

3. S and Q are not married to each other. 10. e

Based on given data, we can draw family tree 11. b


12. a
Solutions 10-12
1) B is the son of C but C is not the mother of B this
implies that C is the Father of B.
2) A and C are married couples this implies that A is the
mother of B.
3) D is daughter of A this implies that B and D are brother
Answers and sisters and both are children of A and C where A is
7. d the mother and C is the father.
Page 163 of 722

Subscribe the Xpress Video Course & Mock Test Package for Bank & Insurance Exams
If there are any suggestions/ errors in our PDFs Feel Free to contact us via this email: admin@exampundit.in
IBPS RRB Clerk Prelims – Ultra Practice Bundle PDF
4) E is brother of C, hence E is the brother in law of A and Answers
paternal uncle of B and D. 16. b
5) F is the brother of A, hence F is the brother in law of C 17. b
and maternal uncle of B and D. Solution 16-17
1) R is the daughter of Q and her son is T. T is not married
2) T’s father is sitting next to P who is a male.
3) There are two couples present at the gathering.
4) R’s parents are present in the gathering.

Answers
13. b
14. c
15. c
Solutions 13-15
From the given information,
1. R is father-in-law of T.
Answers
2. S is son of Q, who is not married to R.
18. d
3. P’s and Q’s gender is same.
19. c
So, Q must be wife of T.
20. b
Based on given data, we can draw family tree
Solution 18-20
From the given information,
1. U is the only daughter of Q, who has three children.
2. W is the only sister in law of U.
3. P is the grandfather of S, whose father is V’s brother,
so V and R are the brother of U.
4. P is the father of V.

Page 164 of 722

Subscribe the Xpress Video Course & Mock Test Package for Bank & Insurance Exams
If there are any suggestions/ errors in our PDFs Feel Free to contact us via this email: admin@exampundit.in
IBPS RRB Clerk Prelims – Ultra Practice Bundle PDF
Based on given data, we can draw family tree Now, there is equal number of males and females. No
single parent has a child. F is father of A, who is married
to D. B is grandmother of C. B has two children. E and D
have same gender. So, we have,
Family Tree:

Answers
21. a
22. d
23. a
Solution 21-23
There is couple in each generation. Q is daughter-in-law Answers:
of P, who has two children. U is the son of R and does not 27. e
have sibling. U is married to V. S is not married. 28. c
Family Tree: 29. c
30. a
Solutions 27-30
1) C and D are brothers while C is brother-in-law of E.
(implying E is wife of D.)
2) B is mother-in-law of F. (implying B is female.)
3) G and H are sons of E and F respectively. (Implying G

Answers: is son of E and D, F is mother or father of H.)

24. d 4) I and J are grand-children of A. (implying A is either

25. a grand-father or grand-mother of I and J.)

26. a 5) J is neither sister nor brother of H.

Solutions 23-26 6) E has one daughter. (implying J is sister of G.)

Page 165 of 722

Subscribe the Xpress Video Course & Mock Test Package for Bank & Insurance Exams
If there are any suggestions/ errors in our PDFs Feel Free to contact us via this email: admin@exampundit.in
IBPS RRB Clerk Prelims – Ultra Practice Bundle PDF
7) There are three sets of couples living together with two
children each. (Implying E and D, B and A, C and F are
the three sets of couples.)

5) H is the son of D and nephew of C.


6) F is the wife of D
7) A is grandmother of G who is the cousin of H.

Answers
31. a
32. b

Solution 31-32
1) A is married to B.
2) B has one son and one daughter.
Answers
33. c
34. b
35. c
Solution 33-35
1) G, C’s sister has two daughters.
2) H is maternal uncle of G’s daughters.
3) C who is the son of A is married to E. 3) B is son of C.
4) E has one son. 4) A is father of D, who is sister to B.

Page 166 of 722

Subscribe the Xpress Video Course & Mock Test Package for Bank & Insurance Exams
If there are any suggestions/ errors in our PDFs Feel Free to contact us via this email: admin@exampundit.in
IBPS RRB Clerk Prelims – Ultra Practice Bundle PDF
Thus, the only members left are E and F. Thus, they are
daughters of G.

Answers:
39. b
Answers:
40. a
36. d
41. b
37. e
Solutions 39-41
38. c
1. F’s mother’s only brother’s father is B.
Solutions 36-38
2. E is the only brother-in-law of H.
1) B is the brother of H, who is the only daughter of A.
3. D’s mother is the only daughter in law of B.
2) C is the son-in-law of D, who is the father of B.
4. Number of males is more than the number of females.
5. G is the only daughter of A.

3) J is the only daughter of I, who is married to the only Answers


son of D. 42. b
4) G is the paternal Grandmother of E, who is the sister of 43. c
F. 44. e

Page 167 of 722

Subscribe the Xpress Video Course & Mock Test Package for Bank & Insurance Exams
If there are any suggestions/ errors in our PDFs Feel Free to contact us via this email: admin@exampundit.in
IBPS RRB Clerk Prelims – Ultra Practice Bundle PDF
Solution 42-44 The final arrangement is as follows:
Sangeeta has two married sons and one married daughter
Ram is Mohan’s son-in-law; Hence Mohan is married to
Sangeeta and Ram is married to the only daughter of
Sangeeta.
Geeta is Shyam’s sister, Geeta has two brothers Shyam
and Sohan.
Sita is not Sohan’s wife, hence Sita is the wife of Shyam Answers
and Meeta is the wife of Sohan. 48. b
49. c
50. a
Solution 48-50
1. U is brother of T who is sister in law of R.
2. S is grandson of P who is father of Q.
3. V is only daughter of R who is wife of U.
4. Q is the son of W who is mother of U.
5. S is the only son of U.

Answers:
45. d
46. c
47. c
Solutions 45-47
1. G’s father’s only sister’s only daughter in law is F.
2. A is the only brother-in-law of D.
3. E is the only sister-in-law of B.
4. Either A or D is the husband of B.

Page 168 of 722

Subscribe the Xpress Video Course & Mock Test Package for Bank & Insurance Exams
If there are any suggestions/ errors in our PDFs Feel Free to contact us via this email: admin@exampundit.in
IBPS RRB Clerk Prelims – Ultra Practice Bundle PDF

8). Number Sequence Questions


Direction (1-5): The following questions are based on e. None
the five three-digit numbers given below. 4) What will be the number obtained if the second digit
234 756 867 123 815 of the smallest number is multiplied with the first digit
1) If 5 is subtracted from even numbers and 1 is added of the largest number?
to odd numbers, then which of the following numbers a. 64
is divisible by 3? b. 16
a. 234 c. 12
b. 815 d. 74
c. 867 e. 34
d. 815 5) If the digits of each number are added together, then
e. 123 sum of the digits of which of the following number is
2) If the digits of the highest and the lowest numbers perfect square?
are reversed, which of the following is the new lowest a. 815
number? b. 123
a. 756 c. 234
b. 815 d. 756
c. 867 e. 867
d. 123 Directions (6-10): Study the following arrangement of
e. 234 numbers and symbols carefully and answer the
3) If the digits in each number are arranged in questions:
ascending order, how many numbers are odd? 64392197426498461234541
a. One 6) How many even numbers are immediately followed
b. Two by a prime number?
c. Three a. One
d. More than three b. Two

Page 169 of 722

Subscribe the Xpress Video Course & Mock Test Package for Bank & Insurance Exams
If there are any suggestions/ errors in our PDFs Feel Free to contact us via this email: admin@exampundit.in
IBPS RRB Clerk Prelims – Ultra Practice Bundle PDF
c. Four 10) How many multiples of 3 are between first ‘2’ from
d. Three left and right end?
e. None of these a. Three
7) If all the even numbers are dropped from the b. Five
arrangement then which of the following will be 5th c. Six
number to the right of the 4th number from the left d. Four
end of the given arrangement? e. None of these
a. 9 Directions (11-15): Study the following information
b. 5 carefully to answer the given questions:
c. 1 245 854 457 652 129
d. 3 11) If we interchange the 1st and 2nd digit of each
e. None of these number then which number will be the lowest among
8) In the given arrangement, how many such numbers them?
are there which are immediately preceded by an odd a. 245
number? b. 854
a. Seven c. 652
b. Three d. 129
c. Four e. 457
d. Nine 12) If we subtract 1 from the 3rd digit and then
e. None of these interchange with the 1st digit, then which number will
9) What is the product of numbers between the first ‘9’ be the greatest among them?
from left end and fifth ‘4’ from right end? a. 129
a. 130 b. 457
b. 125 c. 245
c. 114 d. 652
d. 126 e. 854
e. None of these 13) If all the three digits of each number are arranged
in ascending order and then all the numbers are
Page 170 of 722

Subscribe the Xpress Video Course & Mock Test Package for Bank & Insurance Exams
If there are any suggestions/ errors in our PDFs Feel Free to contact us via this email: admin@exampundit.in
IBPS RRB Clerk Prelims – Ultra Practice Bundle PDF
arranged in decreasing order then find which element value is divided by 3, which number is perfectly
will be 3rd from left end? divided by 3 with the highest quotient?
a. 854 a. 7931
b. 245 b. 6148
c. 652 c. 4389
d. 129 d. 5124
e. 457 e. 2478
14) If all the three digits of each of the numbers are 17) If in each of the given numbers, the fourth digit of
added the resulting sum of which of the following the number is subtracted from the first digit, which
numbers will be a perfect square? number will get the second highest value?
a. 245 a. 6148
b. 854 b. 5124
c. 652 c. 4389
d. 129 d. 7931
e. 457 e. 2478
15) If in each number second and third digits are 18) In each of the given numbers, the fourth digit is
interchanged, then how many even numbers are subtracted from the sum of the first three digits from
there? left then which number will yield highest value?
a. 4 a. 6148
b. 3 b. 2478
c. 2 c. 4389
d. 5 d. 7931
e. 1 e. 5124
Directions (16-20): Answer the questions based on the 19) If all the odd digits in the number are multiplied in
information given below. each of the given numbers, which of the given numbers
4389 5124 7931 2478 6148 will yield a perfect cube?
16) If in each of the given numbers, the second digit is a. 6148
interchanged with the fourth digit and the obtained b. 4389
Page 171 of 722

Subscribe the Xpress Video Course & Mock Test Package for Bank & Insurance Exams
If there are any suggestions/ errors in our PDFs Feel Free to contact us via this email: admin@exampundit.in
IBPS RRB Clerk Prelims – Ultra Practice Bundle PDF
c. 2478 23) How many 8s are preceded by 9 and followed by 6?
d. 7931 a. Two
e. Both (a) and (b) b. Three
20) If the difference between the highest and the lowest c. One
digit in each of the given numbers is considered then d. More than Three
which of the following is the lowest value? e. None
a. 7 24) In the following series of number, find out how
b. 3 many times 3 have appeared?
c. 4 a. Three
d. 8 b. Two
e. 6 c. Four
Direction (21-25): Study the following number d. Five
sequence and answer the questions following it. e. Six
9861579832798627615432521798614 25) In the following series of number, find out how
21) How many even numbers are there in the sequence many times 1 followed by 5 have appeared together?
each of which are immediately followed by an odd a. Three
number? b. One
a. Seven c. Four
b. Nine d. Two
c. Six e. None
d. Eight Directions (26-30): The following questions are based
e. None on the three digits five numbers given below.
22) How many 9s are followed by 8? 545 644 784 527 898
a. Four 26) If the 1st and 3rd digit of each of the number are
b. Three multiplied then the resultant of which of the following
c. Two numbers will be divisible by 5?
d. One a. 644
e. None b. 545 and 527
Page 172 of 722

Subscribe the Xpress Video Course & Mock Test Package for Bank & Insurance Exams
If there are any suggestions/ errors in our PDFs Feel Free to contact us via this email: admin@exampundit.in
IBPS RRB Clerk Prelims – Ultra Practice Bundle PDF
c. 527 and 898 e. 5
d. 545 30) What will be the number obtained if 2nd digit of
e. 644 and 784 the largest number is multiplied with 3rd digit of the
27) If 4 is added to the 1st digit of each number and smallest number?
then the positions of the 2nd and the 3rd digits are a. 64
interchanged, which of the following will be the highest b. 57
number? c. 63
a. 545 d. 54
b. 527 e. 44
c. 898 Direction (31-35): Answer the questions based on the
d. 784 information given below.
e. 644 These questions are based on the three digits five
28) If 2 is subtracted from the 2nd digit of each number numbers.
and then the positions of the 1st and the 3rd digit are 176 489 721 562 723
interchanged, which of the following will be the lowest 31) If in each of the given numbers, third digit is
number? interchanged by first digit then which number gives
a. 545 the highest quotient after being perfectly divisible by
b. 527 3?
c. 898 a. 562
d. 784 b. 721
e. 644 c. 176
29) If in each number 1st and 3rd digits are d. 489
interchanged, then how many even numbers are e. 723
there? 32) If in each of the given numbers, the third digit of
a. 1 the number is subtracted from the second digit, then
b. 2 which number will get the highest value?
c. 3 a. 721
d. 4 b. 489
Page 173 of 722

Subscribe the Xpress Video Course & Mock Test Package for Bank & Insurance Exams
If there are any suggestions/ errors in our PDFs Feel Free to contact us via this email: admin@exampundit.in
IBPS RRB Clerk Prelims – Ultra Practice Bundle PDF
c. 176 234 435 716 901 625
d. 562 36) If all the numbers are arranged in ascending order,
e. 723 what will be the result of the product (multiplication)
33) In each of the given numbers, the second digit is of the first digit of the third number and the third digit
subtracted from the sum of the first and last digits, of the first number from the left end of the new
which number will yield the second highest value? arrangement?
a. 489 a. 35
b. 721 b. 58
c. 562 c. 87
d. 723 d. 24
e. 176 e. 8
34) If the digits of numbers are added and 5 is 37) What will be the number obtained if the highest
subtracted, then which number gives the highest number in the series is added with the second-lowest
number? number in the series?
a. 721 a. 1735
b. 562 b. 643
c. 723 c. 1336
d. 489 d. 638
e. 176 e. 1073
35) If in each number first and third digits are 38) If the highest and the lowest numbers are added,
interchanged, then how many odd numbers are there? what will be the product of the first and third digit of
a. 4 the resulting sum?
b. 3 a. 8
c. 2 b. 5
d. 5 c. 3
e. 1 d. 1
Direction (36-40): The following questions are based e. 4
on the five three-digit numbers given below.
Page 174 of 722

Subscribe the Xpress Video Course & Mock Test Package for Bank & Insurance Exams
If there are any suggestions/ errors in our PDFs Feel Free to contact us via this email: admin@exampundit.in
IBPS RRB Clerk Prelims – Ultra Practice Bundle PDF
39) If 1 is subtracted from the odd digits in each 42) Which digit has least frequency in the above set of
number, which of the following is the new highest figures?
number? a. 5
a. 234 b. 9
b. 435 c. 8
c. 716 d. 6
d. 901 e. 3
e. 625 43) If the number of elements in the series is divided
40) If all the digits in each number are arranged in from the frequency of 3 in the series, what is the
descending order, which of the following number will remainder?
remain unchanged? a. 4
a. 234 b. 7
b. 716 c. 5
c. 625 d. 8
d. 901 e. None
e. none 44) Which digit has the lowest frequency leaving digit
Directions (41-45): Following questions are based on 4 and 8 in the above set of numbers.
the given sequence of numbers: a. 6
9395936964353583839393953959466 b. 9
4696 c. 5
41) How many 3s are there which are preceded by 9 d. Cannot be determined
and immediately followed by 9 in the above set of e. None of these
numbers? 45) There are pairs of adjoining figures which add up
a. 1 to 13. How many such pairs are there?
b. 4 a. 1
c. 5 b. 2
d. 2 c. 3
e. 3 d. 6
Page 175 of 722

Subscribe the Xpress Video Course & Mock Test Package for Bank & Insurance Exams
If there are any suggestions/ errors in our PDFs Feel Free to contact us via this email: admin@exampundit.in
IBPS RRB Clerk Prelims – Ultra Practice Bundle PDF
e. None a. 158
Directions (46-50): The following questions are based b. 159
on the five four-digit numbers given below: c. 160
5689 1676 3648 2941 5937 d. 161
46) If all the digits of each of the numbers are added e. 162
the resulting sum of which of the following numbers is 49) If second and third digits are interchanged within
an odd number? the numbers then what is the difference between the
a. 5689 highest and lowest number of the resultant series?
b. 1676 a. 4103
c. 3648 b. 4003
d. 2941 c. 5103
e. 5937 d. 5003
47) If each of the numbers given in the series are e. 4113
reversed then which of the following numbers is 50) What will be the number obtained if the lowest
second highest number? number in the series is added with the second-highest
a. 5689 number in the series?
b. 1676 a. 7365
c. 3648 b. 6430
d. 2941 c. 1336
e. 5937 d. 6380
48) If first and third digits of each number are e. 1073
removed then what is the sum of the second highest
and second lowest numbers of the resultant series?

Page 176 of 722

Subscribe the Xpress Video Course & Mock Test Package for Bank & Insurance Exams
If there are any suggestions/ errors in our PDFs Feel Free to contact us via this email: admin@exampundit.in
IBPS RRB Clerk Prelims – Ultra Practice Bundle PDF

8). Number Sequence Questions - Detailed Explanation with Answers

Answers: Hence, the new lowest number is 234.


1. b Solution 3
2. e Given series: 234 756 867 123 815
3. b After the digits of each number arranged in ascending
4. b order:
5. c Old New
Solution 1 number number
Given series: 234 756 867 123 815 234 234
1) On subtracting 5 from even numbers and adding 1 to 756 567
the odd numbers: 867 678
229 751 868 124 816 123 123
Old New 815 158
number number
234 229 New series will be: 234 567 678 123 158
756 751 567 and 123 are the odd numbers.
867 868 Hence, there are two odd numbers.
123 124
815 816 Solution 4
Given series: 234 756 867 123 815
Solution 2 Smallest number = 123
Given series: 234 756 867 123 815 Second digit of 123 (smallest number) = 2
Highest number: 867 Largest number = 867
Lowest number: 123 First digit of 867 (highest number) = 8
On reversing highest and lowest number, the new On multiplying them we get, 2 × 8 = 16
arrangement: 234 756 768 321 815 Hence, the number obtained is 16.

Page 177 of 722

Subscribe the Xpress Video Course & Mock Test Package for Bank & Insurance Exams
If there are any suggestions/ errors in our PDFs Feel Free to contact us via this email: admin@exampundit.in
IBPS RRB Clerk Prelims – Ultra Practice Bundle PDF
Solution 5 So, the 5th number to the right of the 4th number from
Given series: 234 756 867 123 815 the left end of the given arrangement is ‘5’.
On adding digits of a number: Hence, option b is correct.
234 → 2 + 3 + 4 = 9 Solution 8
756 → 7 + 5 + 6 = 18 Given series,
867 → 8 + 6 + 7 = 21 64392197426498461234541
123 → 1 + 2 + 3 = 6 So, there are nine numbers, which are immediately
815 → 8 + 1 + 5 = 14 preceded by an odd number.
Here, 9 is a perfect square. Hence, option d is correct.
Hence, the required number is 234. Solution 9
Answers: Given series,
6. c 64392197426498461234541
7. b Product = 2 × 1 × 9 × 7 = 126
8. d Hence, option d is correct.
9. d Solution 10
10. d Given series,
Solution 6 64392197426498461234541
Given series, There are four multiples of ‘3’ between first ‘2’ from left
64392197426498461234541 and right end.
Four even numbers are immediately followed by a prime Hence, option d is correct.
number. Answers:
Hence, option c is correct. 11. d
Solution 7 12. a
Given series, 13. c
64392197426498461234541 14. e
If we dropped even numbers, 15. c
3919791351 Solution 11
Given series:- 245 854 457 652 129
Page 178 of 722

Subscribe the Xpress Video Course & Mock Test Package for Bank & Insurance Exams
If there are any suggestions/ errors in our PDFs Feel Free to contact us via this email: admin@exampundit.in
IBPS RRB Clerk Prelims – Ultra Practice Bundle PDF
Interchanging 1st and 2nd digit of each number, 652 → 6 + 5 + 2 = 11
425 584 547 562 219 129 → 1 + 2 + 9 = 12
So from the above series 219 is the lowest term among 457 → 4 + 5 + 7 = 16
them. Hence, “457” is the number whose sum is a perfect
Hence, “129” is the correct answer. square.
Solution 12 Solution 15
Given series:- 245 854 457 652 129 Given series:- 245 854 457 652 129
Subtracting 1 from 3rd digit, Interchanging second and third number,
244 853 456 651 128 254 845 475 625 192
Interchanging it with 1st digit, Hence, there are two such numbers which are even
442 358 654 165 821 numbers.
So from the above series 821 is the greatest one among Answers:
them. 16. d
Hence, “129” is the correct answer. 17. b
Solution 13 18. d
Given series :- 245 854 457 652 129 19. e
Arranging each number in ascending order, 20. c
245 458 457 256 129 Solution 16
Arranging numbers in decreasing order, Given numbers:
458 457 256 245 129 4389 5124 7931 2478 6148
So, from the above series we can say that “256” comes The second digit interchanged with the fourth digit, we
3rd from left end. get
Hence, “652” is the correct answer. 4983 5421 7139 2874 6841
Solution 14 On dividing these values, the numbers perfectly divisible
Given series: 245 854 457 652 129 by 3 with the highest quotient,
On adding the digits, 4983 – Divisible with quotient 1661
245 → 2 + 4 + 5 = 11 5421 – Divisible with quotient 1807
854 → 8 + 5 + 4 = 17 7139 – Not divisible
Page 179 of 722

Subscribe the Xpress Video Course & Mock Test Package for Bank & Insurance Exams
If there are any suggestions/ errors in our PDFs Feel Free to contact us via this email: admin@exampundit.in
IBPS RRB Clerk Prelims – Ultra Practice Bundle PDF
2874 – Divisible with quotient 958 Difference between the highest and the lowest digit in
6841 – Not divisible each of the given numbers,
So, 1807 is the highest quotient which represents 5124. 64867
Solution 17 Lowest value: 4
Given numbers: Answers:
4389 5124 7931 2478 6148 21. b
The fourth digit of the numbers is subtracted from the 22. a
first digit, we get 23. b
-5 1 6 -6 -2 24. b
The second highest number is 1. 25. d
Solution 18 Solution 21
Given numbers: Given Series: 9 8 6 1 5 7 9 8 3 2 7 9 8 6 2 7 6 1 5 4 3 2 5
4389 5124 7931 2478 6148 21798614
The fourth digit is subtracted from the sum of the first Clearly, an even number followed by an odd number in
three digits, we get the sequence are:
6 4 18 5 3 9861579832798627615432521798614
So, the highest value; 18, which is represented by 7931. Thus, the correct answer is Nine.
Solution 19 Solution 22
Given numbers: Given Series: 9 8 6 1 5 7 9 8 3 2 7 9 8 6 2 7 6 1 5 4 3 2 5
4389 5124 7931 2478 6148 21798614
If all the odd digits in the number are multiplied for each Clearly, there are four 9s followed by 8.
of the given numbers, we get- 9861579832798627615432521798614
27 5 189 7 1 Thus, the correct answer is four.
Perfect cube: 27 and 1 Solution 23
Solution 20 Given Series: 9 8 6 1 5 7 9 8 3 2 7 9 8 6 2 7 6 1 5 4 3 2 5
Given numbers: 21798614
4389 5124 7931 2478 6148 Clearly, there are three 8s preceded by 9 and followed by
6.
Page 180 of 722

Subscribe the Xpress Video Course & Mock Test Package for Bank & Insurance Exams
If there are any suggestions/ errors in our PDFs Feel Free to contact us via this email: admin@exampundit.in
IBPS RRB Clerk Prelims – Ultra Practice Bundle PDF
9861579832798627615432521798614 527: 5*7 = 35
Thus, the correct answer is three. 898: 8*8 = 64
Here 35 and 25 are only divisible by 5.
Solution 24 Hence answer is 527 and 545.
Given Series: 9 8 6 1 5 7 9 8 3 2 7 9 8 6 2 7 6 1 5 4 3 2 5 Solution 27
21798614 Given series: 545 644 784 527 898
Clearly, 3 appears two times in the given number series. Adding 4 to the 1st digit: 945 1044 1184 927 1298
9861579832798627615432521798614 interchanging the 2nd and the 3rd digit: 954 1404 1814
Thus, the correct answer is two. 972 1928
Solution 25 So, 1928 is highest number which came from 898.
Given Series: 9 8 6 1 5 7 9 8 3 2 7 9 8 6 2 7 6 1 5 4 3 2 5 Solution 28
21798614 Given series: 545 644 784 527 898
Clearly, 1 and 5 have appeared two times together in the Subtracting 2 from the 2nd digit: 525 624 764 507 878
given number series. interchanging the 1st and the 3rd digit: 525 426 467 705
9861579832798627615432521798614 878
Thus, the correct answer is two. So 426 is the lowest number, which came from 644.
Answers: Solution 29
26. b Given series: 545 644 784 527 898
27. c Interchanging the 1st and 3rd digits: 545 446 487 725 898
28. e hence 2 even number, which came from 644, 898
29. b Solution 30
30. c Given series: 545 644 784 527 898
Solution 26 Largest no is in the series: 898
Given series: 545 644 784 527 898 Smallest no is in the series: 527
on multiplying 1st and last digits: 2nd digit of largest no: 9
545: 5*5 = 25 3rd digit of smallest no: 7
644: 6*4 = 24 After multiplying this: 9*7 = 63
784: 7*4 = 28 Answers:
Page 181 of 722

Subscribe the Xpress Video Course & Mock Test Package for Bank & Insurance Exams
If there are any suggestions/ errors in our PDFs Feel Free to contact us via this email: admin@exampundit.in
IBPS RRB Clerk Prelims – Ultra Practice Bundle PDF
31. d The 2nddigit is subtracted from the sum of the first three
32. d digits, we get
33. b 05618
34. d So, the second highest value is 6, which is represented by
35. a 721.
Solution 31 Solution 34
Given numbers:176 489 721 562 723 Given numbers:
The third digit interchanged with the first digit, we get 176 489 721 562 723
671 984 127 265 327 If all the number are added and 5 is subtracted, we get,
On dividing these values, the numbers perfectly divisible 9 16 5 8 7
by 3 with the highest quotient, Highest number: 16
671 – Not divisible Solution 35
984 – 328 Given series: 176 489 721 562 723
127– Not divisible After interchanging First and Third digit
265 – Not divisible 617 984 127 265 327
327 – 109 Hence, there are four odd numbers.
So, 328 is the highest quotient which represents 489. Answers:
Solution 32 36. d
Given numbers: 37. c
176 489 721 562 723 38. c
The third digit of the numbers is subtracted from the 2nd 39. d
digit, we get 40. e
1 -1 1 4 -1 Solution 36
The highest number is 4, which is represented by 562. Given series: 234 435 716 901 625
Solution 33 After rearranging the numbers in ascending order:
Given numbers: 234 435 625 716 901
176 489 721 562 723 The first digit of the third number (625) = 6
The third digit of the first number (234) = 4
Page 182 of 722

Subscribe the Xpress Video Course & Mock Test Package for Bank & Insurance Exams
If there are any suggestions/ errors in our PDFs Feel Free to contact us via this email: admin@exampundit.in
IBPS RRB Clerk Prelims – Ultra Practice Bundle PDF
On multiplying the first digit of the third number and the 901 9 and 1
third digit of the first number, we get: 625 2 and 5
6 × 4 = 24
Hence, the resultant product is 24. On subtracting 1 from odd digits:
Solution 37 224 424 606 800 624
Given series: 234 435 716 901 625 Hence, the new highest number is 800.
The highest number in the series = 901 Solution 40
The second-lowest number in the series = 435 Given series: 234 435 716 901 625
On addition of the highest number and second-lowest On rearranging digits in a number in descending order,
number in a series, we get: Old New
901 + 435 = 1336 number number
Hence, the sum will be 1336. 234 432
Solution 38 435 543
Given series: 234 435 716 901 625 716 761
Highest number = 901 901 910
Lowest number = 234 625 652
The sum of the highest and the lowest number = 901 +
234 = 1135; Thus, we get the following arrangement:
The product of the first and third digit of the sum (1135) 432 543 761 910 652
=1×3=3 Here, none of the numbers remain unchanged.
Hence, the product will be 3. Hence, the answer is ‘none’.
Solution 39 Answers:
Given series: 234 435 716 901 625 41. e
Number Odd digits 42. c
234 3 43. d
435 3 and 5 44. c
716 7 and 1 45. b

Page 183 of 722

Subscribe the Xpress Video Course & Mock Test Package for Bank & Insurance Exams
If there are any suggestions/ errors in our PDFs Feel Free to contact us via this email: admin@exampundit.in
IBPS RRB Clerk Prelims – Ultra Practice Bundle PDF
Solution 41 Thus, 8 is the answer.
Given series: 9 3 9 5 9 3 6 9 6 4 3 5 3 5 8 3 8 3 9 3 9 3 9 Solution 44
539594664696 Given series: 9 3 9 5 9 3 6 9 6 4 3 5 3 5 8 3 8 3 9 3 9 3 9
3s are there which are preceded by 9 and immediately 539594664696
followed by 9 Let’s check,
Pattern required: 9 → 3 → 9 We have,
9395936964353583839393953959466 9’s = 10
4696 6’s = 6
Clearly, three such 3s are there. 3’s = 9
Solution 42 5’s = 5
Given series: 9 3 9 5 9 3 6 9 6 4 3 5 3 5 8 3 8 3 9 3 9 3 9 Thus, digit 5 has lowest frequency leaving digit 4 and 8.
539594664696 Solution 45
Let’s check, Given series: 9 3 9 5 9 3 6 9 6 4 3 5 3 5 8 3 8 3 9 3 9 3 9
We have, 539594664696
8’s = 2 Clearly there is two such pair of adjoining figures which
9’s = 10 add up to 13.
6’s = 6 Answers:
3’s = 9 46. c
5’s =5 47. c
4’s = 3 48. b
So, 8’s has least frequency in the above set of figures. 49. a
Solution 43 50. a
Given series: 9 3 9 5 9 3 6 9 6 4 3 5 3 5 8 3 8 3 9 3 9 3 9 Solution 46
539594664696 Given numbers: 5689 1676 3648 2941 5937
Number of elements in the series = 35 …(1) On adding all the digits
Let’s check, 5689 → 28
3's = 9 ….(2) 1676 → 20
When 35 is divided by 9 the remainder will be 8. 3648 → 21
Page 184 of 722

Subscribe the Xpress Video Course & Mock Test Package for Bank & Insurance Exams
If there are any suggestions/ errors in our PDFs Feel Free to contact us via this email: admin@exampundit.in
IBPS RRB Clerk Prelims – Ultra Practice Bundle PDF
2941 → 16 97, 91, 69, 68, 66
5937 → 24 Sum of second highest and second lowest numbers: 91 +
Here, all numbers are even except 21. 68 = 159
Hence, odd number is 3648. Hence, answer is 159.
Solution 47 Solution 49
Given numbers: 5689 1676 3648 2941 5937 Given numbers: 5689 1676 3648 2941 5937
On reversing the numbers Interchanging the second and third digits within the
5689 → 9865 numbers:
1676 → 6761 5689 → 5869
3648 → 8463 1676 → 1766
2941 → 1492 3648 → 3468
5937 → 7395 2941 → 2491
Hence, the second highest number is 3648 5937 → 5397
Solution 48 The highest and lowest numbers are 5869 and 1766
Given numbers: 5689 1676 3648 2941 5937 respectively
Removing first and third digits: The difference is 5869 – 1766 = 4103
5689 → 69 Hence, answer is 4103.
1676 → 66 Solution 50
3648 → 68 Given numbers: 5689 1676 3648 2941 5937
2941 → 91 Adding the lowest number and second highest number
5937 → 97 5689 + 1676 = 7365
Arranging the numbers in descending order: Hence, 7365 is the correct answer.

9). Alphabet Series Questions

Direction (1-5): Following questions are based on five (The new words formed after performing the mentioned
words given below. operations may or may not necessarily be meaningful
DEW BIG RAW FAN DOG English words)
Page 185 of 722

Subscribe the Xpress Video Course & Mock Test Package for Bank & Insurance Exams
If there are any suggestions/ errors in our PDFs Feel Free to contact us via this email: admin@exampundit.in
IBPS RRB Clerk Prelims – Ultra Practice Bundle PDF
1) If alphabets of the given words are arranged 4) If the first alphabets of each word are exchanged by
according to dictionary order and then words are its next alphabet according to English alphabetical
arranged in the order as they would appear in a series, how many meaningful words are formed?
dictionary from left to right, which of the following a. Two
word will be at last position? b. Three
a. RAW c. One
b. FAN d. Four
c. DOG e. None of these
d. DEW 5) If the positions of first and second alphabets of each
e. BIG word are interchanged, which comes third in the order
2) If the positions of first and last alphabets of each of dictionary?
word are interchanged, how many meaningful words a. FAN
are formed? b. DEW
a. Two c. DOG
b. Three d. RAW
c. One e. BIG
d. Four Directions (6-10): The following questions are based
e. None of these on the five three-letter words given below.
3) If the positions of first and last alphabets of each WOF KEH PIL TAX JUV
word are interchanged and arranged from dictionary 6) If words are arranged according to the alphabetical
order then, which word comes last? series from left to right, which word is fourth from the
a. DEW right end?
b. FAN a. JUV
c. DOG b. KEH
d. RAW c. PIL
e. BIG d. WOF
e. TAX

Page 186 of 722

Subscribe the Xpress Video Course & Mock Test Package for Bank & Insurance Exams
If there are any suggestions/ errors in our PDFs Feel Free to contact us via this email: admin@exampundit.in
IBPS RRB Clerk Prelims – Ultra Practice Bundle PDF
7) If in each word, the vowel is replaced by its 10) If the positions of first and third alphabets of each
preceding letter in English alphabetical series and the word are interchanged, which comes second from the
consonant is replaced by its succeeding letter in right?
English alphabetical series, which word thus formed a. FOW
would come second from the left end? b. HEK
a. QHM c. LIP
b. UZY d. XAT
c. LDI e. VUJ
d. XNG Directions (11-15): Study the following arrangement of
e. KTW numbers carefully and answer the questions.
6839386826275435474692625
8) If the third letter of all the words is changed to its 11) How many even numbers are there which are
next letter (succeeding) of the English alphabetical immediately followed by an odd number and
series, then how many words will have more than one immediately preceded by a prime number?
vowel? (Same or different vowel) a. Two
a. Three b. Four
b. Two c. One
c. More than three d. Three
d. None e. None of these
e. One 12) What is the sum of all the numbers between 2nd ‘6’
9) If the positions of the first and second letters of all from the left end and 3rd ‘5’ from the right end in the
the words are interchanged, how many words will given series?
form meaningful English words? a. 24
a. Zero b. 27
b. Four c. 25
c. Two d. 22
d. One e. None of these
e. Three
Page 187 of 722

Subscribe the Xpress Video Course & Mock Test Package for Bank & Insurance Exams
If there are any suggestions/ errors in our PDFs Feel Free to contact us via this email: admin@exampundit.in
IBPS RRB Clerk Prelims – Ultra Practice Bundle PDF
13) How many prime numbers are to the left of 14th 16) How many consonants are there in the above
number from the right end in the given series? arrangement which are immediately followed by a
a. Three vowel and preceded by a consonant?
b. Four a. Two
c. Two b. Three
d. Five c. Four
e. None of these d. Five
14) How many odd numbers are there which are e. None of these
immediately followed and immediately preceded by an 17) Which of the following alphabet is 9th to the right
even number in the given series? of 4th letter from the left end?
a. Two a. W
b. One b. V
c. Four c. T
d. Three d. A
e. None of these e. None of these
15) Which number is in the middle of the numbers, 18) How many letters are there in between the
which are 7th from the left end and 11th from the right alphabet which is 8th from the left end and the
end in the given series? alphabet which 9th from the right end?
a. 8 a. 13
b. 6 b. 12
c. 7 c. 10
d. 2 d. 17
e. None of these e. 14
Directions (16-20): Study the given arrangement of 19) Complete the given series based on the
alphabets and answer the questions based on it. arrangement given.
WTYCOKPQZBMNAERIKLGDHFUJX CKQ, BNE, ILD, ?
VRQTV a. FUJ
b. QVT
Page 188 of 722

Subscribe the Xpress Video Course & Mock Test Package for Bank & Insurance Exams
If there are any suggestions/ errors in our PDFs Feel Free to contact us via this email: admin@exampundit.in
IBPS RRB Clerk Prelims – Ultra Practice Bundle PDF
c. FJV then how many words having two different vowels will
d. XVR be formed?
e. None of these a. Three
20) If all the vowels from the above arrangement are b. Four
dropped then which of the following letter is 13th form c. One
the left end? d. Two
a. L e. Five
b. R 23) How many meaningful words can be made using
c. K all the letters of the word KEDU only once?
d. Z a. One
e. None of these b. Four
Direction (21-25): The following questions are based c. Three
on the five words given below. (The new words formed d. Two
after performing the mentioned operations may or e. Five
may not necessarily be meaningful English words) 24) How many alphabets (as per English alphabetical
KEDU EDRT FHRA ULTP HKED order) would lie between the second letter of the third
21) If the last alphabet in each of the words is changed word from the left and the second letter of the second
to the next alphabet in the English alphabetical series, word from the right?
then how many words having two identical vowels will a. Five
be formed? b. Six
a. Three c. Three
b. Zero d. Two
c. Two e. Four
d. One 25) Which word will be the second last word if the
e. Four words are arranged according to their place in the
22) If the first alphabet in each of the words is changed dictionary taking into consideration the last letter of
to the next alphabet in the English alphabetical series, each word?
a. KEDU
Page 189 of 722

Subscribe the Xpress Video Course & Mock Test Package for Bank & Insurance Exams
If there are any suggestions/ errors in our PDFs Feel Free to contact us via this email: admin@exampundit.in
IBPS RRB Clerk Prelims – Ultra Practice Bundle PDF
b. EDRT alphabetical series, in how many words thus formed
c. FHRA will no vowels appear?
d. ULTP a. None
e. HKED b. One
Directions (26-30): The following questions are based c. Two
on the six words given below. d. Three
MFE HTY ELG PSY NOE LZB e. Five
26) If first two alphabets of each word are 29) How many letters are there in the English
interchanged, then how many meaningful words are alphabetical order between third letter of the word
formed? which is fourth from the right and the second letter of
a. One the word which is fourth from the left of the given
b. Two words?
c. Three a. Ten
d. Four b. Two
e. Five c. Eleven
27) If the first alphabet of each word is interchanged d. Thirteen
with the third alphabet of it, then which word will be e. Three
the second if they are arranged in the order as they 30) If last two alphabets of each word are
would appear in dictionary? interchanged, then how many meaningful words are
a. MFE formed?
b. HTY a. One
c. ELG b. Two
d. PSY c. Three
e. NOE d. Four
28) If in each of the given words, each of the e. Five
consonants is changed to previous letter and each Direction (31-35): Following questions are based on
vowel is changed to next letter in the English five words given below.

Page 190 of 722

Subscribe the Xpress Video Course & Mock Test Package for Bank & Insurance Exams
If there are any suggestions/ errors in our PDFs Feel Free to contact us via this email: admin@exampundit.in
IBPS RRB Clerk Prelims – Ultra Practice Bundle PDF
(The new words formed after performing the mentioned b. Two
operations may or may not necessarily be meaningful c. Three
English words) d. Four
TOB UDC RES PFB YWQ e. None
31) If we interchange both alphabets of each word 34) If in each of the words, all the alphabets are
present at odd positions, how many words ending with arranged in English alphabetical order within the
vowels will be formed? word, how many words will end with a consonant?
a. Four a. Three
b. Three b. Four
c. One c. One
d. Two d. Two
e. None of These e. None
32) If the first and last alphabet of each word is 35) How many letters are there in the English
changed to the previous alphabet and the middle alphabetical series between the first letter of the word
alphabet is changed to its next alphabet in the English which is second from the left and the second letter of
alphabetical order, how many words having only the word which is first from the right of the given
consonants will be formed? words?
a. One a. One
b. Three b. Two
c. Four c. Three
d. Two d. Four
e. Five e. None
33) If the given words are arranged in the order as they Direction (36-40): Study the following alphabet series
would appear in a dictionary from left to right, how and answer the question that follows.
many alphabets are there in English alphabetical THATISAWONDERFULSCENERY
series between first and last letter of the word which 36) If each consonant is substituted with the letter
will be at fourth from right? preceding it in the English alphabetical series and each
a. One vowel is substituted with the next letter of the
Page 191 of 722

Subscribe the Xpress Video Course & Mock Test Package for Bank & Insurance Exams
If there are any suggestions/ errors in our PDFs Feel Free to contact us via this email: admin@exampundit.in
IBPS RRB Clerk Prelims – Ultra Practice Bundle PDF
alphabetical series. How many vowels are present in a. STW
the new arrangement? b. OAD
a. 1 c. URS
b. 2 d. EEY
c. 3 e. IAO
d. 5 40) Which of the following alphabet is 9th to the right
e. 4 of 4th letter from the left end?
37) If seven letters from the right end are placed a. N
between the fourth and the fifth letter from the left end b. D
then which is the 8th letter from the left end in the new c. E
arrangement? d. R
a. I e. None of these
b. S Directions (41-45): Study the following arrangement of
c. N the English alphabet and answer the questions given
d. C below:
e. E VGCLXQDUITHSFPZJRNOWYMBEK
38) Which letter is placed at the immediate left of the A
23rd letter of the English alphabet in the 41) Four of the following five are alike in a certain way
arrangement? based on their position in the above arrangement and
a. W hence form a group. Which one does not belong to that
b. O group?
c. N a. BKY
d. D b. VCX
e. A c. DIX
39) Four of the following five are alike in a certain way d. SPT
based on their positions in the above arrangement and e. NWJ
hence form a group. Which one among the following
does not belong to that group?
Page 192 of 722

Subscribe the Xpress Video Course & Mock Test Package for Bank & Insurance Exams
If there are any suggestions/ errors in our PDFs Feel Free to contact us via this email: admin@exampundit.in
IBPS RRB Clerk Prelims – Ultra Practice Bundle PDF
42) Which of the following letter will be at the c. V
immediate right of the 2nd vowel of the English d. D
alphabetical series? e. A
a. B Directions (46-50): Study the following alphabet series
b. D and answer the questions that follow:
c. K ACEUHDSVZPOJGRYNLKBWT
d. I 46) In the given series, how many vowels are
e. W immediately followed by another vowel?
43) If first six letters from the left end are placed a. One
between the seventh and the eighth letters from the b. Two
right end, what will be the fifth letter to the right of the c. Three
twelfth letter from the left? d. None
a. H e. More than three
b. L 47) Which of the following letters will be third to the
c. S left of the first vowel from the right end?
d. O a. G
e. D b. E
44) Which is the 8th letter from the left of the letter c. V
which is 3rd from the right? d. L
a. Z e. H
b. J 48) If all the letters from the given series are arranged
c. N in alphabetical order from the left, then how many
d. R letters will not change its position?
e. O a. five
45) Which letter is placed at the immediate left of the b. two
7th letter of the English alphabet in the arrangement? c. three
a. W d. one
b. O e. four
Page 193 of 722

Subscribe the Xpress Video Course & Mock Test Package for Bank & Insurance Exams
If there are any suggestions/ errors in our PDFs Feel Free to contact us via this email: admin@exampundit.in
IBPS RRB Clerk Prelims – Ultra Practice Bundle PDF
49) How many letters are there in the English 50) Four of the following five are alike in a certain way
alphabetical series between the sixth letter from the based on their positions in the above arrangement and
right end and the seventh letter from the left end? hence form a group. Which one does not belong to that
a. Two group?
b. Five a. CEH
c. Seven b. VZJ
d. Eight c. DSZ
e. Four d. RYL
e. JGY

9). Alphabet Series Questions - Detailed Explanation with Answers

Answers: Given words are: DEW BIG RAW FAN DOG


1. c On exchanging the alphabets:
2. d WED GIB WAR NAF GOD
3. a The meaningful words are: WED, GIB, WAR, and GOD
4. c Thus, the correct answer is ‘Four’.
5. b Solution 3
Solution 1 Given words are: DEW BIG RAW FAN DOG
Given words are: DEW BIG RAW FAN DOG On exchanging the alphabets:
On arranging alphabets according to the dictionary: DEW WED GIB WAR NAF GOD
BGI ARW AFN DGO When arranged in dictionary order: GIB, GOD, NAF,
Now on arranging words in the order as they appear in a WAR, WED
dictionary from left to right, we get: WED is the last word after arranged in dictionary order.
Left side: AFN ARW BGI DEW DGO : Right side Thus, the correct answer is ‘DEW’.
Thus, DOG is the last word. Solution 4
Solution 2 Given words are: DEW BIG RAW FAN DOG
Page 194 of 722

Subscribe the Xpress Video Course & Mock Test Package for Bank & Insurance Exams
If there are any suggestions/ errors in our PDFs Feel Free to contact us via this email: admin@exampundit.in
IBPS RRB Clerk Prelims – Ultra Practice Bundle PDF
On exchanging the alphabets: XNG LDI QHM UZY KTW
EEW CIG SAW GAN EOG So, the word that is second from the left end: LDI
The meaningful words are: SAW Hence, the answer is LDI.
Thus, the correct answer is ‘RAW’. Solution 8
Solution 5 Hence, the answer is KEH.
Given words are: DEW BIG RAW FAN DOG Given series- WOF KEH PIL TAX JUV
On exchanging the alphabets: The third letter of all the words is changed to its next letter
EDW IBG ARW AFN ODG (succeeding) of the English alphabetical series: WOG
Dictionary order: AFN, ARW, EDW, IBG, ODG KEI PIM TAY JUW
Third word from dictionary order is: EDW So, one word (KEI) will have more than one vowel.
Thus, the correct answer is ‘DEW’. Hence, the answer is one.
Answers: Solution 9
6. b Given series- WOF KEH PIL TAX JUV
7. c Positions of the first and second letters of all the words are
8. e interchanged:
9. a OWF EKH IPL ATX UJV
10. d So, no word will form meaningful English words.
Solution 6 Hence, the answer is zero.
Given series- WOF KEH PIL TAX JUV Solution 10
Words are arranged according to the alphabetical series Given series- WOF KEH PIL TAX JUV
from left to right: Positions of the first and third letters of all the words are
JUV KEH PIL TAX WOF interchanged:
So, KEH is fourth from the right end. FOW HEK LIP XAT VUJ
Solution 7 Hence, second word from the right is XAT.
Given series- WOF KEH PIL TAX JUV Answers:
After replacing the vowels with their preceding letters and 11. a
constants with their succeeding letters in English 12. c
alphabetical series, the resulting series formed would be: 13. b
Page 195 of 722

Subscribe the Xpress Video Course & Mock Test Package for Bank & Insurance Exams
If there are any suggestions/ errors in our PDFs Feel Free to contact us via this email: admin@exampundit.in
IBPS RRB Clerk Prelims – Ultra Practice Bundle PDF
14. a Hence, option a is correct
15. d Solution 15
Solution 11 Given series
Given series 6839386826275435474692625
6839386826275435474692625 ‘2’ is in the middle of 7th number from the left end and
There are two such even numbers which are immediately 11th number from the right end.
followed by an odd number and immediately preceded by Hence, option d is correct.
a prime number. Answers:
Hence, option (a) is correct. 16. b
Solution 12 17. d
Given series 18. a
6839386826275435474692625 19. c
Sum of all the numbers between 2nd ‘6’ from the left end 20. c
and 3rd ‘5’ from the right end is Solution 16
(8 + 2 + 6 + 2 + 7) = 25. Given arrangement:
Hence, option c is correct. WTYCOKPQZBMNAERIKLGDHFUJX
Solution 13 VR Q T V
Given series So, there are three such consonant which are immediately
6839386826275435474692625 followed by a vowel and preceded by a consonant i.e.
There are four prime numbers to the left of 14th element YCO, MNA, HFU.
from the right end in the given series. Hence, option b is correct.
Hence, option b is correct.
Solution 14 Solution 17
Given series Given arrangement
6839386826275435474692625 WTYCOKPQZBMNAERIKLGDHFUJX
There are two such odd numbers, which are immediately VR Q T V
followed and immediately preceded by an even number in 9th to the right + 4th from left = 13th from the left i.e. A.
the given series. Hence, option d is correct.
Page 196 of 722

Subscribe the Xpress Video Course & Mock Test Package for Bank & Insurance Exams
If there are any suggestions/ errors in our PDFs Feel Free to contact us via this email: admin@exampundit.in
IBPS RRB Clerk Prelims – Ultra Practice Bundle PDF
Solution 18 24. c
Given arrangement: 25. b
WTYCOKPQZBMNAERIKLGDHFUJX Solution 21
VRQTV Given: KEDU EDRT FHRA ULTP HKED
So, 8th letter from the left i.e. Q and 9th letter form the 1) On changing the last letter of each word to the next
right i.e. F. letter in the English alphabetical series-
Hence, option a is correct. KEDV EDRU FHRB ULTQ HKEE
Solution 19 2) It is visible that in the series the last word HKEE has
Given arrangement: two identical vowels. So, there is only one word.
WTYCOKPQZBMNAERIKLGDHFUJX Hence, only one word will be formed having two identical
VR Q T V vowels.
Given series: Solution 22
CKQ, BNE, ILD Given: KEDU EDRT FHRA ULTP HKED
So, according to the pattern FJV must be the next set of 1) On changing the first letter of each word to the next
alphabets. alphabet in the English alphabetical series-
Hence, option c is correct. LEDU FDRT GHRA VLTP IKED
Solution 20 2) It is visible that the words IKED and LEDU are the only
Given arrangement: words having two different vowels.
WTYCOKPQZBMNAERIKLGDHFUJX Hence, only two words will be formed having two
VR Q T V different vowels.
So, if all vowels are dropped from the above arrangement Solution 23
then K is the 13th alphabet from the left end. Given: KEDU
Hence, option c is correct. Meaningful words that can be made from KEDU-
DUKE
Answers: There is only one word that can be made using the word.
21. d Hence, the answer is (1) one.
22. d Solution 24
23. a Given: KEDU EDRT FHRA ULTP HKED
Page 197 of 722

Subscribe the Xpress Video Course & Mock Test Package for Bank & Insurance Exams
If there are any suggestions/ errors in our PDFs Feel Free to contact us via this email: admin@exampundit.in
IBPS RRB Clerk Prelims – Ultra Practice Bundle PDF
1) The third word from the left is FHRA and its second Given: MFE HTY ELG PSY NOE LZB
letter is H. 1) On interchanging first and third letter-
2) The second word from the right is ULTP and its second EFM YTH GLE YSP EON BZL
letter is L. 2) Arrangement as per dictionary-
3) The alphabets that lie between H and L are 3 i.e. I, J BZL EFM EON GLE YSP YTH
and K. So, the second word is EFM which is changed form of
Hence three is the correct answer. MFE.
Solution 25 Hence, MFE is the correct answer.
Given: KEDU EDRT FHRA ULTP HKED Solution 28
1) On arranging the words as per the dictionary on the Given: MFE HTY ELG PSY NOE LZB
basis of the last letter- 1) On changing consonants to previous letter and vowel to
FHRA HKED ULTP EDRT KEDU next letter as per English alphabetical series we get:
2) The second last word is EDRT. LEF GSX FKF ORX MPF KYA
Hence the correct answer is EDRT. Note: In English alphabetical series there are 5 vowels i.e.
Answers: A, E, I, O, U else all are consonants
26. d 2) There are three words with no vowel GSX, FKF and
27. a MPF.
28. d Solution 29
29. c Given: MFE HTY ELG PSY NOE LZB
30. a Fourth word from right is ELG and its third letter is G.
Solution 26 Fourth word from the left is PSY and its second letter is
Given: MFE HTY ELG PSY NOE LZB S.
1) On interchanging first two alphabets of each word Letters between G and S as per English alphabetical series
FME THY LEG SPY ONE ZLB are 11 i.e. H, I, J, K, L, M, N, O, P, Q and R.
2) As we can see there are 4 meaningful words so formed Hence, 11 is the correct answer.
in the series: THY, LEG, SPY, ONE Solution 30
Hence, four meaningful words are formed. Given: MFE HTY ELG PSY NOE LZB
Solution 27
Page 198 of 722

Subscribe the Xpress Video Course & Mock Test Package for Bank & Insurance Exams
If there are any suggestions/ errors in our PDFs Feel Free to contact us via this email: admin@exampundit.in
IBPS RRB Clerk Prelims – Ultra Practice Bundle PDF
If last two words are interchanged MEF HYT EGL PYS Note: In English alphabetical series there are 5 vowels i.e.
NEO LBZ A, E, I, O, U else all are consonants.
The meaningful word is NEO= which means New Hence, two words will be formed.
Hence, 1 is the correct answer Solution 33
Answers: Given words are: TOB UDC RES PFB YWQ
31. c On arranging words in the order as they appear in a
32. d dictionary from left to right, we get:
33. e Left side PFB RES TOB UDC YWQ Right side
34. b The fourth word from the right is RES and first and last
35. a letters are R and S respectively.
Solution 31 There are no letters in English alphabetical order between
Given words are: TOB UDC RES PFB YWQ R and S
On interchanging both alphabets of each word present at Hence, none is the correct answer.
odd positions Solution 34
BOT CDU SER BFP QWY Given words are: TOB UDC RES PFB YWQ
Clearly, only one word, CDU ending with vowels will be On arranging alphabets within the word in English
formed. alphabetical order:
Note: In English alphabetical series there are 5 vowels i.e. BOT CDU ERS BFP QWY
A, E, I, O, U else all are consonants. Clearly, four words, BOT, ERS, BFP, and YWQ will end
Hence, only one word will be formed. with a consonant.
Solution 32 Note: In English alphabetical series there are 5 vowels i.e.
Given words are: TOB UDC RES PFB YWQ A, E, I, O, U else all are
On changing the first and last alphabet of each word to the Hence, four words will end with a consonant.
previous alphabet and the middle alphabet to its next Solution 35
alphabet in the English alphabetical order: Given words are: TOB UDC RES PFB YWQ
SPA TEB QFR OGA XXP The second word from the left is UDC and its first letter
Clearly, two words QFR an XXP having only consonants is U
will be formed.
Page 199 of 722

Subscribe the Xpress Video Course & Mock Test Package for Bank & Insurance Exams
If there are any suggestions/ errors in our PDFs Feel Free to contact us via this email: admin@exampundit.in
IBPS RRB Clerk Prelims – Ultra Practice Bundle PDF
First word from the right is YWQ and its second letter is 23rd letter of the English alphabet is W; immediate left to
W W is A.
There is only one letter between U and W in the English Hence, ‘A’ is the correct answer.
alphabetical series and which is V. Solution 39
Hence, one is the correct answer. Given: T H A T I S A W O N D E R F U L S C E N E R
Answers: Y
36. a STW: S - 2 → T, S + 2 → W
37. c OAD: O - 2 → A, O + 2 → D
38. e URS: U - 2 → R, U + 2 → S
39. e EEY: E - 2 → E, E + 2 → Y
40. d IAO: I + 2 → A, A + 2 → O
Solution 36 Hence, IAO is the odd one.
Given: T H A T I S A W O N D E R F U L S C E N E R Solution 40
Y Given: T H A T I S A W O N D E R F U L S C E N E R
New arrangement: S G B S J R B V P M C F Q E V K R Y
BFMFQZ Alphabet which is 9th to the right of 4th letter from the
Hence, there is only one vowel in the new arrangement. left end
Solution 37 THATISAWONDERFULSCENERY
Given: T H A T I S A W O N D E R F U L S C E N E R Hence, R is the correct answer.
Y Answers:
Arrangement: 41. B
Left side T H A T S C E N E R Y I S A W O N D E R F 42. c
U L Right side 43. b
Hence, N is the 8th letter from the left in the new 44. b
arrangement. 45. c
Solution 38 Solution 41
Given: T H A T I S A W O N D E R F U L S C E N E R LeftSide:V G CL X Q D U I T H S F P Z J R N O W Y
Y M B E K A : Right Side
Page 200 of 722

Subscribe the Xpress Video Course & Mock Test Package for Bank & Insurance Exams
If there are any suggestions/ errors in our PDFs Feel Free to contact us via this email: admin@exampundit.in
IBPS RRB Clerk Prelims – Ultra Practice Bundle PDF
BKY: B – 2 = Y, B + 2 = K, Hence, the letter will be L.
VCX: V + 2 = C, C + 2 = X Solution 44
DIX: D – 2 = X, D + 2 = I, Given Series: V G C L X Q D U I T H S F P Z J R N O W
SPT: S – 2 = T, S + 2 = P, YMBEKA
NWJ: N – 2 = J, N + 2 = W. Alphabet which is 8th to the left of 3rd letter from the right
Hence odd one is VCX. end
Solution 42 VGCLXQDUITHSFPZJRNOWYMBEK
Given: A
LeftSide:V G C Hence, J is the correct answer.
L X Q D U I T H S F P Z J R N O W Y M B E K A : Rig
ht Side Solution 45
Second Vowel is E. K is to the immediate right of E. Given Series: V G C L X Q D U I T H S F P Z J R N O W
Hence, K will be to the immediate right of the 2nd vowel YMBEKA
of the English alphabet 7th letter of the English alphabet in the arrangement is G.
Solution 43 Immediate left of G is V.
Given series: Hence, V is the correct answer.
LeftSide:V G C
L X Q D U I T H S F P Z J R N O W Y M B E K A :Rig Answers:
ht Side 46. a
New Arrangement: six letters from the left end are placed 47. c
between the seventh and the eighth letters from the right 48. b
end. 49. e
LeftSide: D U I T H S F P Z J R N O V G C L X Q 50. b
W Y M B E K A :Right Side Solution 46
Left Side: D U I T H S F P Z J R N O V G C L X Q Given series: A C E U H D S V Z P O J G R Y N L K B
W Y M B E K A :Right Side WT
fifth letter to the right of the twelfth letter from the left There are four vowels in the given series: A, E, U, O
is L.
Page 201 of 722

Subscribe the Xpress Video Course & Mock Test Package for Bank & Insurance Exams
If there are any suggestions/ errors in our PDFs Feel Free to contact us via this email: admin@exampundit.in
IBPS RRB Clerk Prelims – Ultra Practice Bundle PDF
Left Side: A C E U H D S V Z P O J G R Y N L K B W Given series: A C E U H D S V Z P O J G R Y N L K B
T : Right Side WT
Only E and U are adjacent to each other. There are four vowels in the given series: A, E, U, O
Therefore, E is immediately followed by U. Left Side: A C E U H D S V Z P O J G R Y N L K B W
Hence, only one vowel is immediately followed by T: Right Side
another vowel. First vowel from the right end: O
The letter third to the left of O: V.
Solution 47 Hence, ‘V’ is the correct answer.

Solution 48
Given Series: A C E U HDSVZPOJGRYNLKBWT
Given series written in English Alphabetical order:-
ABCDEGHJKLNOPRSTUVWYZ
Given Series A C E U H D S V Z P O J G R Y N L K B W T

Given series A B C D E G H J K L N O P R S T U V W Y Z
written in
English
Alphabetical
order

Letters A and R do not change their position.


Hence, two letters do not change their position.

Solution 49 Sixth letter from the right end: N


Given series, Seventh letter from left end: S
Left Side: A C E U H D S V Z P O J G R Y N L K B W In the English alphabetical series, there are four letters
T: Right Side between N and S: O, P, Q, R.
Page 202 of 722

Subscribe the Xpress Video Course & Mock Test Package for Bank & Insurance Exams
If there are any suggestions/ errors in our PDFs Feel Free to contact us via this email: admin@exampundit.in
IBPS RRB Clerk Prelims – Ultra Practice Bundle PDF
Hence, there are ‘four’ letters in the English alphabetical 1) C + 1 → E; E + 2 → H
series between the sixth letter from the right end and the 2) V + 1 → Z; Z + 3 → J
seventh letter from the left end. 3) D + 1 → S; S + 2 → Z
4) R + 1 → Y; Y + 2 → L
Solution 50 5) J + 1 → G; G + 2 → Y
Given series, VZJ does not belong to the group.
Left Side: A C E U H D S V Z P O J G R Y N L K B W Hence, ‘VZJ’ is the correct answer
T: Right Side

10). Coding Decoding Questions

Direction (1-5): Study the following information 2)What is code of “Banks are normal today”?
carefully and answer the questions given below: a. KkdBkdMkdTkd
In a certain code of language, b. KkdLkdMkdTkd
“Banks are digital today” is written as “Lkd, Mkd, Kkd, c. KkdBkdMkdLkd
Tkd” d. KkdMkdXkdVkd
“Amount sent through banks” is written as “Dkd, Kkd, e. None of these
Skd, Fkd” 3)The code “Dkd” represents which of the following
“Digital amount normal today” is written as “Skd, Lkd, word?
Tkd, Bkd” a. Sent
“Today we sent principal” is written as “Dkd, Xkd, Vkd, b. Banks
Lkd” c. Digital
1) What is the code for the word “Through”? d. Amount
a. Bkd e. None of these
b. Vkd 4) What is the code for the word “Principal”?
c. Fkd a. Vkd
d. Either Bkd or Vkd b. Bkd
e. None of these c. Xkd
Page 203 of 722

Subscribe the Xpress Video Course & Mock Test Package for Bank & Insurance Exams
If there are any suggestions/ errors in our PDFs Feel Free to contact us via this email: admin@exampundit.in
IBPS RRB Clerk Prelims – Ultra Practice Bundle PDF
d. Can’t be determined e. eight
e. Tkd 8) What is possible code for ‘create matter’?
5) Which of the following pair is correct? a. ten eight
a. Today-Mkd b. six five
b. Are-Mkd c. three one
c. Sent-Skd d. six nine
d. Digital-Kkd e. None of these
e. All are true 9) What is the code for ‘seem out lot’?
Direction (6-10): Study the following information a. three four five
carefully and answer the questions given below: b. ten eight three
In a certain code of language, c. six nine three
‘create your own destiny’ is written as ‘five six four two’ d. eight three one
‘destiny matter lot own’ is written as ‘nine three two four’ e. either (b) or (d)
‘does own work out’ is written as ‘ten four one seven’ 10) The code ‘one’ is coded for which of the following
‘work seem your matter’ is written as ‘eight seven five word?
nine’ a. does
6) What is the code for ‘destiny’ in the given code b. Seem
language? c. out
a. nine d. either (a) or (c)
b. six e. Your
c. seven Direction (11-15): Study the following information
d. two carefully and answer the questions given below:
e. None of these In a certain language,
7) What is the code for ‘lot’ in the given code language? ‘tree that green work’ is coded as ‘gui, nui, pui, cui’
a. two ‘work around the garden’ is coded as ‘xui, cui, yui, bui’
b. three ‘tree never lie around’ is coded as ‘kui, pui, mui, xui’
c. five ‘that the lie’ is coded as ‘gui, mui, yui’
d. six
Page 204 of 722

Subscribe the Xpress Video Course & Mock Test Package for Bank & Insurance Exams
If there are any suggestions/ errors in our PDFs Feel Free to contact us via this email: admin@exampundit.in
IBPS RRB Clerk Prelims – Ultra Practice Bundle PDF
11) How is the word ‘green’ coded in the given a. That
language? b. Around
a. nui c. Lie
b. mui d. Tree
c. pui e. None of these
d. yui
e. None of these Direction (16-20): Study the following information
12) Which of the following could be coded as ‘yui’? carefully and answer the questions given below:
a. lie In a certain language,
b. never ‘located in remote area’ is coded as ’not, fan, did, can’
c. the ‘area located near forest’ is coded as ’pan, can, lan, did’
d. garden ‘forest along remote villages’ is coded as ’lan, see, fan,
e. None of these how’
13) Which of the following words is coded as ‘kui’ in ‘villages near covered forest’ is coded as ’you, lan, see,
the given language? pan’
a. Work 16) Which of the following is the code for ‘forest’ in the
b. That given language?
c. Garden a. can
d. Never b. how
e. Either (a) or (c) c. you
14) How is the sentence ‘tree work’ coded in the given d. pan
language? e. None of these
a. xuimui 17) Which of the following words is coded as ‘fan’ in
b. pui cui the given language?
c. yuixui a. in
d. guinui b. along
e. Either (a) or (d) c. villages
15)Which of the following could be coded as ‘xui’? d. remote
Page 205 of 722

Subscribe the Xpress Video Course & Mock Test Package for Bank & Insurance Exams
If there are any suggestions/ errors in our PDFs Feel Free to contact us via this email: admin@exampundit.in
IBPS RRB Clerk Prelims – Ultra Practice Bundle PDF
e. None of these a. tui
18) Which of the following is coded as ‘did’? b. kui
a. near c. nui
b. forest d. cui
c. area e. None of these
d. located 22) Which of the following could be coded as ‘cui’?
e. either (c) or (d) a. over
19) What is the code of ‘near’ in the given language? b. every
a. pan c. each
b. did d. here
c. sees e. None of these
d. you 23) Which of the following words is coded as “mui” in
e. None of these the given language?
20) What may be coded as ‘not how’? a. also
a. forest in b. stone
b. area along c. and
c. along in d. over
d. villages in e. None of these
e. None of these 24) How is the sentence ‘here actions’ coded in the
Direction (21-25): Study the following information given language?
carefully and answer the questions given below: a. yui, cui
In a certain language, b. xui, yui
‘stone and pebble there’ is coded as ‘tui mui pui kui’ c. tui, xui
‘there every actions over’ is coded as ‘bui yui kui dui’ d. xui, kui
‘over stone here also’ is coded as ‘xui dui pui nui’ e. None of these
‘also and every each’ is coded as ‘mui bui cui nui’ 25) Which of the following could be coded as ‘kui’?
21) How is the word ‘there’ coded in the given a. and
language? b. there
Page 206 of 722

Subscribe the Xpress Video Course & Mock Test Package for Bank & Insurance Exams
If there are any suggestions/ errors in our PDFs Feel Free to contact us via this email: admin@exampundit.in
IBPS RRB Clerk Prelims – Ultra Practice Bundle PDF
c. pebble a. access
d. here b. your
e. None of these c. handling
Direction (26-30): Study the following information d. easy
carefully and answer the questions given below: e. None of these
In a certain language, 29) What is the code of ‘account’ in the given
‘database access for all’ is coded as ’snr, lmo, kpc, rtp’ language?
‘access your database account’ is coded as ’dkm, rtp, krt, a. psr
snr’ b. rtp
‘bank for account handling’ is coded as ’kpc, bzx, psr, c. kpc
dkm’ d. dkm
‘easy handling database account’ is coded as ’bzx, efn, e. None of these
dkm, rtp’ 30) What may be coded as “snr stp”?
26) Which of the following is the code for ‘handling’ in a. for account
the given language? b. account bank
a. psr c. access data
b. bzx d. your data
c. efn e. database for
d. rtp Direction (31-35): Study the following information
e. None of these carefully and answer the given questions.
27) Which of the following words is coded as ‘kpc’ in In a certain code language,
the given language? ‘shock supervise capital deal’ means ‘ha ma ga ta’
a. for ‘leader victim accuse shock’ means ‘fa ca ma ba’
b. access ‘increase capital supervise victim ’ means ‘ba ga ta ja’
c. bank ‘fresh shock corner deal’ means ‘ka ha ma va’
d. account 31) Which of the following is coded as ‘ca’?
e. None of these a. supervise
28) Which of the following is coded as ‘rtp’? b. leader
Page 207 of 722

Subscribe the Xpress Video Course & Mock Test Package for Bank & Insurance Exams
If there are any suggestions/ errors in our PDFs Feel Free to contact us via this email: admin@exampundit.in
IBPS RRB Clerk Prelims – Ultra Practice Bundle PDF
c. capital Direction (36-40): Study the following information
d. accuse carefully and answer the given questions.
e. cannot be determined In a certain language,
32) What is the code for ‘victim increase’? ‘shut apart from distance’ is coded as ‘three five one six’
a. gaka ‘along with from shall’ is coded as ‘four five two seven’
b. ta ja ‘shall apart gone forever’ is coded as ‘seven three nine
c. faga ten’
d. baja ‘forever with apart shut’ is coded as ‘two three one ten’
e. kaja 36) How is the ‘distance’ coded in the given language?
33) What is the code for ‘corner’ in that language? a. four
a. ja b. seven
b. va c. ten
c. ga d. six
d. ka e. None of these
e. either b or d 37) Which of the following is coded as ‘nine’?
34) Which of the following is coded as ‘faca’ in that a. gone
language? b. forever
a. fresh increase c. apart
b. leader accuse d. shut
c. supervise victim e. None of these
d. corner shock 38) Which of the following words is coded as “three”
e. fresh deal in the given language?
35) What is the code for ‘Capital’? a. shut
a. ba b. from
b. ja c. apart
c. ga d. shall
d. ta e. None of these
e. either c or d
Page 208 of 722

Subscribe the Xpress Video Course & Mock Test Package for Bank & Insurance Exams
If there are any suggestions/ errors in our PDFs Feel Free to contact us via this email: admin@exampundit.in
IBPS RRB Clerk Prelims – Ultra Practice Bundle PDF
39) How is the sentence ‘gone forever’ coded in the 42) What is the code for ‘circle’ in the given code of
given language? language?
a. three one a. ut
b. two seven b. bp
c. three six c. mi
d. nine ten d. lo
e. Cannot be determined e. ja
40) Which of the following could be coded as ‘with’? 43) The code ‘js’ stands for which of the following
a. two word?
b. three a. Either ‘navigation’ or ‘circle’
c. nine b. Either ‘gone’ or ‘place’
d. ten c. Either ‘gone’ or ‘navigation’
e. Cannot be determined d. Either ‘earth’ or ‘circle’
Direction (41-45): Study the following information e. None of these
carefully and answer the given questions. 44) What is the code for ‘earth’ in the given code
In a certain code of language, language?
‘place satellite on earth’ is written as ‘jo ki la si’ a. jo
‘earth moon on circle’ is written as ‘bp la mi jo’ b. mi
‘circle orbit through satellite’ is written as ‘si dv ut bp’ c. la
‘moon navigation gone through’ is written as ‘js mi dv d. Either jo or la
hm’ e. None of these
41) What is the possible code of ‘place moon school’ in 45) What is the code for ‘orbit’ in the given code of
the given code language? language?
a. ki jo mi a. si
b. mi la jo b. dv
c. ki mi si c. ut
d. ki mi xi d. bp
e. ki mi dv e. None of these
Page 209 of 722

Subscribe the Xpress Video Course & Mock Test Package for Bank & Insurance Exams
If there are any suggestions/ errors in our PDFs Feel Free to contact us via this email: admin@exampundit.in
IBPS RRB Clerk Prelims – Ultra Practice Bundle PDF
Direction (46-50): Study the following information d. offers
carefully and answer the given questions. e. None of these
In a code language, 48) Which among the following is correctly matched?
“are there secure promise” is coded as “20E 19G 18C a. will – 22F
21F” b. yard – 24D
“wear donate helped tolerate” is coded as “16F 23D 20H c. system – 23G
20F” d. marinate – 20H
“kinship sentence number for” is coded as “19G 18C 20H e. former – 19G
21F” 49) What is the code for “secure area”?
“fives blue compact pursue” is coded as “21D 21F 20G a. 17G 18F
22E”. b. 21F 18D
46) How is the word “sanction” coded? c. 17F 18G
a. 20H d. 17J 19D
b. 20G e. None on these
c. 20E 50) How is the word “compact” coded?
d. 18H a. 21D
e. None of these b. 21F
47) “21G” is the code for _____. c. 20G
a. around d. 22E
b. envy e. None of these
c. custurd

10). Coding Decoding Questions - Detailed Explanation with Answers:


Answers: 4. d
1. c 5. b
2. c Solutions 1-5
3. a
Page 210 of 722

Subscribe the Xpress Video Course & Mock Test Package for Bank & Insurance Exams
If there are any suggestions/ errors in our PDFs Feel Free to contact us via this email: admin@exampundit.in
IBPS RRB Clerk Prelims – Ultra Practice Bundle PDF
15. b
Solutions 11-15

Answers:
6. d
7. b
8. d
Answers:
9. e
16. e
10. d
17. d
Solutions 6-10
18. e
19. a
20. c
Solutions 16-20

Answers:
11. a
12. c
Answers:
13. d
21. b
14. b
22. c
Page 211 of 722

Subscribe the Xpress Video Course & Mock Test Package for Bank & Insurance Exams
If there are any suggestions/ errors in our PDFs Feel Free to contact us via this email: admin@exampundit.in
IBPS RRB Clerk Prelims – Ultra Practice Bundle PDF
23. c 31. e
24. b 32. d
25. b 33. e
Solutions 21-25 34. b
35. e
Solutions 31-35

Answers:
Answers:
36. d
26. b
37. a
27. a
38. c
28. e
39. d
29. d
40. a
30. c
Solutions 36-40
Solutions 26-30

Answers:
Answers: 41. d

Page 212 of 722

Subscribe the Xpress Video Course & Mock Test Package for Bank & Insurance Exams
If there are any suggestions/ errors in our PDFs Feel Free to contact us via this email: admin@exampundit.in
IBPS RRB Clerk Prelims – Ultra Practice Bundle PDF
42. b 50. c
43. c Solutions 46-50
44. d In each code, the number before the letter, is the highest
45. c number representation of any letter in the word as per
Solutions 41-45 alphabetical series.
The letter is the alphabet (in the alphabetical series) of
the number of letters in the word. Example: If there are
six letters in the word, 6th letter in the alphabetical
series is F.
Example:
‘there’
The letter with the highest value in alphabetical series is
Answers:
T, which represents 20.
46. a
There are five letters in the word, so 5th letter in the
47. c
alphabetical series is E.
48. d
Therefore, the code for ‘profile’ is ‘20E’.
49. b

11). Inequality Questions

Directions (1-50): In the following question, assuming II. F < T


the given statements to be true, find which of the given a) Only I is true
conclusions is/are definitely true and then choose the b) Only II is true
right option accordingly. c) Both I & II are true
1. Statements: d) Neither I nor II is true
R > S > T; R > F > B. e) Either I or II is true
Conclusions: 2. Statement:
I. B > S W>D≥C=X<A<Z≤F
Page 213 of 722

Subscribe the Xpress Video Course & Mock Test Package for Bank & Insurance Exams
If there are any suggestions/ errors in our PDFs Feel Free to contact us via this email: admin@exampundit.in
IBPS RRB Clerk Prelims – Ultra Practice Bundle PDF
Conclusion: 5. Statements:
I. Z > C J ≥ N ≤ T; T = S > R; K > V ≥ J
II. X < W Conclusions:
a) None is true I. V ≥ N
b) Only II is true II. K > J
c) Only I is true a) Only II is true
d) Both I and II are true b) Only I is true
e) Either I or II is true c) Either I or II is true
3. Statement: d) Both I and II are true
L ≥ M = N < O, P < Q ≥ R = S ≥ L e) None of these
Conclusion: 6. Statements:
I. Q > M Q ≤ R; S < T; P > Q; R > S
II. N = Q Conclusions:
a) None is true I. S = Q
b) Only II is true II. T ≥ P
c) Only I is true a) Only I is true
d) Both I and II are true b) Only II is true
e) Either I or II is true c) Either I or II true
4. Statements: d) Neither I nor II is true
D≥E>F=A≤U<L≥T=R e) Both I and II is true
Conclusions: 7. Statement:
I. F < L X < C, W > D, G ≥ W, C = D
II. D > A Conclusion:
a) None is True I. C ≤ G
b) Both I and II are True II. W > X
c) Only II is True a) None is true
d) Only I is True b) Only II is true
e) Either I and II is True c) Both I and II are true
Page 214 of 722

Subscribe the Xpress Video Course & Mock Test Package for Bank & Insurance Exams
If there are any suggestions/ errors in our PDFs Feel Free to contact us via this email: admin@exampundit.in
IBPS RRB Clerk Prelims – Ultra Practice Bundle PDF
d) Only I is true a) Only conclusion I is true.
e) Either I or II is true b) Only conclusion II is true.
8. Statements: c) Either conclusion I or conclusion II is true.
M > A ≤ T < D; R > G = A; O ≥ T = N; d) Neither conclusion I nor conclusion II is true.
Conclusions: e) Both conclusions I and II are true.
I. T < R 11. Statement:
II. N < R U < V < W; Z ≤ T = S < V; X > W ≤ A;
III. A = N Conclusions:
a) Only I and II are true I. S > Z
b) Only I is true II. T < W
c) Only III is true III. T < A
d) Only II is true a) Only I is true
e) None is true b) Only II is true
9. Statements: c) Only III is true
B≥C=D≥E;A≤F≤P=D d) Only II and III are true
Conclusions: e) None is true
I. D ≥ A. 12. Statements:
II. B ≥ F. B < Q = F ≤ N, F ≥ L ≥ S, B ≥ K
a) Only conclusion I is true. Conclusions:
b) Only conclusion II is true. I. N ≥ S
c) Either conclusion I or conclusion II is true. II. F > K
d) Neither conclusion I nor conclusion II is true III. B < F ≥ S
e) Both conclusions I and II are true. a) Only I is true
10. Statements: b) Only II is true
P = S < T ≤ U; Q ≤ U = A ≥ B c) Only I and II are true
Conclusions: d) All I, II and III are true
I. Q = B e) Only II and III are true
II. S ≤ A 13. Statement:
Page 215 of 722

Subscribe the Xpress Video Course & Mock Test Package for Bank & Insurance Exams
If there are any suggestions/ errors in our PDFs Feel Free to contact us via this email: admin@exampundit.in
IBPS RRB Clerk Prelims – Ultra Practice Bundle PDF
D=A<C>B>D e) Both follow
Conclusions: 16. Statements:
I. C > A Y≤U>T; W<Q>P; G>U<P
II. D < B Conclusions:
a) Only II follows I. Q>T
b) Both follow II. G>Y
c) Either I or II follows III. G>T
d) Neither I nor II follows a) Only conclusion II is true.
e) Only I follows b) Both conclusions II and III are true.
14. Statement: c) All the conclusions I, II and III are true.
Z<M≥Y≥N>L d) Only conclusion I is true
Conclusion: e) None is true.
I. Z < L 17. Statements:
II. M > L T > Y ≥ E < C; W > E = N >J; S ≤ Q < J
a) Both follow Conclusions:
b) Only I follows I. C > Q
c) Only II follows II. N ≤ Y
d) Either I or II follows III. W > S
e) Neither I nor II follows a) Only conclusion II is true
15. Statement: b) Only conclusion I is true
D>F≥G≥H≥I>J c) Both conclusions I and III is true
Conclusions: d) All of the conclusions are true
I. D > I e) Both conclusions I and II are true
II. I ≤ F 18. Statements:
a) Only I follows I > O < A; Q ≤ O > R; E > Q ≥ P >C
b) Only II follows Conclusions:
c) Either I or II follows I. I ≥ C
d) Neither I nor I follows II. A > P
Page 216 of 722

Subscribe the Xpress Video Course & Mock Test Package for Bank & Insurance Exams
If there are any suggestions/ errors in our PDFs Feel Free to contact us via this email: admin@exampundit.in
IBPS RRB Clerk Prelims – Ultra Practice Bundle PDF
III. A ≥ E Y ≥ E ≤ D < P; E > S ≥ Q > O > F ≥ B
a) Only conclusion I is true Conclusions:
b) Only conclusion II is true I. Y ≥ F
c) Both conclusions I and III is true II. P ≥ B
d) None of the conclusions is true III. D > F
e) Both conclusions I and II are true a) Only conclusion I is true
19. Statements: O ≥ D ≤ J< L; D > G ≥ B ≥ F > K > T b) Only conclusion III is true
Conclusions: c) Both conclusions I and III is true
I. L ≥ K d) None of the conclusions is true
II. O > T e) Both conclusions I and II are true
III. J ≥ B 22. Statements:
a) Only conclusion II is true P ≤ R < T; G ≥ Z = F; B < M = G ≥ R
b) Only conclusion I is true Conclusions:
c) Both conclusions I and III is true I. F ≤ M
d) None of the conclusions is true II. P ≤ G
e) Both conclusions I and II are true III. B < R
20. Statements: a) Only conclusion II is true.
T ≥ W > S > G; B > N = W ≤ L < U ≤ K b) Both conclusions II and III are true.
Conclusions: c) All the conclusions are true.
I. B > S d) Both conclusions I and II are true.
II. K = T e) None is true.
III. U > G 23. Statement:
a) Only conclusion I is true Z ≥ Q; U > V; Z ≥ O; Q ≥ U; I = Q
b) Only conclusion II is true Conclusions:
c) Both conclusions I and III is true I. U > O
d) None of the conclusions is true II. O > Q
e) Both conclusions I and II are true III. Q > V
21. Statements: a) Both conclusions II and III are true
Page 217 of 722

Subscribe the Xpress Video Course & Mock Test Package for Bank & Insurance Exams
If there are any suggestions/ errors in our PDFs Feel Free to contact us via this email: admin@exampundit.in
IBPS RRB Clerk Prelims – Ultra Practice Bundle PDF
b) Only conclusion I is true Conclusion:
c) Only conclusion III is true I. Y>T
d) Only conclusion II is true II. L>T
e) Both the conclusions I and III are true III. R>Q
24. Statement: a) Only conclusion II is true.
E > K ≤ T; D > X; I ≥ T ≤ F > X b) Both conclusions II and III are true.
Conclusions: c) Only conclusion III follows.
I. I > X d) Only conclusion I is true
II. I ≥ F e) None is true.
III. D > T 27. Statement:
a) Both conclusions II and III are true R≥T>Y=I; O<L; L≤T
b) Only conclusion I is true Conclusions:
c) Only conclusion III is true I. R>O
d) Only conclusion II is true II. L>I
e) None of the conclusions follows III. O>R
25. Statements: a) Only conclusion II is true.
E > T ≤ R; C < R > I; G > E b) Both conclusions II and III are true.
Conclusions: c) Only conclusion III follows.
I. G > C d) Only conclusion I is true
II. T ≤ I e) None is true.
III. G > T 28. Statements:
a) Only conclusion II is true E>Q≤V; T>U<Q; E=C
b) Both conclusions II and III are true Conclusions:
c) Only conclusion III is true I. C>U
d) Only conclusion I is true II. V>U
e) None is true III. T>E
26. Statements: a) Only conclusion II is true.
Q>T; Y≥R<L; Q=J; J<L b) Both conclusions I and II are true.
Page 218 of 722

Subscribe the Xpress Video Course & Mock Test Package for Bank & Insurance Exams
If there are any suggestions/ errors in our PDFs Feel Free to contact us via this email: admin@exampundit.in
IBPS RRB Clerk Prelims – Ultra Practice Bundle PDF
c) Only conclusion III follows. 1. I ≥ O
d) Only conclusion I is true 2. U ≤ I
e) None is true. a) Only 1 follows
29. Statements: b) Only 2 follows
P>Q>R≤T; O≥P; Y<R=X c) Both 1 and 2 follow
Conclusions: d) Neither 1 nor 2 follow
I. O≥X e) Either 1 or 2 follows
II. P>Y 32. Statements:
III. T>P R ≥ K ≥ F, D > F, R ≥ C ≤ S
a) Only conclusion II is true. Conclusions:
b) Both conclusions I and II are true. 1. D > K
c) Only conclusion III follows. 2. R ≥ F
d) Only conclusion I is true a) Only 1 follows
e) None is true. b) Only 2 follows
30. Statements: c) Both 1 and 2 follow
W>B<M; D>V=B; 9>8=W d) Neither 1 nor 2 follow
Conclusions: e) Either 1 or 2 follows
I. 9>B 33. Statements:
II. W>M B < H, X = C ≥ P, X > N < H
III. D>9 Conclusions:
a) Only conclusion II is true. 1. N > B
b) Both conclusions II and III are true. 2. C ≥ H
c) Only conclusion III follows. a) Only 1 follows
d) Only conclusion I is true b) Only 2 follows
e) None is true. c) Both 1 and 2 follow
31. Statements: d) Neither 1 nor 2 follow
I ≥ S = U, O = U, I = M = P e) Either 1 or 2 follows
Conclusions:
Page 219 of 722

Subscribe the Xpress Video Course & Mock Test Package for Bank & Insurance Exams
If there are any suggestions/ errors in our PDFs Feel Free to contact us via this email: admin@exampundit.in
IBPS RRB Clerk Prelims – Ultra Practice Bundle PDF
34. Which of the following symbols should replace the 37. If O≥U>R≤S and T≥U>M<L are true, which of the
question marks respectively so that 'F < P' is definitely following options is neither definitely true nor
true? definitely false?
P?Q=R?S=F<T>U a) O>M
a) < and > b) R<T
b) > and = c) S>L
c) > and > d) T<R
d) > and < e) None of these
e) < and < 38. Statements:
35. Which of the following symbols should replace the W ≤ K, H > T ≥ L, K = C ≥ X, H < Y = X, A > L
question marks respectively so that G > C is definitely Conclusions:
true? 1. A > X
G>F?B=R?A=C<Q>D 2. C > L
a) < and > a) Only 1 follows
b) > and < b) Only 2 follows
c) > and > c) Both 1 and 2 follow
d) > and < d) Neither 1 nor 2 follow
e) < and < e) Either 1 or 2 follows
36. Which of the following order of letters (from left to 39. Statements:
right) in the blanks makes the expression, L>G J ≤ Q, O > P = C, R ≥ X < Q, G = O, R < D ≥ C
definitely true? Conclusions:
____ > ____ = ____ ≤ ____=____ 1. J > C
a) L,A,O,S,G 2. C ≥ Q
b) L,S,G,O,A a) Only 1 follows
c) G,L,O,S,A b) Only 2 follows
d) G,O,A,L,S c) Both 1 and 2 follow
e) L,S,A,G,O d) Neither 1 nor 2 follow
e) Either 1 or 2 follows
Page 220 of 722

Subscribe the Xpress Video Course & Mock Test Package for Bank & Insurance Exams
If there are any suggestions/ errors in our PDFs Feel Free to contact us via this email: admin@exampundit.in
IBPS RRB Clerk Prelims – Ultra Practice Bundle PDF
40. Statements: a) U<T
T = X, S = O < B, S ≥ P = A, X < U < A, R > B b) C≥N
Conclusions: c) X<P
1. A < R d) N<K
2. P < B e) All the options are false
a) Only 1 follows 44. If M>C≥D; S≤C<A≤N and K≥B>A are true, which
b) Only 2 follows of the following options is definitely true?
c) Both 1 and 2 follow a) K>N
d) Neither 1 nor 2 follow b) M<S
e) Either 1 or 2 follows c) B>S
41. If E < G and O < G are definitely true then which d) M=N
of the following symbols should be placed in the blank e) N≥D
spaces respectively? 45. Which of the following symbols should replace the
D>E_F=G>B=N_O question mark in the given expression in order to make
a) <, > the expressions ‘K ≤ H’ and ‘M > J’ definitely true ?
b) <, > H≥I=J?K≤L<M
c) <, = a) >
d) >, > b) ≥
e) None of these c) ≤
42. In which of the given expression does the d) Either < or ≤
expression N> Q and L> P definitely hold True? e) =
a) L>M ≥ N< O = P ≤ Q< R Directions (46 – 50): Relationship between different
b) L> M ≥ N> O = P ≤ Q< R elements is shown in the statements. Find if the
c) L> M ≥ N> O = P ≥ Q< R conclusions also follow or not.
d) L> M ≥ N ≤ O = P ≤ Q< R 46. Statements:
e) L ≥ M ≥ N ≥ O = P ≥ Q< R W > S ≤ Q, D ≤ S, I ≤ B = D
43. If M≤A=T<C; U>P≥T<K and X>A>N are true, Conclusions:
which of the following options is definitely true? I. Q ≥ I
Page 221 of 722

Subscribe the Xpress Video Course & Mock Test Package for Bank & Insurance Exams
If there are any suggestions/ errors in our PDFs Feel Free to contact us via this email: admin@exampundit.in
IBPS RRB Clerk Prelims – Ultra Practice Bundle PDF
II. Q = I c) Either I or II follows
a) Only I follows d) Neither I nor II follow
b) Only II follows e) Both I and II follow
c) Either I or II follows 49. Statements:
d) Neither I nor II follow E ≥ L > K, L > F < D, P > E
e) Both I and II follow Conclusions:
47. Statements: I. P > D
Q < D ≤ A, C > A < K, D ≥ S II. E > F
Conclusions: a) Only I follows
I. K > D b) Only II follows
II. C > S c) Either I or II follows
a) Only I follows d) Neither I nor II follow
b) Only II follows e) Both I and II follow
c) Either I or II follows 50. Statements:
d) Neither I nor II follow Q ≥ X ≥ S, S < A, A > E
e) Both I and II follow Conclusions:
48. Statements: I. E > X
S > N ≥ F, P = N, E > S II. X ≥ E
Conclusions: a) Only I follows
I. E > P b) Only II follows
II. F < E c) Either I or II follows
a) Only I follows d) Neither I nor II follow
c) Only II follows e) Both I and II follow

Page 222 of 722

Subscribe the Xpress Video Course & Mock Test Package for Bank & Insurance Exams
If there are any suggestions/ errors in our PDFs Feel Free to contact us via this email: admin@exampundit.in
IBPS RRB Clerk Prelims – Ultra Practice Bundle PDF

11). Inequality Questions - Solution and detailed Explanation


1. Answer: D) Conclusions:
Given statements: R > S > T;R > F > B I. F < L → True (as D ≥ E > F = A ≤ U < L ≥ T = R)
On combining: B < F < R > S > T II. D > A → True (as D ≥ E > F = A ≤ U < L ≥ T = R)
Conclusions: Therefore, both conclusions follow.
I. B > S → False (as B < F < R > S > T → B < R > S → 5. Answer: D)
thus clear relation between B and S can’t be determined) Statements: J ≥ N ≤ T; T = S > R; K > V ≥ J
II. F < T → False (as B < F < R > S > T → F < R > T → On combining: K > V ≥ J ≥ N ≤ T = S > R
thus clear relation between F and T can’t be determined) Conclusions:
Hence, neither conclusion I nor II is true. I. V ≥ N → True (As V ≥ J ≥ N)
2. Answer: D) II. K > J → True (As K > V ≥ J)
Given Statements: W > D ≥ C = X < A < Z ≤ F Thus, Both I and II are true.
Conclusions: I. Z > C → True (as C = X < A < Z → C < 6. Answer: D)
Z) Given statements: Q ≤ R; S < T; P > Q; R > S
II. X < W → True (as W > D ≥ C = X → W > On combining: P > Q ≤ R > S < T
X) Conclusions:
Hence, both I and II are true. I. S = Q → False (as Q ≤ R > S, so a definite relation
3. Answer: E) between S and Q cannot be determined)
Given Statements: L ≥ M = N < O, P < Q ≥ R = S ≥ L II. T ≥ P → False (P > Q ≤ R > S < T, so a definite relation
On combining: P < Q ≥ R = S ≥ L≥ M = N < O between T and P cannot be determined)
Conclusions: I. Q > M → False (as Q ≥ R = S ≥ L ≥ M → Hence, neither I nor II is true.
Q ≥ M) 7. Answer: B)
II. N = Q → False (as Q ≥ R = S ≥ L ≥ M = N → Q ≥ N) Given Statements: X < C, W > D, G ≥ W, C = D
In statement ‘N = M’ On combining: X < C = D < W ≤ G
Hence, either I or II is true. Conclusions:
4. Answer: B) I. C ≤ G → False (as C = D < W ≤ G → C < G)
Given statements: D ≥ E > F = A ≤ U < L ≥ T = R II. W > X → True (as X < C = D < W → X < W)

Page 223 of 722

Subscribe the Xpress Video Course & Mock Test Package for Bank & Insurance Exams
If there are any suggestions/ errors in our PDFs Feel Free to contact us via this email: admin@exampundit.in
IBPS RRB Clerk Prelims – Ultra Practice Bundle PDF
Hence, only II is true. 11. Answer: D)
8. Answer: E) Statement: U < V < W; Z ≤ T = S < V; X > W ≤ A;
Statements: M > A ≤ T < D; R > G = A; O ≥ T = N; On combining: Z ≤ T = S < V < W < X; W ≤ A; U < V
On combining: M > A ≤ T = N; R > G = A ≤ T < D; O ≥ I. S > Z → False (Z ≤ T = S → Z ≤ S)
T<D II. T < W → True (T = S < V < W → T < W)
Conclusions: III. T < A → True (T = S < V < W ≤ A → T < A)
I. T < R → False (as R > G = A ≤ T → Relation between Hence, Only II and III are true.
T and R can’t be determined) 12. Answer: D)
II. N < R → False (as R > G = A ≤ T = N → Relation Given statements: B < Q = F ≤ N, F ≥ L ≥ S, B ≥ K
between N and R can’t be determined) On combining: K ≤ B < Q < F ≥ L ≥ S; S ≤ L ≤ F ≤ N
III. A = N → False (as A ≤ T = N) Conclusions:
Hence, none of the statements is true. I. N ≥ S ⇒ True (as S ≤ L ≤ F ≤ N → S ≤ N)
9. Answer: E) II. F > K ⇒ True (as K ≤ B < Q < F ≥ L ≥ S → K < F)
Given statements are: B ≥ C = D ≥ E; A ≤ F ≤ P = D. III. B < F ≥ S ⇒ True (as B < Q < F and S ≤ L ≤ F, so we
On combining: B ≥ C = D ≥ E; B ≥ C = D = P ≥ F ≥ A. get, B < Q = F ≥ L ≥ S → B < F ≥ S)
Conclusions: Hence all the conclusions are true.
I. D ≥ A → Clearly follows hence true. 13. Answer: B)
II. B ≥ F → Clearly follows hence true. Given statement: D = A < C > B > D
Hence both I and II true. Conclusions:
10. Answer: D) I. C > A → True (as C > B > D = A → thus C > A)
Given statements are: P = S < T ≤ U; Q ≤ U = A ≥ B. II. D < B → True (as B > D)
On combining: P = S < T ≤ U = A ≥ B; P = S < T ≤ U ≥ Hence, both conclusions follow.
Q. 14. Answer: C)
Conclusions: Given Statements: Z < M ≥ Y ≥ N > L
I. Q = B → The relation between Q and B is not clear. Conclusions:
Hence,the conclusion is false. I. Z < L → False (as Z < M ≥ Y ≥ N > L → relation
II. S ≤ A → Clearly, S < A and doesn’t follow S ≤ A. between Z and L cannot be determined)
Hence, the conclusion is false. II. M > L → True (as M ≥ Y ≥ N > L → M > L)
Page 224 of 722

Subscribe the Xpress Video Course & Mock Test Package for Bank & Insurance Exams
If there are any suggestions/ errors in our PDFs Feel Free to contact us via this email: admin@exampundit.in
IBPS RRB Clerk Prelims – Ultra Practice Bundle PDF
Hence, only II follows. On combining, we get,
15. Answer: E) I > O > R; A > O > R; I > O ≥ Q ≥ P > C; A > O ≥ Q ≥ P
Given Statements: D > F ≥ G ≥ H ≥ I > J > C; A > O ≥ Q < E
Conclusions: Conclusions:
I. D > I → True (as D > F ≥ G ≥ H ≥ I → D > I) I. I ≥ C: False (As, I > O ≥ Q ≥ P > C, so, I > C)
II. I ≤ F → True (as F ≥ G ≥ H ≥ I → F ≥ I) II. A > P: True (As A > O ≥ Q ≥ P, so, A > P)
Hence, both I and II follow. III. A ≥ E: False (As, A > O ≥ Q < E, so, the relation
16. Answer: C) between A and E cannot be determined)
Given statement 19. Answer: A)
Y≤U>T; W<Q>P; G>U<P Given statements: O ≥ D ≤ J < L; D > G ≥ B ≥ F > K > T
On combining, On combining, we get,
G>U<P<Q>W; G>U≥Y; G>U>T O ≥ D> G ≥ B ≥ F > K > T; L > J ≥ D > G ≥ B ≥ F > K >
Conclusions: T
I. Q>T: True (As Q>P>U>T so, Q>T) Conclusions:
II. G>Y: True (As G>U≥Y so, G>Y) I. L ≥ K: False (As, L > J ≥ D > G ≥ B ≥ F > K, so, L > K)
III. G>T: True (As G>U>T so, G>T) II. O > T: True (As O ≥ D > G ≥ B ≥ F > K > T, so, O >
17. Answer: D) T)
Given statements: T > Y ≥ E < C; W > E = N >J; S ≤ Q < III. J ≥ B: False (As, J ≥ D > G ≥ B, so, J > B)
J 20. Answer: C)
On combining, we get, Given statements: T ≥ W > S > G; B > N = W ≤ L< U ≤
T > Y ≥ E = N >J > Q ≥ S; C > E= N >J > Q ≥ S; W > E K
=N>J>Q≥S On combining, we get,
Conclusions: B > N = W > S > G; K ≥ U > L ≥ W > S > G; K ≥ U > L
I. C > Q: True (As, C > E= N > J > Q, so, C > Q) ≥W≤T
II. N ≤ Y: True (As Y ≥ E = N, so, Y ≥ N) Conclusions:
III. W > S: True (As, W > E = N > J > Q ≥ S, so, W > S) I. B > S: True (As, B > N = W >S, so, B > S)
18. Answer: B) II. K = T: False (As K ≥ U > L ≥ W ≤ T, so, the relation
Given statements: I > O < A; Q ≤ O > R; E > Q ≥ P > C between K and T is not determined)
Page 225 of 722

Subscribe the Xpress Video Course & Mock Test Package for Bank & Insurance Exams
If there are any suggestions/ errors in our PDFs Feel Free to contact us via this email: admin@exampundit.in
IBPS RRB Clerk Prelims – Ultra Practice Bundle PDF
III. U > G: True (As, U > L ≥ W > S > G, so, U > G) II. O > Q: False (O ≤ Z ≥ Q ≥ U so, relationship between
21. Answer: B) O and Q can’t be established)
Given statements: Y ≥ E ≤ D < P; E > S ≥ Q > O > F ≥ B III. Q > V: True (Z ≥ Q ≥ U > V, so Q > V)
On combining, we get, 24. Answer: E)
Y ≥ E > S ≥ Q > O > F ≥ B; P > D ≥ E > S ≥ Q > O > F ≥ Given statement
B E > K ≤ T; D > X; I ≥ T ≤ F > X
Conclusions: On combining we get,
I. Y ≥ F: False (As, Y ≥ E > S ≥ Q > O > F, so, Y > F) E > K ≤ T ≤ F > X < D and I ≥ T ≤ F > X
II. P ≥ B: False (As, P > D ≥ E > S ≥ Q > O > F ≥ B, so, P Conclusions:
> B) I. I > X : False (I ≥ T ≤ F > X; so relationship between I
III. D > F: True (As, D ≥ E > S ≥ Q > O > F, so, D > F) and X can’t be established)
22. Answer: D) II. I ≥ F: False (I ≥ T ≤ F > X, relationship between I and
Given statements: P ≤ R <T; G ≥ Z = F; B < M = G ≥ R F can’t be established)
On combining, we get III. D > T: False (E > K ≤ T ≤ F > X < D, relationship
B < M = G ≥ Z = F; P ≤ R ≤ G = M > B between D and T can’t be established)
Conclusions: 25. Answer: C)
I. F ≤ M: True (As M = G ≥ Z = F, so F ≤ M) Given:
II. P ≤ G: True (As P ≤ R ≤ G, so P ≤ G) E > T ≤ R; C < R > I; G > E
III. B < R: False (As R ≤ G = M> B, relation between B On combining:
and R cannot be determined) G > E > T ≤ R > I and G > E > T ≤ R > C
23. Answer: C) Conclusions:
Given statements: I. G > C: False (As G > E > T ≤ R > C so, the relationship
Z ≥ Q; U > V; Z ≥ O; Q ≥ U; I = Q between G and C cannot be established)
On combining, we get, II. T ≤ I: False (As G > E > T ≤ R > C so, the relationship
Z ≥ Q ≥ U > V and O ≤ Z ≥ Q = I between T and I cannot be established)
Conclusions: III. G > T: True (As G > E > T, so, G > T)
I. U > O: False (O ≤ Z ≥ Q ≥ U so, relationship between 26. Answer: A)
U and O can’t be established) Given statement:
Page 226 of 722

Subscribe the Xpress Video Course & Mock Test Package for Bank & Insurance Exams
If there are any suggestions/ errors in our PDFs Feel Free to contact us via this email: admin@exampundit.in
IBPS RRB Clerk Prelims – Ultra Practice Bundle PDF
Q>T; Y≥R<L; Q=J; J<L III. T>E: False (As T>U<Q<E=C so, the relationship
On combining, between T and E cannot be established)
Y≥R<L>J=Q>T 29. Answer: A)
Conclusions: Given statements:
I. Y>T: False (As Y≥R<L>J=Q>T so, the relationship P>Q>R≤T; O≥P; Y<R=X
between Y and T cannot be established) On combining,
II. L>T: True (As Y≥R<L>J=Q>T so, L>T) O≥P>Q>R>Y and T≥R<Q<P
III. R>Q: False (As Y≥R<L>J=Q>T so, the relationship Conclusions:
between R and Q cannot be established) I. O≥X: False (As O≥P>Q>R=X so, the relationship
27. Answer: D) between O and X cannot be established)
Given statement: II. P>Y: True (As O≥P>Q>R>Y so, P>Y)
R≥T>Y=I; O<L; L≤T III. T>P: False (As T≥R<Q<P so, the relationship between
On combining, T and P cannot be established)
R≥T≥l=L>O and I=Y<T≥L>O 30. Answer: D)
Conclusions: Given statements:
I. R>O: True (As R≥T≥l=L>O so, R>O) W>B<M; D>V=B; 9>8=W
II. L>I: False (As I=Y<T≥L>O so, the relationship On combining,
between L and I cannot be established) 9>8=W>B<M and D>V=B<M
III. O>R: False (As R≥T≥l=L>O so, the relationship Conclusions:
between O and R cannot be established) I. 9>B: True (9>8=W>B<M so, 9>B)
28. Answer: B) II. W>M: False (9>8=W>B<M so, the relationship
Given statements, between W and M cannot be established)
E>Q≤V; T>U<Q; E=C III. D>9: False (9>8=W>B=V<D so, the relationship
On combining, between D and 9 cannot be established)
C=E>Q≤V and T>U<Q<E=C 31. Answer: C)
Conclusions: From the given statements we have: O = U = S ≤ I = M =
I. C>U: True (As T>U<Q<E=C so, C>U) P
II. V>U: True (As U<Q≤V so, V>U) O=U=S≤I
Page 227 of 722

Subscribe the Xpress Video Course & Mock Test Package for Bank & Insurance Exams
If there are any suggestions/ errors in our PDFs Feel Free to contact us via this email: admin@exampundit.in
IBPS RRB Clerk Prelims – Ultra Practice Bundle PDF
From this we have, I ≥ O to be True. Using option 4: - P > Q = R < S = F < T > U
U=S≤I We cannot compare P and F. So, option 4 is incorrect.
From this we have, U ≤ I to be True. Using option 5: - P < Q = R < S = F < T > U
32. Answer: B) Here, P < F. So, option 5 is incorrect.
From the given statements we have: D > F ≤ K ≤ R ≥ C ≤ 35. Answer: C)
S Given; G > F ? B = R ? A = C < Q > D
D>F≤K Using option 1: - G > F < B = R > A = C < Q > D
Since we have both "≤" and ">" in the equation, we can’t We cannot compare C and G. So, option 1 is incorrect.
establish the relation between D and K. So, D > K is False. Using option 2: - G > F > B = R < A = C < Q > D
F≤K≤R We cannot compare C and G. So, option 2 is incorrect.
From this we have, R ≥ F to be True. Using option 3: - G > F > B = R > A = C < Q > D
33. Answer: D) Clearly, G > C. So, option 3 is the correct answer.
From the given statements we have: B < H > N < X = C ≥ Using option 4: - G > F > B = R < A = C < Q > D
P We cannot compare C and G. So, option 4 is incorrect.
B<H>N Using option 5: - G > F < B = R < A = C < Q > D
Since we have both "<" and ">" in the equation, we cant We cannot compare C and G. So, option 5 is incorrect.
establish the relation between B and N. So, N > B is False. 36. Answer: B)
H>N<X=C For L>G to be definitely true, L has to be in first position,
Since we have both "<" and ">" in the equation, we cant G has to be in either 2nd or 3rd position.
establish the relation between H and C. So, C ≥ H is False. 37. Answer: C)
34. Answer: C) Option 1: By combining the two statements, we get
Given; P ? Q = R ? S = F < T > U O≥U>M. From this, we can conclude that O>M which is
Using option 1: - P < Q = R > S = F < T > U. definitely true.
We cannot compare P and F. So, option 1 is incorrect. Option 2: By combining the two statements, we get
Using option 2: - P > Q = R = S = F < T > U T≥U>R. From this, we can conclude that T>R which is
Here, P > F. So, option 2 is incorrect. definitely true.
Using option 3: - P > Q = R > S = F < T > U
Clearly, P > F. So, option 3 is the correct answer.
Page 228 of 722

Subscribe the Xpress Video Course & Mock Test Package for Bank & Insurance Exams
If there are any suggestions/ errors in our PDFs Feel Free to contact us via this email: admin@exampundit.in
IBPS RRB Clerk Prelims – Ultra Practice Bundle PDF
Option 3: By combining the two statements, we get B>O=S≥P
S≥R<U>M<L. We can't conclude the relation between S From this we have, P < B to be True.
and L. Thus, S>L is possible 41. Answer: B)
Option 4: By combining the relevant statements, we get From option 1) D > E < F = G > B = N > O
T≥U>R. From this, we can conclude that T>R. Thus, R>T G and E's relation not clear
is definitely false. From option 2) D > E < F = G > B = N > O,
38. Answer: B) Clearly G > E and G >O are definitely true. Hence follows.
From the given statements we have: A > L ≤ T < H < Y = 42. Answer: C)
X≤C=K≥W From option 1) L>M ≥ N< O = P ≤ Q< R
A>L≤T<H<Y=X This lead us to N>Q, Hence does not follow.
Since we have both "≤" and ">" in the equation, we cant From option 2) L> M ≥ N> O = P ≤ Q< R
establish the relation between A and X. So, A > X is False. This lead us to N>Q, Hence does not follow.
L≤T<H<Y=X≤C From option 3) L> M ≥ N> O = P ≥ Q< R
From this we have, C > L to be True. Clearly N> Q and L> P is true. So this is follow.
39. Answer: D) 43. Answer: D)
From the given statements we have: J ≤ Q > X ≤ R < D ≥ Option A: From the statement U>P≥T<K, we can
C=P<O=G conclude that U>T.
J≤Q>X≤R<D≥C Option B: By combining the relevant statements, we get
Since we have both "≤" and "≥" in the equation, we cant N<A=T<C. From this, we can conclude that C>N. We
establish the relation between J and C. So, J > C is False. can't justify the equality sign.
Q>X≤R<D≥C Option C: By combining the relevant statements, we get
Since we have both "≤" and "≥" in the equation, we cant X>A=T≤P. From this, we can't conclude whether X<P or
establish the relation between Q and C. So, C ≥ Q is False. not.
40. Answer: C) Option D: By combining the relevant statements, we get
From the given statements we have: R > B > O = S ≥ P = N<A=T<K. From this, we can conclude that N<K. hence,
A>U>X=T true.
R>B>O=S≥P=A 44. Answer: C)
From this we have, A < R to be True.
Page 229 of 722

Subscribe the Xpress Video Course & Mock Test Package for Bank & Insurance Exams
If there are any suggestions/ errors in our PDFs Feel Free to contact us via this email: admin@exampundit.in
IBPS RRB Clerk Prelims – Ultra Practice Bundle PDF
Option A: By combining the relevant statements, we get From Opition 5: H ≥ I = J = K ≤ L < M. Here K ≤ H and
K≥B>A≤N. From this, we can't conclude the relation M > J both follows. So option 5 is the answer.
between K and N. 46. Answer: A)
Option B: By combining the relevant statements, we get Q ≥ S ≥ D ≥ I.
M>C≥S. From this, we can conclude that M>S. So Q ≥ I
Option C: By combining the relevant statements, we get 47. Answer: E)
S≤C<A<B. From this, we can conclude that B>S. K > A ≥ D , so K > D
Option D: By combining the relevant statements, we get C > A ≥ D ≥ S so C > S
M>C<A≤N. From this, we can't conclude if M is equal to 48. Answer: E)
N or not. E > S > N = P, so E > P
Option E: By combining the relevant statements, we get F ≤ N < S < E, so F < E
D≤C<A≤N. From this, we can conclude that N>D. The 49. Answer: B)
equality is not possible. P > E ≥ L > F < D, so relation can’t be determined between
45. Answer: E) P and D E ≥ L > F, so E > F
From Opition 1: H ≥ I = J > K ≤ L < M. From this K < H 50. Answer: C)
From Opition 2: H ≥ I = J ≥ K ≤ L < M. From this K ≤ H, E < A > S ≤ X, so relation can’t be determined between E
but No relation between M and J and X, but is for sure that either E is greater than X or is
From Opition 3: H ≥ I = J ≤ K ≤ L < M. No relation is true less than equal to X, so either or between the two
From Opition 4: H ≥ I = J ≤/< K ≤ L < M. No relation is
true for H and K

12). Direction Questions

Directions (1-3): Answer the questions based on the and turns left from point R. He then walks for 8m and then
information given below. turns right from point S. He then walks for 5m till point T
Jatin started walking from point P in east direction. After and then turns right and walks for 8m till point U. He then
walking 8m he turns left from point Q. He then walks 3m turns left and walks for 3m and then turns right from point

Page 230 of 722

Subscribe the Xpress Video Course & Mock Test Package for Bank & Insurance Exams
If there are any suggestions/ errors in our PDFs Feel Free to contact us via this email: admin@exampundit.in
IBPS RRB Clerk Prelims – Ultra Practice Bundle PDF
V and walks for 5m till point W. He then turns right from turns right and walks for 4m till point R. He then turns
point W and walks for 8m till point X. right and walks for 15m till point S. He then turns right
1) In which direction is point X with respect to point and walks for 7m till point T.
Q? 4) In which direction is point O with respect to point
a. North-West T?
b. South a. South
c. West b. East
d. North-East c. North-East
e. South-East d. West
2) What is the shortest distance between point S and e. None of these
point X? 5) What is the total distance that Nick covered?
a. 12m a. 45m
b. 13m b. 47m
c. 17m c. 50m
d. 20m d. 52m
e. None of these e. None of these
3) What is the total distance between V and Q? 6) In which direction R with respect with M?
a. 12m a. South
b. 13m b. East
c. 11m c. North-East
d. 15m d. West
e. None of these e. None of these
Direction (4-6): Answer the questions based on the Directions (7-8): Answer the questions based on the
information given below. information given below:
Nick started walking in east direction from point M. After Point A is 4m north of point C. Point F is 10m east of point
walking 4m till point N, he turns right and walks for 6m G, which is 5m south of point H. Point B is north of point
till point O and then turns left and walks for 3m till point D, which is west of point E. Point F is 7m north of point
P. He then turns left and walks for 8m till point Q. He then
Page 231 of 722

Subscribe the Xpress Video Course & Mock Test Package for Bank & Insurance Exams
If there are any suggestions/ errors in our PDFs Feel Free to contact us via this email: admin@exampundit.in
IBPS RRB Clerk Prelims – Ultra Practice Bundle PDF
E. Point C is the middle point of point B and point H. Point c. 10m
B is 6m to the west of H. d. 8m
7) What is the shortest distance between point B and e. None of these
point E? 10) In which direction is point N with respect to point
a. 22m T?
b. 20m a. West
c. 25m b. North
d. 28m c. East
e. None of these d. North-East
8) What is the direction of point G with respect to point e. None of these
D? 11) What is the total distance between T and O?
a. North east a. 11m
b. North west b. 12m
c. South west c. 10m
d. North d. 8m
e. None of these e. None of these
Directions (9-11): Answer the questions based on the Directions (12-14): Answer the questions based on the
information given below. information given below.
Rahul starts walking from point M in south direction. A is sitting 3m north to B. C is sitting 4m west to H, who
After walking 6m he turns left from point N and walks for is sitting 8m south to E. F is sitting 16m north to C. G is
4m till point O. He then turns left and walks for 9m till sitting 8m north to D, who is sitting 4m east to B. I is
point P. He then turns left and walks for 8m till point Q. sitting 6m west to E. F is sitting 4m east to G.
He then turns left and walks for 3m till point R. He then 12) What is the shortest distance between I and H?
turns right and walks for 4m till point S. He then turns left a. 10m
and walks for 6m till point T. b. 15m
9) What is the shortest distance between M and T? c. 12m
a. 11m d. 16m
b. 12m e. None of these
Page 232 of 722

Subscribe the Xpress Video Course & Mock Test Package for Bank & Insurance Exams
If there are any suggestions/ errors in our PDFs Feel Free to contact us via this email: admin@exampundit.in
IBPS RRB Clerk Prelims – Ultra Practice Bundle PDF
13) What is the direction of G with respect to E? 16) What is the shortest distance between point B and
a. North point G?
b. South east a. 10m
c. North West b. 12m
d. South c. 13m
e. None of these d. 15m
14) What is the direction of F with respect to B? e. None of these
a. North east 17) What is the direction of C with respect to F?
b. North West a. North
c. South East b. South east
d. South c. North West
e. None of these d. South West
Directions (15-17): Answer the questions based on the e. None of these
information given below. Directions (18-20): Answer the questions based on the
Anil starts walking in east direction from point A. After information given below.
walking 5m he turns right from point B and walks for 6m. Amit starts walking from point R towards west for 5km to
He then turns left from point C and walks for 4m. He then reach point T. From point T he took right turn and walks
turns left from point D and walks for 8m till point E. He for 8km to reach point Z. From there he again took a right
then turns right and walks for 4m till point F and then turns turn and walks 7km to reach point Y. From point Y, Amit
right. He then walks for 8m till point G and stopped. took a right turn and walks for 4km to reach point K. After
15) What is the shortest distance between point D and that he took a left turn and walks for 5km to reach point J.
point G? 18) Point Y is in which direction with respect to point
a. 5m R?
b. 4m a. Southeast
c. 6m b. Northeast
d. 8m c. Southwest
e. None of these d. Northwest
e. None of these
Page 233 of 722

Subscribe the Xpress Video Course & Mock Test Package for Bank & Insurance Exams
If there are any suggestions/ errors in our PDFs Feel Free to contact us via this email: admin@exampundit.in
IBPS RRB Clerk Prelims – Ultra Practice Bundle PDF
19) If point G is 4km east of point Z, then how far and e. None of these
in which direction is point K with respect to point G? 22) What is the shortest distance between point A and
a. Northeast, 5km point B?
b. Southeast, 2km a. 15m
c. Northwest, 4km b. 5m
d. Southeast, 5km c. 10m
e. East, 6km d. 9m
20) If another point L, which is 4km south of point K, e. None of these
then which of the following statements regarding point 23) What is the total distance covered by Abhishek?
L is true? a. 52m
a. Point L is 3km away from point R b. 54m
b. Point L is in northeast of point Z c. 50m
c. Point L is in south of point Y d. 59m
d. Point L is in southwest of point T e. None of these
e. All are correct Directions (24-25): Answer the questions based on the
Directions (21-23): Answer the questions based on the information given below
information given below. Point P is 5m east of point Q, which is 4m north of point
Abhishek starts walking from point A in east direction. R. Point R is exactly between point S and point T. Point S
After walking 10m he turns right and walks for 13m. He is 4m north of point Q, which is exactly between point U
then turns left and walks for 5m and then turns left. He and point P.
walks for 5m and then turns left and walks for 21m and 24) What is the shortest distance between point U and
stop at point B. point T?
21) In which direction is point A with respect to point a. 10m
B? b. 12m
a. South c. 16m
b. North d. 13m
c. North-East e. None of these
d. North-West
Page 234 of 722

Subscribe the Xpress Video Course & Mock Test Package for Bank & Insurance Exams
If there are any suggestions/ errors in our PDFs Feel Free to contact us via this email: admin@exampundit.in
IBPS RRB Clerk Prelims – Ultra Practice Bundle PDF
25) What is the direction of point P with respect to the Directions (28-30): Answer the questions based on the
point T? information given below:
a. North Eight persons, P, Q, R, S, T, U, V and W are sitting at
b. South west some distance from each other. P is sitting 2m west to Q.
c. North east V is sitting 9m east to W. S is sitting 5m east to R, who is
d. South east sitting 3m north to Q. V is sitting 8m north to U. T is 4m
e. None of these west of U. S is 3m north of T.
Direction (26-27): Answer the following question 28) What is the shortest distance between W and the
based on the information given below. person sitting west to S?
Point A is 8m west of point B. Point E is 1m east of point a. 5m
F. Point G is 2m east of point of H. Point E is 4m north of b. 2m
point D. Point C is 10m west of point D. Point G is 6m c. 8m
north of point F. Point C is 6m south of point B. d. 6m
26) What is the shortest distance between the point A e. 3m
and point C? 29) What is the direction of R with respect to V?
a. 9m a. South
b. 13m b. South east
c. 10m c. North east
d. 12m d. South west
e. None of these e. None of the above
27) Point H is in which direction with respect to point 30) What is the total distance between P and U?
B? a. 15m
a. North-East b. 12m
b. North-West c. 18m
c. South-West d. 11m
d. South-East e. 13m
e. None of these. Directions (31-32): Answer the questions based on the
information given below:
Page 235 of 722

Subscribe the Xpress Video Course & Mock Test Package for Bank & Insurance Exams
If there are any suggestions/ errors in our PDFs Feel Free to contact us via this email: admin@exampundit.in
IBPS RRB Clerk Prelims – Ultra Practice Bundle PDF
Point R is 4m north of point P. Point S is 9m north of point d. South west
Q. Point U is 3m north of point S, which is 6m east of e. None of these
point R. Point Q is 5m west of point T. 34) What is the shortest distance between G and A?
31) What is the shortest distance between point U and a. 15m
point T? b. 10m
a. 13m c. 12m
b. 17m d. 8m
c. 15m e. None of these
d. 12m 35) What is the shortest distance between C and E?
e. None of these a. 5m
32) What is the direction of point Q with respect to b. 10m
point R? c. 12m
a. North east d. 8m
b. South west e. None of these
c. South east Directions (36-38): Answer the questions based on the
d. East information given below.
e. None of these Vani walks 8m from point R towards the east to reach
Directions (33-35): Answer the questions based on the point S. Then she turns left and walks 10m to reach the
information given below: point T. Then she moves 12m east direction and reach the
Seven persons, A, B, C, D, E, F and G are sitting at some stationary shop. Then she moves 10m in south direction
distance from each other. D is sitting 3m north to F, who and then turns left and walks 4m to stop at point D. Karan
is sitting 4m west to G. C is sitting 4m west to B. E is walks 10m from point B towards the east direction and
sitting 4m west to D. A is sitting 9m south to C. B is sitting then he turns left and walks 5m to stop at point D.
3m south to E. 36) What is the distance between Karan’s ending point
33) What is the direction of E with respect to A? and Vani’s starting point?
a. North a. 24m
b. North east b. 20m
c. North West c. 10m
Page 236 of 722

Subscribe the Xpress Video Course & Mock Test Package for Bank & Insurance Exams
If there are any suggestions/ errors in our PDFs Feel Free to contact us via this email: admin@exampundit.in
IBPS RRB Clerk Prelims – Ultra Practice Bundle PDF
d. 23m d. Both (a) and (c)
e. None of these e. None is true.
37) Point B is in which direction with respect to point 40) What is the shortest distance between point G and
T? the point H?
a. South west a. 14m
b. South b. 12m
c. South east c. 10m
d. North west d. 11m
e. None of these e. None of these
38) Stationary shop is in which direction with respect 41) In which direction is C with respect to D?
to point R? a. East
a. South west b. North - West
b. North East c. North - east
c. South east d. South
d. North west e. None of these
e. None of these Directions (42-44): Answer the questions based on the
Direction (39-41): Study the information given below information given below.
carefully and answer the questions that follow. Six persons, P, Q, R, S, T and U are sitting at some
Point A is 4m west of point G, which is 8m north of point distance from each other. P is sitting 5m west to T. R is
B. Point E is 11m south of point D, which is 7m east of sitting 7m north to U, who is sitting 4m east to S. T is
point F. Point H is 10 south of point C, which is 5m north sitting 3m north to Q, who is sitting 2m east to S.
of point F. Point B is 6m west of point H. 42) What is the direction of P with respect to U?
39) Which among the following statements is/are true? a. North
a. Shortest distance between point G and point H is equal b. South east
to the distance between point H and point C. c. North east
b. Shortest distance between point B and point D is less d. North West
than the distance between point D and point E. e. None of these
c. Point A is north east of point H.
Page 237 of 722

Subscribe the Xpress Video Course & Mock Test Package for Bank & Insurance Exams
If there are any suggestions/ errors in our PDFs Feel Free to contact us via this email: admin@exampundit.in
IBPS RRB Clerk Prelims – Ultra Practice Bundle PDF
43) What is the shortest distance between S and the 46) What is the direction of point N with respect to
person sitting in the 2m east of T? point K?
a. 5m a. North east
b. 3m b. South west
c. 6m c. South east
d. 10m d. North west
e. None of these e. None of these
44) What is the total distance between S and R if we 47)What is the direction of point K with respect to
move in straight lines? point L?
a. 15m a. North east
b. 13m b. South east
c. 16m c. South west
d. 11m d. North west
e. None of these. e. None of these
Directions (45-47): Answer the questions based on the Directions (48-50): Answer the questions based on the
information given below. information given below.
Point S is 5m east of point K, which is 4m south of point A person starts walking from point P in east direction.
P. Point R is 6m south of point S, which is the middle point After walking 3m he turns right from point Q and walks
of point A and point R. Point L is 5m east of point A. Point for 4m he turns left from point R. He then walks for 2m
L is 14m east of point N. Point A is in north of R. and then turns left from point S. He walks for 8m and turns
45) What is the shortest distance between point R and right from point T. He walks for 3m and turns right from
the point, which is 2m north of point P? point U and walks for 10m. He then turns right from point
a. 14m V and walks for 8m till point W.
b. 17m 48) What is the shortest distance between P and R?
c. 12m a. 3m
d. 13m b. 6m
e. None of these c. 5m
d. 4m
Page 238 of 722

Subscribe the Xpress Video Course & Mock Test Package for Bank & Insurance Exams
If there are any suggestions/ errors in our PDFs Feel Free to contact us via this email: admin@exampundit.in
IBPS RRB Clerk Prelims – Ultra Practice Bundle PDF
e. None of these 50) In which direction is point P with respect to point
49) In which direction is point T with respect to point W?
V? a. North
a. West b. North-West
b. North-West c. North-East
c. East d. South-West
d. North e. None of these
e. South

12). Direction Questions - Detailed Explanation with Answers

1. d 5. b
2. b 6. c
3. c Solutions 4-6
Solutions 1-3

Answers
4. e
Answers
Page 239 of 722

Subscribe the Xpress Video Course & Mock Test Package for Bank & Insurance Exams
If there are any suggestions/ errors in our PDFs Feel Free to contact us via this email: admin@exampundit.in
IBPS RRB Clerk Prelims – Ultra Practice Bundle PDF
7. b Solutions 12-14
8. a
Solutions 7-8

Answers
15. b

Answers 16. a

9. c 17. d

10. c Solutions 15-17

11. b
Solutions 9-11

Answers
18. b
Answers 19. d
12. a 20. c
13. c Solutions 18-20
14. a
Page 240 of 722

Subscribe the Xpress Video Course & Mock Test Package for Bank & Insurance Exams
If there are any suggestions/ errors in our PDFs Feel Free to contact us via this email: admin@exampundit.in
IBPS RRB Clerk Prelims – Ultra Practice Bundle PDF
Amit starts walking from point R towards west for 5km to
reach point T.
From point T he took right turn and walks for 8km to reach
point Z. From there he again took a right turn and walks
7km to reach point Y.

Answers
24. d
25. c
Solutions 24-25

From point Y, Amit took a right turn and walks for 4km
to reach point K. After that he took a left turn and walks
for 5km to reach point J.

Answers
26. c
27. a
Answers Solutions 26-27
21. c
22. c
23. b
Solutions 21-23

Page 241 of 722

Subscribe the Xpress Video Course & Mock Test Package for Bank & Insurance Exams
If there are any suggestions/ errors in our PDFs Feel Free to contact us via this email: admin@exampundit.in
IBPS RRB Clerk Prelims – Ultra Practice Bundle PDF

Answers
Answers
33. b
28. a
34. a
29. d
35. a
30. d
Solutions 33-35
Solutions 28-30

Answers
36. a
37. c
Answers 38. b
31. a Solutions 36-38
32. c
Solutions 31-32

Page 242 of 722

Subscribe the Xpress Video Course & Mock Test Package for Bank & Insurance Exams
If there are any suggestions/ errors in our PDFs Feel Free to contact us via this email: admin@exampundit.in
IBPS RRB Clerk Prelims – Ultra Practice Bundle PDF
Solutions 42-44

Answers
39. a
40. c Answers

41. b 45. d

Solutions 39-41 46. d


47. c
Solutions 45-47

Answers
42. d
43. a Answers

44. d 48. c
Page 243 of 722

Subscribe the Xpress Video Course & Mock Test Package for Bank & Insurance Exams
If there are any suggestions/ errors in our PDFs Feel Free to contact us via this email: admin@exampundit.in
IBPS RRB Clerk Prelims – Ultra Practice Bundle PDF
49. b
50. a
Solutions 48-50

13). Alpha Numeric Series


Directions (1-5): Study the following sequence
carefully and answer the questions given below it. 2. Four of the following are alike in a particular
pattern and hence form a group. Find the one which
L < X % V 9 7 8 B @ > B 6 N # ? P / does not follow the pattern?
S ^ T U 5 I $ © 4 D F ↑ A) X % <
B) 9 7 8
1. Find the next group in the following series. C) @ > B
9 B @, B # ?, / T U, ____ D) N # 6
A) I 4 D E) # ? N
B) T I $
C) 4 D F 3. How many numbers are there in the sequence which
D) U 5 I is immediately followed by a letter?
E) None of These A) One
Page 244 of 722

Subscribe the Xpress Video Course & Mock Test Package for Bank & Insurance Exams
If there are any suggestions/ errors in our PDFs Feel Free to contact us via this email: admin@exampundit.in
IBPS RRB Clerk Prelims – Ultra Practice Bundle PDF
B) Two 6. How many such symbols are there in the above
C) Four sequence, which are immediately followed by a
D) Three consonant and immediately preceded by a number?
E) More than four A) Three
B) Two
4. Which of the following is the 19th element to the C) Four
right of the 7th element to the left of the 10th element D) One
from the left end when first 20 elements reversed? E) None of These
A) U
B) 5 7. Which is the 14th element to the right of 13th
C) I element from the left end?
D) T A) H
E) None of These B) 4
C) G
D) %
5. How many symbols are there, each of which are
E) 8
immediately followed by a consonant and preceded by
a vowel?
8. Find the next element in the following series.
A) One
L, V, N, J, S, _____
B) Two
A) E
C) Three
B) H
D) Four
C) F
E) None
D) G
E) None of These
Directions (6-8): Study the following alphabet series
and answer the questions given below.
Direction (9-13): Study the following arrangement and
answer the given questions:
L#V3/NW+6JYK↑@ST8C2↓F9E>4H%
G*&
Page 245 of 722

Subscribe the Xpress Video Course & Mock Test Package for Bank & Insurance Exams
If there are any suggestions/ errors in our PDFs Feel Free to contact us via this email: admin@exampundit.in
IBPS RRB Clerk Prelims – Ultra Practice Bundle PDF
A/XV4D@L#BH9&X2BK%R35RJO7M D) None
^A6 E) None of These

9. If all the symbols are dropped from the above 12. Which of the following will be the fourth to the left
arrangement, which of the following will be the sixth of the seventeenth from the left end of the above
from the right end? arrangement?
A) M A) 9
B) A B) &
C) 6 C) X
D) R D) 2
E) J E) None of These

10. How many such vowels are there in above 13. Which of the following is the fourth to the right of
arrangement, each of which is immediately preceded the eleventh to the right of first X in the given
by a consonant and immediately followed by a arrangement?
number? A) %
A) None B) K
B) Three C) B
C) Two D) R
D) One E) None of These
E) None of These
Directions (14-15): Study the following information
11. How many such numbers are there in above carefully to answer the questions given below.
arrangement, each of which is immediately preceded
and also immediately followed by consonant? SNB5P%E7C4#GI$2DG*18L3V@9ZX
A) One &Y
B) Three
C) Two
Page 246 of 722

Subscribe the Xpress Video Course & Mock Test Package for Bank & Insurance Exams
If there are any suggestions/ errors in our PDFs Feel Free to contact us via this email: admin@exampundit.in
IBPS RRB Clerk Prelims – Ultra Practice Bundle PDF
14. How many such consonants are there in the above B) Two
arrangement, each of which is immediately followed C) Three
by a number but not immediately preceded by a D) More than three
vowel? E) One
A) None
B) One 17. If all the numbers are dropped from the series,
C) Two which element will be 5th to the left of 15th from left
D) Four end?
E) Three A) Q
15. Which of the following is the 5th to the left of the B) S
20th from the left end in the above arrangement? C) %
A) $ D) N
B) I E) None of These
C) D
D) 2 18. Which of the following is 8th to the right of 15th
E) G from right end of row?
A) L
Directions (16-20): Study the following B) C
letters/numbers/symbols sequence and answer the C) !
following questions. D) $
E) K
C 4 & 3 A B G 9 # @ 2 Q N S% 5 1 J
D * 0 L V 7 L ! C $ K V 6 I 19. Four of the following are alike in a certain way, so
form a group. Find the odd one out – the one which
16. How many numbers are there in the series, which does not belong to that group.
are immediately preceded by a letter and immediately A) C 4 &
followed by a consonant? B) @ Q S
A) None C) 5 1 J
Page 247 of 722

Subscribe the Xpress Video Course & Mock Test Package for Bank & Insurance Exams
If there are any suggestions/ errors in our PDFs Feel Free to contact us via this email: admin@exampundit.in
IBPS RRB Clerk Prelims – Ultra Practice Bundle PDF
D) D * 0 22. If all the numbers are dropped from the series,
E) V 7 L which element will be 5th to the left of 18th from left
end?
20. The following series follow a certain pattern. What A) $
will come in question mark (?) in the series: B) 8
43B, 9@Q, S5j, *L7, ? C) %
A) LCK D) N
B) 0LV E) None of These
C) V7L
D) !$V 23. Which of the following will be second to the right
E) None of These of 17th from the left end?
A) $
Directions (21-25): Study the following B) 5
letters/numbers/symbols sequence and answer the C) N
following questions. D) 8
E) %
8 1 @ L M C # 7 * E G 6 V B 3 S 5 $ N 8 % ◇P D 1 P 4
YXZ 24. If every second element from the left end dropped
from the series, which element is 4th to left of 11th
21. How many numbers are there in the series, which from the right end in new arrangement?
immediately preceded by a symbols and immediately A) 8
followed by a letter? B) 1
A) One C) @
B) Two D) L
C) Three E) None of These
D) More Than Three
E) None 25. What is the total number of ‘The numbers
immediately follow the letters’ and ‘The symbols
Page 248 of 722

Subscribe the Xpress Video Course & Mock Test Package for Bank & Insurance Exams
If there are any suggestions/ errors in our PDFs Feel Free to contact us via this email: admin@exampundit.in
IBPS RRB Clerk Prelims – Ultra Practice Bundle PDF
immediately followed by a consonant’ together in the D) V
above sequence? E) 3
A) Nine
B) Seven 28. What should come in place of question mark in the
C) Six following on the basis of the above sequence?
D) Five 6#T, CX(, ?
E) Four A) NA1
B) N%V
Directions (26-30): Study the following arrangement C) DV*
and answer questions given: D) Cannot Be Determined
E) None of These
EM@32*VD%!ANQ(BX~9C+TY#LZ6
29. How many such symbols are there in the above
26. If the first fifteen elements in the above passage sequence, each of which is immediately preceded by a
sequence are written in reverse order then which of the consonant and immediately followed by a number?
following will be seventeenth from the right end? A) One
A) V B) Two
B) 2 C) Three
C) D D) More Than Three
D) % E) None of These
E) *
30. Which of the following is sixth to the left of
27. If the above sequence is written in reverse order fourteenth element from the left end?
then which of the following will be fifth to the right of A) D
twelve element from the right end? B) V
A) D C) %
B) * D) *
C) 2 E) None of These
Page 249 of 722

Subscribe the Xpress Video Course & Mock Test Package for Bank & Insurance Exams
If there are any suggestions/ errors in our PDFs Feel Free to contact us via this email: admin@exampundit.in
IBPS RRB Clerk Prelims – Ultra Practice Bundle PDF
Directions (31-35): Study the following arrangement E) A
and answer questions given:
34. How many such numbers are there each of which
82Z*#CD9N01&~MA)6RBY3J+<=FK is immediately preceded by a consonant?
A) Two
31. Which of the following is the 4th element to the left B) Three
of the 14th element from the right end? C) One
A) 1 D) More Than Three
B) & E) None of These
C) M
D) A 35. Four of the five are alike in a certain way based on
E) 0 arrangement, which does not belong to the group?
A) Z # D
32. How many such numbers are there in the above B) N 0 1
arrangement each of which is immediately preceded C) 6 R B
by a consonant and immediately followed by a symbol? D) R B Y
A) One E) None of These
B) Two
C) Three Directions (36-40): Study the following arrangement
D) None and answer questions given:
E) More Than Three T%#CDZ3@0N!K6*AHV8^Y&Q7OJ12
$CB)
33. Which of the following is the 6th letter to the left of
the 14th letter from the left end? 36. Which of the following is the 7th element to the
A) D right of the 12th letter from the right end?
B) N A) $
C) 9 B) C
D) M C) B
Page 250 of 722

Subscribe the Xpress Video Course & Mock Test Package for Bank & Insurance Exams
If there are any suggestions/ errors in our PDFs Feel Free to contact us via this email: admin@exampundit.in
IBPS RRB Clerk Prelims – Ultra Practice Bundle PDF
D) J A) CD3
E) 2 B) T%#
C) 0N!
37. How many such numbers are there in the above D) AHV
arrangement each of which is immediately preceded E) Y&Q
by a consonant and immediately followed by a symbol?
A) One Directions (41-45): Study the following arrangement
B) Two and answer questions given:
C) Four
D) Three (CD~62%SZ8Y)*1KN9@VX$3^R4AS
E) None of These #

38. Which of the following is the 7th element to the left 41. How many such vowels are there in the above
of the 20th element from the left end? arrangement, each of which is immediately followed
A) 5 by a letter and not preceded by a symbol?
B) 4 A) None
C) 6 B) Two
D) 3 C) Three
E) None of These D) More Than Three
39. How many such symbols are there each of which is E) One
immediately preceded by a vowel?
A) One 42. What is the sum of all numbers which are
B) None immediately followed by a letter?
C) Two A) 18
D) Three B) 19
E) None of These C) 20
40. Four of the five are alike in a certain way based on D) 21
arrangement, which does not belong to the group? E) 22
Page 251 of 722

Subscribe the Xpress Video Course & Mock Test Package for Bank & Insurance Exams
If there are any suggestions/ errors in our PDFs Feel Free to contact us via this email: admin@exampundit.in
IBPS RRB Clerk Prelims – Ultra Practice Bundle PDF
Directions (46-50): Study the following arrangement
43. What will come in place of questions mark (?) and answer questions given:
based on the above arrangement?
CD~, %SZ, )*1, ? LG2^I!AR3D7$9K#CE8ZI%MR4U*N+
A) KN9 W
B) R4A
C) 9@V 46. How many such numbers are there in the above
D) X$3 arrangement, each of which is immediately preceded
E) None of These by a symbol and immediately followed by a vowel?
A) One
44. Four of the following five are alike in a certain way B) Two
based on their positions in the above arrangement and C) Three
so form a group. Which is the one that does not belong D) More Than Three
to that group? E) None
A) % 8 *
B) C ~ 2 47. How many such consonants are there in the above
C) 1 9 X arrangement, each of which is immediately preceded
D) $ R S by a number and immediately followed by a letter?
E) None of These A) Two
45. Which of the following is the 7th to the left of the B) Three
20th from the left end of the above arrangement? C) More Than Three
A) * D) One
B) ) E) None of These
C) 1
D) K 48. Which of the following is the 6th to the right of the
E) N 20th from the right end of the above arrangement?
A) #
B) E
Page 252 of 722

Subscribe the Xpress Video Course & Mock Test Package for Bank & Insurance Exams
If there are any suggestions/ errors in our PDFs Feel Free to contact us via this email: admin@exampundit.in
IBPS RRB Clerk Prelims – Ultra Practice Bundle PDF
C) C D) C8I
D) 8 E) IM4
E) X 50. Which of the following is the 5th to the left of the
49. Four of the following five are alike in a certain way 13th from the right end of the above arrangement?
based on their positions in the above arrangement and A) $
so form a group. Which is the one that does not belong B) 9
to that group? C) K
A) L2I D) 7
B) AR3 E) None of These
C) D$K

13). Alpha Numeric Series - Answers with Explanation

Answers (1-5): The series given in this question is: 1) We will look at the first element and the second element
then we came to know that after first element two
L < X % V 9 7 8 B @ > B 6 N # ? P / S elements are left and then third element is taken and when
^ T U 5 I $ © 4 D F ↑ we will look for the relation between second and the third
element third element is immediate next to the second
1) A element.
2) B When next series is taken it starts with the same pattern
3) C but after leaving one element between the last element of
4) A the first series and first element of the next series.
5) E As per this pattern the series which will come in place of
the blank will be I4D.
Solutions (1-5): Answer: A (I4D)
2) In this question we will take one option at a time and
look for the pattern.
Page 253 of 722

Subscribe the Xpress Video Course & Mock Test Package for Bank & Insurance Exams
If there are any suggestions/ errors in our PDFs Feel Free to contact us via this email: admin@exampundit.in
IBPS RRB Clerk Prelims – Ultra Practice Bundle PDF
In first option, first element is between the second and 5) In this question we will look for such symbols which are
third element in the series given. immediately followed by a consonant and preceded by a vowel.

In second option, second element is immediate next to the There are no such pair in the series given, so answer will be
NONE
first element and third element is immediate next to the
Answer: E (None)
second element.
In third option, first element is between the second and
Answers (6-8): The series given in this question is:
third element in the series given.
In fourth and fifth option, first element is between the
L#V3/NW+6JYK↑@ST8C2↓F9E>4H%G*
second and third element in the series given.
&
As per the pattern only option is the second one which
does not follow this pattern, so answer will be 9 7 8.
6) B
Answer: B (9 7 8)
7) D
8) C
3) We will look for such numbers in the series which is being
followed by a letter.
Here, in the series we found four such pairs and the pairs are:
Solutions (6-8):

8B, 6N, 5I and 4D. So answer will be four such pairs


Answer: C (Four) 6) In this question we will have to look for such symbols
which are immediately followed by a consonant and
4) After reversing the first 20 elements in this question the new immediately preceded by a digit.
series will look like this: As per this pattern there are two such pairs and they are: 3/N
and 2↓F. So, answer is Two
^S/P?#N6B>@B879V%X<LTU5I$©4DF↑ Answer: B (Two)
Tenth element from the left end in the new series will be > and 7) In this question we have to find 14th element to the right of
7th element to the left of this > is / and the 19th element to the 13th element from the left end.
right of / will be U. The element which is 13th from the left end is ↑ and the element
Answer: A (U) which is 14th to the right of this element is %
Answer: D (%)

Page 254 of 722

Subscribe the Xpress Video Course & Mock Test Package for Bank & Insurance Exams
If there are any suggestions/ errors in our PDFs Feel Free to contact us via this email: admin@exampundit.in
IBPS RRB Clerk Prelims – Ultra Practice Bundle PDF
8) In this question we have to first identify the pattern Now in the above series we have to find the sixth element
The pattern is after first element that is L one element is left from the right end and that element is J
and the next element is written that is V and after V two Answer: E (J)
elements are left and the third element is written that is N and
after N three elements are left and next element is written that
10) In this question we have to find and look for such
is J and after J four elements are left and fifth elements is
vowels which is immediately preceded by a consonant and
written that is S. Now as we can see each time one element
immediately followed by a number
which is being left is added to the number of left elements, so
Now there is only one such pair and that pair is JO7
after S five elements will be left and the next element will be F
Answer: C (F) Answer: D (One)

Answers (9-13): The series given in this question is: 11) In this question we have to find and look for such
numbers which is immediately preceded and also
A/XV4D@L#BH9&X2BK%R35RJO7M^A immediately followed by consonant.
6 As per this pattern we find there are two such pairs and
they are: V4D and X2B. SO, answer is Two
9) E Answer: C (Two)
10) D
11) C 12) In the series given in the question we have to find the
12) B element which is fourth to the left of the seventeenth from
13) A the left end.
Element which is seventeenth from the left end is K and
Solutions (9-13): the element which fourth to the left of this element is &
Answer: B (&)
9) In this question we will drop all the symbols from the 13) In this question we have to look for that element which
series and the new series will look like this: is fourth to the right of the eleventh to the right of first X
in the series given in the question.
AXV4DLBH9X2BKR35RJO7MA6 Element which is eleventh to the right of first X is X and
the element fourth to the right of X will be %
Page 255 of 722

Subscribe the Xpress Video Course & Mock Test Package for Bank & Insurance Exams
If there are any suggestions/ errors in our PDFs Feel Free to contact us via this email: admin@exampundit.in
IBPS RRB Clerk Prelims – Ultra Practice Bundle PDF
Answer: A (%) 16) E
17) B
Answers (14-15): The series given in this question is: 18) C
19) B
SNB5P%E7C4#GI$2DG*18L3V@9ZX&Y 20) D

14) E Solutions (16-20):


15) D
16) In this question we have to find such numbers which are

Solutions (14-15): immediately preceded by a letter and immediately followed by


a consonant.

14) In this question we have to find such consonants, which is Now, as per this pattern there is only one such pair and that pair

immediately followed by a number but not immediately is: V7L. So, answer will be One

preceded by a vowel Answer: E (One)

As per this pattern there are only 3 pairs and such pairs are: B5,
C4 and L3. So, answer is Three 17) In this question we will drop all the numbers and the new

Answer: E (Three) series will look like this:

15) In this question we have to find the element which 5th to C&ABG#@QNS%JD*0LVL!C$KVI

the left of the 20th from the left end. Now in the new series the element which is 15th from the left

Element which is 20th from the left end is 8 and the element end will be O and the element which is 5th from the left of this

which is 5th to the left of this element is 2. So, answer will be 2 element will be S. So answer will be S

Answer: D (2) Answer: B (S)


18) In this question we have to look for element which is 8th to

Answers (16-20): The series given in this question is: the right of 15th from right end of row.
The element which is 15 from the right end of the row will be
J and the element which is to the right of this element will be !.
C 4 & 3 A B G 9 # @ 2 Q N S% 5 1 J D
Answer: C (!)
* 0 L V 7 L ! C $ K V 6 I

Page 256 of 722

Subscribe the Xpress Video Course & Mock Test Package for Bank & Insurance Exams
If there are any suggestions/ errors in our PDFs Feel Free to contact us via this email: admin@exampundit.in
IBPS RRB Clerk Prelims – Ultra Practice Bundle PDF
19) In this question we will take option one by one and try to 25) A
find the pattern.
In the first option, the pattern is second element is the next Solutions (21-25):
element to the first element and third element is immediate next
to the second element. 21) In this question we have to find such numbers which
In the second option, the pattern is one element is left after the immediately preceded by a symbols and immediately followed
element and the next element is written and after second by a letter.
element one element is left and the next element is written Now as per this pattern there are no such pairs which show the
In the third, fourth and fifth option, the pattern is second above-mentioned pattern.
element is the next element to the first element and third Answer: E (None)
element is immediate next to the second element.
So, answer will be @ Q S 22) In this we will drop all the numbers and the new series will
Answer: B (@ Q S) look like this:

20) In this question we will try to find the pattern of the series. @ L M C # * E G V B S $ N % ◇P D P Y X Z
The pattern is one element is left after the first element and the
next element is written and after second element one element is Now as per the above series we will find out the 18th element
left and the next element is written and between the first from the left end and that element is P and the element which
element series and second element series one element is left is 5th to the left of this element will be $. So, answer will be N
As per this pattern the pair which will come will be: !$V Answer: D (N)
Answer: D (!$V)
23) Now in this question we have look for that element which
Answers (21-25): The series given in this question is: second to the right of 17th from the left end.
Element which is 17th from the right end is 5 and the element
8 1 @ L M C # 7 * E G 6 V B 3 S 5 $ N 8 % ◇P D 1 P 4 Y X which second from this element is N. So answer will be N
Z Answer: C (N)
21) E 24) In this question we will drop every second element from the

22) D left end and the new series will look like this:

23) C
8@M#*GV35N%P14X
24) A
Page 257 of 722

Subscribe the Xpress Video Course & Mock Test Package for Bank & Insurance Exams
If there are any suggestions/ errors in our PDFs Feel Free to contact us via this email: admin@exampundit.in
IBPS RRB Clerk Prelims – Ultra Practice Bundle PDF
Now we have to find the element which 17th from the right end
As per the above series the element which is 11th from the right and that element is *. So, answer is *
end is * and the element which is 4th to this element is 8 Answer: E (*)
Answer: A (8)
27) In this question we have to write the above series in the
25) In this question we will find the numbers immediately reverse order and the new series will look like this:
following the letters and the symbols immediately followed by
a letter 6ZL#YT+C9~XB(QNA!%DV*23@ME
Pairs of the numbers immediately following the letters: G6, B3,
S5, N8, D1 and P4 Now in this question we have to find the element which 12th
Pairs of the symbols immediately followed by a letter: @L, $N from the right end and that element is N and the element which
and ◇P is 5th to the right of this element is V. So answer is V
Answer: A (Nine) Answer: D (V)

Answers (26-30): The series given in this question is: 28) In this question we have to look for the pattern which is
being followed in the series

EM@32*VD%!ANQ(BX~9C+TY#LZ6 The pattern after first element two elements are left and the next

26) E element is written and then after that element one element is
left and the next element is written. Between the two pairs there
27) D
is a gap of one element.
28) B
As per this pattern the pair which will come in place of question
29) B
mark will be: N%V
30) A
Answer: B (N%V)
29) In this question we will look for such symbols which is
Solutions (26-30):
immediately preceded by a consonant and immediately
followed by a number.
26) In this question we will write the first 15 elements in the
Now the pair which follows this pattern is: M@3 and X ~ 9.
reverse order and the new series will look like this:
So, answer is Two
Answer: B (Two)
B(QNA!%DV*23@MEX~9C+TY#LZ6

Page 258 of 722

Subscribe the Xpress Video Course & Mock Test Package for Bank & Insurance Exams
If there are any suggestions/ errors in our PDFs Feel Free to contact us via this email: admin@exampundit.in
IBPS RRB Clerk Prelims – Ultra Practice Bundle PDF
30) In this question we have to find out element which is sixth
to the left of fourteenth element from the left end. 33) In this question we have to find the element which is 6th
Element which is 14th from the left end is ( and the element letter to the left of the 14th letter from the left end.
which is 6th to the left of this element is D The element which is 14th from the left end is M and the
Answer: A (D) element which is 6th from the left of this element is 9
Answer: C (9)
Answers (31-35): The series given in this question is: 34) In this question we have to look for such numbers which is
immediately preceded by a consonant.

82Z*#CD9N01&~MA)6RBY3J+<=FK There are three such pairs which follows this pattern and they
are: D9, N0 and Y3.
Answer: B (Three)
31) E
32) D
35) In this question we have to find common pattern between
33) C
all the options and the option which does not follows that
34) B
pattern will be our answer.
35) A In the first option after first element one element is left and then
the next element is written and after second element one
Solutions (31-35): element is left and the next element is written
31) In this question we have to find 4th letter to the left of the In the second option, second element is immediate next to the
14th letter from the right end. first element and the third element is immediate next to the
The element which is 14th from the right end is M and the second element.
element which is 4th to the left of this element is 0. In the third and fourth option, second element is immediate next
Answer: E (0) to the first element and the third element is immediate next to
the second element.
32) In this question we have to find such numbers which is So, answer will first option
immediately preceded by a consonant and immediately Answer: A (Z#D)
followed by a symbol.
There is no such pair which follows such pattern. So, answer is
Answers (36-40): The series given in this question:
NONE
T%#CDZ3@0N!K6*AHV8^Y&Q7OJ12$
Answer: D (None)
CB)

Page 259 of 722

Subscribe the Xpress Video Course & Mock Test Package for Bank & Insurance Exams
If there are any suggestions/ errors in our PDFs Feel Free to contact us via this email: admin@exampundit.in
IBPS RRB Clerk Prelims – Ultra Practice Bundle PDF
There are no such pairs which follows the above-mentioned
36) E pattern. So answer is NONE

37) D Answer: B (None)

38) C
40) In this question we have to find a common pattern among
39) B
the four options and the option which does not follows this
40) A
patter will be our answer.
In the first option second element is immediate next of the first
Solutions (36-40):
element and there is a gap of one element between the second
and the third element.
36) In this question we have to find the element which is 7th
In the second option, there is no gap between all the three
element to the right of the 12th letter from the right end.
elements means they are the immediate next of each other.
Element which is 12th from the right end is Y and the element
In the third, fourth and fifth option, there is no gap between all
which is 7th to the right of this element is 2
the three elements means they are the immediate next of each
Answer: E (2)
other.
So, answer will first option
37) In this question we have to look for such numbers which is
Answer: A (CD3)
immediately preceded by a consonant and immediately
followed by a symbol
Answers (41-45): The series given in this question is:
There are three such pairs which follows this pattern and they
are: Z3@, K6* and V8^. So answer is Three
(CD~62%SZ8Y)*1KN9@VX$3^R4AS#
Answer: D (Three)
38) In this question we have to find out the element which is 7th
to the left of 20th element from the left end. 41) E
Element which is 20th from the left end is Y and the element 42) D
which is 7th to the left of this element is 6 43) C
Answer: C (6) 44) B
45) A
39) In this question we have to find out such symbols which is
immediately preceded by a vowel. Solutions (41-45):

Page 260 of 722

Subscribe the Xpress Video Course & Mock Test Package for Bank & Insurance Exams
If there are any suggestions/ errors in our PDFs Feel Free to contact us via this email: admin@exampundit.in
IBPS RRB Clerk Prelims – Ultra Practice Bundle PDF
In the third and fourth option, there is a gap of two elements
41) In this question we have to find out such vowels, which is between elements of those pairs
immediately followed by a letter and not preceded by a symbol. So, answer will be second option
As per the above-mentioned pattern there is only such pair Answer: B (C ~ 2)
which follows this pattern and that pair is: AS
Answer: E (One) 45) In this question we have to look for that element which is
7th to the left of the 20th from the left end
42) In this question we have to look for such numbers which is The element which is 20th from the left end is X and the element
immediately preceded by a letter and we have to find the sum which 7th to the left of this element is *
of such numbers. Answer: A (*)
There are three such pairs which follows the above-mentioned
pattern and they are: Z8, N9 and R4. And the sum of such Answers (46-50): The series given in this question is:
numbers means we have to add 8+9+4=21
Answer: D (21) LG2^I!AR3D7$9K#CE8ZI%MR4U*N+W

43) In this question we have to find the pattern which is being


46) E
followed by the series which is being asked in the question.
47) D
The pattern is: The elements are immediate next of each other
48) C
and there is a gap of two elements between two such pairs, so
49) B
the next pair which will come in place of question mark will be:
50) A
9@V
Answer: C (9@V)
Solutions (46-50):

44) In this question we have to find a common pattern among 46) In this question we have to find such numbers which is

the four options and the option which does not follows this immediately preceded by a symbol and immediately followed

patter will be our answer. by a vowel

In the first option there is a gap of two elements between There are no such pairs which follows the above-mentioned

elements of this pair pattern. So, answer will be NONE

In the second option, there is a gap of one element between Answer: E (None)

elements of this pair

Page 261 of 722

Subscribe the Xpress Video Course & Mock Test Package for Bank & Insurance Exams
If there are any suggestions/ errors in our PDFs Feel Free to contact us via this email: admin@exampundit.in
IBPS RRB Clerk Prelims – Ultra Practice Bundle PDF
47) In this question we have to find such consonants which is In the first option, there is a gap of one element between
immediately preceded by a number and immediately followed elements of this pair
by a letter In the second option, there is no gap between the elements of
There is only one such pair which follows this above- this pair
mentioned pattern and that pair is: 8ZI In the third, fourth and fifth option, there is a gap of one element
Answer: D (One) between elements of these pairs
So, answer will be second option
48) In this question we have to find that element which is 6th Answer: B (AR3)
to the right of the 20th from the right end.
The element which is 20th from the right end is D and the 50) In this question, we have to find that element which is 5th
element which is 6th to the right of this element is C to the left of the 13th from the right end
Answer: C (C) The element which is 13th from the right end is E and the
49) In this question we have to find a common pattern among element which is 5th to the left of this element is $
the four options and the option which does not follows this Answer: A ($)
patter will be our answer.

14). Reasoning Data Sufficiency

Directions (1-5): Each of the questions given below


consists of a question and two statements numbered I b) If the data in statement II alone is sufficient to answer
and II given below it. You have to decide whether the the question, while the data in statement I alone is not
data provided in the statements are sufficient to sufficient to answer the question.
answer the question or not. Read both the statements
and answer the question. c) If the data either in statement I alone or in statement II
a) If the data in statement I alone is sufficient to answer alone are sufficient to answer the question.
the question, while the data in statement II alone is not
sufficient to answer the question.

Page 262 of 722

Subscribe the Xpress Video Course & Mock Test Package for Bank & Insurance Exams
If there are any suggestions/ errors in our PDFs Feel Free to contact us via this email: admin@exampundit.in
IBPS RRB Clerk Prelims – Ultra Practice Bundle PDF
d) If the data in both statement I and II together are not II. The principal of the school correctly remembers that
sufficient to answer the question. the difference between the students in class 10 and class 7
is less than nine. There are 33 students in class 7.
e) If the data in both statement I and II together are
necessary to answer the question. 4) Six persons A, B, C, D, E and F are living on
different floors of a building in such a way that the
1) Among A, B, C, D and E sitting around a circular ground floor is numbered as 1, the above 1 is
table facing the centre, who sits second to the right of numbered as 2 and so on till the topmost floor is
C? numbered as 6. How many persons are living between
I. C sits to the immediate right of A. Only two persons A and D?
were sitting between B and A. I. C lives on an even numbered floor but not on the
II. Both D and C are immediate neighbours of B. E sits topmost floor. Only two persons were living between F
second to the right of B. and C. A lives below F.
II. Only four persons living between B and D. No one lives
2) Amongst five bags L, M, N, O and P, each having a between the floors on which B and C lives. F lives
different weight. Which of the bag is heaviest? immediately below D.
I. Only two bags are heavier than O. O is heavier than P
but lighter than M. 5) Six persons J, K, L, M, N and O sitting in a row
II. N is lighter than O but heavier than P. L is heavier than facing north. Who among the following sits third from
O, but not the heaviest. the right end?
I.K sits at an extreme end of the row. J sits second to the
3) How many students are there in class 10 in the right of K. Only one person sits between J and L. N sits
school X? on the immediate right of L.
I. The class teacher of class 10 correctly remembers that II.N sits third to the right of M. Only one person sits
the number of students in her class is more than 21 but less between N and O. O sits to the right of N. L is an
than 32. The number of students in the class is exactly immediate neighbour of N.
divisible by 6.

Page 263 of 722

Subscribe the Xpress Video Course & Mock Test Package for Bank & Insurance Exams
If there are any suggestions/ errors in our PDFs Feel Free to contact us via this email: admin@exampundit.in
IBPS RRB Clerk Prelims – Ultra Practice Bundle PDF
Directions (6-10): Each of the questions below consists II. T sits second to the left of R. S sits fourth to the right
of a question and two statements numbered I and II of P, who is an immediate neighbour of R. S is not an
given below it. You have to decide whether the data immediate neighbour of R.
provided in the statements are sufficient to answer the
question. Read both the statements and give answer 7) In what date does Manish attend the interview in
Mumbai?
a) If the data in statement I alone are sufficient to answer I. Manish’s father correctly remembers that; Manish
the question, while the data in statement II alone are not leaves for Mumbai one of the odd numbered dates after
sufficient to answer the question. 6th but before 12th of the July.
II. Manish’s wife remembers that; Manish attend the
b) If the data in statement II alone are sufficient to answer interview on one of the dates after 8th of July.
the question, while the data in statement I alone are not
sufficient to answer the question. 8) Ten persons are sitting in a straight row and facing
north. How many persons sitting to the right of V?
c) If the data either in statement I alone or in statement II I. S sits third to the right of M and none of them sits at end
alone are sufficient to answer the question. of the row. Only two persons are sitting between V and S.
II. V sits second to the left of A, who sits from either ends
d) If the data in both statement I and II together are not of the row.
sufficient to answer the question.
e) If the data in both statement I and II together are 9) Seven persons J, H, G, F, D, S and A attends the
necessary to answer the question. exams on seven different days of the week starting
6) Six persons N, P, Q, R, S and T are sitting in a from Monday to Sunday. Who among them attend the
circular table and facing center of the table. Who exam on Saturday?
among the following sits opposite to Q? I. J attends the exam on Wednesday. Only one person
I. S sits second to the right of N. T sits immediate left of attends the exam between J and A. Only two persons
P, who is not an immediate neighbour of S and N. attend the exam between S and G.

Page 264 of 722

Subscribe the Xpress Video Course & Mock Test Package for Bank & Insurance Exams
If there are any suggestions/ errors in our PDFs Feel Free to contact us via this email: admin@exampundit.in
IBPS RRB Clerk Prelims – Ultra Practice Bundle PDF
II. G attends the exam on Thursday. Only two persons d) If the data in both the statements I and II together are
attend the exams between G and S. F attends the exam not sufficient to answer the question.
immediately before S.
e) If the data in both the statements I and II together are
10) A, B, C, D, E and F are arranged according to their necessary to answer the question.
weight. Which of the following person is the lightest?
I. B is heavier than C but lighter than E. C is not the 11) On which day of the same week is Sarath exam
lightest. scheduled (Monday being the first day of the week)?
II. D is heavier than only two persons. F is lighter than I. Sarath correctly remembers that his exam is scheduled
only one person. on a day after Tuesday, but before Thursday of the same
Directions (11-15): Each of the following questions week.
below consists of a question and two statements II. Sarath’s father correctly remembers that Sarath’s exam
numbered I and II given below it. You have to decide is scheduled on the third day of the week.
whether the data 12) How many marks has Supriya scored in the test?
provided in the statements are sufficient to answer the (Maximum marks 20)
question. Read both the statements and give answer I. Supriya scored two-digit marks. Her marks were not in
decimals.
a) If the data in statement I alone are sufficient to answer II. Supriya scored more than 9 marks in the test.
the question, while the data in statement II alone are not 13) In which month of the year did Magesh go abroad
sufficient to answer the question. for a vacation?
I. Magesh correctly remembers that he went for a vacation
b) If the data in statement II alone are sufficient to answer in the first half of the year.
the question, while the data in statement I alone are not II. Magesh wife correctly remembers that they went for a
sufficient to answer the question. vacation after 31st March but before 1st May.

c) If the data in either statement I alone or statement II 14) In which direction is point A, with reference to
alone are sufficient to answer the question. point B?

Page 265 of 722

Subscribe the Xpress Video Course & Mock Test Package for Bank & Insurance Exams
If there are any suggestions/ errors in our PDFs Feel Free to contact us via this email: admin@exampundit.in
IBPS RRB Clerk Prelims – Ultra Practice Bundle PDF
I. Point D is to the east of point A. Point A is to the south 16) Seven persons P, Q, R, S, T, U and V are sitting in
of point F. a straight line facing north. Who among the following
II. Point F is to the north-west to point B. Point D is to the sits second to the left of S?
north of point B. I) There are only two persons sitting between R and Q,
who sits to the right of R. V is not an immediate neighbour
15) Among V, W, X, Y and Z seated in a straight line of Q. S is not an immediate neighbour of R, who does not
facing south, who sits at the extreme end of the row? sit at any of the extreme end.
I. W sits second to right of V. Z is not immediate II) T sits third to the left of P and both of them does not
neighbour of Y. sitting at the extreme ends. P is an immediate neighbour
II. V sits third to left of Y. W sits on the immediate right of Q. Not more than one person sits to the left of R. V sits
of X. to the right of S.

Directions (16-20): Each of these questions below 17) How is D related to A?


consist of a question and two statements I and II given I) B is the mother of E, who is the niece of C. F is the
below it. You have to decide which of these statements father-in-law of D.
are sufficient to answer the given questions. Read both II) C is the son of F. D does not have any siblings. A is the
the statements and give answer mother of C.
a) Data in statement I alone is sufficient
b) Data in statement II alone is sufficient 18) What is the code for the word ‘Place’?
I) ‘market place good for’ is coded as ‘mn tk vx az’ and
c) Data in statement I alone or statement II alone is ‘daily market in city’ is coded as ‘ vk rs tk bm’.
sufficient II) ‘good commuters are valid’ is coded as ‘st uw mn cd’
d) Data given in both statement I and statement II together and ‘valid city some issue’ is written as ‘cd po rs qn’
are not sufficient
19) Five persons M, N, O, P and Q are attending the
e) Data in both statement I and statement II together are seminar on five different days of the same week starts
sufficient from Monday to Friday. P attends the seminar on
which of the day?
Page 266 of 722

Subscribe the Xpress Video Course & Mock Test Package for Bank & Insurance Exams
If there are any suggestions/ errors in our PDFs Feel Free to contact us via this email: admin@exampundit.in
IBPS RRB Clerk Prelims – Ultra Practice Bundle PDF
I) Only one person attends the seminar between M and N. b) If the data in statement II alone are sufficient to answer
Only one person attends the seminar after Q. P attends the the question, while the data in statement I alone are not
seminar in one of the days before Q but not on Monday. sufficient to answer the question.
II) Only two persons attend the seminar after M. O attends
the seminar immediately after Q, who does not attend the c) If the data either in statement I alone or in statement II
seminar first day of the week. There are two persons alone are sufficient to answer the question.
attend the seminar between Q and N.
d) If the data in both statement I and II together are not
20) There are 40 students in a class. What is the rank sufficient to answer the question.
of K from top?
I) M is 18th rank from top. L is 13thrank from last and K e) If the data in both statement I and II together are
ranked exactly between M and L. necessary to answer the question.
II) Z is 22nd rank from top. There are only three persons
ranked between Z and K. 21) Seven persons T, L, M, H, D, Q and Z are in a
family. There are three married couples in a family.
Directions (21-22): Each of the questions below How is L related to Q?
consists of a question and two statements numbered I I) M is maternal grandfather of H. Z is brother in law of
and II given below it. You have to decide whether the D. Z has no siblings.
data provided in the statements are sufficient to II) T is only daughter in law of L. L and M are married
answer the question. Read both the statements and couples. D is maternal uncle of H.
give answer
22) Six persons A, B, E, F, G and H are arranged
a) If the data in statement I alone are sufficient to answer according to their height. Find the second shortest
the question, while the data in statement II alone are not person.
sufficient to answer the question. I) A is taller than G but shorter than F. B is taller than E
and H. Only two persons are shorter than G. F is not the
tallest. E is not the shortest.

Page 267 of 722

Subscribe the Xpress Video Course & Mock Test Package for Bank & Insurance Exams
If there are any suggestions/ errors in our PDFs Feel Free to contact us via this email: admin@exampundit.in
IBPS RRB Clerk Prelims – Ultra Practice Bundle PDF
II) More than two persons are taller than E. F is taller than 25) Who among the following was oldest among A, B,
A and G. H is shorter than E. B is taller than A. C, D, E, F?
I) A is older than C but not older than B. D is not older
Directions (23-27): Read the following information than C.
carefully and answer the questions given below. II) D was older than both E and F.
a). Only Statement I is sufficient to answer 26) P, Q, R, S, and T are the five persons in the group.
How many persons are taller than T?
b). Only Statement II is sufficient to answer I) P is taller than Q, who was only shorter than P and T.
II) R is shorter than P, who was shorter than T.
c). Either Statement I or Statement II are sufficient to
answer. 27) On which floor does C lives? If A, B, C, D, E are
living in a building. The lowermost floor of the
d). Neither Statement I nor Statement II are sufficient to building is numbered 1 its above floor numbered 2 and
answer. so on till the topmost floor of the building is numbered
5.
e). Both Statement I and Statement II are sufficient to I) A lives one of the floor below D, who lives in topmost
answer. floor.
II) E lives two floors above C, who neither lives in third
23) How many chocolate does Seetha has? Nor second floor.
I) Abi has 10 chocolates which is five more than Geetha. Direction (28-32): Each of the questions below consists
Seetha has 2 chocolates more than Swetha, who has 2 of a question and two Statements I and II given below
more than Geetha. it. You have to decide whether the data provided in the
II) Swetha has more chocolates than Geetha but less than statements are sufficient to answer the question. Read
Abi, who has 10 chocolates. both the statements and Give answer
24) On which day does Arun took his leave? a) If the data in Statement I alone are sufficient to answer
I) Arun does not took his leave on Wednesday. the question, while the data in Statement II alone are not
II) Arun took his leave either on Monday or Thursday. sufficient to answer the question

Page 268 of 722

Subscribe the Xpress Video Course & Mock Test Package for Bank & Insurance Exams
If there are any suggestions/ errors in our PDFs Feel Free to contact us via this email: admin@exampundit.in
IBPS RRB Clerk Prelims – Ultra Practice Bundle PDF
b) If the data in Statement II alone are sufficient to answer I. D is towards North of N.
the question, while the data in Statement I alone are not II. D is towards South of T which is towards West of K.
sufficient to answer the question
32) Who among B, C, D and W reached office first?
c) If the data either in Statement I alone or in Statement II I. C reached office after W but before B and D.
alone are sufficient to answer the question II. B reached office after D

d) If the data given in both the Statements I and II together Direction (33-50): Each of the following questions
are not sufficient to answer the question below consists of a question and two statements
numbered I and II given below it. You have to decide
e) If the data in both the Statements I and II together are whether the data provided in the statements are
necessary to answer the question sufficient to answer the question. Read both the
statements and give answer.
28) How is ‘of’ written in a code language?
I. ‘central bank of India’ is written as ‘5 3 9 6’ in that code a) If the data in statement I alone are sufficient to answer
language. the question, while the data in statement II alone are not
II. ‘bank of baroda’ is written as ‘7 3 5’ in that code sufficient to answer the question.
language.
b) If the data in statement II alone are sufficient to answer
29) How is D related to E? the question, while the data in statement I alone are not
I. D is sister of M and B. sufficient to answer the question.
II. E is mother of M.
c) If the data in either statement I alone or statement II
30) Among P, Q, R, S and T, who is the heaviest? alone are sufficient to answer the question.
I. R is heavier than only P.
II. S is lighter than Q but heavier than R and T. d) If the data in both the statements I and II together are
not sufficient to answer the question.
31) K is towards which direction of D?
Page 269 of 722

Subscribe the Xpress Video Course & Mock Test Package for Bank & Insurance Exams
If there are any suggestions/ errors in our PDFs Feel Free to contact us via this email: admin@exampundit.in
IBPS RRB Clerk Prelims – Ultra Practice Bundle PDF
e) If the data in both the statements I and II together are 37) Seven boxes are placed one above other which of
necessary to answer the question. the following box is placed immediately above the Box
Hx?
33) Who among Jaya, Kavya, Lokesh and Mani I) Three Box are placed between Box Bx and Fx, which is
reached home first? placed immediately above box Lx.
A) Jaya reached home after Lokesh and before Mani but II) Three box are placed between Box Lx and Box Hx.
did not second last to reach home. Box Lx is placed below box Hx.
B) Kavya reached home after Mani.
38) Who among Priyal, Queen, Rashi and Shivani is
34) Among Akansha, Banya, Caryl, Dinesh and Elvish sitting immediate right to Priyal if all persons are
each one of them has different height, who among the sitting in a straight line facing north?
following is the second shortest? I) Priyal does not sit with to Shivani who does not sit to
I) Dinesh is taller than Akansha and Banya. the extreme right.
II) Elvish is shorter than Caryl and taller than Dinesh. II) None sit to the left of Priyal and on the right of Queen,
35) There are five persons in a family namely Pz, Qz, while only one person sits between Rashi and Queen.
Rz, Sz and Tz. How is Sz related to Pz?
I) Tz is cousin of Pz and son of Qz, who married to Sz. 39) If all persons are facing same direction, then how
II) Rz is father of Pz and brother of Sz. Qz is wife of Sz. many persons are sitting in a row?
36) Among Akansha, Banya, Caryl, Dinesh, Elvish and I) Akansha is on 11th position from the left end of the row.
Farah are sitting in a circle facing center, who among Brijesh is on 8th position from the right end of the row.
the following person sit immediate right of Dinesh? II) Manish is 5th to the right of Akansha and 5th to the left
I) Akansha faces Banya. No person sits between Banya of Brijesh.
and Dinesh.
II) Dinesh sits second to the right of Akansha, Who sits 40) Who is tallest among five friends Akalya, Banya,
immediate left of Caryl. Caryl, Diya and Elvish?
I) Diya is taller than at least two of his friends
II) Elvish is taller than Akalya who is taller than Diya.

Page 270 of 722

Subscribe the Xpress Video Course & Mock Test Package for Bank & Insurance Exams
If there are any suggestions/ errors in our PDFs Feel Free to contact us via this email: admin@exampundit.in
IBPS RRB Clerk Prelims – Ultra Practice Bundle PDF
41) What is Akash’s rank in the class of 30 Students? 45) By using which statements, we can conclude “All H
I) Brijesh, who is 11th from the top in the class. Akash is are T”.
exactly between Brijesh and Caroline, who is 16 from the I) All H are S. No S is P. All S are T.
bottom. II) All H are S. Some P are S. No P are T.
II) Akash who is 10 rank above Brijesh who is 15th rank 46) How is ‘systematic’ written in a code language?
below Caroline. I) ‘systematic money process’ is written as ‘xm nu zx’ and
‘money would fun’ is written as ‘zx zy iz’ in that code
42) Towards which direction was Rashi facing as she language.
stopped running? II) ‘systematic icecream process’ is written as ‘xm nu zm’
I) Rashi run 30 meters towards West, took a left turn and in that code language
run 20 meters. She again took a left turn and stopped after
running 30 meters. 47) Priyanshi is towards which direction from Queen?
II) Rashi run 30 meters towards East, took a right turn and I) Zashn is in south of Queen and west of Srikant, who is
run 20 meters. Then she took a left turn and stopped after in north of Manish.
running 30 meters. II) Manish is south of Srikant and west of Priyanshi.

43) Who is tallest among five friends Priyanka, Queen, 48) How Az is related to Sz?
Risabh, Shivani and Taruna? A) Mz = Rz ≥ Sz > Tz ≥ Kz ≤ Lz
I) Shivani is taller than Risabh but just shorter than B) Az = Bz < Tz = Nz > Fz = Gz
Taruna.
II) Taruna is shorter than Priyanka and Queen is shorter 49) How Px is related to Mx?
than Taruna. I) Qx is daughter-in-law of Mx, who is son of Sx.
II) Tx is mother in law of Px, who is son-in-law of Ux. Ux
44) How is Kz related to Mz? has only one grand-daughter Rx, who is daughter of Qx.
I) Mz is Qz’s father’s only sister. Ux has only one child.
II) Nz is mother Kz who is the daughter of Oz.
50) Who among Az, Bz, Cz, Dz, Ez and Fz is the
shortest?
Page 271 of 722

Subscribe the Xpress Video Course & Mock Test Package for Bank & Insurance Exams
If there are any suggestions/ errors in our PDFs Feel Free to contact us via this email: admin@exampundit.in
IBPS RRB Clerk Prelims – Ultra Practice Bundle PDF
I) Dz and Cz are taller than Az but both shorter than Fz. II) Fz is shorter than Ez but taller than Bz and Dz. while
Bz is taller than Az. Az is not tallest.

14). Reasoning Data Sufficiency - Solution with Explanation

Answers (1-5): Now as per this diagram only I option will get eliminated
as we cannot determine who sits second to the right of C
1) In this question, we have to look after both the as there can be two possible cases.
statements.
I. As per the first statement diagram will be as follows: II. As per the second statement diagram will be as follows:

Now as per this diagram, we cannot determine the person


who sits second to the right of C, so the only statement 2
will get eliminated

Now after both the statements getting individually


eliminated, we will combine both these statements and the
final diagram will look like this:
Page 272 of 722

Subscribe the Xpress Video Course & Mock Test Package for Bank & Insurance Exams
If there are any suggestions/ errors in our PDFs Feel Free to contact us via this email: admin@exampundit.in
IBPS RRB Clerk Prelims – Ultra Practice Bundle PDF
__ > M > O > P > __

From this statement, it is not clear that who is the heaviest,


so this statement will get eliminated and we cannot use
this statement individually

II. As per this statement, diagram will be as follows:

M > L > O > N > P

So from this statement we can conclude that M is the


By this diagram, we can say that person who sits second
heaviest
to the right of C will be D
So, answer will be Only II
So, answer will be Both These Statements
Answer: B (Only II)
Answer: E (Both)
2) In this question, we have to look after both the
3) In this question, we have to analyse both the statements
statements
I. As per this statement only two numbers are there
I. As per this statement, we will have two following cases:
between 21 and 32 which are divisible by 6 that is 24 and
30 but we cannot conclude the number of students in the
CASE I:
class.
M can be the heaviest and P can be lightest

II. From this statement, we cannot conclude that how


M > __ > O > __ > P
many students are there in the class 10 as the difference
between class 7 and class 10 is less than nine, so difference
CASE II:
between both the classes can be any number between 1 to
M will be the second heaviest and P will be the second
8.
lightest
After getting eliminated both the options individually, we
will combine both the statements
Page 273 of 722

Subscribe the Xpress Video Course & Mock Test Package for Bank & Insurance Exams
If there are any suggestions/ errors in our PDFs Feel Free to contact us via this email: admin@exampundit.in
IBPS RRB Clerk Prelims – Ultra Practice Bundle PDF
After combining both the statements, we can conclude that 6
difference between class 7 students and class 10 students 5
is less than 9 and that number must be divisible by 6, so 4 C
there will be only one number that is 30 3
So, answer is 30 2
Answer: E (Both) 1 F

4) In this question also, we will analyse both the CASE II will get eliminated as A lives below F and this
statements. is not possible.
I. As per this statement, we will have two possible cases:

Now in this option, we cannot determine the number of


CASE I persons sitting between A and D, so this option will get
When C is living on floor number 2 eliminated individually.

Floor Name of II. As per this statement also, we will have two case and
Persons they are as follows:
6 CASE I
5 F When B is living on the topmost floor and D is living on
4 the bottommost floor
3
2 C Floor Name of
1 Persons
CASE II 6 B
When C is living on floor number 4 5 C
4
Floor Name of 3
Persons 2

Page 274 of 722

Subscribe the Xpress Video Course & Mock Test Package for Bank & Insurance Exams
If there are any suggestions/ errors in our PDFs Feel Free to contact us via this email: admin@exampundit.in
IBPS RRB Clerk Prelims – Ultra Practice Bundle PDF
1 D 5) In this question we will solve both the statements
individually.
This case will get eliminated as F lives immediately below 1) As per this statement, we will have two possible cases:
D, which is not possible.
CASE I:
CASE II When K sits at extreme left end.
When B is living on the bottom most floor and D is living
on the top most floor
Floor Name of
Persons
6 D
5 F
4 CASE II:
3 When K sits at extreme right end.
2 C
1 B

Now, this option will also get eliminated individually as


This case will get eliminated as J cannot sit second to the
we cannot conclude that where A is sitting
right K.

Now we will combine both the statements and as per both


II. As per this statement, diagram will look like this:
the statements we cannot conclude the place of A Where
A is sitting
So, answer will be both the statements are not sufficient
to answer the question.
Answer: D

Page 275 of 722

Subscribe the Xpress Video Course & Mock Test Package for Bank & Insurance Exams
If there are any suggestions/ errors in our PDFs Feel Free to contact us via this email: admin@exampundit.in
IBPS RRB Clerk Prelims – Ultra Practice Bundle PDF
As per this statement, position of L cannot be determined
as there are 2 possibilities for the position of L but through
this statement we can find, the person, who sits third from
the right end that is N
So, answer is N
Answer: B

Answers (6-10):
6) In this question we will have a look a look at both the CASE II
statements separately.
I. As per this statement, diagram will look like this:

Now as per this statement also, we cannot find the position


and the one, who sits opposite to Q, so this option will also
get eliminated individually.
Now, as per this statement we cannot find that who sits
opposite to Q, so this option will get eliminated Now, we will combine both the statements and the
individually diagram will look like this:

II. As per this statement, we will have two cases and cases CASE II:
will look like this:
CASE I:

Page 276 of 722

Subscribe the Xpress Video Course & Mock Test Package for Bank & Insurance Exams
If there are any suggestions/ errors in our PDFs Feel Free to contact us via this email: admin@exampundit.in
IBPS RRB Clerk Prelims – Ultra Practice Bundle PDF
7) In this question, we will have a look at both these
statements:
I. As per Manish’s father, Manish leaves for Mumbai after
6th and before 12th July but on the odd numbered date, so
between 6 and 12 there are three odd numbers that is 7, 9
and 11
So, this option will get eliminated individually as it is not
clear that on which date Manish leaves for Mumbai.
This case will get eliminated as after combining both the
statements, T sits immediate left of P but in this case this II. As per Manish’s wife Manish attends the interview
is not happening. after 8th July as there are ‘n’ number of possibilities that
on which date Manish will attend the interview
CASE I: So, this option will also get eliminated individually

Now, we will combine both the statements


From both these statements after 8th there are only 2 odd
number that is 9 and 11, so after combining also this is not
clear that Manish leaves for Mumbai on which date
So, answer will be both the statements are not sufficient
to answer the question
Answer: D

Now after combining both the statements, we can say that 8) In this question, we will analyse both the statements

P sits opposite to Q. separately.

So, answer is P I. As per this statement, S sits third to the right of M and

Answer: E none of them sits at end of the row.


By this statement there are multiple possibilities for the
sitting of M and S
Page 277 of 722

Subscribe the Xpress Video Course & Mock Test Package for Bank & Insurance Exams
If there are any suggestions/ errors in our PDFs Feel Free to contact us via this email: admin@exampundit.in
IBPS RRB Clerk Prelims – Ultra Practice Bundle PDF
So, this option will get eliminated individually as we Sunday
cannot decide how many people are sitting to the right of
V. CASE II:
Days Name of Persons
II) As per this statement, diagram will look like this: Monday
Tuesday
Wednesday J
Thursday
Now, if we will look at the diagram, then we can clearly Friday A
see that there are two persons sitting to the right of V Saturday
So, the answer is two Sunday
Answer: E
Now, in this statement only two persons will sit between
9) In this question, we will look after the both the S and G and as per this statement there are multiple
statements separately. possibilities for this sentence, so this option will get
I. As per this statement, we will have two possible cases eliminated individually.
and they are as follows:
II. As per this statement, we will have two possible cases
CASE I: and they will look like this:
CASE I:
Days Name of Persons Days Name of Persons
Monday A Monday S
Tuesday Tuesday
Wednesday J Wednesday
Thursday Thursday G
Friday Friday
Saturday Saturday

Page 278 of 722

Subscribe the Xpress Video Course & Mock Test Package for Bank & Insurance Exams
If there are any suggestions/ errors in our PDFs Feel Free to contact us via this email: admin@exampundit.in
IBPS RRB Clerk Prelims – Ultra Practice Bundle PDF
Sunday From this diagram, we can’t conclude who is the lightest,
so this option will get eliminated individually.
Now, in this case F will attend the exam immediately
before S, which is not possible, so this case will get II. From this statement, we can conclude this diagram:
eliminated.
_>F>_>D>_>_
CASE II:
Days Name of Persons From this diagram as well, we can’t tell that who is the
Monday lightest.
Tuesday So this option will also get eliminated individually.

Wednesday
Thursday G Now, we will combine both the options and after

Friday combining both the options we can conclude this diagram:

Saturday F E>F>B>D>C>A

Sunday S
Now, after both these statements we can conclude that A

Now, from this statement, we can find that F attends the is the lightest.

exam on Saturday. So, answer is both these statements are necessary.

So, answer is F Answer: E

Answer: B
Answers (11-15):

10) We will look into both the statements separately.


I. As per this statement diagram will look like this: 11) We will analyse both the statements separately.
I. From this statement, we can conclude that Sarath’s

E>B>C>_ exam is scheduled on Wednesday.

II. And from this statement also, we can conclude that


Sarath’s exam is scheduled on Wednesday.
Page 279 of 722

Subscribe the Xpress Video Course & Mock Test Package for Bank & Insurance Exams
If there are any suggestions/ errors in our PDFs Feel Free to contact us via this email: admin@exampundit.in
IBPS RRB Clerk Prelims – Ultra Practice Bundle PDF
So from both these statements, answer can be concluded, II. As per this statement Magesh go to abroad after 31st
so answer will either I or II statement is necessary to March and before 1st May and from this we can conclude
answer the question. that Magesh go to abroad in the month of April
Answer: C So, answer is only second statement is alone sufficient to
answer the question
12) We will look after both these statements separately. Answer: B
I. As per these statements Surpiya’s marks can be
anything after 9 till 20 14) In this question we will have a look at both the
So this statement will get eliminated individually. statements separately.
I. As per this statement diagram will look like this:
II. As per second statement, Supriya scored more marks
than 9 and there can be multiple possibilities about the
marks of her
So, this statement will also get eliminated individually
As per this diagram, we cannot conclude that in which
direction is A with respect to B
Now after combining both the statements, then also we
So, this option will get eliminated individually.
cannot conclude the marks of Supriya
So, answer will be both the statements are not sufficient
II. As per this statement diagram will look like this:
to answer the question.
Answer: D

13) In this question we will analyse each statement


separately.
I. As per this statement Magesh go to abroad in the first
half of the year, so there are 6 months available in which
As per this statement, we cannot conclude that in which
he can go.
direction is A with respect to B
So, this statement will get eliminated individually
So, this option will get eliminated individually.
Page 280 of 722

Subscribe the Xpress Video Course & Mock Test Package for Bank & Insurance Exams
If there are any suggestions/ errors in our PDFs Feel Free to contact us via this email: admin@exampundit.in
IBPS RRB Clerk Prelims – Ultra Practice Bundle PDF
As per this statement, it is not clear that who sits at
Now, we will combine both the statements and the extreme end of the row.
diagram will look like this: So this option will get eliminated individually
As per this statement, it is not clear that who sits at
extreme end of the row.
So this option will get eliminated individually

II. According to this statement, there are two possible


cases and diagrams will look like this:

CASE I:

As per seeing the diagram, we can conclude the direction


of A with respect to B and that North West
So, answer will be North West CASE II:

Answer: E

15) In this question we will analyse both the statements


separately.
I. As per this statement, diagram will look like this: After going both the cases, in one case Y is sitting at the
extreme end and in other case V is sitting.
As we cannot draw the exact conclusion that who will be
sitting at the extreme end, this option will also get
eliminated individually.

Page 281 of 722

Subscribe the Xpress Video Course & Mock Test Package for Bank & Insurance Exams
If there are any suggestions/ errors in our PDFs Feel Free to contact us via this email: admin@exampundit.in
IBPS RRB Clerk Prelims – Ultra Practice Bundle PDF
Now we will combine both these statements and the
diagram will look like this:
CASE II will get eliminated because as per first statement
Z is not immediate neighbour of Y.
CASE III:

From this statement it is not clear that who sits second left
So, answer is both the statements are necessary to answer
of S
the question.
So this option will get eliminated individually.
Answer: E

II. As per this statement there are two possible cases and
Answers (16-20):
they will look like this:

16) We will analyse both the statements separately.


CASE I:
I. As per this statement there are three cases possible and
they are as follows:

CASE I:
CASE II:

CASE II: By this statement, we cannot confirm the seat of Q and


other information is also not conclusive
So, this option will get eliminated individually.

Page 282 of 722

Subscribe the Xpress Video Course & Mock Test Package for Bank & Insurance Exams
If there are any suggestions/ errors in our PDFs Feel Free to contact us via this email: admin@exampundit.in
IBPS RRB Clerk Prelims – Ultra Practice Bundle PDF

Now we will combine both these statements and the


diagram will look like this:

Now, as per this statement we cannot establish the relation


between A and D
Now, we can conclude that R sits second to the left S. So, this option will get individually.
So, answer will be both the statement are necessary to
answer the question. Now, we will combine both the statements and the family
Answer: E tree will look like this:

17) We will look at both the statements separately.


I. As per this statement, the family tree will look like this:

Now if we look at the diagram, we cannot conclude the


relation between D and A.
So, this option will get eliminated individually. After looking at the family tree we cannot establish the
gender of D so information here is insufficient to answer
II. As per this statement, family three will look like this: the question
Answer: D

18) We will analyse both the statements separately.


Page 283 of 722

Subscribe the Xpress Video Course & Mock Test Package for Bank & Insurance Exams
If there are any suggestions/ errors in our PDFs Feel Free to contact us via this email: admin@exampundit.in
IBPS RRB Clerk Prelims – Ultra Practice Bundle PDF
I. As per this statement, we can conclude that Monday M/N
Code for Market will be tk Tuesday P
Rest codes for the words are unknown, so this option will Wednesday M/N
get eliminated individually. Thursday Q
Friday M/N
II. As per this statement, we can conclude that
Code for Valid will be cd From this statement, we can conclude that P will attend
Rest codes for the words are unknown, so this option will the seminar on Tuesday
get eliminated individually.
II. As per this statement arrangement will be like this:
Now, we will combine both the statements, and from that
we can conclude that: Days Name of
Market: tk Persons
Valid: cd Monday N
City: rs Tuesday P
Good: mn Wednesday M
Place: vx/az Thursday Q
Friday O
As the code of place is not clear, so information given here
is not sufficient.
From this statement also we can conclude that P attends
Answer: D
the seminar on Tuesday.
So, from these statements answer can be concluded, so we
19) We will look after both the statements separately
can take any of the statement to arrive at the answer.
I. As per this statement arrangement will be like this:
Answer: C

Days Name of 20) We will analyses both the statements separately.


Persons I. According to this statement diagram will look like this:

Page 284 of 722

Subscribe the Xpress Video Course & Mock Test Package for Bank & Insurance Exams
If there are any suggestions/ errors in our PDFs Feel Free to contact us via this email: admin@exampundit.in
IBPS RRB Clerk Prelims – Ultra Practice Bundle PDF
So this option will get eliminated individually

II. As per this statement, diagram will look like this:

As per this statement we can conclude the rank K from the


top.

II. As per this statement we can conclude that:


Z ranks 22nd from the top and there are three people
between Z and K, so K can be before Z and can be after Z As per this statement, we cannot establish the relationship
As this information is not clear, we can conclude that this between L and Q
statement will not be sufficient to answer the question So, this statement will get eliminated individually
Answer: A
Now, we will combine both these statements and the
Answers (21-22): family tree will look like this:

21) We will look after both the statements separately


I. As per this statement family tree will look like this:

L is mother of Q. Thus both statements I and II are


necessary to answer.
21. E (Answer)
As per this diagram we cannot conclude the relation 22) We will analyse both these statements separately.
between Q and L. I. As per this statement arrangement will look like this:
Page 285 of 722

Subscribe the Xpress Video Course & Mock Test Package for Bank & Insurance Exams
If there are any suggestions/ errors in our PDFs Feel Free to contact us via this email: admin@exampundit.in
IBPS RRB Clerk Prelims – Ultra Practice Bundle PDF
II. From this statement, we can conclude that:
B>F>A>G>E>H Abi has 10 chocolates and Swetha has less number of
chocolates than Abi, and there are multiple possibilities
So from this arrangement we can conclude that E is the for the number of chocolates for the Swetha and the
second shortest person. number of chocolates Seetha has no information in this
statement.
II. As per this statement, we can conclude that: So, this option will get eliminated individually.
Answer: A
F>A & G
E>H 24) We will look after both the statements separately
B>A I. As per this statement we cannot conclude when Arun
took his leave
From this arrangement we can conclude that who is the
second shortest person. II. As per this statement also, we cannot conclude the day
So this option will get eliminated individually. when Arun took his leave
Answer: A So, both these statements are not sufficient enough to
answer the question.
Answers (23-27): Answer: D

23) We will analyse both these statements separately. 25) In this question we will solve both the statements
I. From this statement, we can conclude that: separately.
Abi: 10 Chocolates I. According to this statement, we can conclude this:
Geetha: 5 Chocolates
Swetha: 7 Chocolates B>A>C>D
Seetha: 9 Chocolates
From this arrangement, we cannot conclude that who is
From this statement we can arrive at the answer. the oldest as the information about two persons is not
given.
Page 286 of 722

Subscribe the Xpress Video Course & Mock Test Package for Bank & Insurance Exams
If there are any suggestions/ errors in our PDFs Feel Free to contact us via this email: admin@exampundit.in
IBPS RRB Clerk Prelims – Ultra Practice Bundle PDF
So, this statement will be eliminated individually. II. According to this statement, we can conclude this:

II. According to this statement, we can conclude this: T>P>R

D> E and F From the above information it is not clear that how many
persons are taller than T
From this statement as well, we cannot conclude who is So, this option will get eliminated individually.
oldest.
So, this statement will be eliminated individually. Now we will combine both these statements and we can
conclude this:
Now we will combine both these statements and we can
conclude this: T>P>Q>R>S

B>A>C>D>E/F>E/F or

So from this arrangement it is clear that B is the oldest. T>P>Q>S>R


So, answer is both these statements are necessary to From both these cases, we can conclude that no one is
answer the question taller than T
Answer: E So, answer will be both these statements will be necessary
to answer the question
26) We will solve both the statements separately. Answer: E
I. As per this statement, we can conclude that:
T, P>Q 27) We will solve both these statements separately.
I. As per this statement, we can conclude that:
From the above information it is not clear that how many Floor Number of Persons
persons are taller than T 5 D
So, this option will get eliminated individually. 4

Page 287 of 722

Subscribe the Xpress Video Course & Mock Test Package for Bank & Insurance Exams
If there are any suggestions/ errors in our PDFs Feel Free to contact us via this email: admin@exampundit.in
IBPS RRB Clerk Prelims – Ultra Practice Bundle PDF
3 II. As per this statement we cannot get the coder of any
2 word
1 So this option will get eliminated individually
Now, we will combine both these statements, and we can
From this statement, we cannot conclude that below D on conclude that:
which floor A is living Bank/Of: 3/5
So, this option will get eliminated individually
From both these statements, we cannot get the code of
II. From this statement, we can conclude this: “Of”
Floor Number of Persons So, answer will be both the statements are not sufficient

5 to answer the question.

4 Answer: D

3 E
2 29) In this question we will analyse both the statements

1 C separately
I. As per this statement, we can conclude this:

From this statement, we can conclude that C lives on first


floor.
Answer: B From this statement, we cannot conclude the relation
between D and E

Answers (28-32): So, this option will get eliminated individually

28) We will solve both the statements separately. II. As per this statement, we can conclude this:

I. As per this statement we cannot get the coder of any


word
So this option will get eliminated individually

Page 288 of 722

Subscribe the Xpress Video Course & Mock Test Package for Bank & Insurance Exams
If there are any suggestions/ errors in our PDFs Feel Free to contact us via this email: admin@exampundit.in
IBPS RRB Clerk Prelims – Ultra Practice Bundle PDF
From this statement, we cannot conclude the relation
between D and E So, answer is Q is the heaviest
So, this option will get eliminated individually. So, we need both these statements to answer the question
Answer: E
Now, we will combine both these statements, and we can
conclude this: 31) In this question, we will solve both these statements
separately.
I. From this statement we can conclude that D is in North
of N and we cannot conclude that K is in which direction
with respect to D
So, this option will get eliminated individually.
Answer: E

II. From this statement, we can conclude this:


30) We will analyse both the statements separately
I. As per this statement, we can conclude that R is heavier
than only P and the rest are heavier than R so we cannot
conclude that who is the heaviest of them all.
So, this option will get eliminated individually.
So, from this diagram we can say that K is in North East
direction with respect to D
II. As per this statement we, can conclude this:
So, answer will be only second statement is sufficient to
Q>S>R and T
answer the question.
From this analysis, we cannot conclude who is the
Answer: B
heaviest
So this option will also get eliminated individually
32) We will look after both the statements separately
Now, we will combine both these statements and we can
I. As per this statement we can conclude this:
conclude this:

W> C> (B, D)


Q> S> T> R> P
Page 289 of 722

Subscribe the Xpress Video Course & Mock Test Package for Bank & Insurance Exams
If there are any suggestions/ errors in our PDFs Feel Free to contact us via this email: admin@exampundit.in
IBPS RRB Clerk Prelims – Ultra Practice Bundle PDF
34) In this question will have a look at both the statements
From this statement, we can conclude that W reached first. separately.
I. As per this statement, we can say that Dinesh is taller
II. From this statement, we can conclude that D reached than Akansha and Banya but we do not have any
before B but we cannot conclude any information who information about the remaining persons.
reached first. So, this option will get eliminated individually
So this option will get eliminated individually
So, answer will be only I statement is sufficient to answer II. According to this statement, we can conclude this:
the question Caryl > Elvish > Dinesh
Answer: A From this we cannot conclude that who is the second
shortest.
Answers (33-50): So this statement will get eliminated individually.

33) We will analyse both the statements separately. Now, we will combine both the statements and we can
I. As per this statement, we can conclude this: conclude this:
Caryl > Elvish > Dinesh > Akansha/Banya >
Lokesh > Jaya > Mani > Kavya Akansha/Banya
From this statement as well we cannot conclude that who
From this statement, we can say that Lokesh reached first. is the second shortest.
So, answer will be both the statements are not sufficient
II. As per this statement, we can conclude that Kavya to answer the question
reached after Mani, but we cannot say who reached first. Answer: D
So this statement will get eliminated individually
So, answer will be only I statement is sufficient to answer 35) We will analyse both the statements separately
the question I. As per this statement we can conclude this:
Answer: A

Page 290 of 722

Subscribe the Xpress Video Course & Mock Test Package for Bank & Insurance Exams
If there are any suggestions/ errors in our PDFs Feel Free to contact us via this email: admin@exampundit.in
IBPS RRB Clerk Prelims – Ultra Practice Bundle PDF

As per this statement, we cannot conclude the relation


between Sz and Pz as the gender of both these persons is
unknown.
So this option will get eliminated individually. CASE II:

II. As per this statement, we can conclude this:

From both these cases we cannot conclude that who sits


As per this statement, Sz is the uncle of Pz immediate right of Dinesh
So, answer will be only II statement is sufficient to answer So, this option will get eliminated individually
the question
Answer: B II. As per this statement, we can conclude this:

36) We will analyse each statement separately.


I. As per this statement we will have two cases:

CASE I:

Page 291 of 722

Subscribe the Xpress Video Course & Mock Test Package for Bank & Insurance Exams
If there are any suggestions/ errors in our PDFs Feel Free to contact us via this email: admin@exampundit.in
IBPS RRB Clerk Prelims – Ultra Practice Bundle PDF
Answer: E

37) We will analyse both these statements separately.


I. As per this statement we get multiple possibilities.
From this statement, we cannot conclude that which box
is immediately above the box Hx.
So, this option will get eliminated

From this statement also, we cannot conclude who sits II. As per this statement, we can conclude this:
immediate right of Dinesh
So, this option will get eliminated individually.
Boxes Number Boxes Boxes Boxes
7 Hx
Now, we will combine both these statements and we can
6 Hx
conclude that after combining as well Banya will sits to
5 Hx
immediate right of Dinesh.
4
3 Lx
2 Lx
1 Lx

From this statement also, we cannot conclude that which


box is immediately above Hx
So, this option will get eliminated individually

Now, we will combine both these statements and we can


conclude this:
So, answer will be both these statements are necessary to Boxes Number Boxes Boxes
answer the question. 7 Bx

Page 292 of 722

Subscribe the Xpress Video Course & Mock Test Package for Bank & Insurance Exams
If there are any suggestions/ errors in our PDFs Feel Free to contact us via this email: admin@exampundit.in
IBPS RRB Clerk Prelims – Ultra Practice Bundle PDF
6 Hx Bx So, answer will be only II statement is sufficient to answer
5 Hx the question
4 Answer: B
3 Fx
2 Lx Fx 39) We will analyse the statements separately
1 Lx I. As per this statement we can conclude this:

From this statement, we can conclude this, that Bx lives


immediately above Hx
Now in this statement, we cannot conclude that is there
So, answer will be both these statements are necessary to
any between Akansha and Brijesh or not
answer the question.
So, this option will get eliminated individually.
Answer: E
II. As per this statement we can conclude this:

38) In this question we will solve each statement


separately
I. As per this statement, we cannot conclude that who sits
immediate right of Priyal Now from this statement, we cannot conclude the number

So, this option will get eliminated individually of persons sitting in the row.
So, this option will get eliminated individually.

II. As per this statement, we can conclude this:


Now, we will combine both the statements and we can
conclude this:

Now as per this statement, we can conclude that Rashi sits


to the immediate right of Priyal. Now, from this statement we can conclude the number of
persons sitting in the row.
Page 293 of 722

Subscribe the Xpress Video Course & Mock Test Package for Bank & Insurance Exams
If there are any suggestions/ errors in our PDFs Feel Free to contact us via this email: admin@exampundit.in
IBPS RRB Clerk Prelims – Ultra Practice Bundle PDF
So, answer will be both the statements will be needed to 41) We will analyse both the statements separately.
answer the question I. As per this statement, we can conclude this:
Answer: E

40) We will analyse the statements separately.


I. As per this statement, we cannot conclude that who is
the tallest as information about other persons is not given As per this statement, we can count the rank of Akansha
except Diksha. that is 13th rank
So, this option will get eliminated individually.

II. From this statement it is not clear that where is Caryl


II. As per this statement we can conclude this: in the class
Elvish > Akansha > Diksha So, this option will get eliminated individually
Now from this statement we cannot conclude who is the So, answer will be only I is sufficient to answer the
tallest question
So this option will get eliminated individually Answer: A

Now, we will combine these two statements and we can 42) We will analyse both the statements separately
conclude this: I. As per this question we can conclude this:

Elvish > Akansha > Diksha > Banya/Caryl >


Banya/Caryl

Now from this statement, we can conclude that Elvish is


the tallest.
So, answer will be both the statement will be necessary to
answer the question. As per this statement, we can conclude that Rashi is facing
Answer: E East direction.

Page 294 of 722

Subscribe the Xpress Video Course & Mock Test Package for Bank & Insurance Exams
If there are any suggestions/ errors in our PDFs Feel Free to contact us via this email: admin@exampundit.in
IBPS RRB Clerk Prelims – Ultra Practice Bundle PDF
II. As per this statement we can conclude this: From this statement, we cannot conclude that who is the
tallest as information about other persons is not mentioned
So, this option will get eliminated individually

Now, we will combine both these statements and we can


conclude this:

Priyanka > Taruna> Shivani/Queen > Shivani/ Queen


As per this statement, we can conclude that Rashi is facing
> Rishabh
in East direction
So, answer will be we can any of the statements to answer
From this statement, we can conclude that Priyanka is the
the question
tallest
Answer: C
So, answer will be both these statements are necessary to
answer the question
43) In this question we will analyse both the statements
Answer: E
separately.
I. As per this statement, we can conclude that:
44) We will take into consideration both the statements
separately
Taruna > Shivani > Rishabh
I. As per this statement we can conclude this:

From this statement, we cannot conclude that who is the


tallest as information about other persons is not mentioned
So, this option will get eliminated individually

II. As per this statement, we can conclude this:


From this statement we cannot establish any relation
between Kz and Mz
Priyanka > Taruna > Queen
So, this option will get eliminated individually
II. As per this statement, we can conclude this:
Page 295 of 722

Subscribe the Xpress Video Course & Mock Test Package for Bank & Insurance Exams
If there are any suggestions/ errors in our PDFs Feel Free to contact us via this email: admin@exampundit.in
IBPS RRB Clerk Prelims – Ultra Practice Bundle PDF

As per this family tree, we cannot conclude any


relationship between Kz and Mz
So, this option will get eliminated individually
According to this statement, we cannot say that All H are
T.
Now, we will combine both these statements and we
So, this statement will get eliminated individually
cannot establish any common relationship between both
So, answer will be only I statement is sufficient to answer
the statements.
the question.
So, answer will be both the statements are not sufficient
Answer: A
to answer the question
46) We will analyse both the statements separately
Answer: D
I. As per this statement, we can collude this:
Code for Money: ZX
45) We will analyse both the statements separately
I. As per this statement we can conclude this:

From this statement, we can’t derive the code for


Systematic
So, this option will get eliminated individually

II. According to the second statement, we cannot derive


the code for any word.
So, this option will get eliminated individually
From this statement, we can say that All H are T
Now, we will combine both these statements and we can
II. According to this statement we can conclude this: conclude this:
Page 296 of 722

Subscribe the Xpress Video Course & Mock Test Package for Bank & Insurance Exams
If there are any suggestions/ errors in our PDFs Feel Free to contact us via this email: admin@exampundit.in
IBPS RRB Clerk Prelims – Ultra Practice Bundle PDF
Code for Money: ZX So, this statement will get eliminated individually
Code for Systematic/Process: XM/NU
From, this also we can’t get the code for Systematic Now, we will combine both these statements and we can
So, answer will be both the statements are not sufficient conclude this:
to answer the question
Answer: D

47) We will analyse both the statements separately.


I. As per this statement, we can conclude this:

From this statement, we can conclude that Priyanshi is in


South East direction from Queen.
So, answer will be both these statements are necessary to
answer the question
Answer: E
As per this statement, there is no information about
Priyanshi 48) We will analyse both the statements separately
So, this statement will get eliminated individually I. According to this statement, we can conclude this:

II. As per this statement, we can conclude this:


Mz = Rz ≥ Sz > Tz ≥ Kz ≤ Lz

From this statement, we cannot draw any relation between


Az and Mz
So, this statement will get eliminated individually.
According to this statement, we don’t have the
II. According to this statement, we can conclude this:
information of Queen
Az = Bz < Tz = Nz > Fz = Gz
Page 297 of 722

Subscribe the Xpress Video Course & Mock Test Package for Bank & Insurance Exams
If there are any suggestions/ errors in our PDFs Feel Free to contact us via this email: admin@exampundit.in
IBPS RRB Clerk Prelims – Ultra Practice Bundle PDF
So, this statement will get eliminated individually.
From this statement, we cannot draw any relation between
Az and Mz II. We can conclude this from this statement:
So, this statement will get eliminated individually.

Now we will combine both these statements and we can


conclude this:

Az = Bz < Tz < Sz < Rz = Mz


From this statement, we cannot conclude the relationship
From this statement we can conclude that Az is smaller between Px and Mx
than Mz So, this statement will get eliminated individually.
So, answer will be both these statements will be needed to
answer the question Now we will combine both the statements and we can
Answer: E conclude this:

49) In this question we will solve each statement


separately.
I) According to this statement, we can conclude this:

From this diagram, we can conclude that Px is grandson


of Mx
So, answer will be both these statements are necessary to
From this statement, we cannot conclude the relationship
answer the question
between Px and Mx
Answer: E
Page 298 of 722

Subscribe the Xpress Video Course & Mock Test Package for Bank & Insurance Exams
If there are any suggestions/ errors in our PDFs Feel Free to contact us via this email: admin@exampundit.in
IBPS RRB Clerk Prelims – Ultra Practice Bundle PDF

50) We will analyse both the statements separately From this statement, we cannot conclude that who is the
I. According to this statement, we can conclude that: shortest
So, this statement will get eliminated individually.
Fz > Dz and Cz > Az
Bz > Az Now, we will combine both these statements and we can
conclude that:
From this statement, we cannot conclude that who is the
shortest Ez > Fz >> Dz/Cz > Az/Bz
So, this statement will get eliminated individually.
But from this statement we cannot conclude anything
II. As per this statement we can conclude that: So, answer will be both the statements are not sufficient
to answer the question
Ez > Fz > Bz and Dz. Answer: D

15). Days Based Puzzle


Direction (1-5): Study the following information born immediately after the one who works in Dehradun.
carefully and answer the questions given below: The one who works in Bengaluru was born immediately
Seven persons A, B, C, D, E, F and G work in seven before the one who works in Ranchi. C was born two days
different cities i.e. Patna, Palampur, Kochi, Bengaluru, before the one who works in Patna. Both D and F were
Ranchi, Dehradun and Nagpur but not in the same order. born on one of the days after the one who works in Kochi.
They were born on seven different days from Monday to Number of persons born between B and one who works in
Sunday of the same week but again not in the same order. Palampur is same as the number of persons born between
A works in Kochi and was born on Tuesday. Only two B and G. B was born one of the days before G. E works in
persons born were between D and F but D was born on Ranchi and was born on Friday.
one of days before F. The one who works in Nagpur was 1) Who among the following works in Dehradun?

Page 299 of 722

Subscribe the Xpress Video Course & Mock Test Package for Bank & Insurance Exams
If there are any suggestions/ errors in our PDFs Feel Free to contact us via this email: admin@exampundit.in
IBPS RRB Clerk Prelims – Ultra Practice Bundle PDF
a. C b. D – Bengaluru
b. F c. G – Nagpur
c. D d. E – Wednesday
d. G e. B – Saturday
e. None of these Direction (6-10): Study the following information
2) Who among the following was born three days carefully and answer the questions given below.
before E? Seven people L, M, N, O, P, Q and R belong to seven
a. The one who works in Palampur different city/state Chennai, Bihar, Delhi, Odisha, Bengal,
b. The one who works in Bengaluru UP and Punjab. As they belong to different backgrounds
c. The one who works in Kochi they believe in different gods and they offer their respect
d. The one who works in Ranchi by keeping fast for their respective gods on different days
e. None of these of a week from Monday to Sunday.
3) Who among the following was born on Monday? O keeps his fast three days after the person who belongs
a. The one who works in Patna to UP but two days before N. M belongs to either Delhi or
b. The one who works in Ranchi UP. R does not belong to Delhi. L keep his fast two days
c. The one who works in Bengaluru after M but two days before Q, who belongs to Bihar. The
d. The one who works in Palampur person who belongs to Bengal keeps his fast three days
e. None of these after P. The persons who belong to Chennai and Odisha
4) ___ was born on Thursday and works in ___? keep their fast on the first and the last day of the week. R
a. B, Bengaluru keeps his fast four days before N.
b. C, Dehradun 6) Who belongs to Delhi?
c. D, Bengaluru a. M
d. E, Ranchi b. L
e. None of these c. P
5) Four of the following five are related to each other d. O
in some way and thus formed a group. Choose the one e. None of the above
which does not belong to that group. 7) Who keeps his fast on Thursday?
a. C - Tuesday a. M
Page 300 of 722

Subscribe the Xpress Video Course & Mock Test Package for Bank & Insurance Exams
If there are any suggestions/ errors in our PDFs Feel Free to contact us via this email: admin@exampundit.in
IBPS RRB Clerk Prelims – Ultra Practice Bundle PDF
b. O Sunday to Saturday of a week but not in the same order.
c. L One person attended classes only for one day.
d. N Bob attends the classes after Eva. Number of persons
e. Q attends the classes between Diana and Eva is one less than
8) The person who belongs to Odisha keeps his fast on the number of persons attends between Diana and Bob.
which day of the week? Only one person attends classes between Abhay and
a. Monday Cherry, but Cherry does not attend the classes on Sunday.
b. Tuesday Number of persons attending classes before Diana is equal
c. Friday to the number of persons attending classes after Geet. Bob
d. Saturday attends classes immediately before Faizal. Abhay attends
e. Can’t be determined classes before Wednesday, but Abhay does not attend on
9) R belongs to which state? Sunday. Faizal attends classes after Diana.
a. Delhi 11) Who attends the classes on Sunday?
b. Punjab a. Cherry
c. Odisha b. Geet
d. UP c. Eva
e. Chennai d. Bob
10) The person who keeps his fast on the second day of e. None of these
the week belongs to which state? 12) How many persons attend classes between Bob and
a. UP Diana?
b. Delhi a. Two
c. Odisha b. Three
d. Punjab c. Four
e. Bihar d. Five
Direction (11-15): Study the following information e. None of these
carefully and answer the questions given below. 13) Who attends the classes immediately before
Seven persons Abhay, Bob, Cherry, Diana, Eva, Faizal Cherry?
and Geet attended music classes in seven days from a. Eva
Page 301 of 722

Subscribe the Xpress Video Course & Mock Test Package for Bank & Insurance Exams
If there are any suggestions/ errors in our PDFs Feel Free to contact us via this email: admin@exampundit.in
IBPS RRB Clerk Prelims – Ultra Practice Bundle PDF
b. Geet for twice number of days than that of V. Q does not go to
c. Bob Chennai. P does not go for 7 or 2 days. The one, who goes
d. Diana to Bangalore, goes for more days than that of R. V goes
e. Faizal for odd number of days.
14) Which of the following statement is true? 16) Who goes for 3 days?
a. Geet attends the classes on Monday a. P
b. Cherry attends the classes immediately after Faizal b. S
c. Faizal attends the classes on Thursday c. T
d. Abhay attends the classes before Eva d. V
e. Only one person attends the classes between Bob and e. None of these
Cherry 17) To which city does S go?
15) Number of persons, who attend the classes before a. Mumbai
Cherry is one more than the number of persons attend b. Pune
the classes after______. c. Chennai
a. Eva d. Bangalore
b. Diana e. Cannot be determined
c. Geet 18) Who goes for maximum number of days?
d. Faizal a. P
e. Bob b. Q
Direction (16-20): Study the following information c. U
carefully and answer the questions given below. d. V
Seven persons P, Q, R, S, T, U and V go to different cities e. None of these
among Nagpur, Pune, Mumbai, Delhi, Bangalore, 19) Who goes to Chennai?
Chennai and Lucknow but not necessarily in same order. a. P
Each of them goes for different days from 1 to 7. b. Q
T goes to Lucknow for 5 days. R goes for 2 more days c. V
than that of S. V goes to Nagpur. U goes for 1 day. Neither d. U
S nor U goes to Delhi. The one, who goes to Chennai, goes e. None of these
Page 302 of 722

Subscribe the Xpress Video Course & Mock Test Package for Bank & Insurance Exams
If there are any suggestions/ errors in our PDFs Feel Free to contact us via this email: admin@exampundit.in
IBPS RRB Clerk Prelims – Ultra Practice Bundle PDF
20) Who goes for 2 days? d. The one who is learning German
a. R e. None of the above
b. T 22) Who learns Japanese?
c. S a. V
d. P b. P
e. None of these c. S
Direction (21-25): Study the following information d. Q
carefully and answer the questions given below. e. None of the above
Seven friends P, Q, R, S, T, U and V attend different 23) How many persons learn the languages between
language classes, namely French, English, Spanish, the one who learns Spanish and T?
Sanskrit, Chinese, German and Japanese, but not in the a. One
same order from Monday to Sunday in a week. b. Four
P attends a class on Friday. Only two friends attend the c. Two
class between P and the one who is learning Sanskrit. U d. Three
attends the class immediately before S. Neither U nor S is e. None of the above
learning Sanskrit. Only one person attends a class between 24) ___ attends the class immediately before U.
U and the one who is learning Spanish. The one who is a. P
learning Spanish does not have a class on Monday. Q b. The one who learns Sanskrit
attends the class immediately before the one who is c. V
learning French. P is not learning French. Only one person d. The one who learns German
has a class between U and T, where T is above U. R is e. None of the above
learning German. Only two persons have classes between 25) Who attends the class two days after the one who
R and the one who is learning Chinese. T is not learning learns French?
Japanese. a. The one who learns Japanese
21) Who attends the class on Tuesday? b. The one who learns Chinese
a. T c. The one who learns English
b. The one who is learning Japanese d. The one who learns German
c. Q e. The one who learns Spanish
Page 303 of 722

Subscribe the Xpress Video Course & Mock Test Package for Bank & Insurance Exams
If there are any suggestions/ errors in our PDFs Feel Free to contact us via this email: admin@exampundit.in
IBPS RRB Clerk Prelims – Ultra Practice Bundle PDF
e. Friday
Direction (26-30): Read the following information 29) On which day does B visit the doctor?
carefully and answer the questions that follow. a. Monday
There are seven friends A, B, C, D, E, F and G who visit b. Tuesday
an eye doctor every week. However, each one goes on a c. Wednesday
different day of the week, starting from Monday. No two d. Thursday
persons go to the doctor on the same day. e. Friday
A visits the doctor on Monday. D’s turn to visit the doctor 30) How many persons visit the doctor before E?
comes at the very last. Only four persons visit the doctor a. 1
before C. G visits the doctor after C. B’s turn comes after b. 2
E but before F. c. 3
26) Who visits the doctor on Tuesday? d. 4
a. A e. 5
b. B Directions (31-35): Study the following information
c. C and answer the given below questions.
d. D Seven lectures are scheduled to be held in a week. There
e. E is only one lecture on each of the seven days of the week,
27) Who visits the doctor on Saturday? starting from Monday to Sunday. Chemistry is taught
a. E either on Wednesday or on Saturday.
b. F Three lectures are scheduled between Chemistry and
c. G English in that order. Two lectures are scheduled to be
d. D held between English and Computers. Lecture on Physics
e. B is scheduled on the day which is immediately next to the
28) On which day does F visit the doctor? day when lecture on Biology is scheduled. Lecture on
a. Monday Psychology is scheduled to be held after Mathematics.
b. Tuesday Lecture on Psychology is not scheduled on Saturday or
c. Wednesday Sunday.
d. Thursday
Page 304 of 722

Subscribe the Xpress Video Course & Mock Test Package for Bank & Insurance Exams
If there are any suggestions/ errors in our PDFs Feel Free to contact us via this email: admin@exampundit.in
IBPS RRB Clerk Prelims – Ultra Practice Bundle PDF
31) Which of the following lectures is scheduled for 35) On which of the following days is the lecture on
Thursday? Chemistry scheduled?
a. English a. Monday
b. Mathematics b. Sunday
c. Biology c. Saturday
d. Computers d. Wednesday
e. Physics e. Thursday
32) On which of the following days is the lecture on Direction (36-40): Study the following information
Psychology scheduled? carefully and answer the questions given below it.
a. Friday There are seven friends A, B, C, D, E, F and G. Each one
b. Monday goes on a trip on different days from Monday to Sunday
c. Tuesday but not necessarily in the same order.
d. Thursday C travels on the day after the day B travels. G travels on
e. Sunday the day just before the day on which E travels. The number
33) Which of the following combinations of day – of persons between the one who travels on Wednesday
lecture is correct? and B is the same as the number of persons between D and
a. Saturday – Physics G. F travels on Wednesday. E travels on the day which is
b. Monday – Biology 4 days after the day on which D travels. D doesn’t travel
c. Tuesday – English on the day after the day on which F travels.
d. Thursday – English 36) Who goes on Sunday?
e. Sunday – Computers a. B
34) Which of the following lectures is scheduled for b. A
Sunday? c. G
a. Mathematics d. C
b. English e. None of these
c. Biology 37) On which day D travels?
d. Physics a. Tuesday
e. None of these b. Friday
Page 305 of 722

Subscribe the Xpress Video Course & Mock Test Package for Bank & Insurance Exams
If there are any suggestions/ errors in our PDFs Feel Free to contact us via this email: admin@exampundit.in
IBPS RRB Clerk Prelims – Ultra Practice Bundle PDF
c. Sunday P does not move to Goa. Two persons moved between the
d. Monday persons, who moved to Mumbai and Delhi. Q moved to
e. None of these Patna just after S, who does not move on Friday or
38) How many persons go after G? Tuesday. T moves to Nagpur three days after the person,
a. One who moved to Goa. V moved to Bengaluru on Sunday. R
b. Two moves before S’s moving day. U moved after Q but not to
c. Three Mumbai. R does not move to Mumbai or Goa.
d. Four 41) Who moved to Chennai?
e. More than four a. P
39) Who does go on Tuesday? b. R
a. G c. S
b. A d. T
c. C e. Cannot be determined
d. D 42) How many persons moved before the moving day
e. None of these of U?
40) How many persons goes between C and the one a. Three
who goes on Thursday? b. Four
a. Three c. Five
b. One d. One
c. Two e. None of these
d. Four 43) S moved to _________.
e. More than four a. Chennai
Direction (41-45): Study the following information b. Patna
carefully and answer the questions given below it. c. Nagpur
Seven persons P, Q, R, S, T, U and V moved to different d. Goa
cities on different days of a week from Monday to Sunday. e. None of these
Each of them moved to the cities among Delhi, Nagpur, 44) How many persons moved after the one, who
Bengaluru, Goa, Mumbai, Chennai and Patna. moved to Patna?
Page 306 of 722

Subscribe the Xpress Video Course & Mock Test Package for Bank & Insurance Exams
If there are any suggestions/ errors in our PDFs Feel Free to contact us via this email: admin@exampundit.in
IBPS RRB Clerk Prelims – Ultra Practice Bundle PDF
a. Two d. Pari
b. Three e. None of these
c. Four 47) How many persons have meeting before Tarun’s
d. One meeting?
e. None of these a. Five
45) S moved on ________. b. Three
a. Monday c. Four
b. Wednesday d. One
c. Saturday e. None of these
d. Sunday 48) Quam has meeting on ______.
e. Tuesday a. Tuesday
Direction (46-50): Study the following information b. Saturday
carefully and answer the questions given below it. c. Wednesday
Seven persons Pari, Quan, Rinku, Samraat, Tarun, d. Thursday
Utkarsh and Vijay have meetings on different days of a e. None of these
week from Monday to Sunday. 49) Who among the following has meeting on Monday?
At least two persons have meeting before the meeting day a. Pari
of Rinku. Utkarsh has meeting two days after Quam but b. Vijay
not on Thursday or Wednesday. Three persons have c. Quam
meeting between the meeting day of Rinku and Samraat. d. Utkarsh
Rinku does not have meeting on Sunday. Quam does not e. None of these
have meeting on Friday. Vijay has meeting just before 50) How many persons have meeting between Pari’s
Pari but not on Tuesday. and Samraat’s meeting?
46) Who among the following has meeting on a. Three
Saturday? b. Five
a. Utkarsh c. Four
b. Samraat d. One
c. Rinku e. None of these
Page 307 of 722

Subscribe the Xpress Video Course & Mock Test Package for Bank & Insurance Exams
If there are any suggestions/ errors in our PDFs Feel Free to contact us via this email: admin@exampundit.in
IBPS RRB Clerk Prelims – Ultra Practice Bundle PDF

15). Days Based Puzzle - Solution and Detailed Explanation


Solutions 1-5 Saturday
1. A works in Kochi and was born on Tuesday. Only two Sunday F
persons born between D and F but D was born on one of 4. The one who works in Nagpur was born immediately
days before F. after the one who works in Dehradun.
2. Both D and F were born one of the days after the one 5. C was born two days before the one who works in Patna.
who works in Kochi. E works in Ranchi and was born on Case I: When D was born on Wednesday:
Friday. Day Person City
3. The one who works in Bengaluru was born immediately Monday C
before the one who works in Ranchi. Tuesday A Kochi
Case I: When D was born on Wednesday: Wednesday D Patna
Day Person City Thursday Bengaluru
Monday Friday E Ranchi
Tuesday A Kochi Saturday F Dehradun
Wednesday D Sunday Nagpur
Thursday Bengaluru Case II: When D was born on Thursday:
Friday E Ranchi Day Person City
Saturday F Monday C
Sunday Tuesday A Kochi
Case II: When D was born on Thursday: Wednesday Patna
Day Person City Thursday D Bengaluru
Monday Friday E Ranchi
Tuesday A Kochi Saturday Dehradun
Wednesday Sunday F Nagpur
Thursday D Bengaluru
Friday E Ranchi

Page 308 of 722

Subscribe the Xpress Video Course & Mock Test Package for Bank & Insurance Exams
If there are any suggestions/ errors in our PDFs Feel Free to contact us via this email: admin@exampundit.in
IBPS RRB Clerk Prelims – Ultra Practice Bundle PDF
6. Number of persons born between B and one who works 2) The persons who belong to Chennai and Odisha keep
in Palampur is same as the number of persons born their fast either on the first or the last day of the week.
between B and G. 3) M belongs to either Delhi or UP.
7. B was born on one of the days before G, which is not (It clearly implies M does not keep his fast on Monday as
possible in case II, so case II is invalid. he is neither from Chennai nor Odisha, also Q does not
The final arrangement is as follows: keep his fast on Sunday because he is from Bihar and not
Day Person City from Chennai or Odisha (Which means M can’t be having
Monday C Palampur his fast on Wednesday as Q can’t have his fast on Sunday).
Tuesday A Kochi Implies M fast on Tuesday, L on Thursday and Q on
Wednesday D Patna Saturday.)
Thursday B Bengaluru Monday Person City/State
Friday E Ranchi Monday Chennai/Odisha
Saturday F Dehradun Tuesday M Delhi/UP
Sunday G Nagpur Wednesday
Answers: Thursday L
1. b Friday
2. c Saturday Q Bihar
3. d Sunday Chennai/Odisha
4. a
5. c 4) O keeps his fast three days after the person who belongs
Solutions 6-10 to UP but two days before N.
1) L keep his fast two days after M but two days before Q, (Let us suppose that M is from Delhi and the person who
who belongs to Bihar. keep his fast on Wednesday of from UP, then O would be
(Implies Q keeps his fast 4 days after M, hence there are having his fast on Saturday and two days before N, which
three possibilities here which are, M keeps his fast either is not possible as Saturday is the second last day of the
on Monday, Tuesday or Wednesday.) week. Hence our assumption was wrong, the person from
UP can’t have fast on Wednesday or any other day after

Page 309 of 722

Subscribe the Xpress Video Course & Mock Test Package for Bank & Insurance Exams
If there are any suggestions/ errors in our PDFs Feel Free to contact us via this email: admin@exampundit.in
IBPS RRB Clerk Prelims – Ultra Practice Bundle PDF
Wednesday. Implies M is from UP and O keeps his fast Monday P Chennai/Odisha
on Friday and N keeps his fast two days after that, i.e. on Tuesday M UP
Sunday.) Wednesday R Punjab
Monday Person City/State Thursday L Bengal
Monday Chennai/Odisha Friday O Delhi
Tuesday M UP Saturday Q Bihar
Wednesday Sunday N Chennai/Odisha
Thursday L Answers:
Friday O 6. d
Saturday Q Bihar 7. c
Sunday N Chennai/Odisha 8. e
9. b
5) R keeps his fast four days before N. 10. a
(It’s clear that N keeps his fast on Sunday, implies R keeps Solutions 11-15
his fast on Wednesday.) 1. Abhay attends classes before Wednesday, but Abhay
6) The person who belongs to Bengal keeps his fast three does not attend on Sunday, which means Abhay attends
days after P. either on Monday or Tuesday.
(There is only one place left for P to be placed that is on 2. Only one person attends classes between Abhay and
Monday, implies L belongs to Bengal and keeps his fast Cherry.
on Thursday.) Case I:
7) R does not belong to Delhi. Day persons
(As there are only and two options for R and Delhi is one Sunday
of them, implies the other option is the answer, i.e. Punjab. Monday Abhay
Also as Delhi is left to be mapped, we can say O belongs Tuesday
to Delhi as O is the only person left to be mapped to a Wednesday Cherry
state). Thursday
Monday Person City/State Friday

Page 310 of 722

Subscribe the Xpress Video Course & Mock Test Package for Bank & Insurance Exams
If there are any suggestions/ errors in our PDFs Feel Free to contact us via this email: admin@exampundit.in
IBPS RRB Clerk Prelims – Ultra Practice Bundle PDF
Saturday Answers:
11. c
Case II: 12. a
Day Person 13. d
Sunday 14. e
Monday 15. c
Tuesday Abhay Solutions 16-20
Wednesday 1. T goes Lucknow for 5 days.

Thursday Cherry 2. R goes for 2 more days than that of S.

Friday 3. V goes to Nagpur. U goes for 1 day.

Saturday 4. The one, who goes to Chennai, goes for twice number

3. Number of persons attend classes before Diana is equal of days than that of V.

to the number of persons attend classes after Geet. 5. V goes for odd number of days.

4. Bob attends classes immediately before Faizal. i.e. V=3 and Chennai=6 (2*3=6)

5. Bob attends the classes after Eva. R=2+S

6. Number of persons attend between Diana and Eva is (Case-1, S=2 and R=4)

one less than the number of persons attend between Diana (Case-2, S=4 and R=6)

and Bob. This is not possible in case II. So, we have two cases,

So, the final arrangement is as follows. Case 1:.

Day Person Persons Cities Days

Sunday Eva P

Monday Abhay Q

Tuesday Diana R 4

Wednesday Cherry S 2

Thursday Geet T Lucknow 5

Friday Bob U 1

Saturday Faizal V Nagpur 3

Page 311 of 722

Subscribe the Xpress Video Course & Mock Test Package for Bank & Insurance Exams
If there are any suggestions/ errors in our PDFs Feel Free to contact us via this email: admin@exampundit.in
IBPS RRB Clerk Prelims – Ultra Practice Bundle PDF
Case 2: 20. c
Persons Cities Days Solutions 21-25
P 1. Only two friends attend the class between P and the
Q one who is learning Sanskrit.
R Chennai 6 2. U attends the class immediately before S.
S 4 3. Neither U nor S is learning Sanskrit. Only one person
T Lucknow 5 attends a class between U and the one who is learning

U 1 Spanish.

V Nagpur 3 4. Only one person has a class between U and T.

5. Q does not go to Chennai. P does not go for 7 or 2 days. So, U attends immediately before S which means U either

So, case 2 is rejected. attends on Wednesday or Saturday.

7. The one, who goes to Bangalore, goes for more days Case I:

than that of R. Days Persons Languages

8. Neither S nor U goes to Delhi. So, we have, Monday T

Persons Cities Days Tuesday Sanskrit

P Chennai 6 Wednesday U

Q Bangalore 7 Thursday S

R Delhi 4 Friday P Spanish

S Mumbai/Pune 2 Saturday

T Lucknow 5 Sunday

U Pune/Mumbai 1 Case II:

V Nagpur 3 Days Persons Languages

Answers: Monday

16. d Tuesday Sanskrit


17. e Wednesday
18. b Thursday T Spanish
19. a Friday P

Page 312 of 722

Subscribe the Xpress Video Course & Mock Test Package for Bank & Insurance Exams
If there are any suggestions/ errors in our PDFs Feel Free to contact us via this email: admin@exampundit.in
IBPS RRB Clerk Prelims – Ultra Practice Bundle PDF
Saturday U 25. e
Sunday S Solutions26-30
5. R is learning German. Only two persons have classes 1) A visits the doctor on Monday.
between R and the one who is learning Chinese. 2) D’s turn to visit the doctor comes at the very last.
6. T is not learning Japanese. (Thus D visits the doctor on Sunday)
7. Q attends immediately before the one who is learning Day Person
French. Monday A
So, in case II Q must attend the class on Tuesday which Tuesday
means R attends on Monday but then we can’t fix the one Wednesday
who is learning Chinese which means case II is invalid. Thursday
Now, in case I Q must attend on Tuesday which means R Friday
attends on Sunday so, that the one who learns Chinese can Saturday
attend the class on Thursday. Sunday D
The final arrangement is as follows:
Days Persons Languages 3) Only four persons visit the doctor before C.
Monday T English Day Person
Tuesday Q Sanskrit Monday A
Wednesday U French Tuesday
Thursday S Chinese Wednesday
Friday P Spanish Thursday
Saturday V Japanese Friday C
Sunday R German Saturday
Answers: Sunday D
21. c
22. a 4) G visits the doctor after C.
23. d (Since, D visits the doctor at last on Sunday, thus G must
24. b visit the doctor on Saturday)

Page 313 of 722

Subscribe the Xpress Video Course & Mock Test Package for Bank & Insurance Exams
If there are any suggestions/ errors in our PDFs Feel Free to contact us via this email: admin@exampundit.in
IBPS RRB Clerk Prelims – Ultra Practice Bundle PDF
Day Person 1) Chemistry is taught either on Wednesday or on
Monday A Saturday. Three lectures are scheduled between
Tuesday Chemistry and English in that order.
Wednesday Therefore, Chemistry is taught on Wednesday and English
Thursday is taught on Sunday.
Friday C Day Lecture
Saturday G Monday
Sunday D Tuesday
Wednesday Chemistry
5) B’s turn comes after E but before F. Thursday
Day Person Friday
Monday A Saturday
Tuesday E Sunday English
Wednesday B
Thursday F 2) Two lectures are scheduled to be held between English
Friday C and Computers.

Saturday G So, Computers is taught on Thursday.

Sunday D Day Lecture

Answers: Monday

26. e Tuesday
27. c Wednesday Chemistry
28. d Thursday Computers
29. c Friday
30. a Saturday
Solutions 31-35 Sunday English

Page 314 of 722

Subscribe the Xpress Video Course & Mock Test Package for Bank & Insurance Exams
If there are any suggestions/ errors in our PDFs Feel Free to contact us via this email: admin@exampundit.in
IBPS RRB Clerk Prelims – Ultra Practice Bundle PDF
3) Lecture on Physics is scheduled on the day which is Monday Biology
immediately next to the day when lecture on Biology is Tuesday Physics
scheduled. Wednesday Chemistry
Case i) Thursday Computers
Day Lecture Friday Mathematics
Monday Biology Saturday Psychology
Tuesday Physics Sunday English
Wednesday Chemistry
Thursday Computers Case ii)
Friday Day Lecture
Saturday Monday Mathematics
Sunday English Tuesday Psychology
Wednesday Chemistry
Case ii) Thursday Computers
Day Lecture Friday Biology
Monday Saturday Physics
Tuesday Sunday English
Wednesday Chemistry 5) Lecture on Psychology is not scheduled on Saturday or
Thursday Computers Sunday.
Friday Biology The above condition is not satisfied in case i.
Saturday Physics Therefore, the final arrangement will be as shown below:
Sunday English

4) Lecture on Psychology is scheduled to be held after


Mathematics.
Case i)
Day Lecture

Page 315 of 722

Subscribe the Xpress Video Course & Mock Test Package for Bank & Insurance Exams
If there are any suggestions/ errors in our PDFs Feel Free to contact us via this email: admin@exampundit.in
IBPS RRB Clerk Prelims – Ultra Practice Bundle PDF
Day Lecture Thursday
Monday Mathematics Friday
Tuesday Psychology Saturday
Wednesday Chemistry Answers: Sunday
Thursday Computers 31. d
Friday Biology 32. c 3) E travels on the day which is 4 days after the day on
Saturday Physics 33. a which D travels.
Sunday English 34. b 4) The number of persons between the one who travels on
35. d Wednesday and B is the same as the number of persons
Solutions 36-40 between D and G.
1) F travels on Wednesday. 5) G travels on the day just before the day on which E
2) D doesn’t travel on the day after the day on which F travels.
travels. Case 1:
Case 1: Day Person
Day Person Monday D
Monday D Tuesday
Tuesday Wednesday F
Wednesday F Thursday G
Thursday Friday E
Friday Saturday B
Saturday Sunday
Sunday Case 2:
Case 2: Day Person
Day Person Monday
Monday Tuesday D
Tuesday D Wednesday F
Wednesday F Thursday

Page 316 of 722

Subscribe the Xpress Video Course & Mock Test Package for Bank & Insurance Exams
If there are any suggestions/ errors in our PDFs Feel Free to contact us via this email: admin@exampundit.in
IBPS RRB Clerk Prelims – Ultra Practice Bundle PDF
Friday G 2. Q moved to Patna just after S, who does not move on
Saturday E Friday or Tuesday.
Sunday 3. R moves before S’s moving day.
C goes on Sunday. So there will be two cases
Case 1 Case 2
Using statement 5, Case 2 will be eliminated as there is a Days of a Person Cities Person Cities
gap of 2 days between D and G. F travels on Wednesday. Week s s
So, B will travel on Saturday. But E travels on Saturday. Monday
6) C travels on the day after the day B travels. Tuesday Goa
Day Person Wednesda S S Goa
Monday D y
Thursday Q Patna Q Patna
Tuesday A
Friday T
Wednesday F
Saturday T
Thursday G
Sunday V Bengalur V Bengalur
Friday E
u u
Saturday B
4. Two persons moved between the persons, who moved
Sunday C
to Mumbai and Delhi.
Answers:
5. U moved after Q but not to Mumbai.
36. d
6. R does not move to Mumbai or Goa.
37. d
7. P does not move to Goa.
38. c
Days of a Week Persons Cities
39. b
Monday R Chennai
40. c
Tuesday P Mumbai
Solutions 41-45
Wednesday S Goa
1. T move to Nagpur three days after the person, who
Thursday Q Patna
moved to Goa.
Friday U Delhi

Page 317 of 722

Subscribe the Xpress Video Course & Mock Test Package for Bank & Insurance Exams
If there are any suggestions/ errors in our PDFs Feel Free to contact us via this email: admin@exampundit.in
IBPS RRB Clerk Prelims – Ultra Practice Bundle PDF
Saturday T Nagpur 5. Utkarshhave meeting two days after Quam but not on
Sunday V Bengaluru Thursday or Wednesday.
Answers: Case 1 Case 2 Case 3
41. b
Days Persons Persons Persons
42. b
43. d Monday Samraat
44. b Tuesday Samraat
45. b
Wednesday Rinku Quam
Solutions 46-50
1. At least two persons have meeting before the meeting Thursday Quam Quam
day of Rinku.
Friday Rinku Utkarsh
2. Three persons have meeting between the meeting day
of Rinku and Samraat. Saturday Utkarsh Utkarsh Rinku

3. Rinku does not have meeting on Sunday. Sunday Samraat


Case 1 Case 2 Case 3 6. Vijay has meeting just before Pari but not on Tuesday.

Days Persons Persons Persons So, Case 2 and case 3 are rejected.

Days Persons
Monday Samraat
Monday Vijay
Tuesday Samraat
Tuesday Pari
Wednesday Rinku
Wednesday Rinku
Thursday
Thursday Quam
Friday Rinku
Friday Tarun
Saturday Rinku
Saturday Utkarsh
Sunday Samraat

4. Quam does not have meeting on Friday. Sunday Samraat

Page 318 of 722

Subscribe the Xpress Video Course & Mock Test Package for Bank & Insurance Exams
If there are any suggestions/ errors in our PDFs Feel Free to contact us via this email: admin@exampundit.in
IBPS RRB Clerk Prelims – Ultra Practice Bundle PDF
Answers: 48. d
46. a 49. b
47. c 50. c

16). Parallel Row Seating Arrangement


Directions (1-5): Study the given information D) S
carefully and answer the following questions: E) None of these
Twelve persons are sitting in a parallel row. In row 1 six 2. Which among the following does not belong to the
person i.e. A, B, C, D, E and F are sitting and all of them group?
facing North. In row 2, six-person i e. P, Q, R, S, T and U A) R
are sitting and all of them facing South. B) U
Either A or B sits at the end of the row and the person C) Q
between A and B whoever sits at the corner faces R. One D) A
person sits between R and S who does not sit at the end of E) D
the row. P sits to the immediate right of S. Two person sit 3. Who among the following faces E?
between P and T. B and F are immediate neighbours. C A) U
faces the person who sits immediate right of U. E does not B) R
sit at the end of the row. C) S
Three persons sit between A and F. C sits to the left of D D) Person sits between R and S
and to the right of A. D faces the person sits to the E) None
immediate left of T. 4. Which among the following pair sits diagonally
1. Who sits third to the right of T? opposite to each other?
A) R A) R, D
B) One who faces the person who sits immediate left of B) A, D
A C) A, Q
C) One who faces the person who sits to the immediate D) Both A and B
left of C E) Both A and C
Page 319 of 722

Subscribe the Xpress Video Course & Mock Test Package for Bank & Insurance Exams
If there are any suggestions/ errors in our PDFs Feel Free to contact us via this email: admin@exampundit.in
IBPS RRB Clerk Prelims – Ultra Practice Bundle PDF
5. Which among the following statements is/ are true? A) Dog, who is sitting on to the immediate right of vacant
A) B sits between C and F chair.
B) S sits third to the left of Q B) Immediate left to the Teddy
C) D sits third to the right of E C) Both A and B
D) Both A and C D) Archie
E) All are true E) None of These
7) Who is facing Ollie?
A) Charlie
Directions (6-10): Study the given information B) Max
carefully and answer the following questions: C) Buddy
14 chairs are placed in two parallel rows and each row has D) Oscar
7 chairs. Chairs from one row are facing towards north and E) None as the chair is vacant
chairs from another row are facing south. Each chair from 8) Who is sitting on a chair which is placed 3rd to the
either row is facing towards one chair from another row. left of Teddy’s chair?
10 dogs are sitting on these chairs and each row has two A) Charlie
vacant chairs. B) Immediate right to one of the vacant chairs
Buddy is sitting on a chair 3rd to the left of chair of Max, C) Ollie
who is facing vacant chair. In only one row, both vacant D) Both B and C
chairs are placed adjacent to each other. Neither of the E) None of These
vacant chair is placed at the end of the row. Charlie is 9) Who among the following is sitting 2nd to the right
sitting to the immediate right of Max and is facing Archie. of any vacant chair?
Buddy is not facing North direction. Milo is sitting on a A) Teddy
chair which is 2nd to the left of chair of Buddy. Teddy is B) Ollie
facing Oscar, who is sitting on the adjacent chair of C) Archie
Buddy. Toby and Jack are sitting on a chair adjacent to D) Milo
Teddy. Ollie is sitting on one of the chairs. E) Oscar
6) Who among the following is facing Buddy? 10) How many persons are facing vacant chairs?
A) One
Page 320 of 722

Subscribe the Xpress Video Course & Mock Test Package for Bank & Insurance Exams
If there are any suggestions/ errors in our PDFs Feel Free to contact us via this email: admin@exampundit.in
IBPS RRB Clerk Prelims – Ultra Practice Bundle PDF
B) Two D) Surbhi, Yuvraj and Vivek
C) Three E) Tina, Uttara and Qimat
D) Four 12) Which is this statement being correct with respect
E) More Than Four to the position of Xesus?
Directions (11-15): Study the given information A) Opposite to Uttara
carefully and answer the following questions: B) Second to the right of Ritu
12 friends Omkar, Pradeep, Qimat, Ritu, Surbhi, Tina, C) Third to the left of Vivek
Uttara, Vivek, Waqar, Xesus, Yuvraj and Zeus went for a D) To the immediate right of Qimat
dinner that has rows A-N (in sequence) with six seats per E) To the immediate left of Tina
row. 13) Who among the following male is sitting between
Row A is closest to the screen. The group completely two girls?
occupied rows D and E and all they faced the screen. A) Yuvraj
There are five girls and seven boys in this group but none B) Xesus
of the girls occupy the corner seats. It is also known that: C) Zeus
Omkar, Pradeep, Waqar and Zeus occupy the corner seats. D) Tina
Uttara is to the immediate left of Pradeep and opposite E) Ritu
Ritu. However, Uttara can see Ritu but Ritu cannot see 14) Who are the neighbours of Tina?
Uttara. Xesus is second to the left of Waqar and third to A) Vivek and Pradeep
the right of Omkar. Vivek is to the right of Zeus and B) Vivek and Xesus
opposite to Qimat. Yuvraj, who is opposite Surbhi, is the C) Vivek and Uttara
neighbour of Omkar. In row D, the girls have occupied the D) Yuvraj and Xesus
third and fifth seats from the left. In row E, Uttara, is the E) None of These
only girl sitting opposite to a girl. There are three girls in 15) If the following people exchange places (Yuvraj
row E. with Uttara, Xesus with Tina and Vivek with Waqar),
11) Which of these groups sits in the same row? who is third to the right of Surbhi?
A) Yuvraj, Ritu and Xesus A) Omkar
B) Omkar, Yuvraj and Tina B) Qimat
C) Zeus, Surbhi and Ritu C) Ritu
Page 321 of 722

Subscribe the Xpress Video Course & Mock Test Package for Bank & Insurance Exams
If there are any suggestions/ errors in our PDFs Feel Free to contact us via this email: admin@exampundit.in
IBPS RRB Clerk Prelims – Ultra Practice Bundle PDF
D) Waqar A) Theo
E) Yuvraj B) Wiber
C) Arne
Directions (16-20): Study the given information D) Nathan
carefully and answer the following questions: E) None of These
14 friends Adrian, Gerad, Nathan, Arne, Felix, Issac, 18) Who was seated exactly in the middle of 1st row?
Kane, Joshua, Theo, Samuel, Wiber, Elvin, Simon and A) Theo
Owen were seated in two parallel rows. 7 persons were B) Gerad
seated in the 1st row while remaining were seated in the C) Owen
2nd row. Every person from 2nd row was seated exactly D) Felix
behind one of the persons from the 1st row. All of the E) Arne
persons were facing in the same direction that is North. 19) How many persons were seated to the left of
Theo was seated exactly behind the person, who was 2nd Nathan (Only in the row in which Nathan was seated)?
to the left of Arne. Arne is sitting 3rd to the left of Kane. A) One
Neither Theo nor Felix seated at any extreme end. Gerad B) Two
was at a gap of one person from Arne. Adrian was seated C) Three
to the immediate left of Gerad. Adrian, Joshua and Owen D) Four
were seated at the end of the row. Wiber was seated behind E) More Than Four
Arne. Neither Wiber nor Arne was the neighbour of Issac. 20) Who was seated 2nd to the right of Issac?
Simon was the neighbour of Owen. Samuel was seated 2nd A) Arne
to the left of Elvin and was seated behind Nathan. B) Adrian
16) Find the odd one out? C) Joshua
A) Joshua D) Wiber
B) Adrian E) No one
C) Owen
D) Simon Directions (21-25): Study the given information
E) Samuel carefully and answer the following questions:
17) Who was seated 2nd to the left of Felix?
Page 322 of 722

Subscribe the Xpress Video Course & Mock Test Package for Bank & Insurance Exams
If there are any suggestions/ errors in our PDFs Feel Free to contact us via this email: admin@exampundit.in
IBPS RRB Clerk Prelims – Ultra Practice Bundle PDF
Twelve persons are sitting in a parallel row. Person of row B) Raju and No one sits in front of Nikki
1 i.e. Raju, Arya, Ravi, Nikki, Siya and Mahi faces North C) Raju and Pinky
direction and the person of row 2 i.e. Alia, Disha, Varun, D) Arya and No one sits in front of Nikki
Tejas, Dev and Pinky are facing South direction. One seat E) None of these
on each row is vacant. 24) Which of the following pair sit at the end of the ròw
There are three seats between Siya and Raju. One of the 1 and row 2 respectively?
seats between Raju and Siya, who sits at the corner of the A) Alia and Siya
row, is vacant. Raju faces Pinky who sits third to the right B) Tejas and Arya
of Disha. There is only one seat between Mahi and Ravi. C) Arya and Alia
There are only two seats between Varun and Alia who sits D) Both A and C
at the end of the row. Varun and Dev are immediate E) All of the above
neighbours. Tejas faces the person who sits to the 25) Who faces the person who sits third to the right of
immediate right of Ravi. Alia does not face Nikki. Disha?
21) Which among the following pair is/are an A) Person sits fifth from the right end
immediate neighbour of the vacant seat? B) Mahi
A) Pinky- Varun C) Person sits immediate left of Nikki
B) Ravi-Mahi D) Both A and B
C) Alia- Pinky E) Both A and C
D) Both A and C Directions (26-30): Study the given information
E) Both B and C carefully and answer the following questions:
22) Who sits exactly at the middle of the row? Nine persons Gautam, Garuda, Garv, Girish, Gaurav,
A) Varun Ganga, Gitesh, Ganesh and Gopal sit in a room in two
B) Person sits to the immediate right of Raju parallel rows having six seats each, given that each seat in
C) Person sits to the immediate left of Ravi row one is behind the other seat of second row. They are
D) Both A and B facing towards front (or North) and each one sits on
E) Both A and C different seats and the remaining are left vacant. At least
23) Nikki sits to the immediate left of___ and faces___. one seat in each row is vacant.
A) Arya and Alia
Page 323 of 722

Subscribe the Xpress Video Course & Mock Test Package for Bank & Insurance Exams
If there are any suggestions/ errors in our PDFs Feel Free to contact us via this email: admin@exampundit.in
IBPS RRB Clerk Prelims – Ultra Practice Bundle PDF
Ganesh sits immediately behind a vacant seat. Gaurav is A) Immediate Left
sitting at any extreme right end and two persons sit B) Second to the Left
between Gaurav and Girish. Immediate neighbour of C) Immediate Behind
Ganesh is facing Girish’ s seat. Garv doesn’t sit in the D) Immediate Right
same row as Gaurav, but sits to the immediate right of a E) Third to the Right
vacant seat. Gautam sits in immediate front of Ganga, who 29) How many seats are vacant between Gautam and
is sitting second to the left of Garuda. Gitesh sits second Gopal?
to the right of the seat which is immediately behind Gopal. A) One
No two vacant seats are placed immediately front or B) None
behind each other. C) Gautam and Gopal are not in the same row
26) Who sits to the immediate left of Gitesh? D) Two
A) Garuda E) Cannot Be Determined
B) Garv 30) How many persons are sitting between Ganga and
C) Girish Garv?
D) Gaurav A) One
E) Gautam B) Two
27) Who among the following sit/sits at the extreme C) Three
ends of the row? D) Four
I. Gautam E) None of These
II. Garv Directions (31-35): Study the given information
III. Ganga carefully and answer the following questions:
A) Only I Fourteen persons are sitting in a parallel row. Person of
B) Only II row1 i.e. Babita, Sonu, Tappu, Tarak, Madhvi, Komal and
C) Both I and II Roshan are facing North and person of row 2 i.e. Daya,
D) Both II and III Goli, Gogi, Jetha, Anjali, Sodhi and Iyer are facing South.
E) Both I and III Two person sits between Goli and Daya who sits at the
28) What is the position of Garuda with respect to middle of the row. Only one person sits between Roshan
Girish? and Sonu who sits at the end of the row. Tarak faces Daya.
Page 324 of 722

Subscribe the Xpress Video Course & Mock Test Package for Bank & Insurance Exams
If there are any suggestions/ errors in our PDFs Feel Free to contact us via this email: admin@exampundit.in
IBPS RRB Clerk Prelims – Ultra Practice Bundle PDF
Three persons sit between Gogi and Sodhi who is an A) Roshan
immediate neighbour of Daya. Babita sits third from the B) Anjali
left end. Iyer and Anjali are immediate neighbours of each C) Sodhi
other. Iyer faces Roshan. Tappu and Madhavi are D) Iyer
immediate neighbours of each other. E) Jetha
31) Who among the following faces Goli? 35) __ faces the person sits second to the left of Daya.
A) Person sits immediate left of Madhavi A) Sonu
B) Tappu B) Komal
C) Person sits diagonally opposite to Gogi C) Babita
D) All of the above D) Roshan
E) Cannot be determined E) Madhavi
32) If Tappu sits at the corner of the row then who
among the following sits third to the right of Madhavi? Directions (36-40): Study the given information
A) Person faces Daya carefully and answer the following questions:
B) Person faces Anjali Twelve friends Z, Y, X, W, V, U, T, S, R, Q, P and O were
C) Person faces Iyer seated in two rows such that five of them were in row 1
D) Person sits at the end of the row and the rest were seated in row 2. The 2 rows started at an
E) None of these equal gap from the western side wall, thus, the five seats
in row 1 were exactly in line with the first five seats and
33) Which among the following pair sits immediate in front of row 2. They were seated facing the North.
right and immediate left of each other respectively? U was seated 3rd from one of the corners. R was to the
A) Sonu and Komal immediate right of U. Z was seated 3rd to the right of Q,
B) Sodhi and Daya who was seated exactly behind Y. Y was to the immediate
C) Tarak and Babita right of either P or S. X was 2nd to the left of T. Number
D) Jetha and Goli of persons seated to the left of O was same as the number
E) Iyer and Anjali of persons to the right of Y and they were seated in
34) Who among the following does not belong to the different rows. The person seated exactly behind W, who
group? was 2nd to the left of V. Number of persons seated to the
Page 325 of 722

Subscribe the Xpress Video Course & Mock Test Package for Bank & Insurance Exams
If there are any suggestions/ errors in our PDFs Feel Free to contact us via this email: admin@exampundit.in
IBPS RRB Clerk Prelims – Ultra Practice Bundle PDF
left of Q was equal to the number of persons to the right E) Nobody
of S. 40) How many persons were seated between Z and O?
36) Four of the following bears a similar relationship A) None
and hence form a group, who among the following is B) One
not a part of that group? C) Two
A) P D) Three
B) S E) Cannot Be Determined
C) V
D) X Directions (41-45): Study the given information
E) W carefully and answer the following questions:
37) Who among the following was seated 3rd to the left Twelve people are sitting in two parallel rows, containing
of S? six people in each row, such that they are equidistant from
A) Nobody each other. In row 1, D, A, E, B, F and C, are seated and
B) Z facing South, but not necessarily in the same order. In row
C) Q 2, M, I, J, L, N and K are seated facing towards the North,
D) U but not necessarily in the same order.
E) T M sits third to the left of L, who faces C. F, who faces K,
38) What was the position of S with respect to V? sits third to the right of E. N sits second to the left of M.
A) Immediate Left D faces a person, who is not an immediate neighbour of J.
B) Immediate Right A sit third to the left of B. I do not face E.
C) Second to the Left 41) Who among the following are the neighbours of A?
D) Second to the Right A) Person, who sits 2nd to the left of B
E) Third to the Left B) Person, who sits immediate right of C
39) Who was seated exactly in front of O? C) Person, who sits immediate left of F
A) P D) All A, B and C
B) S E) None of These
C) R 42) What is the Position of K with respect to J?
D) W A) Immediate Right
Page 326 of 722

Subscribe the Xpress Video Course & Mock Test Package for Bank & Insurance Exams
If there are any suggestions/ errors in our PDFs Feel Free to contact us via this email: admin@exampundit.in
IBPS RRB Clerk Prelims – Ultra Practice Bundle PDF
B) Third to the Left Twelve people are sitting in two parallel rows, containing
C) Second to the Right six people in each row, such that they are equidistant from
D) Immediate Right each other. In row 1, Q, N, P, M, O and R, are seated and
E) None of These facing South, but not necessarily in the same order. In row
43) Four of the following five are alike in a certain way 2, W, X, Y, Z, V and U are seated facing towards the
and forms a group. Which among the following does North, but not necessarily in the same order.
not belong to that group? R faces W. M does not face Z, who sits to the left of V but
A) Immediate Right of F not to the immediate left of V. Both P and Y are sitting at
B) C the extreme right ends of the row. Three persons it
C) L between Y and U, who does not face M. X sits to the
D) N immediate left of W but does not face O. Two persons sit
E) M between Q and M but none of them sit at extreme ends of
44) Who among the following sits second to the right the line. The one, who faces N sits second to the right of
of C? Z. M does not face U.
A) Person who sits third to the left of B 46) Who among the following faces N?
B) Person who sits second to the left F A) Z
C) Both A and B B) X
D) M C) Y
E) None of These D) U
45) Number of persons to the left of I is same as the E) None of These
number of persons the right of ____. 47) Who sits 2nd to the right of M?
A) B A) P
B) A B) Q
C) D C) O
D) C D) N
E) E E) None of These
Directions (46-50): Study the given information 48) How many persons are sitting between the one,
carefully and answer the following questions: who faces Z and R?
Page 327 of 722

Subscribe the Xpress Video Course & Mock Test Package for Bank & Insurance Exams
If there are any suggestions/ errors in our PDFs Feel Free to contact us via this email: admin@exampundit.in
IBPS RRB Clerk Prelims – Ultra Practice Bundle PDF
A) Two 50) Who among the following faces O?
B) Three A) Y
C) None of These B) Z
D) One C) U
E) Four D) W
49) What is the position of X with respect to Y? E) None of These
A) Third to the Right
B) Immediate Left
C) Third to the Left
D) Immediate Right
E) Second to the Left

16). Parallel Row Seating Arrangement - Answer with Explanation


Answers (1-5): As per this statement, there will be four cases getting
framed and they are as follows:
1) C
2) B CASE I:
3) D
4) E
5) A

Solutions (1-5):

STEP I: Either A or B sits at the end of the row and the


person between A and B whoever sits at the corner faces
CASE II:
R.

Page 328 of 722

Subscribe the Xpress Video Course & Mock Test Package for Bank & Insurance Exams
If there are any suggestions/ errors in our PDFs Feel Free to contact us via this email: admin@exampundit.in
IBPS RRB Clerk Prelims – Ultra Practice Bundle PDF

As per this statement, the arrangement will look like this:

CASE I:

CASE III:

CASE II:

CASE IV:

CASE III:

STEP II: One person sits between R and S who does not
sit at the end of the row.

Page 329 of 722

Subscribe the Xpress Video Course & Mock Test Package for Bank & Insurance Exams
If there are any suggestions/ errors in our PDFs Feel Free to contact us via this email: admin@exampundit.in
IBPS RRB Clerk Prelims – Ultra Practice Bundle PDF

CASE IV: CASE II:

STEP III: P sits to the immediate right of S. Two person


STEP IV: D faces the person sits to the immediate left of
sit between P and T.
T.

As per these statements, CASE III and CASE IV will get


As per this statement, the arrangement will look like this:
eliminated and we will continue with CASE I and CASE
II and they both will look like this:
CASE I:

CASE I:

Page 330 of 722

Subscribe the Xpress Video Course & Mock Test Package for Bank & Insurance Exams
If there are any suggestions/ errors in our PDFs Feel Free to contact us via this email: admin@exampundit.in
IBPS RRB Clerk Prelims – Ultra Practice Bundle PDF

CASE II: STEP VI: B and F are immediate neighbours. C faces the
person who sits immediate right of U.

As per these statements, the final arrangement will look


like this:

CASE I:

STEP V: Three persons sit between A and F.

As per this statement, CASE II will get eliminated, and


we will continue with CASE I and it will look like this:

CASE I:

Answers (6-10):

6) C
7) E
Page 331 of 722

Subscribe the Xpress Video Course & Mock Test Package for Bank & Insurance Exams
If there are any suggestions/ errors in our PDFs Feel Free to contact us via this email: admin@exampundit.in
IBPS RRB Clerk Prelims – Ultra Practice Bundle PDF
8) D
9) A
10) B

Solutions (6-10):

Step II: Milo is sitting on a chair which is 2nd to the left


Step I: Buddy is sitting on a chair 3rd to the left of chair
of chair of Buddy. Teddy is facing Oscar, who is sitting
of Max, who is facing vacant chair. Charlie is sitting to the
on the adjacent chair of Buddy.
immediate right of Max and is facing Archie. Buddy is not
facing North direction. In only one row, both vacant chairs
As per these statements it can be concluded that Milo is
are placed adjacent to each other. Neither of the vacant
sitting at the leftmost position in the row facing South.
chair is placed at the end of the row.

CASE I:
As per these statements there will be two cases and they
are as follows:

CASE I:

CASE II:

CASE II:

Page 332 of 722

Subscribe the Xpress Video Course & Mock Test Package for Bank & Insurance Exams
If there are any suggestions/ errors in our PDFs Feel Free to contact us via this email: admin@exampundit.in
IBPS RRB Clerk Prelims – Ultra Practice Bundle PDF
Step III: Toby and Jack are sitting on a chair adjacent to 14) C
Teddy. In only one row, both vacant chairs are placed 15) E
adjacent to each other. Neither of the vacant chair is Solutions (11-15):
placed at the end of the row.
Step I: Omkar, Pradeep, Waqar and Zeus occupy the
As per these statements, CASE I will be eliminated. So, corner seats. Uttara is to the immediate left of Pradeep and
both vacant chairs which are facing North and placed at opposite Ritu. However, Uttara can see Ritu but Ritu
gap of 1 chair from each other. cannot see Uttara.

As per these statements, all of them are facing the screen,


assume that the screen is in North. Pradeep and Waqar
have to be at extreme right corners. Hence, Omkar and
Zeus are at extreme left corners. Since, Uttara can see Ritu
but Ritu cannot see Uttara, then Uttara is immediate
behind Ritu. Since, Uttara is to the immediate left of
CASE II:
Pradeep, Pradeep is in second row that is Row E.

Thus, the sitting arrangement will be:

Answers (11-15):

Step II: Xesus is second to the left of Waqar and third to


11) A
the right of Omkar.
12) D
13) B

Page 333 of 722

Subscribe the Xpress Video Course & Mock Test Package for Bank & Insurance Exams
If there are any suggestions/ errors in our PDFs Feel Free to contact us via this email: admin@exampundit.in
IBPS RRB Clerk Prelims – Ultra Practice Bundle PDF
As per this statement, Omkar is in row D and Zeus is in
row E.
Thus, the sitting arrangement becomes:

Now, Omkar, Zeus, Waqar and Pradeep are definitely


males.
Since, third and fifth girl from the left in row D are
females, Qimat and Ritu are females.

Step III: Yuvraj, who is opposite Surbhi, is the neighbour Since, Uttara is the only girl sitting opposite to a girl,

of Omkar. Uttara and Ritu are females.


Since Qimat is a female, Vivek has to be the male (as per

As per this statement, the arrangement becomes: the above statement).


Since, there are three females in row E, Surbhi and Tina
are females (Apart from Uttara).
Hence, in row D, Yuvraj and Xesus are males.

Thus, the final arrangement will be:

Step IV: Vivek is to the right of Zeus and opposite


Omkar.

As per this statement it can be concluded that this situation


is only possible when Qimat is between Yuvraj and Xesus.
Thus, the arrangement becomes:

Answers (16-20):

Page 334 of 722

Subscribe the Xpress Video Course & Mock Test Package for Bank & Insurance Exams
If there are any suggestions/ errors in our PDFs Feel Free to contact us via this email: admin@exampundit.in
IBPS RRB Clerk Prelims – Ultra Practice Bundle PDF
16) B
17) D
18) E
19) B
20) E

Solutions (16-20): Step III: Wiber was seated behind Arne. Neither Wiber
nor Arne was the neighbour of Issac. Simon was the
Step I: Theo was seated exactly behind the person, who neighbour of Owen.
was 2nd to the left of Arne. Arne is sitting 3rd to the left of
Kane. Neither Theo nor Felix seated at any extreme end. As per these statements, the arrangement will look like
this:
As per these statements, the diagram will look like this:

Step IV: Samuel was seated 2nd to the left of Elvin and
was seated behind Nathan.
Step II: Gerad was at a gap of one person from Arne.
Adrian was seated to the immediate left of Gerad. Adrian, As per this statement, the final arrangement will look like
Joshua and Owen were seated at the end of the row. this:

As per these statements, we can conclude that Adrian must


be seated at the leftmost position in the 1st row and the
arrangement will look like this:

Page 335 of 722

Subscribe the Xpress Video Course & Mock Test Package for Bank & Insurance Exams
If there are any suggestions/ errors in our PDFs Feel Free to contact us via this email: admin@exampundit.in
IBPS RRB Clerk Prelims – Ultra Practice Bundle PDF

Answers (21-25):
CASE II:

21) E
22) D
23) B
24) E
25) A

Solutions (21-25):

CASE III:
STEP I: There are three seats between Siya and Raju. One
of the seats between Raju and Siya who sits at the corner
of the row is vacant.

As per these statements, there will be six cases being


framed and they are as follows:

CASE I:
CASE IV:

Page 336 of 722

Subscribe the Xpress Video Course & Mock Test Package for Bank & Insurance Exams
If there are any suggestions/ errors in our PDFs Feel Free to contact us via this email: admin@exampundit.in
IBPS RRB Clerk Prelims – Ultra Practice Bundle PDF
As per this statement, CASE I, CASE II and CASE III
will get eliminated and we will continue with the
remaining cases and the arrangements will look like this:

CASE IV:

CASE V:

CASE V:

CASE VI:

CASE VI:

STEP II: Raju faces Pinky who sits third to the right of
Disha.

Page 337 of 722

Subscribe the Xpress Video Course & Mock Test Package for Bank & Insurance Exams
If there are any suggestions/ errors in our PDFs Feel Free to contact us via this email: admin@exampundit.in
IBPS RRB Clerk Prelims – Ultra Practice Bundle PDF

STEP III: There are only two seats between Varun and CASE V:
Alia who sits at the end of the row.

As per this statement, the arrangement will look like this:

CASE IV:

CASE V (A):

CASE IV (A):

CASE VI:

Page 338 of 722

Subscribe the Xpress Video Course & Mock Test Package for Bank & Insurance Exams
If there are any suggestions/ errors in our PDFs Feel Free to contact us via this email: admin@exampundit.in
IBPS RRB Clerk Prelims – Ultra Practice Bundle PDF

CASE VI (A):
CASE V:

STEP IV: Varun and Dev are immediate neighbours.


CASE V (A):
As per this statement, the arrangement will look like this:

CASE IV:

CASE VI:

CASE IV (A):
Page 339 of 722

Subscribe the Xpress Video Course & Mock Test Package for Bank & Insurance Exams
If there are any suggestions/ errors in our PDFs Feel Free to contact us via this email: admin@exampundit.in
IBPS RRB Clerk Prelims – Ultra Practice Bundle PDF

CASE IV (A):

CASE VI (A):

CASE V:
STEP V: There is only one seat between Mahi and Ravi.
Tejas faces the person who sits to the immediate right of
Ravi.

As per these statements, CASE IV, CASE IV (A), CASE


V (A), CASE VI and CASE VI (A) will get eliminated
and we will continue with the remining cases and they will
look like this:
CASE V (A):
CASE IV:

Page 340 of 722

Subscribe the Xpress Video Course & Mock Test Package for Bank & Insurance Exams
If there are any suggestions/ errors in our PDFs Feel Free to contact us via this email: admin@exampundit.in
IBPS RRB Clerk Prelims – Ultra Practice Bundle PDF
CASE V:

CASE VI:

Answers (26-30):

26) B
27) E
28) C
CASE VI (A): 29) A
30) B

Solutions (26-30):

Step I: Gaurav is sitting at any extreme right end and only


one person sits between Gaurav and Girish. Ganesh sits
immediately behind a vacant seat. Immediate neighbour
of Ganesh is facing Girish’ s seat.
STE VI: Alia does not face Nikki.

As per these statements, we can conclude that Gaurav can


As per this statement, the final arrangement will look like
sit on the extreme right of either first row or second row.
this:
Ganesh sits in the row behind the front row, and Girish
Page 341 of 722

Subscribe the Xpress Video Course & Mock Test Package for Bank & Insurance Exams
If there are any suggestions/ errors in our PDFs Feel Free to contact us via this email: admin@exampundit.in
IBPS RRB Clerk Prelims – Ultra Practice Bundle PDF
sits in the front row, hence Gaurav also sits in the front CASE IV:
row. The sitting arrangement and the cases will be like
this:

CASE I:

Step II: Gautam sits in immediate front of Ganga, who is


sitting second to the left of Garuda.

As per this statement, the arrangement will be like this:

CASE II: CASE I:

CASE III: CASE II:

CASE III:

Page 342 of 722

Subscribe the Xpress Video Course & Mock Test Package for Bank & Insurance Exams
If there are any suggestions/ errors in our PDFs Feel Free to contact us via this email: admin@exampundit.in
IBPS RRB Clerk Prelims – Ultra Practice Bundle PDF
CASE I:

CASE IV:

CASE II:

CASE V:

CASE III:

Step III: Garv doesn’t sit in the same row as Gaurav, but
sits to the immediate right of a vacant seat. No two vacant
seats are placed immediately front or behind each other. CASE IV:

As per these statements, we can conclude that Garv


doesn’t sit in the front row. And CASE IV will get
eliminated and the arrangement will look like this this:

Page 343 of 722

Subscribe the Xpress Video Course & Mock Test Package for Bank & Insurance Exams
If there are any suggestions/ errors in our PDFs Feel Free to contact us via this email: admin@exampundit.in
IBPS RRB Clerk Prelims – Ultra Practice Bundle PDF
32) C
33) D
34) A
35) B

Solutions (31-35):
CASE V:

STEP I: Two person sits between Goli and Daya who sits
at the middle of the row.

As per this statement, there will be two cases being framed


and they will look like this:

Step IV: Gitesh sits second to the right of the seat which CASE I:

is immediately behind Gopal.

As per this statement, CASE I, CASE II and CASE III will


get eliminated. So, final arrangement will look like this:

CASE II:

Answers (31-35):

31) E

Page 344 of 722

Subscribe the Xpress Video Course & Mock Test Package for Bank & Insurance Exams
If there are any suggestions/ errors in our PDFs Feel Free to contact us via this email: admin@exampundit.in
IBPS RRB Clerk Prelims – Ultra Practice Bundle PDF
STEP II: Only one person sits between Roshan and Sonu
who sits at the end of the row.

As per this statement, the arrangement will look like this:

CASE I:

CASE II (A):

CASE I (A):

STEP III: Tarak faces Daya.

As per this statement, the arrangement will look like this:

CASE I:

CASE II:

Page 345 of 722

Subscribe the Xpress Video Course & Mock Test Package for Bank & Insurance Exams
If there are any suggestions/ errors in our PDFs Feel Free to contact us via this email: admin@exampundit.in
IBPS RRB Clerk Prelims – Ultra Practice Bundle PDF
CASE I (A): STEP IV: Three persons sit between Gogi and Sodhi who
is an immediate neighbour of Daya.

As per this statement, the arrangement will look like this:

CASE I:

CASE II:

CASE I (A):

CASE II (A):

CASE II:

Page 346 of 722

Subscribe the Xpress Video Course & Mock Test Package for Bank & Insurance Exams
If there are any suggestions/ errors in our PDFs Feel Free to contact us via this email: admin@exampundit.in
IBPS RRB Clerk Prelims – Ultra Practice Bundle PDF

CASE II (A):
CASE I (A):

STEP V: Babita sits third from the left end. Iyer faces CASE II:
Roshan.

As per these statements, CASE I, CASE II and CASE II


(A) will get eliminated and we will continue with CASE
I (A) it will look like this:

CASE I:

CASE II (A):

Page 347 of 722

Subscribe the Xpress Video Course & Mock Test Package for Bank & Insurance Exams
If there are any suggestions/ errors in our PDFs Feel Free to contact us via this email: admin@exampundit.in
IBPS RRB Clerk Prelims – Ultra Practice Bundle PDF
38) A
39) C
40) A

Solutions (36-40):

Step I: Z was seated 3rd to the right of Q, who was seated


STEP VI: Iyer and Anjali are immediate neighbours of exactly behind Y. Y was to the immediate right of either
each other. Tappu and Madhavi are immediate neighbours P or S.
of each other.
As per these statements, there will be three cases and they
As per these statements, the final arrangement will look are as follows:
like this:
CASE I:
CASE I (A):

CASE II:

Answers (36-40):

36) B
37) E

Page 348 of 722

Subscribe the Xpress Video Course & Mock Test Package for Bank & Insurance Exams
If there are any suggestions/ errors in our PDFs Feel Free to contact us via this email: admin@exampundit.in
IBPS RRB Clerk Prelims – Ultra Practice Bundle PDF
CASE III: CASE III:

Step II: U was seated 3rd from one of the corners. R was
CASE III (A):
to the immediate right of U.

As per these statements, CASE II will get eliminated and


the remaining arrangement will look like this:

CASE I:

CASE III: X was 2nd to the left of T. Number of persons


seated to the left of O was same as the number of persons
to the right of Y and they were seated in different rows.

As per these statements, the arrangement will look like


this and we can conclude that CASE III (A) is invalid as
CASE II: O cannot be placed in it:

CASE I:

Page 349 of 722

Subscribe the Xpress Video Course & Mock Test Package for Bank & Insurance Exams
If there are any suggestions/ errors in our PDFs Feel Free to contact us via this email: admin@exampundit.in
IBPS RRB Clerk Prelims – Ultra Practice Bundle PDF
CASE I:

CASE III:

CASE III:

CASE III (A):

Answers (41-45):

41) D
42) B
43) E

CASE IV: The person seated exactly behind W, who was 44) C

2nd to the left of V. Number of persons seated to the left of 45) B

Q was equal to the number of persons to the right of S.


Solutions (41-45):

As per these statements, CASE I is invalid and the final Step I: M sits third to the left of L, who faces C.

arrangement will look like this:

Page 350 of 722

Subscribe the Xpress Video Course & Mock Test Package for Bank & Insurance Exams
If there are any suggestions/ errors in our PDFs Feel Free to contact us via this email: admin@exampundit.in
IBPS RRB Clerk Prelims – Ultra Practice Bundle PDF
As per this statement, there will be three cases being As per this statement, CASE II and CASE III will get
formed and they are as follows: eliminated and we will continue with CASE I and it will
look like this:
CASE I:
CASE I:

CASE II:

CASE II:

CASE III:

CASE III:

Step II: N sits second to the left of M.

Page 351 of 722

Subscribe the Xpress Video Course & Mock Test Package for Bank & Insurance Exams
If there are any suggestions/ errors in our PDFs Feel Free to contact us via this email: admin@exampundit.in
IBPS RRB Clerk Prelims – Ultra Practice Bundle PDF
Step III: F, who faces K, who sits third to the right of E.

As per this statement, the arrangement will look like this:

CASE I:

Answers (46-50):

46) B
47) D
48) A
49) C
50) A
Step IV: A sit third to the left of B.

Solutions (46-50):
As per this statement, the arrangement will look like this:

Step I: P and Y are sitting at the extreme ends of the row.

As per this statement, there will be two cases being framed


and they are as follows:

CASE I:

Step V: D faces a person, who is not an immediate


neighbour of J. I do not face E.

As per these statements, the final arrangement will look


like this:
Page 352 of 722

Subscribe the Xpress Video Course & Mock Test Package for Bank & Insurance Exams
If there are any suggestions/ errors in our PDFs Feel Free to contact us via this email: admin@exampundit.in
IBPS RRB Clerk Prelims – Ultra Practice Bundle PDF

CASE II: Step III: Two persons sit between Q and M but none of
them sit at extreme ends of the line.

As per this statement, the diagram will look like this:

CASE I:

Step II: Three persons it between Y and U, who does not


face M.

As per this statement, the arrangement will look like this:

CASE I:
CASE II:

CASE II:
Page 353 of 722

Subscribe the Xpress Video Course & Mock Test Package for Bank & Insurance Exams
If there are any suggestions/ errors in our PDFs Feel Free to contact us via this email: admin@exampundit.in
IBPS RRB Clerk Prelims – Ultra Practice Bundle PDF
Step IV: X sits to the immediate left of W but does not Step V: M does not face Z, who sits to the left of V but
face O. R faces W. not to the immediate left of V. The one, who faces N sits
second to the right of Z. M does not face U.
As per this statement, the arrangement will look like this:
As per this statement, CASE II will get eliminated and the
CASE I: arrangement will look like this:

CASE I:

CASE II:

CASE II:

17). Flat & Floors Based Puzzle

Directions (1 –5): Study the following information There are eight persons i.e. Jiya, Tanu, Puja, Riya, Isha,
carefully to answer the given questions: Diva, Ranu and Tia who lives in a building having three
Page 354 of 722

Subscribe the Xpress Video Course & Mock Test Package for Bank & Insurance Exams
If there are any suggestions/ errors in our PDFs Feel Free to contact us via this email: admin@exampundit.in
IBPS RRB Clerk Prelims – Ultra Practice Bundle PDF
floors such that ground floor is numbered as 1 and above A) Floor number and the flat number of Tia is same
it is 2 and so on up to top floor numbered as 3. Each of the B) There are no floors between Tia and Isha
floor consist of 3 flats as flat-1, flat-2 and flat-3. Flat-1 of C) Tia lives to the immediate west of Riya
floor-2 is immediately above flat-1 of floor-1 and D) Both A and B
immediately below flat-1 of floor-3 and in the same way E) All of the above
flat-2 of floor-2 is immediately above flat-2 of floor-1 and 3. Who lives to the immediate east of Ranu?
immediately below flat-2 of floor-3 and so on. Flat-2 is in A) Tanu
east of flat-1 and flat-3 is in east of flat-2. One of the flats B) No-One
is Vacant. C) Diva
Note: If A lives above (or below) B, then they are living D) Isha
in same flat unless stated otherwise. E) None of these
Diva lives to the immediate west of Isha on floor-2. Riya 4. Which of the following statements is/are not true?
lives in the same flat in which Diva lives. Tanu who lives A) Puja lives immediately above the one who lives to the
in an odd numbered flat, does not live above Tia. immediate east of Diva
Immediate west flat of Tanu is vacant. Puja and Riya live B) Ranu and Jiya shared the same floor
in the same floor while Puja and Ranu shared the same C) Tanu and Ranu shared the same flat
flat. Jiya lives in an even numbered floor. Tia and Tanu do D) All are true
not share the same floor. E) None is true
1. Who among the following lives immediate above the 5. Isha lives to the _______of the one who lives towards
vacant flat? north-east of Ranu?
A) Jiya A) Both B and C
B) Diva B) All C, D and E
C) One who lives to the immediate south of Riya and in C) East
the same flat as Riya D) South
D) Both A and C E) South-East
E) Both B and C Direction (6-10): Study the following information
2. Which of the following statements is/are true carefully to answer the given questions:
regarding Tia?
Page 355 of 722

Subscribe the Xpress Video Course & Mock Test Package for Bank & Insurance Exams
If there are any suggestions/ errors in our PDFs Feel Free to contact us via this email: admin@exampundit.in
IBPS RRB Clerk Prelims – Ultra Practice Bundle PDF
There are ten persons i.e Rupa, Ritz, Rita, Ruhi, Rozi, C) Person lives to the west of Ritz is to the north of Rupa
Riky, Ritu, Razi, Riaz and Riku, who lives in a building D) Both A and C
having five floors such that ground floor is numbered as 1 E) All of the above
and above it is 2 and so on up to top floor numbered as 5. 8. Riaz lives in which direction with respect to Riku
Each of the floor consist of 2 flats as flat-A, flat-B. Flat-A and on which floor?
of floor-2 is immediately above flat-A of floor-1 and A) South-East, Floor-4
immediately below flat-A of floor-3 and in the same way B) South-West, Floor-3
flat-B of floor-2 is immediately above flat-B of floor-1 C) South-East, Floor-3
and immediately below flat-B of floor-3 and so on. Flat-B D) South-West, Floor-4
is in east of flat-A. E) None of these
Note: If A lives above (or below) B, then there are 9. Which of the following statements is/are false
living in same flat unless stated otherwise. regarding Ruhi?
Ritu lives to the East of the one who lives immediately A) Ruhi lives to the east of Riku
above of Rupa. Riaz and Riky don’t live in same floor. B) Rupa lives four floors below from Ruhi
Rupa lives in an odd numbered floor. Ruhi lives west of C) There only one floor in between Ruhi and Ritz
Riku, who lives second to above the flat of Ritz. Rita lives D) Both A and C are False
two flats below of Razi who lives on Floor-4 and in flat- E) Both A and B are False
A. Riaz lives north-west of Rozi. 10. On which floor and in which flat does Ritu lives?
6. How many floors are there in between Riky and the A) Floor-4, Flat-A
one who lives east of Rita? B) Floor-5, Flat-B
A) 1 C) Floor-3, Flat-A
B) 2 D) Floor-2, flat-B
C) 3 E) None of the above
D) 4
E) 5 Direction (11-15): Study the following information
7. Which of the following statements is/are true? carefully to answer the given questions:
A) Rozi lives to the South-East of Riaz There are Twelve persons i.e. Nisa, Nora, Naaz, Neli,
B) Razi lives immediate below of Ruhi Nemu, Niki, Neha, Noor, Nini, Nimo, Nita and Niti who
Page 356 of 722

Subscribe the Xpress Video Course & Mock Test Package for Bank & Insurance Exams
If there are any suggestions/ errors in our PDFs Feel Free to contact us via this email: admin@exampundit.in
IBPS RRB Clerk Prelims – Ultra Practice Bundle PDF
lives in a building having Four floors such that ground B) Nimo and Neha
floor is numbered as 1 and above it is 2 and so on up to C) Nita and Neli
top floor numbered as 3. Each of the floor consist of 3 flats D) Both B and C
as flat-1, flat-2 and flat-3. Flat-1 of floor-2 is immediately E) All of the above
above flat-1 of floor-1 and immediately below flat-1 of 13. Who among the following sits to the extreme west
floor-3 and in the same way flat-2 of floor-2 is on Floor - 2 and extreme east on Floor-4?
immediately above flat-2 of floor-1 and immediately A) Nisa and Nini
below flat-2 of floor-3 and so on. Flat-2 is in east of flat-1 B) Nimo and Naaz
and flat-3 is in east of flat-2. C) Nita and Nora
Note: If A lives above (or below) B, then there are D) Neha and Nora
living in same flat unless stated otherwise. E) None of the above
Nora lives immediately above of Noor who lives to the 14. How many Floors are in between the Niki and Neli?
immediate East of Niki in an odd numbered floor. Neha A) 2Floors
lives to the west of Nimo who lives to the immediate B) 0 Floor
above of Neli on Floor-2 and in Flat-3. Nini lives north- C) 1Floor
west of Noor and immediate west of Nora. Nemu lives D) Either A or B
immediately below of Naaz who lives in North-West E) Either B or C
direction of Nisa who lives immediately above of Niti. 15. Which of the following statement is/are correct
11. Which of the following pair have same Flat No. and regarding Nemu?
same Floor No.? A) Nemu sits to the west of Noor
A) Nora and Noor B) Nimo sits towards South-East of Nemu
B) Nisa and Nita C) Nemu is in North-West direction of Nisa
C) Neli and Nita D) Both B and C
D) Both A and B E) All of the above
E) Both B and C
12. Which of the following pair sits towards South- Direction (16-20): Study the following information
West and South-East of Niki respectively? carefully to answer the given questions:
A) Naaz and Nora
Page 357 of 722

Subscribe the Xpress Video Course & Mock Test Package for Bank & Insurance Exams
If there are any suggestions/ errors in our PDFs Feel Free to contact us via this email: admin@exampundit.in
IBPS RRB Clerk Prelims – Ultra Practice Bundle PDF
There are nine persons i.e. P, Q, R, S, T, U, V, W and X 16. Who lives to the East of the one who likes Orange?
who lives in a building having Three floors such that A) The one who likes grey
ground floor is numbered as 1 and above it is 2 and so on B) P
up to top floor numbered as 3. Each of the floors consist C) One who sits to the south of T
of 3 flats as flat-1, flat-2 and flat-3. Flat-1 of floor-2 is D) One who likes Pink
immediately above flat-1 of floor-1 and immediately E) All of the above
below flat-1 of floor-3 and in the same way flat-2 of floor- 17. Which of the following statements is/are not true?
2 is immediately above flat-2 of floor-1 and immediately A) X likes green and lives to the west of R
below flat-2 of floor-3 and so on. Flat-2 is in east of flat-1 B) U lives on Floor 3 and Flat-2 and likes Grey
and flat-3 is in east of flat-2.Each of them likes different C) Both who likes Black and red live in the same flat
colours i.e. Red, Yellow, White, Black, Blue, Pink, Green, D) Both B and C
Grey and Orange. E) None is true
Note: If A lives above (or below) B, then there are 18. Who among the following lives on Floor-2?
living in same flat unless stated otherwise. A) One who likes grey
Number of Flats towards west of T is equal to the number B) One who lives immediate east of W
of flats towards east of V, but in different floors. X who C) One who lives immediate west of the one likes pink
likes Green lives to the immediate west of R who likes colour.
Yellow and lives in Flat-3 of Floor-1. U does not live on D) All of the above
Floor-2 but lives to the East of the one who likes Red who E) Both A and C
does not live in an even numbered flat. T and P lives in 19. Who likes Black?
the same flat which is Flat-3. Q lives northwest of S. The A) R
one who likes Pink doesn’t live in topmost floor. The one B) One lives on Flat-3 of Floor-1
who likes Blue colour and S lives in the same flat. W does C) One who lives immediate west of X
not like Red and lives one of the extreme west flat. The D) Both A and B
one who likes Orange lives to the immediate west of the E) All of the above
one who likes Grey and T likes White. Persons like
Orange colour and Black colour lives in the same flat and 20. Which of the following statements is/ are true
W does not like Black colour. regarding W?
Page 358 of 722

Subscribe the Xpress Video Course & Mock Test Package for Bank & Insurance Exams
If there are any suggestions/ errors in our PDFs Feel Free to contact us via this email: admin@exampundit.in
IBPS RRB Clerk Prelims – Ultra Practice Bundle PDF
A) W lives on the same Flat and Floor number number of Neel, both are living in even numbered floor.
B) W lives to the south-east of U There is only one person lives on the floor in which Nitu
C) W likes orange and sits to the north of V lives. Zoya lives north-west of either Bela or Aman.
D) All are true Neither Bela nor Aman lives west of Prem.
E) None is true 21. How many persons live between Zoya’s floor and
Pari’s floor?
Direction (21-25): Study the following information A) 2
carefully to answer the given questions: B) 3
There are ten persons i.e. Zoya, Hema, Neel, Zeba, Pari, C) 4
Roma, Aman, Nitu, Prem and Bela who lives in a building D) 5
having Six floors such that ground floor is numbered as 1 E) None of these
and above it is 2 and so on up to top floor numbered as 3. 22. Which of the following pair live on Floor-4?
Each of the floor consist of 2 flats as flat-1, flat-2. Flat-1 A) Zoya and the one who lives immediately below of the
of floor-2 is immediately above flat-1 of floor-1 and vacant flat
immediately below flat-1 of floor-3 and in the same way B) One who lives immediately above and below of Prem
flat-2 of floor-2 is immediately above flat-2 of floor-1 and and Roma respectively
immediately below flat-2 of floor-3 and so on. Flat-2 is in C) Zeba and the one who lives immediately below of the
east of flat-1. Two flats are vacant but they are not vacant flat
adjacent. D) Both A and B
Note: If A lives above (or below) B, then there are E) Both A and C
living in same flat unless stated otherwise. 23. Who among the following has/have same flat and
Only two persons live below Pari who lives immediately floor numbers?
below of Prem, who lives in Flat-2. Zeba lives towards A) Pari
South-East of Nitu who lives to the immediate south of B) Bela
Roma and share the same flat as of Roma. Hema and C) Aman
Roma shared the same floor which is an even numbered D) Both A and B
such that Hema lives east of Roma. No flat in ground floor E) Only A and Either B or C
is Vacant. Zoya’s floor number is twice of the floor 24. Which of the following statements is/are true?
Page 359 of 722

Subscribe the Xpress Video Course & Mock Test Package for Bank & Insurance Exams
If there are any suggestions/ errors in our PDFs Feel Free to contact us via this email: admin@exampundit.in
IBPS RRB Clerk Prelims – Ultra Practice Bundle PDF
A) Hema lives to the west of Roma Shreya lives in an even numbered Floor in Flat-B. Sonam
B) Prem lives to the east of Neel who speaks Kannada lives in Flat-A of an odd numbered
C) Either Bela or Aman lives on Flat-1 of Floor-1 floor. Sonam lives south-west of Shreya. Persons
D) Both A and C speaking Tamil and Bengali shared the same Floor. Sakshi
E) Both B and C lives second to above Surbhi in Flat-B. The one who
25. Who lives to the south- west of Pari? speaks Urdu lives north-east of Suman. Shikha speaks
A) Bela Hindi lives to the west of Sakshi. There is a gap of one
B) Aman Floor between the both who speaks Tamil & Punjabi and
C) None both of them live in an even numbered floor in Flat-A.
D) Either A or B Sunita speaks Tamil and lives above the one who speaks
E) Cannot be determined Punjabi. One who speaks Marathi lives immediately
below of Shreya.
Direction: (26-30): Study the following information 26. Who among of the following lives to the East of the
carefully to answer the given questions: one who speaks Hindi?
There are eight persons i.e. Suman, Sanjhi, Sonam, A) One who speaks Urdu
Surbhi, Sakshi, Sunita, Shreya and Shikha who lives in a B) One who live in Flat-B of Floor-3
building having Four floors such that ground floor is C) Sakshi
numbered as 1 and above it is 2 and so on up to top floor D) All of the above
numbered as 3. Each of the floor consist of 2 flats as flat- E) Both A and C
A, flat-B. Flat-A of floor-2 is immediately above flat-A of 27. Who among the following lives in Flat-B of Floor-
floor-1 and immediately below flat-A of floor-3 and in the 2?
same way flat-B of floor-2 is immediately above flat-B of A) One who speaks English
floor-1 and immediately below flat-B of floor-3 and so on. B) One who speaks Marathi
Flat-B is in east of flat-A. Each of them speaks different C) One who speaks Urdu
languages i.e. Bengali, Tamil, Marathi, Urdu, English, D) One who speaks Punjabi
Hindi, Kannada and Punjabi. E) None of these
Note: If A lives above (or below) B, then there are living
in same flat unless stated otherwise.
Page 360 of 722

Subscribe the Xpress Video Course & Mock Test Package for Bank & Insurance Exams
If there are any suggestions/ errors in our PDFs Feel Free to contact us via this email: admin@exampundit.in
IBPS RRB Clerk Prelims – Ultra Practice Bundle PDF
28. Number of person’s lives above Suman in same flat and flat-3. Flat-1 of floor-2 is immediately above flat-1 of
is equal to the number of person lives below ________ floor-1 and in the same way flat-2 of floor-2 is
in same flat? immediately above flat-2 of floor-1 and so on. Flat-2 is in
A) One who speaks Urdu east of flat-1 and flat-3 is in east of flat-2. Each of them
B) One who lives in Flat-B of Floor-3 has different test series i.e. Gradeup, Oliveboard,
C) One who speaks English Adda247, Testbook, IBPS Guide and Exampundit.
D) Both A and C
E) Both A and B Note: If A lives above (or below) B, then there are living
29. Which of the following statements is/are not true? in same flat unless stated otherwise.
A) Person who speaks Punjabi lives to the west Shreya
B) Sanjhi lives to the east of the one who speaks Tamil Person who has Testbook lives to the extreme west of the
C) Person speaking Hindi and Kannada shared the same one who has Adda247 and to the South-west of the one
Floor who has Gradeup. Akbar has Exampundit shared the same
D) All are True Flat and Floor number. Ashish and Akash shared the same
E) None is True floor and none of them has Testbook. Neither Ashish nor
30. Which of the following pair shared the same Flat? Akash shared the same flat with the one who has
A) Suman and the person who speaks English Testbook. Arnav who has Gradeup lives to the North-East
B) Sunita and the person who speaks Kannada of Ashish. Person who has IBPS Guide and the Person
C) Sakshi and the person who speaks Hindi having Exampundit shared the same flat. Aditya lives in
D) Both A and B the same Flat and Floor number.
E) Both A and C 31. Who among the following lives on Floor 2?
A) Person lives to the east of Akbar
Direction (31-35): Study the following information B) Person having series of Oliveboard
carefully to answer the given questions: C) Person having series of Adda247
There are Six persons i.e. Akash, Akbar, Ashish, Arnav, D) Both A and B
Aditya and Aarav who lives in a building having Two E) Both B and C
floors such that ground floor is numbered as 1 and above 32. Ashish has a series of _______ and sits to the west
it is 2. Each of the floor consist of 3 flats as flat-1, flat-2 of ____
Page 361 of 722

Subscribe the Xpress Video Course & Mock Test Package for Bank & Insurance Exams
If there are any suggestions/ errors in our PDFs Feel Free to contact us via this email: admin@exampundit.in
IBPS RRB Clerk Prelims – Ultra Practice Bundle PDF
A) Testbook, Akash a building having Three floors such that ground floor is
B) IBPS Guide, Aditya numbered as 1 and above it is 2 and so on up to top floor
C) IBPS Guide, Akash numbered as 3. Each of the floor consist of 4 flats as flat-
D) Testbook, Aditya 1, flat-2, flat-3 and flat-4. Flat-1 of floor-2 is immediately
E) None of these above flat-1 of floor-1 and immediately below flat-1 of
33. Which of the following statements is/are correct? floor-3 and in the same way flat-2 of floor-2 is
A) Arnav has series of Gradeup immediately above flat-2 of floor-1 and immediately
B) Aditya has series of Adda247 below flat-2 of floor-3 and so on. Flat-2 is in east of flat-
C) Aarav has series of IBPS Guide 1, flat-3 is in east of flat-2 and flat-4 is in east of flat-3.
D) All of the above Two flats are vacant but they are not adjacent. Each of
E) Both A and C them works in different banks i.e. BOB, BOI, Canara,
34. Who among the following pair lives on the same Indian bank, SBI, PNB, UCO, BOM, UBI and OBC.
Floor and Flat number? Note: If A lives above (or below) B, then there are living
A) Aarav and the one who has series of Exampundit in same flat unless stated otherwise.
B) Akbar and the one who has series of Testbook
C) Aarav and the one who has series of Testbook Nishu shared the same floor and flat number. Komal who
D) Akbar and the one who has series of Exampundit lives in flat-1 of floor-2 lives immediately below to the
E) None of these one who works in BOI. Nishu lives immediately above of
35. Who among the following has series of Oliveboard? Rahul who works in Canara bank and lives to the
A) Person who lives to the west of Arnav immediate east of Tani. Tani and Chhaya shared the same
B) Person who lives to the north of Ashish flat, Similarly, Komal and Amit shared the same flat.
C) Person who lives to the north-west of Akash Manish who works in UCO bank lives in flat-4 of floor-1
D) Both A and B and shared the flat with Mohini. Tani lives on the floor
E) Both A and C which has one vacant flat. Sejal who works in PNB lives
Direction: (36-40): Study the following information to the east of the Nitin who lives in an odd numbered flat,
carefully to answer the given questions: does not works in BOI. The one who works in OBC lives
There are ten persons i.e. Nitin, Nishu, Manish, Mohini, to the west of the person works in SBI. Neither Chhaya
Rahul, Amit, Sejal, Komal, Tani and Chhaya who lives in Nor Tani works in SBI. Amit doesn’t live west of Chhaya.
Page 362 of 722

Subscribe the Xpress Video Course & Mock Test Package for Bank & Insurance Exams
If there are any suggestions/ errors in our PDFs Feel Free to contact us via this email: admin@exampundit.in
IBPS RRB Clerk Prelims – Ultra Practice Bundle PDF
Person works in BOB lives towards the North-East of the 39. How many person lives above Sejal, combining all
person works in UBI and the person works in BOM lives flats?
towards the South-East of the person works in Indian A) 7
bank. B) 6
36. Who among the following lives to the extreme east C) 5
of floor-2? D) 4
A) Person works in UCO bank E) 3
B) Person works in Indian Bank 40. Which among the following flat is vacant?
C) Person works in BOI A) FLAT-4 of floor-3
D) Person works in UBI B) FLAT-2 of floor-3
E) None of these C) FLAT-3 of floor-1
37. Person lives east of Komal is equal to the person D) FLAT-3 of floor-2
lives west of the_________ E) None of the above
A) Person works in Canara bank
B) Persons live after the vacant flat Direction: (41-45): Study the following information
C) Person works in UBI carefully to answer the given questions:
D) Person works in BOM There are eight persons i.e. Dev, Ved, Devesh, Vedi,
E) Person works in BOB Durga, Vishu, Vivek and Dolly who lives in a building
38. Which among the following statements is/are having three floors such that ground floor is numbered as
incorrect regarding Chhaya? 1 and above it is 2 and so on up to top floor numbered as
A) Chhaya lives immediately below the person works in 3. Each of the floor consist of 3 flats as flat-1, flat-2, flat-
BOB 3. Flat-1 of floor-2 is immediately above flat-1 of floor-1
B) Chhaya lives to the immediate east of the person works and immediately below flat-1 of floor-3 and in the same
in PNB way flat-2 of floor-2 is immediately above flat-2 of floor-
C) Chhaya lives in flat-2 of floor-1 1 and immediately below flat-2 of floor-3 and so on. Flat-
D) All are Correct 2 is in east of flat-1, flat-3 is in east of flat-2.
E) None is Correct

Page 363 of 722

Subscribe the Xpress Video Course & Mock Test Package for Bank & Insurance Exams
If there are any suggestions/ errors in our PDFs Feel Free to contact us via this email: admin@exampundit.in
IBPS RRB Clerk Prelims – Ultra Practice Bundle PDF
Note: If A lives above (or below) B, then there are living 44. Who among the following lives in an even
in same flat unless stated otherwise. Left and right are numbered flat?
taken as per north direction. A) Devesh, Dolly
Dev lives in flat-1 and to the immediate left of Ved who B) Durga, Ved
lives below of Vishu. Vedi lives on the same flat and floor C) Vivek, Dev
number. Vishu lives in flat-2 of one of the floors. Vivek D) Vishu, Vedi
lives to the left of Durga who lives on floor-3. Dolly lives E) Devesh, Durga
below of Vedi in the same flat as of Vedi. Devesh lives to 45. Number of persons lives east of Devesh is equal to
the left of Dolly. Flat towards east of Ved is empty. the number of person lives west of_______
41. On which of the following floor does Vishu lives? A) Dolly
A) Floor-3 B) Dev
B) Floor-2 C) Vishu
C) Floor-1 D) Devesh
D) Immediate below of Durga’s Floor E) None of these
E) Both B and D Direction: (46-50): Study the following information
42. Who among the following lives to the North-west of carefully to answer the given questions:
Dolly? There are Ten persons i.e. Panchi, Palak, Payal, Parul,
A) Vivek Prerna, Pankhuri, Pihu, Piku, Prachi and Pankhi who lives
B) Devesh in a building having Four floors such that ground floor is
C) Durga numbered as 1 and above it is 2 and so on up to top floor
D) Both A and C numbered as 3. Each of the floor consist of 3 flats as flat-
E) Both A and B 1, flat-2, flat-3. Flat-1 of floor-2 is immediately above flat-
43. Which of the following statements is/are true? 1 of floor-1 and immediately below flat-1 of floor-3 and
A) Dev lives in an odd numbered flat in the same way flat-2 of floor-2 is immediately above
B) Vedi lives in an even numbered floor flat-2 of floor-1 and immediately below flat-2 of floor-3
C) Devesh lives in even numbered flat and so on. Flat-2 is in east of flat-1, flat-3 is in east of flat-
D) Both A and B 2. Two vacant flats are on odd numbered floors and are on
E) Both A and C even numbered flats.
Page 364 of 722

Subscribe the Xpress Video Course & Mock Test Package for Bank & Insurance Exams
If there are any suggestions/ errors in our PDFs Feel Free to contact us via this email: admin@exampundit.in
IBPS RRB Clerk Prelims – Ultra Practice Bundle PDF
Note: If A lives above (or below) B, then there are living D) Both A and B
in same flat unless stated otherwise. E) All are true
Palak lives on an odd numbered flat and in an even 48. How many floors are in between Pankhi and Pihu?
numbered floor. The flat between Prerna and Piku is A) 1 Floor
vacant and Prerna lives one of the floors above the floor B) No Floor
on which Piku lives. Panchi lives on the floor immediately C) 2 Floors
above Prachi in the same flat. Pihu and Payal live on the D) Either A or B
same floor and no one else lives on that floor. Parul lives E) Either B or C
immediately to the west of the flat in which Piku lives. 49. Which of the followings statements is/are correct
Pihu and Pankhi live in the same flats which are numbered regarding Payal?
1. Pankhi and Prachi do not live on an odd-numbered A) She lives in odd numbered flat and in an even
floor. Panchi does not live on flat number 1. numbered floor
46. Who among the following lives in the same B) She lives in even numbered flat and in an odd
numbered floor and flat? numbered floor
A) Panchi and Pankhi C) She lives in an even numbered floor and flat
B) Piku and Pihu D) She lives in an odd numbered floor and flat
C) Prachi and Pihu E) None is correct
D) Pankhuri and Panchi 50. Who among the following lives on floor 2?
E) None of these A) Pankhuri and Parul
47. Which of the following statements is/are correct? B) Piku and Prachi
A) Prachi lives in an even numbered floor and in an odd C) Prachi and Parul
numbered flat D) Both A and B
B) Prerna lives in an even numbered floor and flat E) Both B and C
C) Payal lives to the east of the vacant floor

17). Flat & Floors Based Puzzle – Solution with detailed Explanation
Answers (1-5):

Page 365 of 722

Subscribe the Xpress Video Course & Mock Test Package for Bank & Insurance Exams
If there are any suggestions/ errors in our PDFs Feel Free to contact us via this email: admin@exampundit.in
IBPS RRB Clerk Prelims – Ultra Practice Bundle PDF
1) E
2) D
3) B
4) E
5) B
Solutions (1-5):
Step I:
Diva lives to the immediate west of Isha on floor-2.
Jiya lives in an even numbered floor.
As per these lines two cases will arrive:
Case-1 Case-2
Floor Flat-1 Flat-2 Flat-3 Floor Flat-1 Flat-2 Flat-3
3 3
2 DIVA ISHA JIYA 2 JIYA DIVA ISHA
1 1

Riya lives in the same flat in which Diva lives. (By using this line we get 2 more cases)
Case-1 Case-2
Floor Flat-1 Flat-2 Flat-3 Floor Flat-1 Flat-2 Flat-3
3 RIYA 3 RIYA
2 DIVA ISHA JIYA 2 JIYA DIVA ISHA
1 1

Case-1-A Case-2-A
Floor Flat-1 Flat-2 Flat-3 Floor Flat-1 Flat-2 Flat-3
3 3
2 DIVA ISHA JIYA 2 JIYA DIVA ISHA

Page 366 of 722

Subscribe the Xpress Video Course & Mock Test Package for Bank & Insurance Exams
If there are any suggestions/ errors in our PDFs Feel Free to contact us via this email: admin@exampundit.in
IBPS RRB Clerk Prelims – Ultra Practice Bundle PDF
1 RIYA 1 RIYA

Step II: Tanu who lives in an odd numbered flat, does not live above Tia.
Immediate west flat of Tanu is vacant.
Also Tia and Tanu do not share the same floor.
Brief Hints: Tanu must live in FLAT-3 of floor-1 (odd number) and lives below Tia.
Case-1 Case-2
Floor Flat-1 Flat-2 Flat-3 Floor Flat-1 Flat-2 Flat-3
3 RIYA 3 RIYA
2 DIVA ISHA JIYA 2 JIYA DIVA ISHA
1 VACANT TANU 1 VACANT TANU
Note: Immediate west flat of Tanu is vacant (this condition not possible in Case-2A, hence rejected)
Case-1-A Case-2-A (REJECTED)
Floor Flat-1 Flat-2 Flat-3 Floor Flat-1 Flat-2 Flat-3
3 3
2 DIVA ISHA JIYA 2 JIYA DIVA ISHA
1 RIYA VACANT TANU 1 RIYA
Step III:Puja and Riya live in the same floor while Puja and Ranu shared the same flat.
Note: Case-1 and Case-1-A gets rejected since we can’t satisfy the above said condition
Case-1 (REJECTED) Case-2
Floor Flat-1 Flat-2 Flat-3 Floor Flat-1 Flat-2 Flat-3
3 RIYA 3 PUJA RIYA TIA
2 DIVA ISHA JIYA 2 JIYA DIVA ISHA
1 VACANT TANU 1 RANU VACANT TANU
Case-1-A (REJECTED)
Floor Flat-1 Flat-2 Flat-3
3

Page 367 of 722

Subscribe the Xpress Video Course & Mock Test Package for Bank & Insurance Exams
If there are any suggestions/ errors in our PDFs Feel Free to contact us via this email: admin@exampundit.in
IBPS RRB Clerk Prelims – Ultra Practice Bundle PDF
2 DIVA ISHA JIYA
1 RIYA VACANT TANU

Case 2 is the final result. [Final Arrangement]


Case-2
Floor Flat-1 Flat-2 Flat-3
3 PUJA RIYA TIA
2 JIYA DIVA ISHA
1 RANU VACANT TANU
Answers (6-10):
6) A
7) E
8) B
9) A
10) D
Solutions (6-10):
Step I:
Ritu lives to the East of the one who lives immediately above of Rupa.
Rupa lives in an odd numbered floor.

As per these lines two Cases will Arrive:

Case-1 Case-2
Floor Flat-A Flat-B Floor Flat-A Flat-B
5 5
4 4 RITU
3 3 RUPA

Page 368 of 722

Subscribe the Xpress Video Course & Mock Test Package for Bank & Insurance Exams
If there are any suggestions/ errors in our PDFs Feel Free to contact us via this email: admin@exampundit.in
IBPS RRB Clerk Prelims – Ultra Practice Bundle PDF
2 RITU 2
1 RUPA 1

Rita lives two flats below of Razi who lives on Floor-4 and in flat-A.
Case-1 Case-2
Floor Flat-A Flat-B Floor Flat-A Flat-B
5 5
4 RAZI 4 RAZI RITU
3 3 RUPA
2 RITA RITU 2 RITA
1 RUPA 1

Ruhi lives west of Riku, who lives second to above the flat of Ritz
Note: Here we get one more sub case from case-1
Case-1 Case-1-A Case-2
Floor Flat-A Flat-B Floor Flat-A Flat-B Floor Flat-A Flat-B
5 5 RUHI RIKU 5 RUHI RIKU
4 RAZI 4 RAZI 4 RAZI RITU
3 RUHI RIKU 3 RITZ 3 RUPA RITZ
2 RITA RITU 2 RITA RITU 2 RITA
1 RUPA RITZ 1 RUPA 1
Riaz lives north-west of Rozi
Note: Case-2 rejected since Riaz lives north-west of Rozi is not possible.

Case-1 Case-1-A Case-2 (REJECTED)


Floor Flat-A Flat-B Floor Flat-A Flat-B Floor Flat-A Flat-B
5 RIAZ 5 RUHI RIKU 5 RUHI RIKU

Page 369 of 722

Subscribe the Xpress Video Course & Mock Test Package for Bank & Insurance Exams
If there are any suggestions/ errors in our PDFs Feel Free to contact us via this email: admin@exampundit.in
IBPS RRB Clerk Prelims – Ultra Practice Bundle PDF
4 RAZI ROZI 4 RAZI 4 RAZI RITU
3 RUHI RIKU 3 RIAZ RITZ 3 RUPA RITZ
2 RITA RITU 2 RITA RITU 2 RITA
1 RUPA RITZ 1 RUPA ROZI 1
Riaz and Riky don’t live in same floor.
Note: Case-2 rejected since Riaz and Riky don’t live in same floor is not possible.

Case-1 (REJECTED) Case-1-A


Floor Flat-A Flat-B Floor Flat-A Flat-B
5 RIAZ 5 RUHI RIKU
4 RAZI ROZI 4 RAZI RIKY
3 RUHI RIKU 3 RIAZ RITZ
2 RITA RITU 2 RITA RITU
1 RUPA RITZ 1 RUPA ROZI
The final table arrangement is:
Case-1-A
Floor Flat-A Flat-B
5 RUHI RIKU
4 RAZI RIKY
3 RIAZ RITZ
2 RITA RITU
1 RUPA ROZI
Answers (11-15):
11) B
12) C
13) D
14) C
15) E
Page 370 of 722

Subscribe the Xpress Video Course & Mock Test Package for Bank & Insurance Exams
If there are any suggestions/ errors in our PDFs Feel Free to contact us via this email: admin@exampundit.in
IBPS RRB Clerk Prelims – Ultra Practice Bundle PDF
Solutions (11-15):
Step I:Nora lives immediately above of Noor who lives to the immediate East of Niki in an odd numbered floor.
As per these lines four cases will form:
Case-1 Case-2
Floor Flat-1 Flat-2 Flat-3 Floor Flat-1 Flat-2 Flat-3
4 4
3 3
2 NORA 2 NORA
1 NIKI NOOR 1 NIKI NOOR

Case-1-A Case-2-A
Floor Flat-1 Flat-2 Flat-3 Floor Flat-1 Flat-2 Flat-3
4 NORA 4 NORA
3 NIKI NOOR 3 NIKI NOOR
2 2
1 1
Neha lives to the west of Nimo who lives to the immediate above of Neli on Floor-2 and in Flat-3.
Case-2 gets rejected since above said condition is not possible. Also we get two more sub cases from Case-1-A and
Case-2-A
Case-1 Case-2 (REJECTED)
Floor Flat-1 Flat-2 Flat-3 Floor Flat-1 Flat-2 Flat-3
4 4
3 3
2 NEHA NORA NIMO 2 NORA
1 NIKI NOOR NELI 1 NIKI NOOR

Case-1-A Case-2-A

Page 371 of 722

Subscribe the Xpress Video Course & Mock Test Package for Bank & Insurance Exams
If there are any suggestions/ errors in our PDFs Feel Free to contact us via this email: admin@exampundit.in
IBPS RRB Clerk Prelims – Ultra Practice Bundle PDF
Floor Flat-1 Flat-2 Flat-3 Floor Flat-1 Flat-2 Flat-3
4 NORA 4 NORA
3 NIKI NOOR 3 NIKI NOOR
2 NEHA NIMO 2 NEHA NIMO
1 NELI 1 NELI

Case-1-A (I) Case-2-A (II)


Floor Flat-1 Flat-2 Flat-3 Floor Flat-1 Flat-2 Flat-3
4 NORA 4 NORA
3 NIKI NOOR 3 NIKI NOOR
2 NEHA NIMO 2 NEHA NIMO
1 NELI 1 NELI

Nini lives north-west of Noor and immediate west of Nora


Case-1 gets rejected since above said condition is not possible
Case-1 (Rejected)
Floor Flat-1 Flat-2 Flat-3
4
3
2 NEHA NORA NIMO
1 NIKI NOOR NELI

Case-1-A Case-2-A
Floor Flat-1 Flat-2 Flat-3 Floor Flat-1 Flat-2 Flat-3
4 NINI NORA 4 NINI NORA
3 NIKI NOOR 3 NIKI NOOR

Page 372 of 722

Subscribe the Xpress Video Course & Mock Test Package for Bank & Insurance Exams
If there are any suggestions/ errors in our PDFs Feel Free to contact us via this email: admin@exampundit.in
IBPS RRB Clerk Prelims – Ultra Practice Bundle PDF
2 NEHA NIMO 2 NEHA NIMO
1 NELI 1 NELI
Case-1-A (I) Case-2-A (II)
Floor Flat-1 Flat-2 Flat-3 Floor Flat-1 Flat-2 Flat-3
4 NINI NORA 4 NINI NORA
3 NIKI NOOR 3 NIKI NOOR
2 NEHA NIMO 2 NEHA NIMO
1 NELI 1 NELI
Nemu lives immediately below of Naaz who lives in North-West direction of Nisa who lives immediately above of
Niti.
Note: Case-1-A, Case-2-A and Case-1-A (I) gets rejected since above said condition is not possible
Case-1-A (REJECTED) Case-2-A (REJECTED)
Floor Flat-1 Flat-2 Flat-3 Floor Flat-1 Flat-2 Flat-3
4 NINI NORA 4 NINI NORA
3 NIKI NOOR 3 NIKI NOOR
2 NEHA NIMO 2 NEHA NIMO
1 NELI 1 NELI

Case-1-A (I) (REJECTED) Case-2-A (II)


Floor Flat-1 Flat-2 Flat-3 Floor Flat-1 Flat-2 Flat-3
4 NINI NORA 4 NAAZ NINI NORA
3 NIKI NOOR 3 NEMU NIKI NOOR
2 NEHA NIMO 2 NEHA NISA NIMO
1 NELI 1 NITI NELI
Finally NITA lives in 1st floor of FLAT-2 and we get the final arrangement
Case-2-A (II)
Floor Flat-1 Flat-2 Flat-3

Page 373 of 722

Subscribe the Xpress Video Course & Mock Test Package for Bank & Insurance Exams
If there are any suggestions/ errors in our PDFs Feel Free to contact us via this email: admin@exampundit.in
IBPS RRB Clerk Prelims – Ultra Practice Bundle PDF
4 NAAZ NINI NORA
3 NEMU NIKI NOOR
2 NEHA NISA NIMO
1 NITA NITI NELI
The final table arrangement is:
Case-2-A (II)
Floor Flat-1 Flat-2 Flat-3
4 NAAZ NINI NORA
3 NEMU NIKI NOOR
2 NEHA NISA NIMO
1 NITA NITI NELI
Answers (16-20):
16) E
17) D
18) D
19) D
20) C
Solutions (16-20):
X who likes Green lives to the immediate west of R who likes Yellow and lives in Flat-3 of Floor-1.

Floor Flat-1 Color Flat-2 Color Flat-3 Color


3
2
1 X GREEN R YELLOW
T and P lives in the same flat which is Flat-3.
Note: Here we get one more case.
Case-1

Page 374 of 722

Subscribe the Xpress Video Course & Mock Test Package for Bank & Insurance Exams
If there are any suggestions/ errors in our PDFs Feel Free to contact us via this email: admin@exampundit.in
IBPS RRB Clerk Prelims – Ultra Practice Bundle PDF
Floor Flat-1 Color Flat-2 Color Flat-3 Color
3 T
2 P
1 X GREEN R YELLOW

Case-2
Floor Flat-1 Color Flat-2 Color Flat-3 Color
3 P
2 T
1 X GREEN R YELLOW

Number of Flats towards west of T is equal to the number of flats towards east of V, but in different floors.
Note: Here we get one more possibility from each case. Thus 4 we have total cases
Case-1
Floor Flat-1 Color Flat-2 Color Flat-3 Color
3 T
2 V P
1 X GREEN R YELLOW
Case-1 (A)
Floor Flat-1 Color Flat-2 Color Flat-3 Color
3 T
2 P
1 V X GREEN R YELLOW

Case-2

Page 375 of 722

Subscribe the Xpress Video Course & Mock Test Package for Bank & Insurance Exams
If there are any suggestions/ errors in our PDFs Feel Free to contact us via this email: admin@exampundit.in
IBPS RRB Clerk Prelims – Ultra Practice Bundle PDF
Floor Flat-1 Color Flat-2 Color Flat-3 Color
3 V P
2 T
1 X GREEN R YELLOW
Case-2 (A)
Floor Flat-1 Color Flat-2 Color Flat-3 Color
3 P
2 T
1 V X GREEN R YELLOW
U does not live on Floor-2 but lives to the East of the one who likes Red who does not live in an even numbered flat.
W does not like Red and lives one of the extreme west flat.
Note: U must live on Flat-2 of Floor-3 (only possibility in all cases). Also we get only possibility for W in each case,
except case-2
Case-1
Floor Flat-1 Color Flat-2 Color Flat-3 Color
3 RED U T
2 V P
1 W X GREEN R YELLOW
Case-1 (A)
Floor Flat-1 Color Flat-2 Color Flat-3 Color
3 RED U T
2 W P
1 V X GREEN R YELLOW

Case-2
Floor Flat-1 Color Flat-2 Color Flat-3 Color
3 V RED U P

Page 376 of 722

Subscribe the Xpress Video Course & Mock Test Package for Bank & Insurance Exams
If there are any suggestions/ errors in our PDFs Feel Free to contact us via this email: admin@exampundit.in
IBPS RRB Clerk Prelims – Ultra Practice Bundle PDF
2 W/ T
1 W/ X GREEN R YELLOW
Case-2 (A)
Floor Flat-1 Color Flat-2 Color Flat-3 Color
3 RED U P
2 W T
1 V X GREEN R YELLOW
The one who likes Blue colour and S lives in the same flat.
Q lives northwest of S
Note: Case-2 gets rejected since Q lives northwest of S is not possible.

Case-1
Floor Flat-1 Color Flat-2 Color Flat-3 Color
3 Q RED U BLUE T
2 V S P
1 W X GREEN R YELLOW
Case-1 (A)
Floor Flat-1 Color Flat-2 Color Flat-3 Color
3 Q RED U BLUE T
2 W S P
1 V X GREEN R YELLOW

Case-2 [REJECTED]
Floor Flat-1 Color Flat-2 Color Flat-3 Color
3 V RED U P
2 W/ T
1 W/ X GREEN R YELLOW

Page 377 of 722

Subscribe the Xpress Video Course & Mock Test Package for Bank & Insurance Exams
If there are any suggestions/ errors in our PDFs Feel Free to contact us via this email: admin@exampundit.in
IBPS RRB Clerk Prelims – Ultra Practice Bundle PDF
Case-2 (A)
Floor Flat-1 Color Flat-2 Color Flat-3 Color
3 Q RED U BLUE P
2 W S T
1 V X GREEN R YELLOW
The one who likes Orange lives to the immediate west of the one who likes Grey and T likes White.
Persons like Orange colour and Black colour lives in the same flat and W does not like Black colour.
The one who likes Pink doesn’t live in topmost floor.
Note: T likes White as given. Finally P likes Pink in case-1-A
Case-1 gets rejected since W doesn’t like Black
Case-2-A gets rejected since the one who likes Pink doesn’t live in topmost floor

Case-1 (REJCETED)
Floor Flat-1 Color Flat-2 Color Flat-3 Color
3 Q RED U BLUE T WHITE
2 V ORANGE S GREY P
1 W BLACK X GREEN R YELLOW
Case-1 (A)
Floor Flat-1 Color Flat-2 Color Flat-3 Color
3 Q RED U BLUE T WHITE
2 W ORANGE S GREY P PINK
1 V BLACK X GREEN R YELLOW

Case-2 (A) (REJCETED)


Floor Flat-1 Color Flat-2 Color Flat-3 Color
3 Q RED U BLUE P PINK

Page 378 of 722

Subscribe the Xpress Video Course & Mock Test Package for Bank & Insurance Exams
If there are any suggestions/ errors in our PDFs Feel Free to contact us via this email: admin@exampundit.in
IBPS RRB Clerk Prelims – Ultra Practice Bundle PDF
2 W ORANGE S GREY T WHITE
1 V BLACK X GREEN R YELLOW
Final arrangement
Case-1 (A)
Floor Flat-1 Color Flat-2 Color Flat-3 Color
3 Q RED U BLUE T WHITE
2 W ORANGE S GREY P PINK
1 V BLACK X GREEN R YELLOW
Answers (21-25):
21) E (only 1 person)
22) A
23) E
24) C
25) D
Solutions (21-25):
Only two persons live below Pari who lives immediately below of Prem, who lives in Flat-2
No flat in ground floor is Vacant.
Floor Flat-1 Flat-2
6
5
4
3 PREM
2 PARI
1

Hema and Roma shared the same floor which is an even numbered such that Hema lives east of Roma.
Note: Here we get one more case.

Page 379 of 722

Subscribe the Xpress Video Course & Mock Test Package for Bank & Insurance Exams
If there are any suggestions/ errors in our PDFs Feel Free to contact us via this email: admin@exampundit.in
IBPS RRB Clerk Prelims – Ultra Practice Bundle PDF
Case-1 Case-2
Floor Flat-1 Flat-2 Floor Flat-1 Flat-2
6 6 ROMA HEMA
5 5
4 ROMA HEMA 4
3 PREM 3 PREM
2 PARI 2 PARI
1 1
Zeba lives towards South-East of Nitu who lives to the immediate south of Roma and share the same flat as of Roma.
Note: Here we get one more case from case-2
Case-1 Case-2 Case-2-A
Floor Flat-1 Flat-2 Floor Flat-1 Flat-2 Floor Flat-1 Flat-2
6 6 ROMA HEMA 6 ROMA HEMA
5 5 NITU 5 NITU
4 ROMA HEMA 4 ZEBA 4
3 NITU PREM 3 PREM 3 PREM
2 PARI 2 PARI 2 PARI
1 ZEBA 1 1 ZEBA
Zoya’s floor number is twice of the floor number of Neel, both are living in even numbered floor
There is only one person lives on the floor in which Nitu lives.
Note: Neel2nd floor and Zoya4th floor, only possibility. Thus case-1 gets eliminated.
Case-1 (REJECTED) Case-2 Case-2-A
Floor Flat-1 Flat-2 Floor Flat-1 Flat-2 Floor Flat-1 Flat-2
6 6 ROMA HEMA 6 ROMA HEMA
5 5 NITU VACANT 5 NITU VACANT
4 ROMA HEMA 4 ZOYA ZEBA 4 ZOYA/ ZOYA/
3 NITU PREM 3 PREM 3 PREM

Page 380 of 722

Subscribe the Xpress Video Course & Mock Test Package for Bank & Insurance Exams
If there are any suggestions/ errors in our PDFs Feel Free to contact us via this email: admin@exampundit.in
IBPS RRB Clerk Prelims – Ultra Practice Bundle PDF
2 PARI 2 NEEL PARI 2 NEEL PARI
1 ZEBA 1 1 ZEBA
Zoya lives north-west of either Bela or Aman
Neither Bela nor Aman lives west of Prem.
Note: Case-2-A gets eliminated since Zoya lives north-west of either Bela or Aman is not possible.
Position of Aman and Bela is not confirmed.

Case-2 Case-2-A (REJECTED)


Floor Flat-1 Flat-2 Floor Flat-1 Flat-2
6 ROMA HEMA 6 ROMA HEMA
5 NITU VACANT 5 NITU VACANT
4 ZOYA ZEBA 4 ZOYA/ ZOYA/
3 VACANT PREM 3 PREM
2 NEEL PARI 2 NEEL PARI
1 AMAN/BELA BELA/AMAN 1 ZEBA
FINAL Arrangement
Case-2
Floor Flat-1 Flat-2
6 ROMA HEMA
5 NITU VACANT
4 ZOYA ZEBA
3 VACANT PREM
2 NEEL PARI
1 AMAN/BELA BELA/AMAN
Answers (26-30):
26) D
27) A
28) E
Page 381 of 722

Subscribe the Xpress Video Course & Mock Test Package for Bank & Insurance Exams
If there are any suggestions/ errors in our PDFs Feel Free to contact us via this email: admin@exampundit.in
IBPS RRB Clerk Prelims – Ultra Practice Bundle PDF
29) C
30) B
Solutions (26-30):
Step I:
Shreya lives in an even numbered Floor in Flat-B.
One who speaks Marathi lives immediately below of Shreya.

CASE-1
Floor Flat-A Language Flat-B Language
4
3
2 SHREYA
1 MARATHI
CASE-2
Floor Flat-A Language Flat-B Language
4 SHREYA
3 MARATHI
2
1
Sonam who speaks Kannada lives in Flat-A of an odd numbered floor.
Sonam lives south-west of Shreya.
Sakshi lives second to above Surbhi in Flat-B.
Case-2 we get one more possibility
CASE-1
Floor Flat-A Language Flat-B Language
4
3 SAKSHI

Page 382 of 722

Subscribe the Xpress Video Course & Mock Test Package for Bank & Insurance Exams
If there are any suggestions/ errors in our PDFs Feel Free to contact us via this email: admin@exampundit.in
IBPS RRB Clerk Prelims – Ultra Practice Bundle PDF
2 SHREYA
1 SONAM KANNADA SURBHI MARATHI

CASE-2
Floor Flat-A Language Flat-B Language
4 SHREYA
3 SONAM KANNADA SAKSHI MARATHI
2
1 SURBHI

CASE-2-A
Floor Flat-A Language Flat-B Language
4 SHREYA
3 SAKSHI MARATHI
2
1 SONAM KANNADA SURBHI

Shikha speaks Hindi lives to the west of Sakshi


Note: Case-2 gets rejected since Shikha speaks Hindi lives to the west of Sakshi is not possible.
CASE-1
Floor Flat-A Language Flat-B Language
4
3 SHIKHA HINDI SAKSHI
2 SHREYA
1 SONAM KANNADA SURBHI MARATHI

CASE-2 (REJECTED)

Page 383 of 722

Subscribe the Xpress Video Course & Mock Test Package for Bank & Insurance Exams
If there are any suggestions/ errors in our PDFs Feel Free to contact us via this email: admin@exampundit.in
IBPS RRB Clerk Prelims – Ultra Practice Bundle PDF
Floor Flat-A Language Flat-B Language
4 SHREYA
3 SONAM KANNADA SAKSHI MARATHI
2
1 SURBHI

CASE-2-A
Floor Flat-A Language Flat-B Language
4 SHREYA
3 SHIKHA HINDI SAKSHI MARATHI
2
1 SONAM KANNADA SURBHI
There is a gap of one Floor between the both who speaks Tamil & Punjabi and both of them live in an even numbered
floor in Flat-A.
Sunita speaks Tamil and lives above the one who speaks Punjabi.
Persons speaking Tamil and Bengali shared the same Floor.
CASE-1
Floor Flat-A Language Flat-B Language
4 SUNITA TAMIL BENGALI
3 SHIKHA HINDI SAKSHI
2 PUNJABI SHREYA
1 SONAM KANNADA SURBHI MARATHI

CASE-2-A
Floor Flat-A Language Flat-B Language
4 SUNITA TAMIL SHREYA BENGALI
3 SHIKHA HINDI SAKSHI MARATHI

Page 384 of 722

Subscribe the Xpress Video Course & Mock Test Package for Bank & Insurance Exams
If there are any suggestions/ errors in our PDFs Feel Free to contact us via this email: admin@exampundit.in
IBPS RRB Clerk Prelims – Ultra Practice Bundle PDF
2 PUNJABI
1 SONAM KANNADA SURBHI
The one who speaks Urdu lives north-east of Suman.
Case-2-A gets eliminated since above said condition is possible.

CASE-2-A (REJECTED)
Floor Flat-A Language Flat-B Language
4 SUNITA TAMIL SHREYA BENGALI
3 SHIKHA HINDI SAKSHI MARATHI
2 PUNJABI
1 SONAM KANNADA SURBHI
Finally, Sanjhi lives in Flat-B of Floor-4 and Shreya speaks English.
CASE-1
Floor Flat-A Language Flat-B Language
4 SUNITA TAMIL SANJHI BENGALI
3 SHIKHA HINDI SAKSHI URDU
2 SUMAN PUNJABI SHREYA ENGLISH
1 SONAM KANNADA SURBHI MARATHI
FINAL ARRANGEMENT
CASE-1
Floor Flat-A Language Flat-B Language
4 SUNITA TAMIL SANJHI BENGALI
3 SHIKHA HINDI SAKSHI URDU
2 SUMAN PUNJABI SHREYA ENGLISH
1 SONAM KANNADA SURBHI MARATHI
Answers (31-35):
31) D
32) C
Page 385 of 722

Subscribe the Xpress Video Course & Mock Test Package for Bank & Insurance Exams
If there are any suggestions/ errors in our PDFs Feel Free to contact us via this email: admin@exampundit.in
IBPS RRB Clerk Prelims – Ultra Practice Bundle PDF
33) A
34) B
35) E
Solutions (31-35):
Step I:
Person who has Textbook lives to the extreme west of the one who has Adda247 and to the South-west of the one
who has Gradeup.
As per these lines four cases will arrive:

Case-1
Floor Flat-1 Test series Flat-2 Test series Flat-3 Test series
2 GRADEUP
1 TEXTBOOK ADDA247
Case-2
Floor Flat-1 Test series Flat-2 Test series Flat-3 Test series
2 GRADEUP
1 TEXTBOOK ADDA247

Case-1-A
Floor Flat-1 Test series Flat-2 Test series Flat-3 Test series
2 GRADEUP
1 TEXTBOOK ADDA247
Case-2-A
Floor Flat-1 Test series Flat-2 Test series Flat-3 Test series
2 GRADEUP
1 TEXTBOOK ADDA247
Step II:Akbar has series of Exampundit shared the same Flat and Floor number.
Note: Akbar lives either on FLAT-1 of Floor-1 (OR) FLAT-2 of Floor-2. Thus Case-1-A and Case-2-A gets rejected.
Page 386 of 722

Subscribe the Xpress Video Course & Mock Test Package for Bank & Insurance Exams
If there are any suggestions/ errors in our PDFs Feel Free to contact us via this email: admin@exampundit.in
IBPS RRB Clerk Prelims – Ultra Practice Bundle PDF
Case-1
Floor Flat-1 Test series Flat-2 Test series Flat-3 Test series
2 AKBAR EXAMPUNDIT GRADEUP
1 TEXTBOOK ADDA247
Case-2
Floor Flat-1 Test series Flat-2 Test series Flat-3 Test series
2 AKBAR EXAMPUNDIT GRADEUP
1 TEXTBOOK ADDA247

Case-1-A (REJECTED)
Floor Flat-1 Test series Flat-2 Test series Flat-3 Test series
2 GRADEUP
1 TEXTBOOK ADDA247
Case-2-A (REJECTED)
Floor Flat-1 Test series Flat-2 Test series Flat-3 Test series
2 GRADEUP
1 TEXTBOOK ADDA247
Step III:
Ashish and Akash shared the same floor and none of them has Textbook.
Neither Ashish nor Akash shared the same flat with the one who has Textbookseries.
Note: As per above condition both Ashish and Akash lives Flat-2 and Flat-3 of Floor 2 or 1.
Case-1
Floor Flat-1 Test series Flat-2 Test series Flat-3 Test series
2 AKBAR EXAMPUNDIT GRADEUP
1 TEXTBOOK ASHISH/AKASH ADDA247 AKASH/ASHISH
Case-2

Page 387 of 722

Subscribe the Xpress Video Course & Mock Test Package for Bank & Insurance Exams
If there are any suggestions/ errors in our PDFs Feel Free to contact us via this email: admin@exampundit.in
IBPS RRB Clerk Prelims – Ultra Practice Bundle PDF
Floor Flat-1 Test series Flat-2 Test series Flat-3 Test series
2 AKBAR EXAMPUNDIT GRADEUP
1 TEXTBOOK ASHISH/AKASH AKASH/ASHISH ADDA247
Step IV:
Arnav who has Gradeup lives to the North-East of Ashish.
Aditya lives in the same Flat and Floor number.
Note: Aditya lives on FLAT-1 of Floor-1, only possibility in both cases.
Case-1
Floor Flat-1 Test series Flat-2 Test series Flat-3 Test series
2 AKBAR EXAMPUNDIT ARNAV GRADEUP
1 ADITYA TEXTBOOK ASHISH ADDA247 AKASH
Case-2
Floor Flat-1 Test series Flat-2 Test series Flat-3 Test series
2 AKBAR EXAMPUNDIT ARNAV GRADEUP
1 ADITYA TEXTBOOK ASHISH AKASH ADDA247
Step-V: Person who has IBPS Guide and the Person having Exampundit shared the same flat.
Case-1 gets rejected since above said condition is not possible.
Finally, Aarav has Olivboard and lives in Flat-1 of Floor-2

Case-1 (REJECTED)
Floor Flat-1 Test series Flat-2 Test series Flat-3 Test series
2 AKBAR EXAMPUNDIT ARNAV GRADEUP
1 ADITYA TEXTBOOK ASHISH ADDA247 AKASH
Case-2
Floor Flat-1 Test series Flat-2 Test series Flat-3 Test series
2 AARAV OLIVEBOARD AKBAR EXAMPUNDIT ARNAV GRADEUP
1 ADITYA TEXTBOOK ASHISH IBPS-GUIDE AKASH ADDA247
The final arrangement will be:
Page 388 of 722

Subscribe the Xpress Video Course & Mock Test Package for Bank & Insurance Exams
If there are any suggestions/ errors in our PDFs Feel Free to contact us via this email: admin@exampundit.in
IBPS RRB Clerk Prelims – Ultra Practice Bundle PDF
Case-2
Floor Flat-1 Test series Flat-2 Test series Flat-3 Test series
2 AARAV OLIVEBOARD AKBAR EXAMPUNDIT ARNAV GRADEUP
1 ADITYA TEXTBOOK ASHISH IBPS-GUIDE AKASH ADDA247
Answers (36-40):
36) E
37) A
38) B
39) B
40) B
Solutions (36-40):
Komal who lives in flat-1 of floor-2 lives immediately below to the one who works in BOI.
Floor Flat-1 BANKS Flat-2 BANKS Flat-3 BANKS Flat-4 BANKS
3 BOI
2 KOMAL
1
Nishu shared the same floor and flat number.
Nishu lives immediately above of Rahul who works in Canara bank and lives to the immediate east of Tani.
Note: Nishu can live on Flat-1 of Floor-1 (OR) Flat-2 of Floor-2 (OR) Flat-3 of Floor-3 as per 1st condition. Nishu can’t
live on Flat-1 of Floor-1 as per 2nd condition.

CASE-1
Floor Flat-1 BANKS Flat-2 BANKS Flat-3 BANKS Flat-4 BANKS
3 BOI
2 KOMAL NISHU
1 TANI RAHUL CANARA
CASE-2

Page 389 of 722

Subscribe the Xpress Video Course & Mock Test Package for Bank & Insurance Exams
If there are any suggestions/ errors in our PDFs Feel Free to contact us via this email: admin@exampundit.in
IBPS RRB Clerk Prelims – Ultra Practice Bundle PDF
Floor Flat-1 BANKS Flat-2 BANKS Flat-3 BANKS Flat-4 BANKS
3 BOI NISHU
2 KOMAL TANI RAHUL CANARA
1
Tani and Chhaya shared the same flat, Similarly, Komal and Amit shared the same flat.
Amit doesn’t live west of Chhaya
Note: Case-1 gets rejected since above said 1st condition is not possible.
Case-2 we get one more possibility.

CASE-1 (REJECTED)
Floor Flat-1 BANKS Flat-2 BANKS Flat-3 BANKS Flat-4 BANKS
3 BOI
2 KOMAL NISHU
1 TANI RAHUL CANARA
CASE-2
Floor Flat-1 BANKS Flat-2 BANKS Flat-3 BANKS Flat-4 BANKS
3 BOI CHHAYA NISHU
2 KOMAL TANI RAHUL CANARA
1 AMIT
CASE-2-A
Floor Flat-1 BANKS Flat-2 BANKS Flat-3 BANKS Flat-4 BANKS
3 AMIT BOI NISHU
2 KOMAL TANI RAHUL CANARA
1 CHHAYA
Tani lives on the floor which has one vacant flat.
Manish who works in UCO bank lives in flat-4 of floor-1 and shared the flat with Mohini.
CASE-2

Page 390 of 722

Subscribe the Xpress Video Course & Mock Test Package for Bank & Insurance Exams
If there are any suggestions/ errors in our PDFs Feel Free to contact us via this email: admin@exampundit.in
IBPS RRB Clerk Prelims – Ultra Practice Bundle PDF
Floor Flat-1 BANKS Flat-2 BANKS Flat-3 BANKS Flat-4 BANKS
3 BOI CHHAYA NISHU MOHINI
2 KOMAL TANI RAHUL CANARA VACANT
1 AMIT MANISH UCO
CASE-2-A
Floor Flat-1 BANKS Flat-2 BANKS Flat-3 BANKS Flat-4 BANKS
3 AMIT BOI NISHU MOHINI
2 KOMAL TANI RAHUL CANARA VACANT
1 CHHAYA MANISH UCO
Sejal who works in PNB lives to the east of the Nitin who lives in an odd numbered flat, does not works in BOI.
Note: Nitin must lives flat-1. Nitin can’t live in flat-3 since Sejal who works in PNB lives to the east of the Nitin. Case-
2 gets rejected since Nitin does not works in BOI. Also Flat-2 of floor-3 must be vacant since all 10 persons are
occupied.
CASE-2 (REJECTED)
Floor Flat-1 BANKS Flat-2 BANKS Flat-3 BANKS Flat-4 BANKS
3 NITIN BOI CHHAYA NISHU MOHINI
2 KOMAL TANI RAHUL CANARA VACANT
1 AMIT MANISH UCO
CASE-2-A
Floor Flat-1 BANKS Flat-2 BANKS Flat-3 BANKS Flat-4 BANKS
3 AMIT BOI VACANT NISHU MOHINI
2 KOMAL TANI RAHUL CANARA VACANT
1 NITIN CHHAYA SEJAL PNB MANISH UCO
The one who works in OBC lives to the west of the person works in SBI.
Neither Chhaya Nor Tani works in SBI.
CASE-2-A
Floor Flat-1 BANKS Flat-2 BANKS Flat-3 BANKS Flat-4 BANKS

Page 391 of 722

Subscribe the Xpress Video Course & Mock Test Package for Bank & Insurance Exams
If there are any suggestions/ errors in our PDFs Feel Free to contact us via this email: admin@exampundit.in
IBPS RRB Clerk Prelims – Ultra Practice Bundle PDF
3 AMIT BOI VACANT NISHU OBC MOHINI SBI
2 KOMAL TANI RAHUL CANARA VACANT
1 NITIN CHHAYA SEJAL PNB MANISH UCO
Person works in BOB lives towards the North-East of the person works in UBI and the person works in BOM lives
towards the South-East of the person works in Indian bank.
CASE-2-A
Floor Flat-1 BANKS Flat-2 BANKS Flat-3 BANKS Flat-4 BANKS
3 AMIT BOI VACANT NISHU OBC MOHINI SBI
2 KOMAL INDIAN TANI BOB RAHUL CANARA VACANT
1 NITIN UBI CHHAYA BOM SEJAL PNB MANISH UCO
The Final result is:
CASE-2-A
Floor Flat-1 BANKS Flat-2 BANKS Flat-3 BANKS Flat-4 BANKS
3 AMIT BOI VACANT NISHU OBC MOHINI SBI
2 KOMAL INDIAN TANI BOB RAHUL CANARA VACANT
1 NITIN UBI CHHAYA BOM SEJAL PNB MANISH UCO

Answers (41-45):
41) E
42) D
43) A
44) B
45) A
Solutions (41-45):
Vishu lives in flat-2 of one of the floors.
Dev lives in flat-1 and to the immediate left of Ved who lives below of Vishu.
Case-1 Case-2

Page 392 of 722

Subscribe the Xpress Video Course & Mock Test Package for Bank & Insurance Exams
If there are any suggestions/ errors in our PDFs Feel Free to contact us via this email: admin@exampundit.in
IBPS RRB Clerk Prelims – Ultra Practice Bundle PDF
Floor Flat-1 Flat-2 Flat-3 Floor Flat-1 Flat-2 Flat-3
3 VISHU 3 VISHU
2 DEV VED 2
1 1 DEV VED

Case-3
Floor Flat-1 Flat-2 Flat-3
3
2 VISHU
1 DEV VED
Flat towards east of Ved is empty.
Case-1 Case-2
Floor Flat-1 Flat-2 Flat-3 Floor Flat-1 Flat-2 Flat-3
3 VISHU 3 VISHU
2 DEV VED VACANT 2
1 1 DEV VED VACANT

Case-3
Floor Flat-1 Flat-2 Flat-3
3
2 VISHU
1 DEV VED VACANT
Vedi lives on the same flat and floor number.
Note: Vedi lives of Flat-1 of Floor-1 (OR) Flat-2 of Floor-2 (OR) Flat-3 of Floor-3. Thus we get sub cases for case-1
and case-2
Case-1 Case-2
Floor Flat-1 Flat-2 Flat-3 Floor Flat-1 Flat-2 Flat-3

Page 393 of 722

Subscribe the Xpress Video Course & Mock Test Package for Bank & Insurance Exams
If there are any suggestions/ errors in our PDFs Feel Free to contact us via this email: admin@exampundit.in
IBPS RRB Clerk Prelims – Ultra Practice Bundle PDF
3 VISHU 3 VISHU
2 DEV VED VACANT 2 VEDI
1 VEDI 1 DEV VED VACANT

Case-1-A Case-2-A
Floor Flat-1 Flat-2 Flat-3 Floor Flat-1 Flat-2 Flat-3
3 VISHU VEDI 3 VISHU VEDI
2 DEV VED VACANT 2
1 1 DEV VED VACANT

Case-3
Floor Flat-1 Flat-2 Flat-3
3 VEDI
2 VISHU
1 DEV VED VACANT
Dolly lives below of Vedi in the same flat as of Vedi.
Vivek lives to the left of Durga who lives on floor-3.
Case-1 and Case-2 gets rejected since 1st condition is not possible.
Case-1 (REJECTED) Case-2 (REJECTED)
Floor Flat-1 Flat-2 Flat-3 Floor Flat-1 Flat-2 Flat-3
3 VISHU 3 VISHU
2 DEV VED VACANT 2 VEDI
1 VEDI 1 DEV VED VACANT
Case-1-A and Case-2-A gets rejected since 2nd condition is not possible.
Case-1-A (REJECTED) Case-2-A (REJECTED)
Floor Flat-1 Flat-2 Flat-3 Floor Flat-1 Flat-2 Flat-3

Page 394 of 722

Subscribe the Xpress Video Course & Mock Test Package for Bank & Insurance Exams
If there are any suggestions/ errors in our PDFs Feel Free to contact us via this email: admin@exampundit.in
IBPS RRB Clerk Prelims – Ultra Practice Bundle PDF
3 VISHU VEDI 3 VISHU VEDI
2 DEV VED VACANT 2 DOLLY
1 DOLLY 1 DEV VED VACANT
Case-3
Floor Flat-1 Flat-2 Flat-3
3 VIVEK DURGA VEDI
2 VISHU DOLLY
1 DEV VED VACANT
Devesh lives to the left of Dolly.
Case-3
Floor Flat-1 Flat-2 Flat-3
3 VIVEK DURGA VEDI
2 DEVESH VISHU DOLLY
1 DEV VED VACANT
FINAL ARRANGEMENT
Case-3
Floor Flat-1 Flat-2 Flat-3
3 VIVEK DURGA VEDI
2 DEVESH VISHU DOLLY
1 DEV VED VACANT
Answers (46-50):
46) B
47) E
48) C
49) D
50) E
Solutions (46-50):
Two vacant flats are on odd numbered floors and are on even numbered flats.
Page 395 of 722

Subscribe the Xpress Video Course & Mock Test Package for Bank & Insurance Exams
If there are any suggestions/ errors in our PDFs Feel Free to contact us via this email: admin@exampundit.in
IBPS RRB Clerk Prelims – Ultra Practice Bundle PDF
Floor Flat-1 Flat-2 Flat-3
4
3 VACANT
2
1 VACANT

The flat between Prerna and Piku is vacant and Prerna lives one of the floors above the floor on which Piku lives.
Parul lives immediately to the west of the flat in which Piku lives.
Floor Flat-1 Flat-2 Flat-3
4 PRERNA
3 VACANT
2 PARUL PIKU
1 VACANT

Pihu and Pankhi live in the same flats which are numbered 1.
Pihu and Payal live on the same floor and no one else lives on that floor.
Pankhi and Prachi do not live on an odd-numbered floor.
Note: Here we get one more case with respect to Pihu and Payal.
Case-1 Case-2
Floor Flat-1 Flat-2 Flat-3 Floor Flat-1 Flat-2 Flat-3
4 PANKHI PRERNA 4 PANKHI PRERNA
3 PANKHI VACANT 3 PIHU VACANT PAYAL
2 PARUL PIKU 2 PARUL PIKU
1 PIHU VACANT PAYAL 1 PANKHI VACANT
Panchi does not live on flat number 1.
Panchi lives on the floor immediately above Prachi in the same flat.
Pankhi and Prachi do not live on an odd-numbered floor.
Note: Case-2 gets rejected since Prachi do not live on an odd-numbered floor.
Page 396 of 722

Subscribe the Xpress Video Course & Mock Test Package for Bank & Insurance Exams
If there are any suggestions/ errors in our PDFs Feel Free to contact us via this email: admin@exampundit.in
IBPS RRB Clerk Prelims – Ultra Practice Bundle PDF

Case-1 Case-2 [REJECTED]


Floor Flat-1 Flat-2 Flat-3 Floor Flat-1 Flat-2 Flat-3
4 PANKHI PRERNA 4 PANKHI PRERNA
3 VACANT PANCHI 3 PIHU VACANT PAYAL
2 PARUL PIKU PRACHI 2 PARUL PIKU
1 PIHU VACANT PAYAL 1 VACANT
Palak lives on an odd numbered flat and in an even numbered floor.
Finally, Pankhuri lives in flat-1 of Floor-3
Case-1
Floor Flat-1 Flat-2 Flat-3
4 PANKHI PRERNA PALAK
3 PANKHURI VACANT PANCHI
2 PARUL PIKU PRACHI
1 PIHU VACANT PAYAL
FINAL ARRANGEMENT
Case-1
Floor Flat-1 Flat-2 Flat-3
4 PANKHI PRERNA PALAK
3 PANKHURI VACANT PANCHI
2 PARUL PIKU PRACHI
1 PIHU VACANT PAYAL

18). Certain Number- Circle Seating

Page 397 of 722

Subscribe the Xpress Video Course & Mock Test Package for Bank & Insurance Exams
If there are any suggestions/ errors in our PDFs Feel Free to contact us via this email: admin@exampundit.in
IBPS RRB Clerk Prelims – Ultra Practice Bundle PDF
Directions (1-5): Study the following information (c) Y
carefully and answer the questions given below: (d) W
Certain numbers of persons are sitting around a circular (e) None of these
table. All the persons are facing towards the centre. There 4. What is the position of P with respect to Z?
are two person sit between U and T. Either O or S is (a) Second to the left
neighbour of U. Q sits third to the right of P. S is 3rd to (b) Third to the left
the left of T. V is immediate right to W. X sits to the (c) Second to the right
immediate left of Y. Z sits second to right of S. Z sit next (d) Fourth to the right
to N. The number of persons sitting between Q and R is (e) None of these
same as between P and Q. O and P is sitting next to each 5. W is an immediate neighbour of which of the
other. There is only two persons sit between V and Y. V following?
sits third to the right of U. The number of persons sitting (a) U
between N and T is same as between O and T. (b) Q
1. What is the position of N with respect to U? (c) Both (b) and (d)
(a) Fifth to left (d) V
(b) Immediate right (e) Cannot be determined
(c) Immediate left Directions (6-10): Study the following information
(d) Second to right carefully and answer the questions given below:
(e) None of these A certain number of people are sitting around a circle
2. How many persons are sitting in the circular table? facing the center. At most one person is sitting between D
(a) Twelve and B. One person is sitting between C and E. A sits to the
(b) Fourteen immediate left of B. Six people are sitting between B and
(c) Nine E. Five people are sitting between C and D. Three people
(d) Thirteen are sitting between C, who is not an immediate neighbour
(e) None of these of F and A. E sits third to the right of F. D is not an
3. Who is sitting seventh to the right of Z? immediate neighbour of B.
(a) N 6. How many persons are sitting in the circular table?
(b) R (a) Ten
Page 398 of 722

Subscribe the Xpress Video Course & Mock Test Package for Bank & Insurance Exams
If there are any suggestions/ errors in our PDFs Feel Free to contact us via this email: admin@exampundit.in
IBPS RRB Clerk Prelims – Ultra Practice Bundle PDF
(b) Fifteen (e) Only II
(c) Thirteen 10. Which of the following statement is false?
(d) Nine (a) D sits fourth to the right of A.
(e) Twelve (b) F sits third to left of E.
7. Who among the following is sitting at a gap of three (c) B sits fourth to right of C.
seats from A? (d) F sits immediate right D.
(I) C (e) None of these
(II) F Directions (11-15): Study the following information
(III) E carefully and answer the questions given below:
(a) Only I Certain number of persons sits around a circular table
(b) Only III facing the center. Only four persons are sit between P and
(c) Only II and III T, who is on the immediate left of Q. S is sixth to the right
(d) Only I and II of U. R is at a gap of two persons from U, who is adjacent
(e) Only II to T. Only one person sits between R and V. V sits fifth to
8. What is the position of A with respect to C? the right of Q. V sits on the immediate left of P.
(a) Immediate left 11. How many people are sitting in the circular table?
(b) Fourth to the right (a) Nine
(c) Immediate right (b) Eleven
(d) Fourth to the left (c) Twelve
(e) None of these (d) Ten
9. How many persons sit between E and A? (e) Cannot be determine
(I) Five Q12. How many known people are sitting between S
(II) Seven and U, when counted from left of S?
(III) Six (a) Three
(a) Only I (b) One
(b) Only III (c) Two
(c) Only I and III (d) Four
(d) Only I and II (e) Five
Page 399 of 722

Subscribe the Xpress Video Course & Mock Test Package for Bank & Insurance Exams
If there are any suggestions/ errors in our PDFs Feel Free to contact us via this email: admin@exampundit.in
IBPS RRB Clerk Prelims – Ultra Practice Bundle PDF
13. What is the position of R with respect to Q? and E. Not more than four persons sit between G and F
(a) Third to the left from left of F. Only three persons sit between A and C.
(b) Third to the right 16. Who is sitting second to the right of A?
(c) Fifth to the left (a) F
(d) Ninth to the right (b) B
(e) Eight to right (c) D
14. Who amongst the followings is third to the right of (d) E
P? (e) Cannot be determined
(a) R 17. Who sits at immediate right of E?
(b) S (a) P
(c) T (b) U
(d) Q (c) O
(e) Cannot be determined (d) D
15. How many people can sit between T and V? (e) Cannot be determined
(a) Three 18. How many persons sitting in a circle?
(b) Five (a) Seven
(c) Two (b) Nine
(d) Four (c) Eleven
(e) One (d) Ten
Directions (16-20): Study the following information (e) None of these
carefully and answer the questions given below: 19. Who among the following person sits sixth to the
A certain number of persons are sitting around a circular right of F?
table facing towards the center. Only six persons sit (a) A
between E and B. G is not an immediate neighbor of E. G (b) D
sits fourth to the left of B. B is an immediate neighbor of (c) B
C. Only three persons sit between A and C. There are five (d) E
persons sit between E and D from the left of E. F sits (e) None of these
second to the left of E. Only three persons sit between C
Page 400 of 722

Subscribe the Xpress Video Course & Mock Test Package for Bank & Insurance Exams
If there are any suggestions/ errors in our PDFs Feel Free to contact us via this email: admin@exampundit.in
IBPS RRB Clerk Prelims – Ultra Practice Bundle PDF
20. How many persons are sitting between C and G, (e) More than five
when counting right of C? 23. What is the position of H with respect to D?
(a) Two (a) Third to the left
(b) One (b) Immediate to the left
(c) Five (c) Fifth to the left
(d) Four (d) Seventh to the right
(e) More than five (e) Eight to right
Directions (21-25): Study the following information 24. If M sits immediate left of A then who sits fourth to
carefully and answer the questions given below: the left of M?
A certain number of persons are sitting around a circular (a) B
table facing towards the center. D sits second to the right (b) E
B. G sits fourth to the right of F. There are two person sits (c) D
between A and H. D sits third to the left of E. F sits seventh (d) F
to the right of B. C sits immediate right of F. Only three (e) Cannot be determined
people sit between A and B. Only four people sit between 25. How many people can sit between H and G from
B and C. C is not an immediate neighbor of A. left of H?
21. How many people are sitting in the circular table? (a) Three
(a) Nine (b) Five
(b) Eleven (c) Two
(c) Twelve (d) Four
(d) Thirteen (e) One
(e) Cannot be determine Directions (26-30): Study the following information
22. How many people are sitting between F and B, carefully and answer the questions given below:
when counted from left of F? A certain number of persons are sitting around a circular
(a) Three table facing towards the center. Q is to the immediate left
(b) One of B. H is sitting fifth to the right of B. D sits second to
(c) Two the left of Q. D is not an immediate neighbor of H.Only
(d) Five three persons are sitting between D and H. P is sitting fifth
Page 401 of 722

Subscribe the Xpress Video Course & Mock Test Package for Bank & Insurance Exams
If there are any suggestions/ errors in our PDFs Feel Free to contact us via this email: admin@exampundit.in
IBPS RRB Clerk Prelims – Ultra Practice Bundle PDF
to the right of M, who is an immediate neighbor of Q. Only (III) Six
seven persons are sitting between Y and M from left of (a) Only I
M.Z sits fourth to the right of P. (b) Only III
26. How many persons are sitting in the circular table? (c) Either I or III
(a) Ten (d) EitherI or II
(b) Fifteen (e) Only II
(c) Thirteen 30. Which of the following statement is false?
(d) Nine (a) Y sits fifth to the right of D.
(e) Twelve (b) Z sits third to left of M.
27. Who among the following is sitting at a gap of three (c) B sits fourth to right of Z.
seats from P? (d) P sits immediate right Y.
(I) Z (e) None of these
(II) Q Directions (31-35): Study the following information
(III) D carefully and answer the questions given below:
(a) Only I A certain number of persons are sitting around a circular
(b) Only III table facing towards the center. J is sitting fifth to the left
(c) Only II and III of K. There is three seats between K and L. There is five
(d) Only I and II persons sit between M and O. J is an immediate neighbor
(e) Only II of P. K sits fourth to the right of P. There is three seats
28. What is the position of Y with respect to D? between J and M. M is an immediate neighbor of L. N sits
(a) Immediate left fourth to the right of O. Not more than 13 people and not
(b) Fourth to the right less than 10 sit in the circular table.
(c) Fifth to the right 31. If B sits third to the right of J, then what is the
(d) Fourth to the left position of L with respect to J?
(e) None of these (a) Second to the left
29. How many persons sit between Z and Y? (b) Immediate right
(I) Four (c) Immediate left
(II) Seven (d) Sixth to the right
Page 402 of 722

Subscribe the Xpress Video Course & Mock Test Package for Bank & Insurance Exams
If there are any suggestions/ errors in our PDFs Feel Free to contact us via this email: admin@exampundit.in
IBPS RRB Clerk Prelims – Ultra Practice Bundle PDF
(e) None of these A certain number of persons are sitting around a circular
32. How many persons are sitting in the circular table? table facing towards the center. More than ten persons sit
(a) Twelve in this circular table. E sits next to D but not an immediate
(b) Fourteen neighbor of H. There are four seats between F and H.
(c) Nine There are six persons sit between A and H. H sits fifth to
(d) Thirteen the left of G. A sits third to the left of F. B sits at a gap of
(e) None of these three seats from G. H is not an immediate neighbor of B.
33. Who is sitting seventh to the right of P? C sits second to the left of H. Three persons sit between C
(a) J and D.
(b) O 36. Who is sitting second to the right of D?
(c) M (a) F
(d) N (b) B
(e) None of these (c) D
34. What is the position of K with respect to M? (d) E
(a) Second to the left (e) Cannot be determined
(b) Third to the left 37. Who is sitting second to the left of A?
(c) Second to the right (a) G
(d) Fourth to the right (b) B
(e) None of these (c) C
35. P is an immediate neighbour of which of the (d) D
following? (e) Cannot be determined
(a) J 38. How many persons sitting in a circle?
(b) O (a) Seventeen
(c) Both (a) and (b) (b) Ninteen
(d) L (c) Eleven
(e) (d) Fifteen
Directions (36-40): Study the following information (e) None of these
carefully and answer the questions given below:
Page 403 of 722

Subscribe the Xpress Video Course & Mock Test Package for Bank & Insurance Exams
If there are any suggestions/ errors in our PDFs Feel Free to contact us via this email: admin@exampundit.in
IBPS RRB Clerk Prelims – Ultra Practice Bundle PDF
39. Who among the following person sits sixth to the (e) None of these
right of E? 42. How many persons are sitting in the circular table?
(a) A (a) Twelve
(b) D (b) Fourteen
(c) B (c) Nine
(d) C (d) Thirteen
(e) None of these (e) None of these
40. How many persons are sitting between B and D, 43. Who is sitting sixth to the right of B?
when counting right of D? (a) D
(a) Two (b) F
(b) Six (c) A
(c) Five (d) G
(d) Four (e) None of these
(e) Three 44. What is the position of G with respect to A?
(a) Second to the left
Directions (41-45): Study the following information (b) Third to the right
carefully and answer the questions given below: (c) Second to the right
A certain number of persons are sitting around a circular (d) Fourth to the right
table facing towards the center. There are two persons sit (e) None of these
between A and G. There is only one person sit between G 45. A is an immediate neighbour of which of the
and D. There is only person between B and C who does following?
not sit next to D. F sits third to the right of G. A is an (a) B
immediate neighbor of B. D sits third to the left of B. (b) C
41. What is the position of D with respect to F? (c) Both (a) and (b)
(a) Fifth to left (d) D
(b) Immediate right (e) Cannot be determined
(c) Immediate left Directions (46-50): Study the following information
(d) Second to right carefully and answer the questions given below:
Page 404 of 722

Subscribe the Xpress Video Course & Mock Test Package for Bank & Insurance Exams
If there are any suggestions/ errors in our PDFs Feel Free to contact us via this email: admin@exampundit.in
IBPS RRB Clerk Prelims – Ultra Practice Bundle PDF
A certain number of persons are sitting around a circular 48. How many persons sitting in a circle?
table facing towards the center. There are more than eight (a) Seven
persons sit in the circular table. D sits second to the left of (b) Nine
A. C sits third to the left of G. G sits fourth to the left of (c) Eleven
D. C sits fourth to the right of E. H sits second to the left (d) Ten
of D. There are two person sits between H and B, who is (e) None of these
an immediate neighbor of G. F sits at a gap of three seats 49. Who among the following person sits fifth to the
from left of B. There are two persons sit between A and right of F?
B. (a) A
46. Who is sitting second to the right of C? (b) G
(a) F (c) B
(b) B (d) E
(c) A (e) None of these
(d) E 50. How many persons are sitting between F and G,
(e) Cannot be determined when counting right of F?
47. Who sits at immediate right of G? (a) Two
(a) P (b) One
(b) U (c) Four
(c) O (d) Five
(d) D (e) More than five
(e) Cannot be determined

18). Certain Number- Circle Seating - Solution and Detailed Explanation

SOLUTION (1-5): 2. Two persons are sitting between U and T.


Explanation in detail: 3. Either O or S is neighbour of U.
1. S is 3rd to the left of T.
Page 405 of 722

Subscribe the Xpress Video Course & Mock Test Package for Bank & Insurance Exams
If there are any suggestions/ errors in our PDFs Feel Free to contact us via this email: admin@exampundit.in
IBPS RRB Clerk Prelims – Ultra Practice Bundle PDF

4. The number of persons sitting between N and T 7. O and P is sitting next to each other.
is same as between O and T. 8. X sits to the immediate left of Y.
5. Z sits second to right of S. 9. Y sits second to left of N.
6. Z sit next to N.
In CASE1 Z is not an immediate neighbor of N So,
CASE1 is cancelled out.

Page 406 of 722

Subscribe the Xpress Video Course & Mock Test Package for Bank & Insurance Exams
If there are any suggestions/ errors in our PDFs Feel Free to contact us via this email: admin@exampundit.in
IBPS RRB Clerk Prelims – Ultra Practice Bundle PDF
Final arrangement:

10. The number of persons sitting between Q and R


is same as between P and Q.
11. Q sits third to the right of P. 1. a
2. d
3. E (V is sitting 7th right of Z)
4. c
5. c

SOLUTION (6-10):
Explanation in detail:
1. E sits third to the right of F.
2. Six people are sitting between B and E.
3. A sits to the immediate left of B.

12. V sits third to the right of U.


13. V is immediate right to W.
14. There is only two person sit between V and Y.

Page 407 of 722

Subscribe the Xpress Video Course & Mock Test Package for Bank & Insurance Exams
If there are any suggestions/ errors in our PDFs Feel Free to contact us via this email: admin@exampundit.in
IBPS RRB Clerk Prelims – Ultra Practice Bundle PDF

6. Five people are sitting between C and D.


7. D is not an immediate neighbour of B.
8. One person is sitting between D and B.

4. Three people are sitting between C, who is not an


immediate neighbour of F and A.
5. One person is sitting between C and E.

Page 408 of 722

Subscribe the Xpress Video Course & Mock Test Package for Bank & Insurance Exams
If there are any suggestions/ errors in our PDFs Feel Free to contact us via this email: admin@exampundit.in
IBPS RRB Clerk Prelims – Ultra Practice Bundle PDF
There is gap of two between B and D in CASE2 so
CASE2 is cancelled out.
Final arrangement:

6. c
7. d
8. b
9. c
10. e [option a & c are False i.e. both a & c)

SOLUTION (11-15):
Explanation in detail: 4. V sits fifth to the right of Q.
1. Only four persons are sit between P and T 5. V sits on the immediate left of P.
2. T is on the immediate left of Q. In CASE1 V sits on the immediate right of P so CASE1
3. U is adjacent to T. is cancelled out.

Page 409 of 722

Subscribe the Xpress Video Course & Mock Test Package for Bank & Insurance Exams
If there are any suggestions/ errors in our PDFs Feel Free to contact us via this email: admin@exampundit.in
IBPS RRB Clerk Prelims – Ultra Practice Bundle PDF

Final arrangement:

6. S is sixth to the right of U.


7. R is at a gap of two persons from U.
8. Only one person sits between R and V.

11. c
12. C
Page 410 of 722

Subscribe the Xpress Video Course & Mock Test Package for Bank & Insurance Exams
If there are any suggestions/ errors in our PDFs Feel Free to contact us via this email: admin@exampundit.in
IBPS RRB Clerk Prelims – Ultra Practice Bundle PDF
13. c
14. e
15. b

SOLUTION (16-20):
Explanation in detail:
1. G sits fourth to the left of B.
2. B is an immediate neighbor of C.

3. Only three persons sit between C and E.


4. G is not an immediate neighbor of E.

Page 411 of 722

Subscribe the Xpress Video Course & Mock Test Package for Bank & Insurance Exams
If there are any suggestions/ errors in our PDFs Feel Free to contact us via this email: admin@exampundit.in
IBPS RRB Clerk Prelims – Ultra Practice Bundle PDF
In CASE1 and CASE1a there is no such space for E so,
CASE1 and CASE1a is cancelled out.

7. Only six persons sit between E and B from right of


E.
8. There are five persons sit between E and D from
the left of E.

5. F sits second to the left of E.


6. Not more than four persons sit between G and F
from left of F.
In CASE2 there is more than four persons sit
between F and G So, CASE2 is cancelled out.

Page 412 of 722

Subscribe the Xpress Video Course & Mock Test Package for Bank & Insurance Exams
If there are any suggestions/ errors in our PDFs Feel Free to contact us via this email: admin@exampundit.in
IBPS RRB Clerk Prelims – Ultra Practice Bundle PDF
Final arrangement:

16. c
17. e 4. F sits seventh to the right of B.
18. d 5. C sits immediate right of F.
19. b
20. e

SOLUTION (21-25):
Explanation in detail:

1. Only three people sit between A and B.


2. Only four people sit between B and C.
3. C is not an immediate neighbor of A.

Page 413 of 722

Subscribe the Xpress Video Course & Mock Test Package for Bank & Insurance Exams
If there are any suggestions/ errors in our PDFs Feel Free to contact us via this email: admin@exampundit.in
IBPS RRB Clerk Prelims – Ultra Practice Bundle PDF

F sits second to right of C, so CASE1 is cancelled


out.
6. D sits second to the right B.
7. G sits fourth to the right of F.
8. There are twopersons sit between A and H.
9. D sits third to the left of E. 21. d
22. e
23. b
24. e
25. c

SOLUTION (26-30):
Explanation in detail:
1. Q is to the immediate left of B.
2. H is sitting fifth to the right of B.
3. D sits second to the left of Q.
4. D is not an immediate neighbor of H.
Final arrangement:

Page 414 of 722

Subscribe the Xpress Video Course & Mock Test Package for Bank & Insurance Exams
If there are any suggestions/ errors in our PDFs Feel Free to contact us via this email: admin@exampundit.in
IBPS RRB Clerk Prelims – Ultra Practice Bundle PDF
7. Only seven persons are sitting between Y and M
from left of M.
8. Z sits fourth to the right of P.

5. Only three persons are sitting between D and H.


6. P is sitting fifth to the right of M, who is an
immediate neighbor of Q.
Final arrangement:

Page 415 of 722

Subscribe the Xpress Video Course & Mock Test Package for Bank & Insurance Exams
If there are any suggestions/ errors in our PDFs Feel Free to contact us via this email: admin@exampundit.in
IBPS RRB Clerk Prelims – Ultra Practice Bundle PDF
26. e
27. d
28. c
29. c
30. c
SOLUTION (31-35):
Explanation in detail:
1. K sits fourth to the right of P.
2. There is three seats between K and L.

5. There is three seats between J and M.


6. M is an immediate neighbor of L.
NOTE: Not more than 13 people and not less than
10 sit in the circular table.

3. J is sitting fifth to the left of K.


4. J is an immediate neighbor of P.

7. N sits fourth to the right of O.

Page 416 of 722

Subscribe the Xpress Video Course & Mock Test Package for Bank & Insurance Exams
If there are any suggestions/ errors in our PDFs Feel Free to contact us via this email: admin@exampundit.in
IBPS RRB Clerk Prelims – Ultra Practice Bundle PDF
8. There is five persons sit between M and O. 34. b
35. c

SOLUTION (36-40):
Explanation in detail:

1. A sit third to the left of F.


2. There are four seats between F and H.

Final arrangement:

31. e
32. a
33. c

Page 417 of 722

Subscribe the Xpress Video Course & Mock Test Package for Bank & Insurance Exams
If there are any suggestions/ errors in our PDFs Feel Free to contact us via this email: admin@exampundit.in
IBPS RRB Clerk Prelims – Ultra Practice Bundle PDF
3. There are six persons sit between A and H. 4. H sits fifth to the left of G.
NOTE: More than persons it in this circular table. 5. B sits at a gap of three seats from G.
So, CASE2 is cancelled out. 6. H is not an immediate neighbor of B.

7. C sits second to the left of H.


8. Three persons sit between C and D.
9. E sits next to D but not an immediate neighbor of
H.

Page 418 of 722

Subscribe the Xpress Video Course & Mock Test Package for Bank & Insurance Exams
If there are any suggestions/ errors in our PDFs Feel Free to contact us via this email: admin@exampundit.in
IBPS RRB Clerk Prelims – Ultra Practice Bundle PDF
Final arrangement:

3. There are two persons sit between A and G.


36. e
4. G is not an immediate neighbor of D.
37. a
38. d
39. c
40. b
SOLUTION (41-45):
Explanation in detail:

1. A is an immediate neighbor of B.
2. D sits third to the left of B.

Page 419 of 722

Subscribe the Xpress Video Course & Mock Test Package for Bank & Insurance Exams
If there are any suggestions/ errors in our PDFs Feel Free to contact us via this email: admin@exampundit.in
IBPS RRB Clerk Prelims – Ultra Practice Bundle PDF
5. There is only one person sit between G and D. Final arrangement:
6. There is only person between B and C who does
not sit next to D.
In CASE1 C sits next to D so, CASE1 is
cancelled out.

41. c
42. c
43. a
44. b
45. c

7. F sits third to the right of G. SOLUTION (46-50):


Explanation in detail:
1. D sits second to the left of A.
2. H sits second to the left of D.

Page 420 of 722

Subscribe the Xpress Video Course & Mock Test Package for Bank & Insurance Exams
If there are any suggestions/ errors in our PDFs Feel Free to contact us via this email: admin@exampundit.in
IBPS RRB Clerk Prelims – Ultra Practice Bundle PDF
3. There are two persons sits between H and B,
4. B is an immediate neighbor of G.
5. G sits fourth to the left of D.
6. There are two persons sit between A and B.
7. There are more than eight persons sit in the
circular table.

Final arrangement:

8. C sits fourth to the right of E.


46. b
9. C sits third to the left of G.
47. e
10. F sits at a gap of three seats from left of B.
48. d
49. b
50. c

Page 421 of 722

Subscribe the Xpress Video Course & Mock Test Package for Bank & Insurance Exams
If there are any suggestions/ errors in our PDFs Feel Free to contact us via this email: admin@exampundit.in
IBPS RRB Clerk Prelims – Ultra Practice Bundle PDF

19). Floor Based Puzzle

Direction (1-5): Study the following information (c) D


carefully and answer the question given below: (d) H
There are eight people A, B, C, D, E, F, G and H are living (e) None of these
in a 8 floor Building on different floors from top to bottom Q4. Four of the following five are alike in a certain way
(such as ground floor is numbered as 1 and top floor is based on the given arrangement and thus form a
numbered as 8) but not necessarily in the same order. group. Which is the one that does not belong to that
There is a gap of three floors between B and D and both group?
of them live on odd number of floors. F lives just above H (a) C
who lives on an even numbered floor. A lives on floor (b) A
number 6. Only one person lives between B and E. D lives (c) H
above A. Three persons live between C and H. (d) B
Q1. Who lives on floor number five? (e) G
(a) C Q5. Who lives on 4TH floor?
(b) F (a) D
(c) D (b) B
(d) G (c) E
(e) None of these (d) F
Q2. How many persons live between C and B? (e) None of these
(a) One Direction (6-10): Study the following information
(b) Three carefully and answer the question given below:
(c) Five There are eight people A, B, C, D, E, F, G and H are living
(d) Four in a 8 floor Building on different floors from top to bottom
(e) None of these (such as ground floor is numbered as 1 and top floor is
Q3. Who lives immediately above E? numbered as 8) but not necessarily in the same order. F
(a) G lives on an odd number floor but does not live on 3rd floor.
(b) A H lives immediate below F. More than two persons live
Page 422 of 722

Subscribe the Xpress Video Course & Mock Test Package for Bank & Insurance Exams
If there are any suggestions/ errors in our PDFs Feel Free to contact us via this email: admin@exampundit.in
IBPS RRB Clerk Prelims – Ultra Practice Bundle PDF
between G and H. There are six persons live between A Q9. What is the difference between the floor number
and G. D lives immediate above E but live below B. C of A and H?
does not live above F. E does not live immediate above G. (a) One
Q6. Who among the following live on the odd floor? (b) Three
I. F (c) Five
II. B (d) Two
III. C (e) None of these
(a) Only I Q10. Who lives immediately above F?
(b) Only II (a) G
(c) Only I and III (b) A
(d) Only I and II (c) H
(e) Only III (d) D
Q7. Who among the following live immediate below C? (e) C
(a) D Direction (11-15): Study the following information
(b) A carefully and answer the question given below:
(c) G There are seven people A, B, C, D, E, F and G are living
(d) B in a 7-floor building on different floors from top to bottom
(e) None of these (such as ground floor is numbered as 1 and top floor is
Q8. Four of the following five are alike in a certain way numbered as 7) but not necessarily in the same order. A
based on the given arrangement and thus form a life on the 2nd floor. E live immediately above F. B live
group. Which is the one that does not belong to that above F. A life immediately above D and immediately
group? below G. There are four persons live between D and C.
(a) G 11. Who among the following lives on ground floor?
(b) A (a) A
(c) H (b) D
(d) D (c) B
(e) C (d) G
(e) None of these
Page 423 of 722

Subscribe the Xpress Video Course & Mock Test Package for Bank & Insurance Exams
If there are any suggestions/ errors in our PDFs Feel Free to contact us via this email: admin@exampundit.in
IBPS RRB Clerk Prelims – Ultra Practice Bundle PDF
12. Who among the following lives immediately below There are Eleven people A, B, C, D, E, F, G, H, I, J and K
B? are living in a 11 floor building on different floors from
(a) G top to bottom (such as ground floor is numbered as 1 and
(b) D top floor is numbered as 11) but not necessarily in the
(c) C same order. A live above F. I live above E but not
(d) F immediately. E does not live immediately above or
(e) None of these immediately below C.G lives on 7th floor. Two person live
13. How many persons live between D and B? between G and H. D live immediately above H. There is
(a) One as many person living above D as below B. Five persons
(b) Three live between F and K, who lives below G. Only one person
(c) Five live between G and C.J lives below C
(d) Two 16. Who among the following live immediately above
(e) None of these C?
14. Who among the following lives on Top floor? (a) D
(a) E (b) I
(b) A (c) F
(c) F (d) E
(d) C (e) None of these
(e) None of these 17. How many persons are there between E and B?
15. Which of the following combination is false? (a) One
(a) Floor no. 2 – A (b) Two
(b) Floor no. 7 – B (c) Three
(c) Floor no. 5 – C (d) Four
(d) Floor no. 3 – G (e) None of these
(e) None of these 18. Who among the following lives on the topmost
Direction (16-20): Study the following information floor?
carefully and answer the question given below: (a) G
(b) C
Page 424 of 722

Subscribe the Xpress Video Course & Mock Test Package for Bank & Insurance Exams
If there are any suggestions/ errors in our PDFs Feel Free to contact us via this email: admin@exampundit.in
IBPS RRB Clerk Prelims – Ultra Practice Bundle PDF
(c) J Q21. Who lives on floor number Two?
(d) D (a) C
(e) None of these (b) F
19. Which of the following combinations is/are true? (c) D
(a) Floor no. 2 – D (d) E
(b) Floor no. 7 – F (e) None of these
(c) Floor no. 5 – C Q22. How many persons live between G and H?
(d) Floor no. 8 – E (a) One
(e) None of these (b) Three
20. Which of the following live just below J? (c) Two
(a) K (d) Four
(b) E (e) None of these
(c) B Q23. Who lives immediate below B?
(d) I (a) G
(e) None of these (b) A
Direction (21-25): Study the following information (c) D
carefully and answer the question given below: (d) H
There are eight people A, B, C, D, E, F, G and H are living (e) No One
in a 8 floor Building on different floors from top to bottom Q24. Four of the following five are alike in a certain
(such as ground floor is numbered as 1 and top floor is way based on the given arrangement and thus form a
numbered as 8) but not necessarily in the same order. No group. Which is the one that does not belong to that
two persons according to alphabetical order live adjacent group?
to each other (A does not live just above or below B, B (a) C
does not live just above or below A and C, and so on). D (b) E
live either 2nd or 7th floor. Only two persons live between (c) F
C and F. E lives immediately below C. G live above F but (d) B
not at the topmost floor. Only one person sit between B (e) G
and F. C live on 6th floor. Q25. Who lives on 4TH floor?
Page 425 of 722

Subscribe the Xpress Video Course & Mock Test Package for Bank & Insurance Exams
If there are any suggestions/ errors in our PDFs Feel Free to contact us via this email: admin@exampundit.in
IBPS RRB Clerk Prelims – Ultra Practice Bundle PDF
(a) D Q27. Who among the following live immediate below
(b) H G?
(c) E (a) D
(d) F (b) A
(e) None of these (c) C
Direction (26-30): Study the following information (d) B
carefully and answer the question given below: (e) None of these
There are eight people A, B, C, D, E, F, G and H are living Q28. Four of the following five are alike in a certain
in a 8 floor Building on different floors from top to bottom way based on the given arrangement and thus form a
(such as ground floor is numbered as 1 and top floor is group. Which is the one that does not belong to that
numbered as 8) but not necessarily in the same order. A group?
lives on an odd floor below 6th floor. There are two (a) G
persons live between A and B. G lives on one of the odd (b) F
floor but not at bottom most floor. There is only one (c) B
person live between B and D, who live just below E. The (d) D
number of persons living between F and C is same as the (e) C
number of person living between F and D. C live on of the Q29. What is the difference between the floor number
floor below F. of B and G?
Q26. Who among the following live on the Even floor? (a) Five
I. F (b) Three
II. B (c) Two
III. C (d) One
(a) Only I (e) None of these
(b) Only II Q30. Who lives immediately above H?
(c) Only I and III (a) G
(d) Only I and II (b) A
(e) All I, II, III (c) B
(d) D
Page 426 of 722

Subscribe the Xpress Video Course & Mock Test Package for Bank & Insurance Exams
If there are any suggestions/ errors in our PDFs Feel Free to contact us via this email: admin@exampundit.in
IBPS RRB Clerk Prelims – Ultra Practice Bundle PDF
(e) C (a) G
Direction (31-35): Study the following information (b) C
carefully and answer the question given below: (c) D
There are eight people A, B, C, D, E, F, G and H are living (d) H
in a 8 floor Building on different floors from top to bottom (e) No One
(such as ground floor is numbered as 1 and top floor is Q34. Four of the following five are alike in a certain
numbered as 8) but not necessarily in the same order. A way based on the given arrangement and thus form a
lives on an even-numbered floor but not on the floor group. Which is the one that does not belong to that
numbered second or fourth. Only three floors are there group?
between A and B. Only two people live between C and E. (a) F
F lives on a floor above D. There are equal numbers of (b) C
floors between the floors on which E and B live and (c) E
between the floors on which A and E live. H lives (d) H
immediately below D’s floor. G lives immediately below (e) G
A’s floor. Q35. Who lives on 6th floor?
Q31. Who lives on floor number Two? (a) D
(a) C (b) A
(b) F (c) E
(c) A (d) F
(d) E (e) G
(e) None of these Direction (36-40): Study the following information
Q32. How many persons live between D and F? carefully and answer the question given below:
(a) One There are eight people S, T, U, V, W, X, Y and Z are living
(b) Three in a 8 floor Building on different floors from top to bottom
(c) Two (such as ground floor is numbered as 1 and top floor is
(d) Four numbered as 8) but not necessarily in the same order.
(e) None of these Only one person lives between the floors of U and V who
Q33. Who lives immediate below B? lives on any floor below the floor of W. Z lives on the
Page 427 of 722

Subscribe the Xpress Video Course & Mock Test Package for Bank & Insurance Exams
If there are any suggestions/ errors in our PDFs Feel Free to contact us via this email: admin@exampundit.in
IBPS RRB Clerk Prelims – Ultra Practice Bundle PDF
floor immediately above the floor on which T lives. U group. Which is the one that does not belong to that
lives on the third numbered floor. Only one person lives group?
between the floors of W and X. W lives above X. Only (a) T
two persons live between the floors of U and Y. (b) X
Q36. ________ is at the bottommost position. (c) W
(a) Z (d) Y
(b) Y (e) Z
(c) Either (a) or (b) Q40. What is the difference between the floor number
(d) U of Y and X?
(e) None of these (a) One
Q37. Who among the following live on the odd floor? (b) Three
I. W (c) Four
II. T (d) Two
III. V (e) None of these
(a) Only I Direction (41-45): Study the following information
(b) Only II carefully and answer the question given below:
(c) Only II and III There are eight people A, B, C, D, J, K, L and M are living
(d) Only I and II in a 8 floor Building on different floors from top to bottom
(e) Only III (such as ground floor is numbered as 1 and top floor is
Q38. Who among the following live immediate below numbered as 8) but not necessarily in the same order.
X? J lives immediately above the floor on which C lives but
(a) Z not on an even numbered floor. D lives on the floor which
(b) V is a prime number but not on the 3rd floor. K lives
(c) T immediately below the floor on which B lives. The person
(d) Y lives between A and C is not more than three and not less
(e) None of these than two. A live on the topmost floor. There is one person
Q39. Four of the following five are alike in a certain living between the floors on which J and D lives. M lives
way based on the given arrangement and thus form a immediately above the floor on which B lives.
Page 428 of 722

Subscribe the Xpress Video Course & Mock Test Package for Bank & Insurance Exams
If there are any suggestions/ errors in our PDFs Feel Free to contact us via this email: admin@exampundit.in
IBPS RRB Clerk Prelims – Ultra Practice Bundle PDF
41. Who among the following live immediately above (b) D
M? (c) J
(a) D (d) B
(b) C (e) None of these
(c) J Direction (46-50): Study the following information
(d) L carefully and answer the question given below:
(e) None of these There are seven people A, B, C, D, E, F, G are living in a
42. How many persons are there between A and M? 7 floor Building on different floors from top to bottom
(a) One (such as ground floor is numbered as 1 and top floor is
(b) Two numbered as 7) but not necessarily in the same order. G
(c) Three lives on one of the odd numbered floor above D but not
(d) Four on top floor. F lives immediately before C, who lives in
(e) None of these bottom most floor. A lives on an odd numbered floor but
43. Who among the following lives on the bottom most not on the floor numbered 3. B lives on immediately
floor? below A. Only one person lives between B and E and they
(a) L lives on even numbered floor. G lives above E.
(b) C 46. Who among the following live on the even floor?
(c) J I. F
(d) K II. B
(e) None of these III. C
44. Which of the following combinations is/are false? (a) Only I
(a) Floor no. 2 – K (b) Only II
(b) Floor no. 7 – D (c) Only I and II
(c) Floor no. 5 – J (d) Only II and III
(d) Floor no. 8 – A (e) Only III
(e) None of these
45. Which of the following live just below L? 47. Who among the following live immediately below
(a) K C?
Page 429 of 722

Subscribe the Xpress Video Course & Mock Test Package for Bank & Insurance Exams
If there are any suggestions/ errors in our PDFs Feel Free to contact us via this email: admin@exampundit.in
IBPS RRB Clerk Prelims – Ultra Practice Bundle PDF
(a) D 49. What is the difference between the floor number of
(b) A A and C?
(c) G (a) One
(d) B (b) Three
(e) No one (c) Six
48. Four of the following five are alike in a certain way (d) Two
based on the given arrangement and thus form a (e) None of these
group. Which is the one that does not belong to that 50. Who lives immediately above G?
group? (a) B
(a) G (b) A
(b) A (c) E
(c) E (d) D
(d) D (e) C
(e) C

19). Floor Based Puzzle - Solution and Detailed Explanation

SOLUTION (1-5):
Explanation in detail:
1. A lives on floor number 6.
2. There is a gap of three floors between B and D and both
of them live on odd number of floors.
3. D lives above A.

Page 430 of 722

Subscribe the Xpress Video Course & Mock Test Package for Bank & Insurance Exams
If there are any suggestions/ errors in our PDFs Feel Free to contact us via this email: admin@exampundit.in
IBPS RRB Clerk Prelims – Ultra Practice Bundle PDF
2. d
3. a
4. d
5. e

SOLUTION (6-10):
Explanation in detail:
1. F lives on an odd number floor but does not live on 3rd
floor.
2. H lives immediate below F.

4. F lives just above H who lives on an even numbered


floor.
5. Only one person lives between B and E.
6. Three persons live between C and H.

3. More than two persons live between G and H.


4. There are six persons live between A and G.

1. b

Page 431 of 722

Subscribe the Xpress Video Course & Mock Test Package for Bank & Insurance Exams
If there are any suggestions/ errors in our PDFs Feel Free to contact us via this email: admin@exampundit.in
IBPS RRB Clerk Prelims – Ultra Practice Bundle PDF
Final Arrangement:

5. D lives immediate above E but live below B.


6. C does not live above F.
7. E does not live immediate above G. 6. d
7. c
8. a
9. d
10. b
SOLUTION (11-15):
Explanation in detail:
1. A lives on the 2nd floor.
2. A lives immediately above D and immediately below
G.

So CASE2 is cancelled out.


Page 432 of 722

Subscribe the Xpress Video Course & Mock Test Package for Bank & Insurance Exams
If there are any suggestions/ errors in our PDFs Feel Free to contact us via this email: admin@exampundit.in
IBPS RRB Clerk Prelims – Ultra Practice Bundle PDF

3. There are four persons live between D and C.


11. b
4. D live below C.
12. c
13. c
14. e
15. c
SOLUTION (16-20):
Explanation in detail:
1. G lives on 7th floor.
2. Two person live between G and H.
3. D live immediately above H.

5. E live immediately above F.


6. B live above F.

Page 433 of 722

Subscribe the Xpress Video Course & Mock Test Package for Bank & Insurance Exams
If there are any suggestions/ errors in our PDFs Feel Free to contact us via this email: admin@exampundit.in
IBPS RRB Clerk Prelims – Ultra Practice Bundle PDF

4. There is as many person living living above D as below


B. 7. Only one person live between G and C.
8. I live above E but not immediately.
9. E does not live immediately above or immediately
below C.

There is no floor empty for B in CASE2 so CASE2 is


cancelled out.
5. A live above F.
6. Five persons live between F and K, who lives below G.
16. b
17. a
18. d
19. c

Page 434 of 722

Subscribe the Xpress Video Course & Mock Test Package for Bank & Insurance Exams
If there are any suggestions/ errors in our PDFs Feel Free to contact us via this email: admin@exampundit.in
IBPS RRB Clerk Prelims – Ultra Practice Bundle PDF
20. b Note: G live on 7th floor not on 4th floor because no two
SOLUTION (21-25): persons according to alphabetical order live adjacent
Explanation in detail: to each other.
1. C live on 6th floor. 5. Only one person sit between B and F.
2. E lives immediately below C. 6. D live either 2nd or 7th floor.

H cannot sit on topmost floor because no two persons


according to alphabetical order live adjacent to each
3. Only two persons live between C and F. other.
4. G live above F but not at the topmost floor.

21. c

Page 435 of 722

Subscribe the Xpress Video Course & Mock Test Package for Bank & Insurance Exams
If there are any suggestions/ errors in our PDFs Feel Free to contact us via this email: admin@exampundit.in
IBPS RRB Clerk Prelims – Ultra Practice Bundle PDF
22. c
23. e
24. a
25. b
SOLUTION (26-30):
Explanation in detail:
1. A lives on an odd floor below 6th floor.
2. There are two persons live between A and B.

There is no floor empty for E in CASE3B So, CASE3B


is cancelled out.
There is no floor empty for F in CASE2 So, CASE2 is
cancelled out.
5. C live on of the floor below F.
6. G lives on one of the odd floor but not at bottom most
floor.
So, CASE1is cancelled out.
3. There is only one person live between B and D, who
live just below E.
4. The number of persons living between F and C is same
as the number of person living between F and D.

Page 436 of 722

Subscribe the Xpress Video Course & Mock Test Package for Bank & Insurance Exams
If there are any suggestions/ errors in our PDFs Feel Free to contact us via this email: admin@exampundit.in
IBPS RRB Clerk Prelims – Ultra Practice Bundle PDF
SOLUTION (31-35):
Explanation in detail:
1. A lives on an even-numbered floor but not on the floor
numbered second or fourth.
2. Only three floors are there between A and B.

Final arrangement:

3. There are equal numbers of floors between the floors on


which E and B live and between the floors on which A and
E live.
4. Only two people live between C and E.

26. e
27. c
28. a
29. a
30. e
Page 437 of 722

Subscribe the Xpress Video Course & Mock Test Package for Bank & Insurance Exams
If there are any suggestions/ errors in our PDFs Feel Free to contact us via this email: admin@exampundit.in
IBPS RRB Clerk Prelims – Ultra Practice Bundle PDF
F lives on a floor below D in CASE2B so CASE2B is
cancelled out.
Final arrangement:

5. F lives on a floor above D.


6. H lives immediately below D’s floor.
7. G lives immediately below A’s floor.

31. e
32. c
33. b
34. c
35. c
SOLUTION (36-40):
Explanation in detail:
1. U lives on the third numbered floor.
2. Only two persons live between the floors of U and Y.

There is no flat empty for H and D in CASE2A so


CASE2A is cancelled out.

Page 438 of 722

Subscribe the Xpress Video Course & Mock Test Package for Bank & Insurance Exams
If there are any suggestions/ errors in our PDFs Feel Free to contact us via this email: admin@exampundit.in
IBPS RRB Clerk Prelims – Ultra Practice Bundle PDF

3. Only one person lives between the floors of W and X. There is no space for Z and T in CASE1A so CASE1A

4. W lives above X. is cancelled out.


Final arrangement:

5. Only one person lives between the floors of U and V.


6. U lives on one of the floor below the floor of W. 36. e

7. Z lives on the floor immediately above the floor on 37. c

which T lives. 38. b


39. a
40. c
SOLUTION (41-45):

Page 439 of 722

Subscribe the Xpress Video Course & Mock Test Package for Bank & Insurance Exams
If there are any suggestions/ errors in our PDFs Feel Free to contact us via this email: admin@exampundit.in
IBPS RRB Clerk Prelims – Ultra Practice Bundle PDF
Explanation in detail: 5. The person lives between A and C is not more than three
1. D lives on the floor which is a prime number but not on and not less than two.
the 3rd floor.
2. A lives on the topmost floor.

In CASE2A and CASE2B the person lives between A


and C is more than three and less than two.
So, CASE2A and CASE2B is cancelled out.
3. J lives immediately above the floor on which C lives
6. M lives immediately above the floor on which B lives.
but not on an even numbered floor.
7. K lives immediately below the floor on which B lives.
4. There is one person living between the floors on which
J and D lives.

41. b
In CASE1 J live in even floor so CASE1 is cancelled
42. d
out.
43. d
Page 440 of 722

Subscribe the Xpress Video Course & Mock Test Package for Bank & Insurance Exams
If there are any suggestions/ errors in our PDFs Feel Free to contact us via this email: admin@exampundit.in
IBPS RRB Clerk Prelims – Ultra Practice Bundle PDF
44. a There is no floor empty for F in CASE1 so CASE1 is
45. c cancelled out.
SOLUTION (46-50):
Explanation in detail:
1. G lives on one of the odd numbered floor above D but
not on top floor.
2. Only three persons lives between C and G.

4. A lives on an odd numbered floor but not on the floor


numbered 3.
5. B lives on immediately below A.
6. Only one person lives between B and E and they lives
on even numbered floor.
3. F lives immediately before C.

46. c
47. e
Page 441 of 722

Subscribe the Xpress Video Course & Mock Test Package for Bank & Insurance Exams
If there are any suggestions/ errors in our PDFs Feel Free to contact us via this email: admin@exampundit.in
IBPS RRB Clerk Prelims – Ultra Practice Bundle PDF
48. c 50. a
49. c

20). BOX Based Puzzle

Directions (1 – 5): Study the following information 3. Four of the following five are alike in a certain way
carefully and answer the questions given below: and hence form a group find which one of the following
Seven boxes were kept one above the other from top to does not belong to that group?
bottom. Only two boxes are kept between B and U. V is a) A-T
kept just above S. Only three boxes are kept between S b) L-T
and T. More than three boxes are kept below T. U is kept c) T-B
above T. L is kept above B and below A. d) B-S
1. Which of the following box is kept at the top most e) L-V
position? 4. Which of the following is true regarding L?
a) B I. Only three boxes are kept below L
b) L II. U is kept below L
c) S III. S is kept above L
d) A a) Both I and II
e) None of these b) Only II
2. Number of boxes kept below B is same as number of c) Only I
boxes placed above_______? d) Both I and III
a) T e) All I, II and III
b) L 5. Which of the following pair of boxes are not adjacent
c) A to each other?
d) U a) A-U
e) None of these b) L-B
c) S-V
Page 442 of 722

Subscribe the Xpress Video Course & Mock Test Package for Bank & Insurance Exams
If there are any suggestions/ errors in our PDFs Feel Free to contact us via this email: admin@exampundit.in
IBPS RRB Clerk Prelims – Ultra Practice Bundle PDF
d) U-L b) F
e) B-V c) H
Directions (6 – 10): Study the following information d) C
carefully and answer the questions given below: e) D
Eight boxes were kept one above the one in the form of a 9. Which of the following statement is true?
stack. Three boxes were kept between A and G. One box a) Two boxes were kept between A and F
was kept between C and D. Not more than two boxes were b) Other than the given options
kept between B and C. No boxes were kept between A and c) Box B was kept immediately above box G
D. Three boxes were kept between E and F. Two boxes d) Box A was kept at the top of the stack
were kept between B and F. Box H was kept immediately e) Two boxes were kept between D and E
below box E. E was kept at the top. F is kept above B. 10. Four of the following were in a certain group,
6. How many boxes were kept between G and E? which of the following that does not belong to that
a) Four group?
b) More than four a) AE
c) Three b) BC
d) Two c) AF
e) One d) DF
7. Which of the following box was kept immediately e) BC
above C? Directions (11 – 15): Study the following alphanumeric
a) H series carefully and answer the questions given below:
b) B
c) G Seven boxes A, B, C, D, E, F and G are placed one above
d) D another in a stack. All the boxes are different in colours
e) F i.e. Red, Green, Blue, Orange, Violet, White and Grey but
8. The number of boxes kept below box G was the same not necessarily in same order. Three boxes are placed
as between the number of boxes kept above box between box D and violet colour box. Box E is placed just
_______ below violet colour box. Red colour box is placed just
a) E above grey colour box. White colour box is placed just
Page 443 of 722

Subscribe the Xpress Video Course & Mock Test Package for Bank & Insurance Exams
If there are any suggestions/ errors in our PDFs Feel Free to contact us via this email: admin@exampundit.in
IBPS RRB Clerk Prelims – Ultra Practice Bundle PDF
above box A, which is orange colour. One box is placed d) E
between box D and box G. Box D and box E is neither red e) None of these
in colour nor grey in colour. Box A is not placed below 15. Four of the following five are alike in certain way
box E. Three boxes are placed between box F and box B. and hence they from a group, find the one that does
One box is placed between box B and Green colour box. not belong to that group?
11. Which of the following colours is box E? a) F-White
a) Violet b) A-Red
b) Green c) D-Grey
c) Blue d) C-Green
d) Red e) B-Violet
e) None of these Directions (16 – 20): Study the following information
12. Which of the following box is red in colour? and answer the given questions.
a) G Seven boxes, A, B, C, D, E, F, and G are kept one above
b) B another in different stacks but not necessarily in the same
c) D order. These stacks are numbered 1 to 7 from bottom to
d) C the top respectively.
e) None of these G is kept at an odd prime numbered position. Three boxes
13. How many boxes are placed between box D and box are stacked in between A and F, A is kept at a position
C? below G. Box B is kept immediately below box D. Box C
a) One is not adjacent to box A. Three boxes are kept between D
b) Three and G, D is kept above G.
c) Two 16. How many boxes are kept between B and E?
d) Four a) 2
e) None b) 1
14. Which of the following box is blue in colour? c) 3
a) G d) 4
b) D e) None of the above
c) C
Page 444 of 722

Subscribe the Xpress Video Course & Mock Test Package for Bank & Insurance Exams
If there are any suggestions/ errors in our PDFs Feel Free to contact us via this email: admin@exampundit.in
IBPS RRB Clerk Prelims – Ultra Practice Bundle PDF
17. Which of the following box is kept at the lowermost Ten boxes are placed in ten shelves from bottom to top
position? such that shelf number 1 is the bottommost shelf and shelf
a) E above the bottommost shelf is numbered as 2 and so on.
b) C Box A is placed on shelf number 5. Only three boxes are
c) F placed between the box A and J. Box E is placed
d) A immediately above the box J. Number of boxes placed
e) None of the above between box E and A is same as the number of boxes
18. Which of the following box is adjacent to box G? placed between box J and C. Box B is placed on even
a) B number shelf but immediately below the box F. Box B is
b) F not placed on sixth shelf. Only one box is placed between
c) A box F and G. Box G and X are placed adjacent to each
d) C other. Box D is placed on one of the shelves above the
e) None of the above shelf on which box W is placed.
19. In which position box E is kept? 21. Which box is placed on shelf number 9?
a) 2nd a) D
b) 3rd b) F
c) 5th c) J
d) 7th d) G
e) None of the above e) None of these
20. Box F is kept at the ________________ position. 22. Which box is placed two places below the box G?
a) 7th a) A
b) 4th b) X
c) 6th c) B
d) 2nd d) E
e) 5th e) None of these
Directions (21 – 25): Answer the questions based on the 23. __ box is placed on 2nd shelf?
information given below. a) Box W
b) Box B
Page 445 of 722

Subscribe the Xpress Video Course & Mock Test Package for Bank & Insurance Exams
If there are any suggestions/ errors in our PDFs Feel Free to contact us via this email: admin@exampundit.in
IBPS RRB Clerk Prelims – Ultra Practice Bundle PDF
c) Box G adjacent to X nor W is adjacent to V. Locker T is stacked
d) Box E below locker V. R is one of the locker.
e) None of these 26. If all the lockers are arranged alphabetically from
24. Box W is placed immediately below the box__? top to bottom, how many positions will remain the
a) Box D same?
b) Box G a) 3
c) Box F b) 2
d) Box B c) 1
e) Box C d) 0
25. Four of the following five are related to each in e) None of the above
some way and thus formed a group. Choose the one 27. How many lockers are kept below R?
which does not belong to that group. a) 2
a) Box D b) 1
b) Box B c) 3
c) Box A d) 4
d) Box X e) 5
e) Box E 28. Which of locker is kept immediately above V?
Directions (26 – 30): Study the following information a) W
and answer the given questions. b) Q
In a corporate office nine locker boxes for the new c) S
employees are installed in a single column. Locker W is d) X
stacked immediately above locker S. V is kept at an odd e) None of the above
numbered position above the locker X, which is kept 29. How many lockers are kept between W and T?
below Q. There are four boxes in between X and Q. U is a) 3
kept at the top of all the boxes. Three lockers are there b) 5
between P and V. Locker T, which is neither adjacent to S c) 2
nor Q, is kept immediately above locker P. Neither S is d) 4
e) 1
Page 446 of 722

Subscribe the Xpress Video Course & Mock Test Package for Bank & Insurance Exams
If there are any suggestions/ errors in our PDFs Feel Free to contact us via this email: admin@exampundit.in
IBPS RRB Clerk Prelims – Ultra Practice Bundle PDF
30. Which locker is stacked immediately above W? d) One
a) U e) None of these.
b) R 33. The box immediately above D has ___ pens.
c) Q a) 8
d) V b) 5
e) None of the above c) 7
Directions (31 – 35): Answer the questions based on the d) 9
information given below: e) None of these.
Six boxes A, G, C, D, E and F are kept one above the other 34. Which among the following doesn’t form a group?
like a stack. Each of these boxes contains different number a) F, A
of pens among 2, 5, 6, 7, 8 and 9. The bottommost box is b) A, E
numbered 1 and the topmost box is numbered 6. c) E, D
Box 5 contains 8 pens and is two boxes above D. Number d) D, F
of pens in D is in prime number. Box with 5 pens is e) C, G
immediately above the box with 2 pens. Box with 9 pens 35. How many boxes are between the box with highest
is immediately above G. Box with 6 pens is below box 4.C number of pens and the one with lowest number of
is immediately above the box with 6 pens. F is two boxes pens?
above E but doesn’t contain 9 pens. a) One
31. The sum of number of pens of F and A is ____. b) Four
a) 13 c) Three
b) 11 d) Two
c) 14 e) None of these.
d) 15 Directions (36 – 40): Study the following information
e) None of these. and answer the given questions.
32. How many boxes are below A? A certain number of boxes are placed one above the
a) Three another in such a way that box at the bottom is numbered
b) Two as 1 and box just above it is numbered 2 and so on. There
c) Four are not more than 14 boxes in the arrangement.
Page 447 of 722

Subscribe the Xpress Video Course & Mock Test Package for Bank & Insurance Exams
If there are any suggestions/ errors in our PDFs Feel Free to contact us via this email: admin@exampundit.in
IBPS RRB Clerk Prelims – Ultra Practice Bundle PDF
Only 2 boxes are kept between P and Q, where Q is kept 39. How many boxes are kept between T and R?
below P. Only 3 boxes are kept between R and Q. S is kept a) None
between Q and J, which is placed immediately below P. b) 1
Only 3 boxes are kept between R and B which is kept c) 2
either at the top or at the bottom of the stack. Only 2 boxes d) 3
are kept between R and T, which is not placed adjacent to e) More than 3
B. Box D is placed exactly between E and F. U is placed 40. Which among the following is kept at numbered 7?
exactly between T and W. W is placed above R. a) E
36. Which among the following boxes is placed b) T
immediately above R? c) W
a) E d) U
b) F e) D
c) W Directions (41 – 45): Study the following information
d) U carefully and answer the questions given below:
e) D Eight boxes - P, Q, R, S, T, U, V and W were kept one
37. How many boxes are kept below F? above the one in the form of a stack. Three boxes were
a) none kept between P and Q. No boxes were kept between P and
b) 1 V. Four boxes were kept between V and W. Two boxes
c) 2 were kept between S and Q. Box T was kept immediately
d) 3 below box R. More than two boxes were kept above box
e) Cannot be determined U.
38. If T is related to S and U is related to Q. In the same 41. Which of the following box is kept at the top of the
way, R is related to? stack?
a) B a) V
b) F b) P
c) E c) S
d) W d) R
e) None of these e) Q
Page 448 of 722

Subscribe the Xpress Video Course & Mock Test Package for Bank & Insurance Exams
If there are any suggestions/ errors in our PDFs Feel Free to contact us via this email: admin@exampundit.in
IBPS RRB Clerk Prelims – Ultra Practice Bundle PDF
42. How many boxes were kept between R and U? Eight boxes are kept one above the other. More than five
a) Two boxes are kept above box P. Box D is kept immediately
b) Three above box R and immediately below box G. Two boxes
c) None are kept between box R and box Z. Box N is kept below Z
d) One and above box L. Box T is kept below box L but not
e) Four immediate below. At least four boxes are kept below box
43. Which of the following box was kept immediately R.
after W? 46. Which of the following is not true?
a) Q a) Box P is kept above box T
b) V b) Box T is kept at the bottom
c) P c) There are two boxes kept below box L
d) S d) Box R is kept immediately above box L
e) T e) All are incorrect
44. How many boxes were kept below box T? 47. Number of boxes kept below box G is same as
a) Three number of boxes kept above_______?
b) Two a) N
c) More than three b) L
d) One c) P
e) None d) R
45. The number of boxes kept above box R was the e) None of these
same as the number of boxes kept below box ______ 48. How many boxes are kept above box Z?
a) W a) Two
b) U b) Three
c) S c) None
d) Q d) Five
e) T e) Four
Directions (46 – 50) Study the following information 49. Which box is kept exactly between box N and box
carefully and answer the questions given below: P?
Page 449 of 722

Subscribe the Xpress Video Course & Mock Test Package for Bank & Insurance Exams
If there are any suggestions/ errors in our PDFs Feel Free to contact us via this email: admin@exampundit.in
IBPS RRB Clerk Prelims – Ultra Practice Bundle PDF
a) L a) D
b) T b) Z
c) Z c) R
d) R d) T
e) None of these e) None of these
50. If box P is related to box L, box N is related to box
R, then in the same manner box G is related to
_________?

20). BOX Based Puzzle - Solution and Detailed Explanation

(1 – 5): Common Explanation: Only two boxes are kept between B and U. U is kept above
From the given statements, More than three boxes are kept T. So case-3 is eliminated. L is kept above B and below
below T. Only three boxes are kept between S and T. V is A. So case-2 is eliminated. Hence the final arrangement
kept just above S. So we have three possible cases i.e. is:
case-1, case-2 and case-3: Boxes
Case – 1 Case – 2 Case – 3 A
Boxes Boxes Boxes U
T T
T L
T B
V V
V S S
V S
S 1. Answer: D)
2. Answer: A)

Page 450 of 722

Subscribe the Xpress Video Course & Mock Test Package for Bank & Insurance Exams
If there are any suggestions/ errors in our PDFs Feel Free to contact us via this email: admin@exampundit.in
IBPS RRB Clerk Prelims – Ultra Practice Bundle PDF
3. Answer: B) B
4. Answer: C) No boxes were kept between A and D.
5. Answer: D) Box
(6 – 10): Common Explanation: E
Three boxes were kept between E and F. Two boxes were H
kept between B and F. Box H was kept immediately below A
box E. E was kept at the top. F is kept above B. D
Box F
E C
H G
B

F 6. Answer: B)
7. Answer: E)
8. Answer: C)
B 9. Answer: E)
Three boxes were kept between A and G. One box was 10. Answer: D)
kept between C and D. Not more than two boxes were kept
between B and C. (11 – 15): Common Explanation:
Box From the given statement, three boxes are placed between
E box D and violet colour box. Box E is placed just below
H violet colour box. White colour box is placed just above
A/G box A, which is orange colour. One box is placed between
D box D and box G. Box A is not placed below box E. Here
F we get two possibilities i.e. Case-1 and case-2.
C
A/G

Page 451 of 722

Subscribe the Xpress Video Course & Mock Test Package for Bank & Insurance Exams
If there are any suggestions/ errors in our PDFs Feel Free to contact us via this email: admin@exampundit.in
IBPS RRB Clerk Prelims – Ultra Practice Bundle PDF
G is kept at an odd prime numbered position.
Serial Number Boxes
7 D
6 B
5
4
3 G
Now, Red colour box is placed just above grey colour box. 2
Box D and box E is neither red in colour nor grey in 1
colour. Here case-2 ruled out. Three boxes are placed Three boxes are staked in between A and F, A is kept at a
between box F and box B. One box is placed between box position below G.
B and Green colour box. So, the final arrangement will be: Box C is not adjacent to box A.
Serial Number Boxes
7 D
6 B
5 F
4 C
3 G
11. Answer: B) 2 E
12. Answer: A) 1 A
13. Answer: B) Therefore, this is the final arrangement.
14. Answer: E) 16. Answer: C)
15. Answer: C) 17. Answer: D)
(16 – 20): Common Explanation: 18. Answer: D)
Box B is kept immediately below box D. 19. Answer: A)
Three boxes are kept between D and G, D is kept above 20. Answer: E)
G. (21 – 25): Common Explanation:

Page 452 of 722

Subscribe the Xpress Video Course & Mock Test Package for Bank & Insurance Exams
If there are any suggestions/ errors in our PDFs Feel Free to contact us via this email: admin@exampundit.in
IBPS RRB Clerk Prelims – Ultra Practice Bundle PDF
Box A is placed on shelf number 5. Only three boxes are 2
placed between the box A and J. 1
Box E is placed immediately above the box J. Box B is placed on even number shelf but immediately
Number of boxes placed between box E and A is same as below the box F.
the number of boxes placed between box J and C. Box B is not placed on sixth shelf. Only one box is placed
Case I: When box J is placed below the box A: between box F and G.
Shelf Person Case I: When box J is placed below the box A:
10 Shelf Person
9 10
8 9 F
7 8 B
6 7 G
5 A 6
4 C 5 A
3 4 C
2 E 3
1 J 2 E
Case II: When box J is placed above the box A: 1 J
Shelf Person Case II: When box J is placed above the box A:
10 E Shelf Person
9 J 10 E
8 9 J
7 8
6 7
5 A 6
4 C 5 A
3 4 C

Page 453 of 722

Subscribe the Xpress Video Course & Mock Test Package for Bank & Insurance Exams
If there are any suggestions/ errors in our PDFs Feel Free to contact us via this email: admin@exampundit.in
IBPS RRB Clerk Prelims – Ultra Practice Bundle PDF
3 F V is kept at an odd numbered position above the locker X,
2 B which is kept below Q. There are four boxes in between
1 G X and Q
G and X are placed adjacent to each other which is not S Lockers Lockers Lockers
possible in case II so, case II is invalid. No. (Case I) (Case II) (Case III)
Box D is placed on one of the shelves above the shelf on 9 U U U
which box W is placed. 8 Q
The final arrangement is as follows: 7 \V Q
Shelf Person 6 Q
10 D 5 \V \V \V
9 F 4
8 B 3 X \V \V
7 G 2 X
6 X 1 X
5 A Locker W is stacked immediately above locker S.
4 C Locker T, which is neither adjacent to S nor Q, is kept
3 W immediately above locker P.
2 E Locker T is stacked below locker V.
1 J Neither S is adjacent to X nor W is adjacent to V.
Case III will become invalid.
21. Answer: B) S Lockers Lockers Lockers
22. Answer: A) No. (Case Ia) (Case Ib) (Case II)
23. Answer: D) 9 U U U
24. Answer: E) 8 Q Q W
25. Answer: C) 7 W V S
(26 – 30): Common Explanation: 6 S Q
U is kept at the top of all the boxes. 5 V V

Page 454 of 722

Subscribe the Xpress Video Course & Mock Test Package for Bank & Insurance Exams
If there are any suggestions/ errors in our PDFs Feel Free to contact us via this email: admin@exampundit.in
IBPS RRB Clerk Prelims – Ultra Practice Bundle PDF
4 3 V
3 X X 2 W
2 T W 1 X
1 P S X
Three lockers are there between P and V. R is one of the 26. Answer: B)
locker. 27. Answer: C)
Therefore, case Ib and II are invalid. Finally, we have 28. Answer: C)
S No. Lockers 29. Answer: D)
9 U 30. Answer: C)
8 Q (31 – 35): Common Explanation:
7 W As Box 5 contains 8 pens and is two floors above D.
6 S Number of pens in D is prime numbered.

5 V Box with 5 pens is immediately above the box with 2 pens,

4 R so D contains either 2 or 5 or 7 pens.

3 X Case I Case II Case III

2 T 6

1 P 5 (8) (8) (8)


4 (5)

If all the lockers are arranged alphabetically from top to 3 D(2) D(5) D(7)

bottom, it will be: 2 (2) (5)

S No. Lockers 1 (2)

9 P Box with 9 pens is immediately above G.

8 Q Box with 6 pens is below box 4, this is not possible in case

7 R III, so case III is rejected.

6 S C is immediately above the box with 6 pens.

5 T F is two boxes above E but doesn’t contain 9 pens, so case

4 U II is rejected, so F contains 7 pens, and E contains 5 pens.

Page 455 of 722

Subscribe the Xpress Video Course & Mock Test Package for Bank & Insurance Exams
If there are any suggestions/ errors in our PDFs Feel Free to contact us via this email: admin@exampundit.in
IBPS RRB Clerk Prelims – Ultra Practice Bundle PDF
The final table is given below:
6 F(7)
5 A(8)
4 E(5)
3 D(2)
2 C(9)
1 G(6)

Case 2:
31. Answer: D)
32. Answer: C)
33. Answer: B)
34. Answer: D)
35. Answer: E)
(36 – 40): Common Explanation:
a) Only 2 boxes are kept between P and Q, where Q is kept
below P.

c) S is kept between Q and J, which is placed immediately


below P.
Case 1:

b) Only 3 boxes are kept between R and Q.


(so, we get 2 possible case here).
Case 1:

Page 456 of 722

Subscribe the Xpress Video Course & Mock Test Package for Bank & Insurance Exams
If there are any suggestions/ errors in our PDFs Feel Free to contact us via this email: admin@exampundit.in
IBPS RRB Clerk Prelims – Ultra Practice Bundle PDF

Case 2:

Case 2:

d) Only 3 boxes are kept between R and B.


e) B is kept ether at top or bottom. f) Only 2 boxes are kept between R and T, which is not
Case 1: placed adjacent to B.
(so, case 1 gets eliminated here)
Case 2:

Page 457 of 722

Subscribe the Xpress Video Course & Mock Test Package for Bank & Insurance Exams
If there are any suggestions/ errors in our PDFs Feel Free to contact us via this email: admin@exampundit.in
IBPS RRB Clerk Prelims – Ultra Practice Bundle PDF

36. Ansewr: C)
g) U is placed exactly between T and W.
37. Ansewr: E)
Case 2:
38. Ansewr: E)
39. Ansewr: C)
40. Ansewr: D)
(41 – 45): Common Explanation:
Three boxes were kept between P and Q. No boxes were
kept between P and V. Four boxes were kept between V
and W.
Case 1 Case 2 Case 3 Case 4
Box Box Box Box
P W
V P W
V Q
h) Box D is placed exactly between E and F. Q
Q
Q V

Page 458 of 722

Subscribe the Xpress Video Course & Mock Test Package for Bank & Insurance Exams
If there are any suggestions/ errors in our PDFs Feel Free to contact us via this email: admin@exampundit.in
IBPS RRB Clerk Prelims – Ultra Practice Bundle PDF
W P V S
W P
Two boxes were kept between S and Q. Box T was kept 41. Answer: B)
immediately below box R. More than two boxes were kept 42. Answer: A)
above box U. 43. Answer: D)
Hence, case 2 and case 3 are invalid. 44. Answer: C)
Case 1 Case 4 45. Answer: B)
Box Box (46 – 50): Common Explanation:
P S More than five boxes are kept above box P. Box D is kept
V W immediately above box R and immediately below box G.
R U Two boxes are kept between box R and box Z. At least

T Q four boxes are kept below box R. 1Hence, we have five

Q R cases:

U T
W V
S P
Case 4 is invalid
So, the final solution is
Case 1
Box
P Box N is kept below Z and above box L. Hence Case-2,

V Case-4 and Case-5 are eliminated. Box T is kept below

R box L but not immediate below. Hence Case-3 is

T eliminated. So the final arrangement is:

Q
U
W

Page 459 of 722

Subscribe the Xpress Video Course & Mock Test Package for Bank & Insurance Exams
If there are any suggestions/ errors in our PDFs Feel Free to contact us via this email: admin@exampundit.in
IBPS RRB Clerk Prelims – Ultra Practice Bundle PDF
46. Answer: D)
47. Answer: C)
48. Answer: C)
49. Answer: A)
50. Answer: B)

21). Month Based Puzzle

Directions (1-5): Study the following information to 2) If H is related to April, R is related to December, in
answer the given questions: the same way, M is related to which of the following
Seven persons M, R, S, T, Q, K and H are attending the month?
seminar in different months January, February, April, a) August
August, September, November and December in a year b) September
but not necessarily in the same order. c) November
Only two persons attend the seminar before M. Only one d) February
person attends the seminar between R and M. The number e) None of these
of persons attends the seminar after R is one more than the 3) How many persons attend the seminar between M
number of persons attends the seminar before K. T attends and T?
the seminar immediately before Q. As many persons a) 1
attend the seminar before H is same as after Q. b) 2
1) How many persons attend the seminar after S? c) 3
a) 1 d) 4
b) 2 e) None of these
c) 3 4) Who among the following person attends the
d) 4 seminar immediately before R?
e) None of these a) Q

Page 460 of 722

Subscribe the Xpress Video Course & Mock Test Package for Bank & Insurance Exams
If there are any suggestions/ errors in our PDFs Feel Free to contact us via this email: admin@exampundit.in
IBPS RRB Clerk Prelims – Ultra Practice Bundle PDF
b) T a) Two
c) K b) Three
d) S c) Four
e) None of these d) Five
4) Four of the following five are alike in a certain way e) None
and so form a group. Find the one which does not 7) Four of the following five are alike in a certain way
belong to the group? and hence form a group. Which of the following one
a) H-M does not belong to the group?
b) T-S a) V
c) Q-R b) X
d) K-S c) Z
e) S-Q d) T
Directions (6-10): Study the following information e) S
carefully and answer the below questions. 8) The number of persons born before W is one less
Eight persons- S, T, U, V, W, X, A and Z were than the number of persons born after ____?
born in the same year of different months such as January, a) S
March, April, May, July, September, October and b) Z
December but not necessarily in the same order. The c) A
consecutive alphabetical name of the persons was not born d) X
in consecutive months. e) None of these
S was born in the month which has an even 9) Who among the following was born after S but
number of days. Only three persons were born between S before T?
and T. Two persons were born between U and V who was a) U
born an adjacent month to S.X was born in July. U was b) Z
born before V. The number of persons born after U was c) W
equal to the number of persons born before Z. d) X
6) How many persons were born between A and the e) None
one who was born in October?
Page 461 of 722

Subscribe the Xpress Video Course & Mock Test Package for Bank & Insurance Exams
If there are any suggestions/ errors in our PDFs Feel Free to contact us via this email: admin@exampundit.in
IBPS RRB Clerk Prelims – Ultra Practice Bundle PDF
10) In a certain way, A is related to T in the same way 12) Who among the following person visits a sanctuary
V is related to X then who among the following person three months before F?
is related to Z? a) The one who visits immediately before A
a) A b) H
b) T c) The one who visits immediately after B
c) X d) G
d) S e) None of these
e) None 13) Which among the following is correctly paired?
Directions (11-15): Answer the questions based on the a) March- E
information given below. b) October- H
Eight persons A, B, C, D, E, F, G and H are visiting a c) July- A
sanctuary on different months January, March, April, d) April- G
May, July, September, October and November in a year e) All are incorrect
but not necessarily in the same order. 14) Who among the following person visits a sanctuary
Three persons visit a sanctuary between C and B, who two months after A?
doesn’t visit in the month of 31 days. G visits just after C. a) E
One person visits between G and the one, who visits b) H
immediately after A. A visit after C. No one visits a c) F
sanctuary between D and F, who visits in the month of 30 d) B
days but not in April. E visits sanctuary before H. e) None of these
11) Who among the following person visits a sanctuary 15) How many persons visit a sanctuary between E and
in May? F?
a) The one who visits immediately before E a) Six
b) C b) Two
c) The one who visits immediately before H c) Four
d) G d) Three
e) None of these e) None of these

Page 462 of 722

Subscribe the Xpress Video Course & Mock Test Package for Bank & Insurance Exams
If there are any suggestions/ errors in our PDFs Feel Free to contact us via this email: admin@exampundit.in
IBPS RRB Clerk Prelims – Ultra Practice Bundle PDF
Direction (16-20): Study the following information d) The one who born immediately before Y
carefully and answer the given questions: e) None of these
Seven persons were born in different months January, 19) Which of the following statement is true?
March, May, June, July, August and September but not a) Less than one person was born between P and B
necessarily in the same order. b) Two persons were born between R and A
Two persons were born between T and B. T was born after c) More than three persons were born between T and
B. As many persons born before T was same as after R. R A
does not born in March. H was born immediately before d) More than two persons were born before Y
P. Only one person was born between A and H. Y was not e) None is true
the eldest person. 20) Four of the following five are alike in a certain way
16) How many persons were born between Y and A? and so form a group. Find the one who does not belong
a) None to the group?
b) As many persons born after A a) Y
c) 2 b) A
d) As many persons born between R and T c) H
e) None of these d) B
17) If Y is related to P, H is related to A then, in the e) P
same way T is related to who among the following? Directions (21-25): Study the following information
a) R carefully and answer the below questions.
b) Y Eight persons-A, B, C, D, E, F, G, and H are belonging to
c) B different countries they visit India on the different month
d) No one of the same year (2020) in January, February, March,
e) None of these May, July, September, November, and December but not
18) Who among the following person was born necessarily in the same order.
immediately after A? A is visiting in a month which has less than 31 days. Two
a) Y persons visit between A and the one who visits
b) The one who born immediately before H immediately before H. H visits after A. C visits in a month
c) H which has an average number of days of the visiting
Page 463 of 722

Subscribe the Xpress Video Course & Mock Test Package for Bank & Insurance Exams
If there are any suggestions/ errors in our PDFs Feel Free to contact us via this email: admin@exampundit.in
IBPS RRB Clerk Prelims – Ultra Practice Bundle PDF
months of A and F. Both G and C visit in consecutive 24) As per the given arrangement, A is related to May
months. The number of persons visits before D is same as and D is related to March following a certain pattern,
the number of persons visits after E who visits before D.F G is related to which of the following?
visits neither the first nor the last month of the year. a) September
21) Who among the following person visits India in b) November
July? c) February
a) E d) January
b) G e) March
c) One who visits between F and D. 25) In which of the following month does E visit India?
d) One who visits immediately before H a) September
e) H b) November
22) Four of the following are alike in a certain way. c) May
Who among the following one does not belong to the d) July
group? e) March
a) F Directions (26-30): Study the following information
b) One who visits immediately before A carefully and answer the below questions
c) One who visits on December month Eight persons- Aadi, Abhay, Anil, Arya, Anand, Asha,
d) E Ajay and Arjun are going to their native places on
e) One who visits immediately after H different months-January, February, March, May, June,
23) How many people visit India between the month in August, September and December of the year 2020 but not
which D and H have visited? necessarily in the same order.
a) Two Asha going to native in the month has even number of
b) Three days. One person goes between Asha and Arjun. Arya
c) No one does not go in the month having an odd number of days.
d) Five The number of persons goes before Arjun is the same as
e) None of these the number of persons goes after Anand. Aadi goes
immediately before Abhay. Ajay goes one of the months
before Anil.
Page 464 of 722

Subscribe the Xpress Video Course & Mock Test Package for Bank & Insurance Exams
If there are any suggestions/ errors in our PDFs Feel Free to contact us via this email: admin@exampundit.in
IBPS RRB Clerk Prelims – Ultra Practice Bundle PDF
26) In which of the following month does Ajay goes to 30) How many persons are going to their native place
his native place? between the one who goes in September and Aadi?
a) June a) 1
b) May b) 2
c) August c) 3
d) January d) 4
e) March e) 5
27) Who among the following person goes immediately Direction (31-35): Study the following information to
before Anand? answer the given questions:
a) Arya Seven persons H, M, T,R, P, L and D are born in different
b) Abhay months January, March, May, June, July, September and
c) Asha November but not necessarily in the same order.
d) Anil Note: All persons are born on the same date in the same
e) Aadi year.
28) Four of the following five persons are alike in a As many persons are born before R is same as after D.
certain way and hence form a group. Find the odd one Only one person is born between M and R. M is elder than
who does not belong to the group? R. T is born after R. Two persons are born between T and
a) Ajay P.L is born after H.
b) Aadi 31) If T was born immediately after R, then how many
c) Arjun persons were born between P and R?
d) Arya a) None
e) Anand b) 1
29) Which of the following combination is true? c) 2
a) June-Asha d) 3
b) May-Ajay e) None of these
c) January-Aadi 32) If H is related to D, R is related to L, in the same
d) December-Anil way, M is related to which of the following?
e) All are true a) P
Page 465 of 722

Subscribe the Xpress Video Course & Mock Test Package for Bank & Insurance Exams
If there are any suggestions/ errors in our PDFs Feel Free to contact us via this email: admin@exampundit.in
IBPS RRB Clerk Prelims – Ultra Practice Bundle PDF
b) T Eight persons namely Abhijit, Babul, Hemant, Kamal,
c) M Kabir, Radhey, Roopal and Rudra were born in eight
d) Cannot be determined different months of a year but not necessarily in the same
e) None of these order. They were born in a month viz. January, March,
33) Four of the following five are alike in a certain way April, May, July, August, October and November.
based on the group. Find the one which does not belong Hemant was born in the month which has 30 days. Three
to the group? persons were born between Hemant and Kabir. Rudra was
a) The one who was born immediately after M born just before Kabir. Only one person was born between
b) D Rudra and Radhey, who was born after Roopal. Three
c) The one who was born in January persons were born between Roopal and Abhijit. Roopal
d) R neither the eldest person nor born after Abhijit. Kamal was
e) The one who was born immediately after D neither born in January nor born in March.
34) Which of the following person was born 36. Who among the following person was born in July?
immediately before P, if T was not the youngest? a) The One who was born just before Kamal
a) D b) The One who was born between Radhey and
b) The one who was born in March Rudra
c) The one who was born in September c) Radhey
d) H d) Kamal
e) None of these e) None of these
35) If H is related to June, R is related to July, then in 37. How many persons were born after Abhijit?
the same way D is related to which of the following? a) One
a) September b) Two
b) June c) Three
c) March d) Seven
d) November e) None of these
e) None of these 38. Who among the following person was born in May?
Directions (36-40): Study the following information a) Abhijit
carefully and answer the questions given below it. b) Radhey
Page 466 of 722

Subscribe the Xpress Video Course & Mock Test Package for Bank & Insurance Exams
If there are any suggestions/ errors in our PDFs Feel Free to contact us via this email: admin@exampundit.in
IBPS RRB Clerk Prelims – Ultra Practice Bundle PDF
c) Kamal attends after L. L attends a seminar on a month which has
d) Kabir an even number of days.
e) None of these 41) How many persons attend the seminar between L
39. How many persons were born between Rudra and and P?
Abhijit? a) 4
a) Two b) 3
b) None c) 5
c) Three d) 1
d) Four e) None of these
e) None of these 42) How many persons attend a seminar between K
40. Four of the following five are alike in a certain way and the one who attends in November?
to form a group. Find the one who does not belong to a) 3
the group? b) 5
a) The One who was born just before Roopal c) 2
b) Kamal d) 1
c) The One who was born just after Kamal e) None of these
d) Radhey 43) Who among the following attends the seminar in
e) One who was born between Babul and Rudra January?
Direction (41-45): Study the following information a) K
carefully and answer the below questions. b) N
Seven persons - J, K, L, M, N, O and P are attending the c) P
seminar on different month’s viz., January, April, June, d) M
August, October, November and December but not e) None of these
necessarily in the same order. 44) The seminar on how many people are changed
M attends a seminar in November. K attends seminar on a when all of them are made to attend a seminar in
month which has 30 days. P attends seminar immediately alphabetical order from January to December?
before or after M. Three persons attend seminar between a) 6
P and N. Three persons attend between L and O, who b) 4
Page 467 of 722

Subscribe the Xpress Video Course & Mock Test Package for Bank & Insurance Exams
If there are any suggestions/ errors in our PDFs Feel Free to contact us via this email: admin@exampundit.in
IBPS RRB Clerk Prelims – Ultra Practice Bundle PDF
c) 3 d) C
d) 5 e) A
e) None of these 47) How many persons were born between H and C?
45) Four of the following five are alike in a certain way a) 1
thus form a group. Find the one which does not belong b) 2
to the group? c) 3
a) K d) 4
b) J e) None of these
c) O 48) Four of the following five are alike in a certain way
d) P and hence they form a group. Which one of the
e) N following does not belong to that group?
Direction (46-50): Study the following information a) The one who was born on March
carefully and answer the below questions. b) The one who was born on May
Eight persons A to H were born on eight different months c) The one who was born on February
of the same year i.e. February, March, April, May, June, d) The one who was born on July
July, November and December. e) The one who was born on December
A was born on the month having 30 days but not in 49) Who among the following was born in July?
November. Only one person born between A and the one a) E
who was born in June. E was the youngest person. Three b) D
persons were born between the one who was born on c) C
March and G. D was born immediately before G. C and F d) G
were born on consecutive months but F was not the elder e) None of these
one. B was not born in May. 50) Which of the following statements is true about E?
46) Who among the following person was born in a) E was born on December
April? b) E was born immediately after B
a) G c) Only one person was born between E and G
b) D d) None is true
c) H e) All are true
Page 468 of 722

Subscribe the Xpress Video Course & Mock Test Package for Bank & Insurance Exams
If there are any suggestions/ errors in our PDFs Feel Free to contact us via this email: admin@exampundit.in
IBPS RRB Clerk Prelims – Ultra Practice Bundle PDF

21). Month Based Puzzle - Solution and Detailed Explanation

Directions (1-5): T attends the seminar immediately before Q.


Solution: Case-1 gets eliminated since as many persons attend the
seminar before H is same as after Q.

Explanation: Directions (6-10):

 Only two persons attend the seminar before M. 6) Answer: E

 Only one person attends the seminar between R 7) Answer: E

and M. 8) Answer: C

 The number of persons attends the seminar after R 9) Answer: D

is one more than the number of persons attends the 10) Answer: B

seminar before K. Solution:

As many persons attend the seminar before H is same as


after Q.
Page 469 of 722

Subscribe the Xpress Video Course & Mock Test Package for Bank & Insurance Exams
If there are any suggestions/ errors in our PDFs Feel Free to contact us via this email: admin@exampundit.in
IBPS RRB Clerk Prelims – Ultra Practice Bundle PDF
Explanation:  The number of persons born after U was equal to
 S was born in the month which has an even the number of persons born before Z.
number of days.  The consecutive alphabetical name of the persons
 Only three persons were born between S and T. was not born in consecutive months.
 Case-2 gets eliminated we have no place for Z.

 Two persons were born between U and V who was


born an adjacent month to S.X was born in July. U
was born before V.
Our Final arrangement becomes,

Page 470 of 722

Subscribe the Xpress Video Course & Mock Test Package for Bank & Insurance Exams
If there are any suggestions/ errors in our PDFs Feel Free to contact us via this email: admin@exampundit.in
IBPS RRB Clerk Prelims – Ultra Practice Bundle PDF
Directions (11-15): One person visits between G and the one, who
11) Answer: C visits immediately after A. A visits after C. Case-
12) Answer: B 3 gets eliminated we have no place for A.
13) Answer: D
14) Answer: D
15) Answer: A
Solution:

No one visits a sanctuary between D and F, who


visits in the month of 30 days but not in April.
E visits sanctuary before H. Case-1 gets eliminated
since F visits in the month of 30 days but not in
April.

Explanation:
Three persons visit a sanctuary between C and B,
who doesn’t visit in the month of 31 days.
G visits just after C.

Direction (16-20):
16) Answer: a
17) Answer: a
Page 471 of 722

Subscribe the Xpress Video Course & Mock Test Package for Bank & Insurance Exams
If there are any suggestions/ errors in our PDFs Feel Free to contact us via this email: admin@exampundit.in
IBPS RRB Clerk Prelims – Ultra Practice Bundle PDF
18) Answer: b
19) Answer: e
20) Answer: d
Solution:

Directions (21-25):
21) Answer: d
22) Answer: e
23) Answer: c
Explanation 24) Answer: a
 Two persons were born between T and B. 25) Answer: c
 As many persons born before T was same as after Solution:
R.
 T was born after B. R does not born in March.

 A is visiting in a month which has less than 31


 H was born immediately before P. days.
 Only one person was born between A and H.  two persons visit between A and the one who visits
 Y was not the eldest person. Case-1 gets immediately before H.
eliminated Because Y is not the eldest person.  H visits after A.

Page 472 of 722

Subscribe the Xpress Video Course & Mock Test Package for Bank & Insurance Exams
If there are any suggestions/ errors in our PDFs Feel Free to contact us via this email: admin@exampundit.in
IBPS RRB Clerk Prelims – Ultra Practice Bundle PDF
 From the above conditions,  F visits neither the first nor the last month of the
year.

 C visits in a month which has an average number


of days of visiting months of A and F.
 Both G and C visit in consecutive months.
Directions (26-30):
26) Answer: b
27) Answer: c
28) Answer: d
29) Answer: e
30) Answer: e
Solution:

 The number of persons visits before D is same as


the number of persons visits after E who visits
before D.

Page 473 of 722

Subscribe the Xpress Video Course & Mock Test Package for Bank & Insurance Exams
If there are any suggestions/ errors in our PDFs Feel Free to contact us via this email: admin@exampundit.in
IBPS RRB Clerk Prelims – Ultra Practice Bundle PDF

 The Number of persons goes before Arjun is the


same as the number of persons goes after Anand.
 Aadi goes immediately before Abhay.
Explanation:
 Asha going to native in the month has even
number of days
 One person goes between Asha and Arjun.
 Arya does not go in the month having an odd
number of days.
 Case-2 and case-3 are eliminated we have no place
for Arya.

Ajay goes one of the months before Anil.

Page 474 of 722

Subscribe the Xpress Video Course & Mock Test Package for Bank & Insurance Exams
If there are any suggestions/ errors in our PDFs Feel Free to contact us via this email: admin@exampundit.in
IBPS RRB Clerk Prelims – Ultra Practice Bundle PDF
 As many persons are born before R is same as after
D.
 Only one person is born between M and R. M is
elder than R.

 T is born after R.
 Two persons are born between T and P.
 L is born after H.

Direction (31-35):
31) Answer: D
32) Answer: D
33) Answer: E
34) Answer: C
Case 2 will be dropped because two persons are born
35) Answer: D
between T and P.
Explanation
Case 3 will be dropped because T is born after R.
(Directions 36–40):
36) Answer: C
37) Answer: B
38) Answer: D
39) Answer: A
Solution: 40) Answer: C
Solution:

Page 475 of 722

Subscribe the Xpress Video Course & Mock Test Package for Bank & Insurance Exams
If there are any suggestions/ errors in our PDFs Feel Free to contact us via this email: admin@exampundit.in
IBPS RRB Clerk Prelims – Ultra Practice Bundle PDF
 Only one person was born between Rudra and
Radhey, who was born after Roopal.
 Here, we have one more case from Case-1. Here,
given that Radhey was born after Roopal, because
of this Radhey was not born in January.

Explanation:
 Hemant was born in the month which has 30 days.
 Three persons were born between Hemant and
Kabir.
 Rudra was born just before Kabir.  Three persons were born between Roopal and
Abhijit. From the previous statement, Roopal was
 Here, we have two cases.
born before Radhey.
 Roopal neither the eldest person nor born after
Abhijit.

Page 476 of 722

Subscribe the Xpress Video Course & Mock Test Package for Bank & Insurance Exams
If there are any suggestions/ errors in our PDFs Feel Free to contact us via this email: admin@exampundit.in
IBPS RRB Clerk Prelims – Ultra Practice Bundle PDF
Kamal was neither born in January nor born in March.  M attends a seminar in November.
 K attends seminar on a month which has 30 days.

 P attends seminar immediately before or after M.


 Three persons attend a seminar between P and N.

Direction (41-45):
41) Answer: D
42) Answer: A
43) Answer: B
44) Answer: D  Three persons attend between L and O, who attend
45) Answer: A after L.
Solution:  L attends a seminar on a month which has an even
number of days.

Explanation So, the final arrangement is,

Page 477 of 722

Subscribe the Xpress Video Course & Mock Test Package for Bank & Insurance Exams
If there are any suggestions/ errors in our PDFs Feel Free to contact us via this email: admin@exampundit.in
IBPS RRB Clerk Prelims – Ultra Practice Bundle PDF
 Only one person born between A and the one who
was born in June.
 E was the youngest person. From the statements,
we get,

Direction (46-50):
46) Answer: e
47) Answer: b
48) Answer: c
49) Answer: d
 Three persons were born between the one who was
50) Answer: e
born on March and G.
Solution:
 D was born immediately before G.
 C and F were born on consecutive months but F
was not the elder one.
 B was not born in May.
 From the statements, we get

Explanation:
 A was born on the month having 30 days but not
in November.

Page 478 of 722

Subscribe the Xpress Video Course & Mock Test Package for Bank & Insurance Exams
If there are any suggestions/ errors in our PDFs Feel Free to contact us via this email: admin@exampundit.in
IBPS RRB Clerk Prelims – Ultra Practice Bundle PDF

22). Month and Date Based Puzzle

Direction (1-5): Study the following information d) E – 9th June


carefully and answer the questions given below. e) B –13th June
Eight persons B to I appear for a bank exam on four 3) Who among the following person appears the exam
different dates viz. 9th, 13th, 22nd and 30th of two different on 13th June?
months- May and June of the same year but not a) H
necessarily in the same order. No two persons appear an b) B
exam on the same date of same month. c) C
G appears the exam on a date which is the sum of H's d) I
appears date and B's appears date but no two of them e) D
appear the exam consecutively. E appears the exam before 4) How many days before I appears the exam with
I who is not appears the exam consecutively with H. C respect to F?
appears the exam after D. F appears the exam immediately a) 21
before the one who appears four days before D. b) 8
1) On which of the following day does E appear the c) 17
exam? d) 13
a) 22nd May e) 4
b) 30th June 5) Who among the following person appears the exam
c) 9th June just after E?
d) 22nd June a) H
e) 9th May b) I
2) Which of the following combination is true? c) B
a) I – 22nd May d) C
b) C – 30th May e) G
c) H – 30th May

Page 479 of 722

Subscribe the Xpress Video Course & Mock Test Package for Bank & Insurance Exams
If there are any suggestions/ errors in our PDFs Feel Free to contact us via this email: admin@exampundit.in
IBPS RRB Clerk Prelims – Ultra Practice Bundle PDF
Directions (6-10): Answer the questions based on the a) P
information given below. b) The one who attends immediately after R
Ten people viz. P, Q, R, S, T, U, V, W, X and Y are c) Y
scheduled to attend a meeting on 12th and 15th of five d) S
different months viz. April, May, June, July and August e) The one who attends immediately before T
in the same year, but not necessarily in the same order. No 8. How many persons attend the meeting between S
two persons attend the meeting on the same date of the and T?
same month. a) Same as the number of persons attends a meeting
X and V attend the meeting on the month having less than between Q and Y.
31 days but not on the same month or the same date. Only b) Three
one person attends the meeting after R. Y attends the c) More than three
meeting before R but after V. S attends the meeting four d) Same as the number of persons attends a meeting
persons before Y but after X. Neither S nor Y attends the between X and U.
meeting on 12th. P attends the meeting on the month e) None
having less than 31 days and immediately before U. Q 9. Four of the following five are alike in a certain way
attends the meeting before T on the same date but on and hence form a group. Find the one that does not
consecutive months. There are two persons between W’s belong to the group.
meeting and T’s meeting. a) P
6. The person Q attends the meeting on which of the b) U
following date? c) S
a) 15th June d) Y
b) 12th April e) R
c) 15th April 10. If U is related to V and Q is related to Y, then who
d) 12th June among the following person is related to S?
e) 12th July a) Q
7. Who among the following person attends the b) X
meeting on 15th June? c) R

Page 480 of 722

Subscribe the Xpress Video Course & Mock Test Package for Bank & Insurance Exams
If there are any suggestions/ errors in our PDFs Feel Free to contact us via this email: admin@exampundit.in
IBPS RRB Clerk Prelims – Ultra Practice Bundle PDF
d) P 13. Which of the following number shows that the sum
e) Y of birth dates of I and K?
Directions (11-15): Study the following information a) 38
carefully and answer the questions given below. b) 41
Eight persons viz. I, J, K, L, M, N, O, and P are born on c) 36
four different dates viz. 8th, 13th, 16th, and 21st of two d) 37
different months viz. September and October in the same e) None of the above
year, but not necessarily in the same order. No two 14. Which of the following pair is the eldest and the
persons were born on the same date of the same month. youngest among all?
Both M and N were born on different even dates of the a) P, K
different months. P was born immediately before I. L was b) M, K
born on a month having less than 31 days but not before I. c) P, L
K was born after O in the same month. J was born on a d) M, O
prime date but before N. e) None of these
11. Who among the following person was born on 16th 15. How many persons were born after O?
October? a) Four
a) J b) Three more than the number of persons born after L.
b) K c) One
c) N d) No one
d) I e) Two less than the number of persons born before J.
e) O Directions (16-20): Study the following information
12. On which of the following date does P was born? carefully and answer the following questions.
a) 16th September Ten persons P, Q, R, S, T, U, V, W, X and Y were born
b) 13th October either on 8th or 27th in different months i.e. January,
c) 16th October February, March, April and May of 2019 but not
d) 13th September necessarily in the same order. No two persons born on the
e) 8th September same date of the same month.

Page 481 of 722

Subscribe the Xpress Video Course & Mock Test Package for Bank & Insurance Exams
If there are any suggestions/ errors in our PDFs Feel Free to contact us via this email: admin@exampundit.in
IBPS RRB Clerk Prelims – Ultra Practice Bundle PDF
P was born in the month having even days but not the least 19) Who among the following is the youngest among
number of days. Four people were born between P and W. all?
Three people were born between W and S. T was born in a) U
the month having the least number of days. Two persons b) V
were born in between T and Q. U and V were born in the c) P
same month but not in March, U was older than V. Three d) Q
persons were born between U and R. X was not born in e) None of these
the month having the least or odd number of days. 20) Four of the following form a group following a
16) R was born in which of the following month? certain pattern. Which among the following
a) The month having even number of days statements doesn’t belong to the group?
b) January a) UV
c) The month having the least number of days b) YT
d) March c) RS
e) None of these d) XU
17) Who among the following pairs was born in April? e) QW
a) RS Directions (21-25): Study the following information
b) SP carefully and answer the below questions.
c) PX Seven persons L, M, N, O, P, Q and R were giving a
d) TR petition to the collector on different dates either 16th or
e) None of these 21stof different months January, April, August and
18) How many persons were born in between Y and U? September but not necessarily in the same order. In one of
a) 4 the given dates of the month, no one was given a petition.
b) 6 P and M gave the petition in the same month but neither
c) 3 in January nor in April. Q gave petition on an odd
d) 2 numbered date of the month which had even number of
e) 5 days. R gave petition in either April or August. Only one
person gave a petition between N and R who gave before

Page 482 of 722

Subscribe the Xpress Video Course & Mock Test Package for Bank & Insurance Exams
If there are any suggestions/ errors in our PDFs Feel Free to contact us via this email: admin@exampundit.in
IBPS RRB Clerk Prelims – Ultra Practice Bundle PDF
N. L and P gave on the same date but in different months. c) Only I and II
Only one petition was given between Q and O, who gave d) Only II
before Q. No petition was given on a date immediately e) None of these
after N had given the petition. L was not the second person 24) On which of the following date and month did L
to give a petition. give petition?
21) Who among the following persons gave petition on a) January 16th
21st August? b) April 21st
a) L c) April 16th
b) No one d) August 16th
c) Q e) September 21st
d) P 25) How many persons gave a petition between O and
e) O P?
22) Four of the following are alike in a certain way. a) One
Who among the following one does not belong to that b) Two
group? c) Three
a) L d) None
b) R e) More than three
c) N Directions (26-30): Study the following information
d) P carefully and answer the below questions.
e) Q Eight persons- Amir, Abdul, Ramya, Usha, Sanjay, Vijay,
23) Which of the following statements is/are false with Vino, &Sudha are shifting their houses on different dates
respect to N? 3, 7, 10, 17, 18, 21, 23, 27 in October but not necessarily
I. N gives a petition in August in the same order.
II. Three petitions were given between L and N Ramya is shifting on October 10. Only one person is
III. No one gives immediately after N shifting between Ramya and Vino. More than three
a) Only I and III persons are shifting between Sanjay and Vijay who shifted
b) Only I after sanjay on prime numbered date. The difference

Page 483 of 722

Subscribe the Xpress Video Course & Mock Test Package for Bank & Insurance Exams
If there are any suggestions/ errors in our PDFs Feel Free to contact us via this email: admin@exampundit.in
IBPS RRB Clerk Prelims – Ultra Practice Bundle PDF
between the shifting dates of Sanjay and Sudha is twice d) Only II
the difference between the shifting dates of Vino and e) None of these
Vijay. Abdul is shifting one of the dates before Usha. 29) On which of the following date does Abdul shifting
Amir doesn’t shift the house on non-prime numbered date. his house?
26) Who among the following person is shifting the a) October 21st
house on October 17th? b) October 17th
a) Sanjay c) October 23rd
b) Sudha d) October 7th
c) Vijay e) October 18th
d) Abdul 30) How many persons are shifted between Sudha and
e) Usha Usha?
27) Four of the following five are alike in a certain way. a) One
Who among the following one does not belong to the b) Two
group? c) Three
a) Usha d) None
b) Vijay e) More than three
c) Sanjay Directions (31-35): Answer the questions based on the
d) Amir information given below.
e) Vino Ten students viz. A, C, D, H, J, K, L, M, P and S from the
28) Which of the following statement(s) is/are true with same school joints the same college for graduation in May
respect to Sanjay? and October of different years viz. 1994, 1995, 1997, 1998
I. Sanjay shifted his house immediately before Ramya and 1999, but not necessarily in the same order. No two
II. Two persons are shifted between Sanjay and Vino students join in the same month of the same year.
III. Sanjay shifted on an odd numbered date J, who joints the college on an even year, joints the college
a) Only I and III immediately before M but not in the same year. Only three
b) Only III students join the college between M and A. L joints two
c) Only I and II persons before A and is three persons earlier than the one

Page 484 of 722

Subscribe the Xpress Video Course & Mock Test Package for Bank & Insurance Exams
If there are any suggestions/ errors in our PDFs Feel Free to contact us via this email: admin@exampundit.in
IBPS RRB Clerk Prelims – Ultra Practice Bundle PDF
who joints immediately before S. Only one person joints a) One
between P and H and both join before A. P doesn’t joint b) Two
before K. D joints two persons after H. C doesn’t joint c) Three
immediately after M. d) Four
31. On which of the following month and year does C e) Cannot be determined
join the college? 35. Who among the following person joins the college
a) May 1999 in October 1997?
b) October 1998 a) P
c) October 1999 b) The one immediately joins before P
d) May 1998 c) H
e) Cannot be determined d) The one immediately joins after A
32. If all of them are jointed for a full time four years e) Cannot be determined
course, then on which year does L graduate? Directions (36-40): Study the following information
a) May 2001 carefully to answer the given question:
b) May 2000 Ten persons viz. A, B, C, D, K, L, M, N, O, and P have
c) May 2002 seminar in five different months viz. February, April,
d) October 2001 June, August, December, on 2 different dates viz. 13 and
e) October 2000 18 of each month but not necessarily in the same order.
33. How many persons join between M and S? N have seminar on 18th of the month after June. There are
a) As many persons between J and H four persons who have a seminar between O and M. There
b) Two is one person who has a seminar between C and A. A’s
c) Three seminar is just after the seminar of L but not in the same
d) As many persons between K and D month. There are four persons who have a seminar
e) More than four between P and K. There are two persons who have a
34. What is the difference between the graduation year seminar between N and M. The number of persons has a
of P and J, if they joined for a full time three years seminar after P is same as the number of persons have a
course? seminar before B.

Page 485 of 722

Subscribe the Xpress Video Course & Mock Test Package for Bank & Insurance Exams
If there are any suggestions/ errors in our PDFs Feel Free to contact us via this email: admin@exampundit.in
IBPS RRB Clerk Prelims – Ultra Practice Bundle PDF
36.Who among the following has a seminar on 13th c) O
June? d) M
a) D e) D
b) B 40.How many persons have a seminar between B and
c) O P?
d) M a) 5
e) C b) 4
37.The number of persons have seminar before A is c) 2
same as the number of persons have seminar after d) 6
____? e) 3
a) D Directions (41-45): Answer the questions based on the
b) N information given below:
c) B Eight persons have exams on different dates either on 8th
d) O or 19th of June, July, August and September in 2020.
e) None of these Q’s exam is immediately before P’s exam who has an
38.Four of the following five are alike in a certain way exam on 8th July. Three persons have exams between P’s
and hence they form a group. Which one of the and R’s exam. S’s exam is immediately after W’s exam.
following does not belong to that group? T’s exam is in a month, which has 31 days. V’s exam is
a) B after U’s exam. More than two persons have an exam
b) K between T’s and U’s exam.
c) O 41. How many persons have exams before T’s exam?
d) L A. Three
e) N B. Four
39.Who among the following has a seminar before C. Two
April? D. Five
a) C E. None of these
b) B 42. Find the odd one out.

Page 486 of 722

Subscribe the Xpress Video Course & Mock Test Package for Bank & Insurance Exams
If there are any suggestions/ errors in our PDFs Feel Free to contact us via this email: admin@exampundit.in
IBPS RRB Clerk Prelims – Ultra Practice Bundle PDF
A. The one whose exam is on 8th June Eight persons are born in four different months of two
B. The one whose exam is on 19th July dates. Months are March, June, July and August. In two
C. R different dates 8th or 15th. Two persons are born on each
D. The one whose exam is on 8th July month. No two persons are born on same date.
E. S F was born in odd number date in the month which has
43. Who among the following person has an exam odd number of days. D was born immediately after F.
immediately before W’s exam? Number of persons born before D is same as the number
A. The one whose exam is on 19th August of persons born after C. G was born immediately before
B. Q C. Only three persons were born between G and A. A was
C. The one whose exam is on 8th July not born in July. E was born immediately before H. B is
D. R one of the persons.
E. None of these
44. V has an exam on which date? 46.In which of the following month C was born?
A. 19th August a) May
B. 8th August b) June
C. 19th July c) August
D. 19th September d) July
E. None of these e) None of these
45. How many persons have an exam between V and 47.In which of the following month H was born?
W? a) May
A. Two b) June
B. Four c) August
C. One d) July
D. Three e) None of these
E. None of these 48.Four of the following five are alike in a certain way
Directions (46-50): Answer the questions based on the thus forms a group. Find the one which does not belong
information given below: to the group?

Page 487 of 722

Subscribe the Xpress Video Course & Mock Test Package for Bank & Insurance Exams
If there are any suggestions/ errors in our PDFs Feel Free to contact us via this email: admin@exampundit.in
IBPS RRB Clerk Prelims – Ultra Practice Bundle PDF
a) A d) E
b) F e) H
c) D 50. How many persons are born between D and H?
d) E a) 1
e) H b) 5
49.Who was born on July? c) 3
a) A d) 4
b) F e) None
c) G

22). Month and Date Based Puzzle - Solution and Detailed Explanation

Direction (1-5): G appears the exam on a date which is the sum of H’s
1) Answer: E appears date and B’s appears date but no two of them
2) Answer: A appear the exam consecutively.
3) Answer: E Here we get four possibilities.
4) Answer: B
5) Answer: C
Solution:

Date 9th 13th 22nd 30th


Month
May E B I F
June H D G C

Explanation:
F appears the exam immediately before the one who
appears four days before D. C appears the exam after D.

Page 488 of 722

Subscribe the Xpress Video Course & Mock Test Package for Bank & Insurance Exams
If there are any suggestions/ errors in our PDFs Feel Free to contact us via this email: admin@exampundit.in
IBPS RRB Clerk Prelims – Ultra Practice Bundle PDF
1. X and V attend the meeting on the month having
less than 31 days but not on the same month or the
same date.
2. Only one person attends the meeting after R.

We cannot fix the position of D in case (1) and case (2).


So these cases are eliminated.
E appears the exam before I who is not appear the exam
consecutively with H. 3. Y attends the meeting before R but after V.
4. S attends the meeting four persons before Y but
after X.
5. Neither S nor Y attends the meeting on 12th.
We cannot fix the position of I in case (3). So this case 6. Hence X and V are fixed.
is eliminated.
6.Answer: d) 12th June
7.Answer: e
8.Answer: a
9.Answer: a) P
10.Answer: b) X 7. P attends the meeting on the month having less
Solution: than 31 days and immediately before U.
8. Q attends the meeting before T on the same date
but on consecutive months.

Explanation:

Page 489 of 722

Subscribe the Xpress Video Course & Mock Test Package for Bank & Insurance Exams
If there are any suggestions/ errors in our PDFs Feel Free to contact us via this email: admin@exampundit.in
IBPS RRB Clerk Prelims – Ultra Practice Bundle PDF
1. Both M and N were born on different even dates
of the different months.

9. There are two persons between W’s meeting and


T’s meeting. Hence case 2 gets eliminated.

2. P was born immediately before I. L was born on a


month having less than 31 days but not before I.

Directions (11-15):
11.Answer: c) N
12.Answer: d) 13th September
13.Answer: d)37
14.Answer: b) M, K
3. K was born after O in the same month. J was born
15.Answer: e) two less than the number of persons
on a prime date but before N. Hence, case 2 gets
born before J.
eliminated.
Solution:

16) Answer: D
Explanation:

Page 490 of 722

Subscribe the Xpress Video Course & Mock Test Package for Bank & Insurance Exams
If there are any suggestions/ errors in our PDFs Feel Free to contact us via this email: admin@exampundit.in
IBPS RRB Clerk Prelims – Ultra Practice Bundle PDF
17) Answer: C
18) Answer: E
19) Answer: B
20) Answer: D
Solution:

 U and V were born in the same month but not in


March, U was older than V.
 Three persons were born between U and R.
 X was not born in the month having the least or

Explanation: odd number of days.


Case 2 will be eliminated as X cannot be placed.
 From the given statements, P was born in the
month having even days but not the least number
of days.
 Four people were born between P and W.
 Three people were born between W and S.
 T was born in the month having the least number
of days.
 Two persons were born in between T and Q.

Page 491 of 722

Subscribe the Xpress Video Course & Mock Test Package for Bank & Insurance Exams
If there are any suggestions/ errors in our PDFs Feel Free to contact us via this email: admin@exampundit.in
IBPS RRB Clerk Prelims – Ultra Practice Bundle PDF
 P and M gave the petition in the same month but
neither in January nor in April.
We have two cases,

21.Answer: b  Q gave petition on an odd numbered date of the


22.Answer: e month which had even number of days.
23.Answer: e  Only one petition was given between Q and O who
24.Answer: a
gave before Q.
25.Answer: c
 From the above conditions, there are three more
Solution:
possibilities.

 R gave petition in either April or August.


 Only one person gave petition between N and R
Explanation: who gave before N.

Page 492 of 722

Subscribe the Xpress Video Course & Mock Test Package for Bank & Insurance Exams
If there are any suggestions/ errors in our PDFs Feel Free to contact us via this email: admin@exampundit.in
IBPS RRB Clerk Prelims – Ultra Practice Bundle PDF
 No petition was given on a date after N had given Case2
the petition. October 3 Amir
 L was not the second person to give a petition. October 7 Sanjay
 Case- 1b, case-1a case-2b gets eliminated October 10 Ramya
October 17 Sudha
October 18 Vino
October 21 Abdul
October 23 Vijay
October 27 Usha

L and P gave on the same date but in different months Explanation:


 Ramya is shifting on October 10.
 Only one person is shifting between Ramya and
Vino.
 From the above condition, there are two
possibilities.
Case1 Case2
October 3 Vino
October 7
October 10 Ramya Ramya
Directions (26-30)
October 17
26.Answer: b
October 18 Vino
27.Answer: e
October 21
28.Answer: e
October 23
29.Answer: a
October 27
30.Answer: c
Solution:

Page 493 of 722

Subscribe the Xpress Video Course & Mock Test Package for Bank & Insurance Exams
If there are any suggestions/ errors in our PDFs Feel Free to contact us via this email: admin@exampundit.in
IBPS RRB Clerk Prelims – Ultra Practice Bundle PDF
 More than three persons are shifting between October 21
Sanjay and Vijay who shifted after Sanjay on October 23 Vijay Vijay Vijay
prime numbered date. October 27
 From the above condition, there are three Amir doesn’t shift the house on non-prime numbered date.
possibilities. Abdul is shifting one of the dates before Usha.
Case1 Case2 Case2a From the above condition, case2 shows the final
October 3 Vino Sanjay arrangement.
October 7 Sanjay Sanjay Case2
October 10 Ramya Ramya Ramya October 3 Amir
October 17 October 7 Sanjay
October 18 Vino Vino October 10 Ramya
October 21 October 17 Sudha
October 23 Vijay Vijay Vijay October 18 Vino
October 27 October 21 Abdul
October 23 Vijay
 The difference between the shifting dates of October 27 Usha
Sanjay and Sudha is twice the difference between 31.Answer: c) October 1999
the shifting dates of Vino and Vijay. 32.Answer: (a) May 2001
 From the above condition, case1 and case2a were 33.Answer: e) More than four
dropped. 34.Answer: a) One
Case1 Case2 Case2a 35.Answer: c) H
October 3 Vino Sanjay Solution:
October 7 Sanjay Sanjay
October 10 Ramya Ramya Ramya
October 17 Sudha
October 18 Vino Vino Explanation:

Page 494 of 722

Subscribe the Xpress Video Course & Mock Test Package for Bank & Insurance Exams
If there are any suggestions/ errors in our PDFs Feel Free to contact us via this email: admin@exampundit.in
IBPS RRB Clerk Prelims – Ultra Practice Bundle PDF
1. J, who joints the college on an even year, joints the
college immediately before M but not in the same
year. Only three students join the college between
M and A.

Directions (36-40):
36. Answer: d
37. Answer: b
38. Answer: d
2. L joints two persons before A and is three persons 39. Answer: a
earlier than the one who joints immediately before 40. Answer: c
S. Solution:

3. Only one person joints between P and H and both


join before A. P doesn’t joint before K.

4. D joints two persons after H. C doesn’t joint Explanation:


immediately after M. Hence, Case 2 gets  N has a seminar on 18th of the month after June.
eliminated.  Here we get 2 possibilities i.e. Case 1 and Case 2.
 There are two persons who have a seminar
between N and M.

Page 495 of 722

Subscribe the Xpress Video Course & Mock Test Package for Bank & Insurance Exams
If there are any suggestions/ errors in our PDFs Feel Free to contact us via this email: admin@exampundit.in
IBPS RRB Clerk Prelims – Ultra Practice Bundle PDF
 There are four persons who have a seminar
between O and M.

From this Condition Case 2 is eliminated. So, the final

 A’s seminar is just after the seminar of L but not arrangement is-

in the same month.


 There is one person who has a seminar between C
and A.
 There are four persons who have a seminar
between P and K.
 The number of persons has a seminar after P is the
same as the number of persons have a seminar
before B.

Directions (41-45):
41. Answer D
42. Answer B
43. Answer C
44. Answer D
45. Answer D

Page 496 of 722

Subscribe the Xpress Video Course & Mock Test Package for Bank & Insurance Exams
If there are any suggestions/ errors in our PDFs Feel Free to contact us via this email: admin@exampundit.in
IBPS RRB Clerk Prelims – Ultra Practice Bundle PDF
Solution:

Explanation:  T’s exam is in a month, which has 31 days, so T’s


 Q’s exam is immediately before P’s exam who has exam is either on 19th August or 19th July.
an exam on 8th July.  V’s exam is after U’s exam.
 Three persons have exam between P’s and R’s  More than two persons have exam between T’s
exam. and U’s exam, this is not possible in case II, so
 S’s exam is immediately after W’s exam, so S’s case II is rejected.
exam is either on 8th August or 19th August.  The final table is given below:

Directions (46-50):
46.Answer: d

Page 497 of 722

Subscribe the Xpress Video Course & Mock Test Package for Bank & Insurance Exams
If there are any suggestions/ errors in our PDFs Feel Free to contact us via this email: admin@exampundit.in
IBPS RRB Clerk Prelims – Ultra Practice Bundle PDF
47.Answer: c
48.Answer: c
49. Answer: c
50. Answer: d
Solution: G was born immediately before C. Only three persons
were born between G and A.

Explanation:
 F was born in odd number date in the month which
E was born immediately born before H. A was not born in
has odd number of days.
July.
 D was born immediately after F.

So the final arrangement is,


Number of persons born before D is same as the number
of persons born after C.

Page 498 of 722

Subscribe the Xpress Video Course & Mock Test Package for Bank & Insurance Exams
If there are any suggestions/ errors in our PDFs Feel Free to contact us via this email: admin@exampundit.in
IBPS RRB Clerk Prelims – Ultra Practice Bundle PDF

23). Profession Based Puzzle


Directions (1-5): Answer the questions based on the 2) Who is the immediate senior to the one who is a
information given below. Manager?
There are eight employees of the company named as A, B, a) F
C, D, E, F, G and H. They have different designations b) E
starting from clerk, PO, Assistant Manager, Manager, c) A
Deputy General Manager, General Manager, Managing d) C
Director and Chief Managing Director. All the e) None of these
designations given are to be considered in a given order 3) Which of the following statement/statements is/are
(as Clerk is considered as Junior-most rank and Chief true?
Managing Director is considered as the Senior-most rank). a) B is the Managing Director.
Only three persons are junior to C. A is neither MD nor b) A is immediate junior to the General Manager.
Clerk. F’s rank is either immediate above or immediate c) F is an Assistant Manager.
below to the person who is a Manager. E is three ranks d) Both a and b
senior to F. B is immediate senior to D. Both G and H are e) Both b and c
junior to A, but both are not immediate junior to him. H is 4) Four of the following five pairs of designations and
senior to G. employees have certain relationship between each
1) Which of the following statement/statements is/are other. Which one does not belong to the group?
not true? a) A- Managing Director
a) A is Deputy General Manager. b) C- General Manager
b) The one who is Managing Director is immediate senior c) E- Chief Managing Director
to B. d) H- Assistant Manager
c) The one who is PO is immediate junior to F. e) F- Deputy General Manager
d) E is General Manager. 5) Who among the following are senior most and
e) Both b and c junior most employees respectively?
a) E, G

Page 499 of 722

Subscribe the Xpress Video Course & Mock Test Package for Bank & Insurance Exams
If there are any suggestions/ errors in our PDFs Feel Free to contact us via this email: admin@exampundit.in
IBPS RRB Clerk Prelims – Ultra Practice Bundle PDF
b) E, D a) Two persons are designated between the one who is
c) B, G travelling to Shimla and Daman.
d) A, D b) The one who travels to Kullu is designated immediate
e) None of these next to Probationary Officer.
Directions (6-10): Study the following information c) One person is designated between the one who is
carefully and answer the below questions. travelling to Coorg and Daman.
Six bankers hold different designations in the same bank d) The one who is travelling to Goa is designated
viz.- General Manager (GM), Deputy General Manager immediate next to Clerk.
(DGM), Manager (MG), Assistant Manager (AM), e) All of the above statements are true.
Probationary Officer (PO), and Clerk. They all are 8) Who among the following person is travelling
travelling to different tourist places viz.- Daman, Shimla, immediate next to Coorg?
Goa, Leh, Kullu, and Coorg. Designations are given in a) Assistant Manager
descending order from General Manager to Clerk. b) Clerk
Only two persons are designated between the one who c) Manager
travels to Goa and Coorg. The one who is travelling to d) General Manager
Coorg is designated immediate next to the one who is e) None of these
travelling to Leh. The Assistant Manager is travelling to 9) Four out of five are related to each other in some
Daman. The one who is travelling to Kullu is neither a way and thus form a group, find the odd among the
Manager nor the General Manager. five?
6) Who among the following person travels to Shimla? a) Shimla – Coorg
a) Assistant Manager b) Goa – Kullu
b) Clerk c) Daman – Coorg
c) Probationary Officer d) Leh – Daman
d) General Manager e) Coorg – Leh
10) Which of the following combination is true?
e) None of these a) GM – Coorg
7) Which of the following statement is/are not true? b) Clerk – Shimla
c) PO – Daman
Page 500 of 722

Subscribe the Xpress Video Course & Mock Test Package for Bank & Insurance Exams
If there are any suggestions/ errors in our PDFs Feel Free to contact us via this email: admin@exampundit.in
IBPS RRB Clerk Prelims – Ultra Practice Bundle PDF
d) DGM – Goa C. The one who is designated as CTO
e) Manager – Leh D. N
Directions (11-15): Study the following information E. None of these
carefully and answer the given questions: 13. Who among the following person is the Clerk?
There are seven employees i.e., H, I, J, K, L, M and N of A. I
a company and all of them are working on seven different B. J
designations of a company viz. CEO, CFO, CTO, C. K
Managing Director, General Manager, Assistant Manager D. M
and Clerk but not necessarily in the same order. All the E. None of these
designations given are to be considered in a given order 14. How many persons are junior to I?
(CEO is considered as Senior-most and Clerk is A. Two
considered as the Junior-most). B. As many persons designated between K and L.
Only two persons are senior to I. Only one person C. Three
designated between I and N. There are more than two D. As many persons designated between N and J.
persons designated between J and N. J is not the junior- E. None of these
most employee. There are more than two persons 15. Who among the following person is the second
designated between K and J. K is senior to J. L is neither junior-most employee?
a General Manager nor Clerk. M is junior to H. A. J
11. Find the odd one? B. M
A. I, H C. N
B. N, L D. H
C. K, L E. None of these
D. H, M Directions (16-20): Study the following information
E. N, I carefully and answer the below questions.
12. Who among the following person is just senior to Eight persons A, B, C, D, E, F, G and H are working in a
L? bank with different grade positions such as Clerk, Officer,
A. K Manager, Senior Manager (SM), Chief Manager,
B. H Assistant General Manager(AGM), Deputy General
Page 501 of 722

Subscribe the Xpress Video Course & Mock Test Package for Bank & Insurance Exams
If there are any suggestions/ errors in our PDFs Feel Free to contact us via this email: admin@exampundit.in
IBPS RRB Clerk Prelims – Ultra Practice Bundle PDF
Manager(DGM), and General Manager(GM). Their II. F holds aposition higher than B
positions are given in ascending order such that General III. E holds a position higher than G
Manager is the highest position & Clerk is the lowest a) Only I and II
position. b) Only I
A is a Manager. Only one person holds a position between c) Only II and III
A and C. Atmost one person holding a position between d) Only II
C and G. Only two persons hold a position between C and e) None of these
D. The number of persons holding a higher position than 19) How many persons are holding aposition higher
D is one less than the number of people holding a lower than H?
position than F. E holds a position higher than F. E is a) Three
neither Deputy General Manager nor Senior Manager. H b) Two
is Junior to D but not a Clerk . c) One
16) Who among the following person is a Clerk? d) Five
a) A e) Four
b) G 20)If B is related to Officer and G is related to General
c) F Manager in a certain way, then who among the
d) B following is related to Deputy General Manager?
e) D a) F
17) Four of the following are alike in a certain way. b) E
Who among the following one does not belong to that c) D
group? d) A
a) A-Manager e) None of the above
b) C-Chief Manager Direction (21-25): Study the following information
c) G-General Manager carefully and answer the questions below.
d) E-Assistant general manager 7 persons A, B, C, D, E, F and G are working in a company
e) B- Clerk in different designations as follows: Chairman, MD, CFO,
18) Which of the following statements is/are true? CEO, lead, manager and supervisor but not necessarily in
I. A is a manager the same order. They are taking leave on different days.
Page 502 of 722

Subscribe the Xpress Video Course & Mock Test Package for Bank & Insurance Exams
If there are any suggestions/ errors in our PDFs Feel Free to contact us via this email: admin@exampundit.in
IBPS RRB Clerk Prelims – Ultra Practice Bundle PDF
Chairman is the highest position where the Supervisor is b) Manager
the lowest position in a company. c) Lead
A takes leave as fourth and immediately before the person d) CEO
who is a supervisor. Two persons take leave between the e) Chairman
person who is a supervisor and B who is a lead. The 24.How many persons are taking leave after D?
number of persons takes leave after CEO is the same as a) Same as the number of persons takes leave between G
the number of persons takes leave before him. Only one and B
person takes leave between supervisor and CFO. Three b) Same as the number of persons takes leave between C
persons take leave between the chairman and B. Four and B
persons take leave between Lead and C. More than 4 c) Same as the number of persons takes leave between B
persons take a leave between C and G. C is neither a CFO and A
nor a manager. F neither take a leave before A nor after E. d) Same as the number of persons takes leave after E
21.What is the designation of the person who took e) No one
leave immediately before A? 25.Which of the following statement is not correct
a) Manager regarding person C?
b) Lead a) C’s designation is MD
c) CFO b) C took leave after E who is chairman
d) Supervisor c) C took leave before G
e) None of these d) Number of persons took leave before C is 3
22.How many persons are taking leave between B and e) Both c and d
F?
a) Same as the number of persons take leave after A Directions (26-30): Study the following information
b) Same as the number of persons takes leave before B carefully and answer the below questions
c) Same as the number of persons takes leave before F Eight friends Kavi, Kavya, Malar, Mani, Kalai, Karthi,
d) Same as the number of persons take leave after F Kamal, and Mala belong to the different designations.
e) None of these. According to their salary, the designation of the people in
23.What is the designation of G? increasing order are as Clerk, Teacher, Engineer, Lawyer,
a) MD Doctor, Businessman, CEO, and Chairman. (clerk has the
Page 503 of 722

Subscribe the Xpress Video Course & Mock Test Package for Bank & Insurance Exams
If there are any suggestions/ errors in our PDFs Feel Free to contact us via this email: admin@exampundit.in
IBPS RRB Clerk Prelims – Ultra Practice Bundle PDF
lowest salary and the chairman has the highest salary in III. Designation of Kavya is Lawyer
the group). a) Only I and III
The number of people gets less salary than Kalai is twice b) Only III
as that of the number of people gets less salary than Mani. c) Only I and II
Kalai gets more salary than Kavi but just lesser than d) Only II
Kavya. Malar gets just more salary than Kamal. Kavi is e) All are true
not a teacher but gets a low salary than Mani. The number 29) What is the designation of Mani?
of people gets more salary than Kavya is same as the a) Teacher
number of people gets a low salary than Kamal. The salary b) Doctor
of Mala is more than Karthi but she is neither a CEO nor c) Lawyer
a Businessman. d) Engineer
26) Who among the following person is Businessman? e) None of the above
a) Karthi 30) Who among the following gets just more salary
b) Mani than Kamal?
c) Kavi a) The one who is Doctor
d) Malar b) The one who is Teacher
e) Mala c) The one who is a Businessman
27) Four of the following five pairs of persons are alike d) Kavya
in a certain way. Who among the following one does e) Kalai
not belong to the group? Directions (31-35): Study the following information
a) Mala-Karthi carefully and answer the questions given below.
b) Kamal-Kavya Seven persons viz. T, U, V, W, X, Y, and Z are working
c) Kalai-Mani in the police department in which their designations are
d) Kavya-Kalai Commissioner of Police (CP), Deputy Commissioner of
e) Mani-Malar Police (DCP), Assistant Commissioner of Police (ACP),
28) Which of the following statement is/are true? Superintendent of Police (SP), Inspector of Police (IP),
I. Mala gets the highest salary among them Sergeant and Constable but not necessarily in the same
II. Karthi gets more salary than Kamal order.
Page 504 of 722

Subscribe the Xpress Video Course & Mock Test Package for Bank & Insurance Exams
If there are any suggestions/ errors in our PDFs Feel Free to contact us via this email: admin@exampundit.in
IBPS RRB Clerk Prelims – Ultra Practice Bundle PDF
Note: The designations are at decreasing order from CP is 34. How many persons are junior to Y?
the highest designation whereas constable is the lowest a) None
designation b) As many persons are senior to U
V is three designations higher than the one who is c) Two
designated immediately under Z. The one who designated d) Three
as the least is second lower than U. Only one person is e) As many persons are senior to V
designated between X and T where X is higher designated 35. How many persons are designated between V and
than T. Y is neither a Constable nor a DCP, as well as X W?
is not designated immediately above or below Z. a) One
31. Who among the following person is the Deputy b) Two
Commissioner of Police? c) Three
a) The one who is immediately senior to T d) Four
b) Z e) None
c) X Directions (36-40): Study the following information
d) The one who is immediately junior to U carefully and answer the below questions.
e) None of these Six persons namely – P, Q, R, S, T, and U holds a different
32. How many persons are senior to Z? designation in Income tax department viz.- Commissioner
a) One (COM), Additional Deputy Commissioner (ADC), Joint
b) Two Commissioner (JC), Deputy Commissioner (DC),
c) Three Assistant Commissioner (AC), and Income tax Officer
d) Four (ITO). Each person likes different colours viz.- Yellow,
e) None Brown, Green, Black, Red, and Blue. Designations are
33. What is the designation of W? given in decreasing order from Commissioner to Income
a) Sergeant tax Officer.
b) Constable Only two persons are designated higher to the one who
c) Inspector of Police likes Black. Three persons are designated between P and
d) Superintendent of Police S, who likes Brown. P is designated higher to the one who
e) None of these likes Yellow but not as much as the one who likes Black.
Page 505 of 722

Subscribe the Xpress Video Course & Mock Test Package for Bank & Insurance Exams
If there are any suggestions/ errors in our PDFs Feel Free to contact us via this email: admin@exampundit.in
IBPS RRB Clerk Prelims – Ultra Practice Bundle PDF
The one who likes Brown is designated immediate next to a) As many as designated above the one who likes Red.
Q. Neither Q nor R like Black. Two persons are b) Two
designated between T and the person designated c) As many as designated between the one who likes
immediate next to R. One person is designated between U Black and S.
and the one who likes Green. Deputy Commissioner d) Three
doesn’t like Red. e) Either A or C
36) What is the designation of the one who likes Red? 40) Which of the following statement is/are not true?
a) ADC I. The one who likes Red is designated immediately
b) JC previous to Commissioner.
c) AC II. Deputy Commissioner is designated immediately
d) COM next to T.
e) None of these III. Two persons are designated between Q and the one
37) Who among the following person likes Blue? who likes Black.
a) ITO a) Both I and III
b) The one who is designated immediate next to Q. b) Only III
c) JC c) Both II and III
d) R d) Only II
e) None of these e) None of these
38) How many persons are designated lower to the one Directions (41-45): Study the following information
who likes Green? carefully and answer the questions given below.
a) Three Eight persons P, Q, R, S, T, U, V and W are working in a
b) As many as designated between U and the one who company with different grade positions such as CEO, MD,
likes Brown. GM, DGM, Senior Manager, Manager, Assistant Manager
c) Two and Clerk but not necessarily in the same order. Their
d) As many as person designated higher to S. positions are given in descending order, such as CEO as
e) None of these the highest position and Clerk as the lowest position.
39) How many persons are designated between T and Consecutive alphabetical name of the persons doesn’t
the one who likes Blue? hold a consecutive grade position.
Page 506 of 722

Subscribe the Xpress Video Course & Mock Test Package for Bank & Insurance Exams
If there are any suggestions/ errors in our PDFs Feel Free to contact us via this email: admin@exampundit.in
IBPS RRB Clerk Prelims – Ultra Practice Bundle PDF
Two persons hold a position between W and T. Number 44. V is a __________?
of persons holds a position higher than V is twice as that A. Assistant Manager
of the number of persons holds a position higher than P. B. Clerk
As many persons ranking higher than T is same as that of C. GM
lower than V. R’s grade position is immediately higher D. DGM
than U. The grade position of Q is higher than S. E. SM
41. How many persons have a higher-grade position 45. As many persons ranking higher than _____ is
than R? same as that of lower than _____?
A. 2 A. Q, P
B. 4 B. R, Q
C. 3 C. P,T
D. 1 D. U,R
E. R holds the highest position E. None of those given as an option
42. Who among the following person is General Directions (46-50): Study the following information
Manager? carefully and answer the below questions.
A. R There are eight employees of the educational institute
B. U named as M, N, O, P, Q, R, S and T. They have different
C. V designations starting from Clerk, Head Clerk, Assistant
D. S Professor, Professor, HOD, Supervisor, Principal and
E. P Director. All the designations given are to be considered
43. The number of persons holds a position higher than in a given order (as Clerk is considered as Junior-most
R is same as that of the number of persons hold a rank and Director is considered as the Senior-most rank).
position lower than _____? Only two persons are junior rank to O. S is three ranks
A. Q senior to Q. Q’s rank is neither immediate above nor
B. T immediate below to O. R is immediate senior rank to M.
C. V P is a Head Clerk. N is two ranks junior to M. T is not a
D. S Supervisor.
E. W 46) Which of the following statement is not true?
Page 507 of 722

Subscribe the Xpress Video Course & Mock Test Package for Bank & Insurance Exams
If there are any suggestions/ errors in our PDFs Feel Free to contact us via this email: admin@exampundit.in
IBPS RRB Clerk Prelims – Ultra Practice Bundle PDF
a) R is a Principal. d) Both a and b
b) The one who is Supervisor is immediate senior rank to e) Both b and c
Q. 49) Four of the following five pairs of designations and
c) The one who is Assistant Professor is immediate junior employees have a certain relationship with each other.
rank to N. Which one does not belong to the group?
d) Q is a Clerk. a) S- Supervisor
e) All are true. b) M- HOD
47) Who is the immediate junior rank to the one who c) Q- Assistant Professor
is Professor? d) O- Clerk
a) R e) N- Head Clerk
b) Q 50) Who among the following persons are junior-most
c) O and senior-most employees respectively?
d) S a) T, S
e) None of these b) Q, R
48) Which of the following statement is/are true? c) S, T
a) S is Director. d) R, Q
b) M is immediate senior rank to HOD. e) None of these
c) Only two persons are senior rank than Q.

23). Profession Based Puzzle - Solutions


Directions (1-5):
1) Answer: B
2) Answer: C
3) Answer: E
4) Answer: D
5) Answer: C
Solutions
Page 508 of 722

Subscribe the Xpress Video Course & Mock Test Package for Bank & Insurance Exams
If there are any suggestions/ errors in our PDFs Feel Free to contact us via this email: admin@exampundit.in
IBPS RRB Clerk Prelims – Ultra Practice Bundle PDF
Explanation:
 Only three persons are junior to C.
 A is neither MD nor Clerk.
 F’s rank is either immediate above or immediate
below to the person who is a Manager.
 E is three ranks senior to F.
.
 Here, we have to take two cases i.e. Case (1) and
Directions (6-10):
Case (2).
6. Answer: D
7. Answer: B
8. Answer: D
9. Answer: C
10. Answer: E
Solution:

 B is immediate senior to D.
 From the above statement, we have to take three
cases more i.e. Case (1a), Case (2a) and Case (2b).
 Both G and H are junior to A, but both are not
immediate junior to him
Explanation:
 H is senior to G.
We have:
 Only two persons are designated between the one
who travels to Goa and Coorg.
 The Assistant Manager is travelling to Daman.

Page 509 of 722

Subscribe the Xpress Video Course & Mock Test Package for Bank & Insurance Exams
If there are any suggestions/ errors in our PDFs Feel Free to contact us via this email: admin@exampundit.in
IBPS RRB Clerk Prelims – Ultra Practice Bundle PDF
 The one who is travelling to Coorg is designated
immediate next to the one who is travelling to Leh.
That means, in case (1) the one who is travelling
to Leh is a Clerk, in case (2) the one who is
travelling to Leh is a Manager.
Based on the above given information we have:

Case (1) is not valid as the one who is travelling to


Kullu is neither a Manager nor the General Manager.
Directions (11-15):
11. Answer B
12. Answer C
13. Answer D
14. Answer D
Again, we have: 15. Answer A
 The one who is travelling to Kullu is neither a Solution:
Manager nor the General Manager.
That means, in case (2) the one who is travelling
to Kullu is a Clerk, case (1) is not valid.
Based on the above given information we have the final
arrangement as follows:

Explanation:
 Only two persons are senior to I.

Page 510 of 722

Subscribe the Xpress Video Course & Mock Test Package for Bank & Insurance Exams
If there are any suggestions/ errors in our PDFs Feel Free to contact us via this email: admin@exampundit.in
IBPS RRB Clerk Prelims – Ultra Practice Bundle PDF
 Only one person designated between I and N. 19.Answer: e
20.Answer: b
Solution:
Grade positions Persons
Clerk B
Officer F
Manager A

 There are more than two persons designated Senior Manager H

between J and N. Chief Manager C

 J is not the junior-most employee. Assistant General E

 K is senior to J. Manager
Deputy General G
 There are more than two persons designated
Manager
between K and J.
General Manager D
 L is neither General Manager nor Clerk.
Explanation:
 M is junior to H.
 A is a Manager.
 Hence, the final arrangement is:
 Only one person holds a position between A and
C.
 From the above condition, there are two
possibilities.
Grade positions Case1 Case2
Clerk C
Officer
Manager A A
Directions (16-20):
Senior Manager
16.Answer: d
17.Answer: c Chief Manager C

18.Answer: a
Page 511 of 722

Subscribe the Xpress Video Course & Mock Test Package for Bank & Insurance Exams
If there are any suggestions/ errors in our PDFs Feel Free to contact us via this email: admin@exampundit.in
IBPS RRB Clerk Prelims – Ultra Practice Bundle PDF
Assistant General
Manager
Deputy General
Manager
General Manager

 Only two persons hold a position between C & D.


 From the above condition, there are three
possibilities.

 E holds a position higher than F.


 E is neither Deputy General Manager nor Senior
Manager.
 From the above conditions, case1 is dropped.

 The number of persons holding a higher position


than D is one less than the number of people
holding a lower position than F.
 Atmost one person holding a position between C
and G.
 H is Junior to D but not a Clerk .
 From the above conditions, Case2 is dropped.
 Case1a shows the final arrangement.
Grade positions Case2 Case1a

Page 512 of 722

Subscribe the Xpress Video Course & Mock Test Package for Bank & Insurance Exams
If there are any suggestions/ errors in our PDFs Feel Free to contact us via this email: admin@exampundit.in
IBPS RRB Clerk Prelims – Ultra Practice Bundle PDF
Clerk C B  A takes leave as fourth and immediately before the
Officer F person who is a supervisor.
Manager A A  Two persons take leave between the person who is
Senior Manager D H a supervisor and B who is a lead.
Chief Manager C Persons Designations
Assistant General F E
Manager B Lead
Deputy General G
Manager A
General Manager E D Supervisor
Direction (21-25):
21) Answer: c
22) Answer: d
23) Answer: b  Only one person took leave between supervisor
24) Answer: b and CFO. here two possible cases are formed i.e.,
25) Answer: e case 1 and case 2
Solution:  Three persons take between the chairman and
Persons Designations person B.
G Manager  The number of persons taking leave after the CEO
B lead is the same as the number of persons taking leave
D CFO before him.
A CEO CASE 1 CASE 2
F Supervisor Persons Designations Persons Designations
E Chairman
C MD B Lead B lead
Explanation: CFO
 Here we have statements, A CEO A CEO

Page 513 of 722

Subscribe the Xpress Video Course & Mock Test Package for Bank & Insurance Exams
If there are any suggestions/ errors in our PDFs Feel Free to contact us via this email: admin@exampundit.in
IBPS RRB Clerk Prelims – Ultra Practice Bundle PDF
Supervisor Supervisor B Lead
Chairman Chairman D CFO
CFO A CEO
F Supervisor
 Four persons take leave between Lead and C. E Chairman
 C is neither CFO nor manager. C MD
 Here case 2 is violated because C cannot be CFO. Based on the given information, we have the final
 Also, C’s designation is MD as it cannot be arrangement as follows:
Manager in case 1. Persons Designations
CASE 1 CASE 2 G Manager
Persons Designations Persons Designations B lead
D CFO
B Lead B Lead A CEO
CFO F Supervisor
A CEO A CEO E Chairman
Supervisor Supervisor C MD
Chairman Chairman
C MD C CFO Directions (26-30):
 More than 4 persons took leave between C and G. 26.Answer: d
Hence, G can be the Manager 27.Answer: e

 F neither takes leave before A nor after E. 28.Answer: e

 From this we can get, F cannot be placed before A 29.Answer: a

also E cannot be placed before F and A. 30.Answer: c

 Hence E took leave immediately before C. Solution:


Clerk Kavi

Persons Designations Teacher Mani


G Manager Engineer Kalai
Page 514 of 722

Subscribe the Xpress Video Course & Mock Test Package for Bank & Insurance Exams
If there are any suggestions/ errors in our PDFs Feel Free to contact us via this email: admin@exampundit.in
IBPS RRB Clerk Prelims – Ultra Practice Bundle PDF
Lawyer Kavya  From the above condition, case2 was dropped.

Doctor Kamal Case1 Case2 Case3

Businessman Malar Clerk Kavi Kamal

CEO Karthi Teacher Mani

Chairman Mala Engineer Kalai Mani

Explanation: Lawyer Kavya Mani

 The number of people gets less salary than Kalai Doctor Kamal Kalai
is twice as that of the number of people gets less Businessman Kavya
salary than Mani. Kalai
CEO
 From the above condition, there are three
Chairman Kavya
possibilities.
Case1 Case2 Case3
 Kavi is not a teacher but gets a low salary than
Clerk
Mani.
Teacher Mani Case1 Case3
Engineer Kalai Mani
Clerk Kavi Kamal
Lawyer Mani Mani
Teacher
Doctor Kalai Engineer Kalai Kavi
Businessman Lawyer Kavya Mani
CEO Kalai Doctor Kamal
Chairman Businessman
 Kalai gets more salary than Kavi but just lesser CEO Kalai
than Kavya.
Chairman Kavya
 The number of people gets more salary than Kavya
is same as the number of people gets a low salary
 Malar gets just more salary than Kamal.
than Kamal.

Page 515 of 722

Subscribe the Xpress Video Course & Mock Test Package for Bank & Insurance Exams
If there are any suggestions/ errors in our PDFs Feel Free to contact us via this email: admin@exampundit.in
IBPS RRB Clerk Prelims – Ultra Practice Bundle PDF
 The salary of Mala is more than Karthi but she is
neither a CEO nor a Businessman.
From the above condition, case3 was dropped. Case1
shows the final arrangement.

Case1 Case3

Clerk Kavi Kamal

Teacher Mani Malar

Engineer Kalai Kavi

Lawyer Kavya Mani


Explanation:
Doctor Kamal 1. V is three designations higher than the one who is
Businessman Malar designated immediately under Z.

CEO Karthi Kalai

Chairman Mala Kavya

Directions (31-35):
31.Answer: a)
32.Answer: c)
33.Answer: b)
34.Answer: e)
35.Answer: d)
Solution:

2. The one designated the least is second lower than


U. Hence, Case 2 gets eliminated.

Page 516 of 722

Subscribe the Xpress Video Course & Mock Test Package for Bank & Insurance Exams
If there are any suggestions/ errors in our PDFs Feel Free to contact us via this email: admin@exampundit.in
IBPS RRB Clerk Prelims – Ultra Practice Bundle PDF

3. Only one person is designated between X and T Directions (36-40):

where X is higher designated than T. 36. Answer: A


37. Answer: D
38. Answer: B
39. Answer: E
40. Answer: C
Solution:

4. Y is neither a Constable nor DCP, as well as X is


not designated immediately above or below Z.
Hence, Case 1, 4 and 4a get eliminated. Explanation:
We have:
Page 517 of 722

Subscribe the Xpress Video Course & Mock Test Package for Bank & Insurance Exams
If there are any suggestions/ errors in our PDFs Feel Free to contact us via this email: admin@exampundit.in
IBPS RRB Clerk Prelims – Ultra Practice Bundle PDF
 Only two persons are designated higher to the one  Deputy Commissioner doesn’t like Red.
who likes Black. That means, R likes Blue, and Q likes Red.
 Three persons are designated between P and S, Based on the above given information we have the final
who likes Brown. arrangement as follows:
 P is designated higher to the one who likes Yellow
but not as much as the one who likes Black.
 The one who likes Brown is designated immediate
next to Q.
 Neither Q nor R like Black.
That means, P is Assistant Commissioner, and Q
is Additional Deputy Commissioner.
Based on the above given information we have:
Directions (41-45):
41. Answer: E
42. Answer: E
43. Answer: D
44. Answer: E
45. Answer: A
Solution

Again, we have:
 Two persons are designated between T and the
person designated immediate next to R.
That means, R is deputy commissioner and T likes
Yellow.
 One person is designated between U and the one
who likes Green.
Explanation:
That means, P likes Green.
Page 518 of 722

Subscribe the Xpress Video Course & Mock Test Package for Bank & Insurance Exams
If there are any suggestions/ errors in our PDFs Feel Free to contact us via this email: admin@exampundit.in
IBPS RRB Clerk Prelims – Ultra Practice Bundle PDF
The number of persons holds a position higher than V is R’s grade position is immediately higher than U. Case 3
twice as that of the number of persons holds a position is eliminated.
higher than P.

As many persons ranking higher than T is same as that of


lower than V.
The grade position of Q is higher than S.

Two persons hold a position between W and T. So the case


1 gets eliminated.

The final arrangement is-

Page 519 of 722

Subscribe the Xpress Video Course & Mock Test Package for Bank & Insurance Exams
If there are any suggestions/ errors in our PDFs Feel Free to contact us via this email: admin@exampundit.in
IBPS RRB Clerk Prelims – Ultra Practice Bundle PDF
There are eight employees of the educational institute
named as M, N, O, P, Q, R, S and T. They have different
designations starting from Clerk, Head Clerk, Assistant
Professor, Professor, HOD, Supervisor, Principal and
Director.
Clerk is considered as junior-most rank and Director is
considered as senior-most rank.
Only two persons are junior rank to O.
S is three ranks senior to Q.
Q’s rank is neither immediate above nor immediate below

Directions (46-50): to O.

46) Answer: D R is immediate senior rank to M.

47) Answer: C P is a Head Clerk.

48) Answer: D N is two ranks junior to M.

49) Answer: B
50) Answer: A
Solutions

T is not a Supervisor.

Explanation:

Page 520 of 722

Subscribe the Xpress Video Course & Mock Test Package for Bank & Insurance Exams
If there are any suggestions/ errors in our PDFs Feel Free to contact us via this email: admin@exampundit.in
IBPS RRB Clerk Prelims – Ultra Practice Bundle PDF
Case (1) is the final answer.

24). Syllogism Questions

Directions (1-50): In each of the questions below are I. Few Bike are Truck.
given some statements followed by some Conclusions. II. All jeeps are Bike is possibility.
You have to take the given statements to be true even, (a) If Only conclusion I follows.
if they seem to be at variance from commonly known (b) If Only conclusion II follows.
facts. Read all the conclusions and then decide which (c) If Either conclusion I or II follows.
of the given conclusions logically follows from the (d) If Neither conclusion I nor II follows.
given statements disregarding commonly known (e) If Both conclusions I and II follow.
facts. 2. Statements:
(a) If Only conclusion I follows. All Power are Strong.
(b) If Only conclusion II follows. Some Strong are not Leader.
(c) If Either conclusion I or II follows. All Leader are Fat.
(d) If Neither conclusion I nor II follows. Conclusions:
(e) If Both conclusions I and II follow. I. All Leader are Power is a possibility
1. Statements: II. Some Fat are Strong
Few Bike are Car. (a) If Only conclusion I follows.
Some Truck is Jeep. (b) If Only conclusion II follows.
Only a few Jeep are Car. (c) If Either conclusion I or II follows.
Conclusions: (d) If Neither conclusion I nor II follows.
Page 521 of 722

Subscribe the Xpress Video Course & Mock Test Package for Bank & Insurance Exams
If there are any suggestions/ errors in our PDFs Feel Free to contact us via this email: admin@exampundit.in
IBPS RRB Clerk Prelims – Ultra Practice Bundle PDF
(e) If Both conclusions I and II follow. No Plastic are Cup.
3. Statements: Conclusions:
All Sun are Moon. I. Some Fiber are Plastic.
Some Star are Moon. II. Some Plates are Plastic.
No Star are Planet. (a) If Only conclusion I follows.
Conclusions: (b) If Only conclusion II follows.
I. Some Sun are Star. (c) If Either conclusion I or II follows.
II. Some Planet are Sun is possibility. (d) If Neither conclusion I nor II follows.
(a) If Only conclusion I follows. (e) If Both conclusions I and II follow.
(b) If Only conclusion II follows. 6. Statements:
(c) If Either conclusion I or II follows. Only a few Bowls are Spoons.
(d) If Neither conclusion I nor II follows. Some Spoons are Forks.
(e) If Both conclusions I and II follow. No Forks are Plates.
4. Statements: Conclusions:
Few Cup are Plates. I. Few Forks are Bowls.
Some Plates are Fiber. II. Few Plates are Bowls.
No Plastic are Cup. (a) If Only conclusion I follows.
Conclusions: (b) If Only conclusion II follows.
I. Some Fiber are Cup is possibility. (c) If Either conclusion I or II follows.
II. Some Plastic are Plates is possibility. (d) If Neither conclusion I nor II follows.
(a) If Only conclusion I follows. (e) If Both conclusions I and II follow.
(b) If Only conclusion II follows. 7. Statements:
(c) If Either conclusion I or II follows. Some Red is Black.
(d) If Neither conclusion I nor II follows. Some Black is Yellow.
(e) If Both conclusions I and II follow. All Yellow is Green.
5. Statements: Conclusions:
Few Cup are Plates. I. Few Red is Yellow.
Some Plates are Fiber. II. Few Black is Green.
Page 522 of 722

Subscribe the Xpress Video Course & Mock Test Package for Bank & Insurance Exams
If there are any suggestions/ errors in our PDFs Feel Free to contact us via this email: admin@exampundit.in
IBPS RRB Clerk Prelims – Ultra Practice Bundle PDF
(a) If Only conclusion I follows. (e) If Both conclusions I and II follow.
(b) If Only conclusion II follows. 10. Statements:
(c) If Either conclusion I or II follows. No Black is Paper.
(d) If Neither conclusion I nor II follows. All Pencils are Black.
(e) If Both conclusions I and II follow. Few Pens are not Pencils.
8. Statements: Conclusion:
Some Date are Month I: All Black is Pen.
No Date is Year II: Some Paper is not Pencil.
Only Week are Year (a) If Only conclusion I follows.
Conclusion: (b) If Only conclusion II follows.
I: Some Week are not Date (c) If Either conclusion I or II follows.
II: Some Month are not Week. (d) If Neither conclusion I nor II follows.
(a) If Only conclusion I follows. (e) If Both conclusions I and II follow.
(b) If Only conclusion II follows. 11. Statements:
(c) If Either conclusion I or II follows. All North are South.
(d) If Neither conclusion I nor II follows. All South are East.
(e) If Both conclusions I and II follow. All East are West.
9. Statements: Conclusion:
Few Police are Force. I: All North is West.
No force is Army. II: Some West is South.
All Army are Wise. (a) If Only conclusion I follows.
Conclusion: (b) If Only conclusion II follows.
I: Some Force is not Wise (c) If Either conclusion I or II follows.
II: Some Police are Army. (d) If Neither conclusion I nor II follows.
(a) If Only conclusion I follows. (e) If Both conclusions I and II follow.
(b) If Only conclusion II follows. 12. Statements:
(c) If Either conclusion I or II follows. Few Doctors are Ideal.
(d) If Neither conclusion I nor II follows. Few women are doctors.
Page 523 of 722

Subscribe the Xpress Video Course & Mock Test Package for Bank & Insurance Exams
If there are any suggestions/ errors in our PDFs Feel Free to contact us via this email: admin@exampundit.in
IBPS RRB Clerk Prelims – Ultra Practice Bundle PDF
All Ideal is Perfect. (a) If Only conclusion I follows.
Conclusion: (b) If Only conclusion II follows.
I: All Doctors are Perfect. (c) If Either conclusion I or II follows.
II: Few Women are Ideal is possibility. (d) If Neither conclusion I nor II follows.
(a) If Only conclusion I follows. (e) If Both conclusions I and II follow.
(b) If Only conclusion II follows. 15. Statements:
(c) If Either conclusion I or II follows. No Cat is Rat.
(d) If Neither conclusion I nor II follows. No Rat is Bat.
(e) If Both conclusions I and II follow. Only a few Bat are Cow.
13. Statements: Conclusions:
All Cricketer are Batsmen I. Few Cat are Cow is a possibility.
Few Batsmen are Keeper. II. All Bat are Cow is possibility.
All Keeper are Fielder. (a) If Only conclusion I follows.
Conclusion: (b) If Only conclusion II follows.
I: All Fielder can be Cricketer. (c) If Either conclusion I or II follows.
II: Few Batsmen are Fielder. (d) If Neither conclusion I nor II follows.
(a) If Only conclusion I follows. (e) If Both conclusions I and II follow.
(b) If Only conclusion II follows. 16. Statements:
(c) If Either conclusion I or II follows. Some Light are Bulb.
(d) If Neither conclusion I nor II follows. No Bulb is Current.
(e) If Both conclusions I and II follow. Only a few Current are Glow.
14. Statements: Conclusions:
All File are Complete. I. Some light are Current is a possibility.
Few File are Check. II. Some Glow are Bulb is possibility.
Only a few Check are wrong (a) If Only conclusion I follows.
Conclusion: (b) If Only conclusion II follows.
I: All Check are wrong is a possibility. (c) If Either conclusion I or II follows.
II: All Complete are Check. (d) If Neither conclusion I nor II follows.
Page 524 of 722

Subscribe the Xpress Video Course & Mock Test Package for Bank & Insurance Exams
If there are any suggestions/ errors in our PDFs Feel Free to contact us via this email: admin@exampundit.in
IBPS RRB Clerk Prelims – Ultra Practice Bundle PDF
(e) If Both conclusions I and II follow. All Bad is Disaster.
17. Statements: Conclusions:
Few School are College. I. Some Good are not Disaster.
Some School is not Centre. II. Some Disaster can never be movie.
No College are Open. (a) If Only conclusion I follows.
Conclusions: (b) If Only conclusion II follows.
I. Some Centre are Open. (c) If Either conclusion I or II follows.
II. Some School are Centre. (d) If Neither conclusion I nor II follows.
(a) If Only conclusion I follows. (e) If Both conclusions I and II follow.
(b) If Only conclusion II follows. 20. Statements:
(c) If Either conclusion I or II follows. Only Grapes are Mango.
(d) If Neither conclusion I nor II follows. Some grapes are Orange.
(e) If Both conclusions I and II follow. No Orange is Litchi
18. Statements: Conclusions:
Some Men are Smart I. Some Mango are Orange.
Some Smart are Handsome II. All Orange can be Grapes.
No Handsome is Attitude (a) If Only conclusion I follows.
Conclusion: (b) If Only conclusion II follows.
I: All Smart is Men (c) If Either conclusion I or II follows.
II: All Men are Handsome (d) If Neither conclusion I nor II follows.
(a) If Only conclusion I follows. (e) If Both conclusions I and II follow.
(b) If Only conclusion II follows. 21. Statements:
(c) If Either conclusion I or II follows. Some Subject are Reasoning.
(d) If Neither conclusion I nor II follows. All Reasoning are Logical.
(e) If Both conclusions I and II follow. No Logical is Critical.
19. Statements: Conclusions:
All Movies are Good. I. Some Subject are Critical is possibility.
No Good is Bad. II. Few Logical are Subject.
Page 525 of 722

Subscribe the Xpress Video Course & Mock Test Package for Bank & Insurance Exams
If there are any suggestions/ errors in our PDFs Feel Free to contact us via this email: admin@exampundit.in
IBPS RRB Clerk Prelims – Ultra Practice Bundle PDF
(a) If Only conclusion I follows. (c) If Either conclusion I or II follows.
(b) If Only conclusion II follows. (d) If Neither conclusion I nor II follows.
(c) If Either conclusion I or II follows. (e) If Both conclusions I and II follow.
(d) If Neither conclusion I nor II follows. 24. Statements:
(e) If Both conclusions I and II follow. Some File are Copy.
22. Statements: Some Copy are Eraser.
Some Tea are Strong. No Pen is File.
No Tea is a Coffee. Conclusions:
Some Coffee are not Strong. I. Some File are Eraser
All Coffee are not Hot. II. No File is Eraser
Conclusions: (a) If Only conclusion I follows.
I. Some Tea are Hot. (b) If Only conclusion II follows.
II. Some Hot are Strong. (c) If Either conclusion I or II follows.
(a) If Only conclusion I follows. (d) If Neither conclusion I nor II follows.
(b) If Only conclusion II follows. (e) If Both conclusions I and II follow.
(c) If Either conclusion I or II follows. 25. Statements:
(d) If Neither conclusion I nor II follows. Some File are Copy.
(e) If Both conclusions I and II follow. Some Copy are Eraser.
23. Statements: No Pen is File.
Some Tea are Strong. Conclusions:
No Tea is a Coffee. I. Some Copy are Pen is possibility.
Some Coffee are not Strong. II. No Copy are Pen.
All Coffee are not Hot. (a) If Only conclusion I follows.
Conclusions: (b) If Only conclusion II follows.
I. Some Strong are Hot is possibility. (c) If Either conclusion I or II follows.
II. Some Coffee are Strong is possibility. (d) If Neither conclusion I nor II follows.
(a) If Only conclusion I follows. (e) If Both conclusions I and II follow.
(b) If Only conclusion II follows. 26. Statements:
Page 526 of 722

Subscribe the Xpress Video Course & Mock Test Package for Bank & Insurance Exams
If there are any suggestions/ errors in our PDFs Feel Free to contact us via this email: admin@exampundit.in
IBPS RRB Clerk Prelims – Ultra Practice Bundle PDF
Some Boys are Students. No Brown is White.
All students are Talented. Conclusions:
Some Girls are Talented. I. All Green being Brown is a possibility.
Conclusions: II. Only a few Green are Whites.
I. Some Boys are Talented. (a) If Only conclusion I follows.
II. Some Girls are Students. (b) If Only conclusion II follows.
(a) If Only conclusion I follows. (c) If Either conclusion I or II follows.
(b) If Only conclusion II follows. (d) If Neither conclusion I nor II follows.
(c) If Either conclusion I or II follows. (e) If Both conclusions I and II follow.
(d) If Neither conclusion I nor II follows. 29. Statements:
(e) If Both conclusions I and II follow. No Black is White.
27. Statements: Some White are Green.
Some Boys are Students. Some Brown is Green.
All students are Talented. No Brown is White.
Some Girls are Talented. Conclusions:
Conclusions: I. No Black is Brown.
I. Some Boys are Girls. II. Only a few Brown are Green.
II. Some Students are Talented. (a) If Only conclusion I follows.
(a) If Only conclusion I follows. (b) If Only conclusion II follows.
(b) If Only conclusion II follows. (c) If Either conclusion I or II follows.
(c) If Either conclusion I or II follows. (d) If Neither conclusion I nor II follows.
(d) If Neither conclusion I nor II follows. (e) If Both conclusions I and II follow.
(e) If Both conclusions I and II follow. 30. Statements:
28. Statements: No Goa is Pune.
No Black is White. Some Mumbai is Navi.
Some White are Green. All Nashik are Mumbai.
Some Brown is Green. Only a few Mumbai are Goa.
Conclusions:
Page 527 of 722

Subscribe the Xpress Video Course & Mock Test Package for Bank & Insurance Exams
If there are any suggestions/ errors in our PDFs Feel Free to contact us via this email: admin@exampundit.in
IBPS RRB Clerk Prelims – Ultra Practice Bundle PDF
I. Some Goa is Navi. Conclusions:
II. All Nashik can never be Goa. I. Some Wide is Light.
(a) If Only conclusion I follows. II. All Long can be Light.
(b) If Only conclusion II follows. (a) If Only conclusion I follows.
(c) If Either conclusion I or II follows. (b) If Only conclusion II follows.
(d) If Neither conclusion I nor II follows. (c) If Either conclusion I or II follows.
(e) If Both conclusions I and II follow. (d) If Neither conclusion I nor II follows.
31. Statements: (e) If Both conclusions I and II follow.
No Goa is Pune. 33. Statements:
Some Mumbai is Navi. No Road is Wide.
All Nashik are Mumbai. Some Road is Long.
Only a few Mumbai are Goa. Only a few Light is Fuse.
All Fuse are Long.
Conclusions: Conclusions:
I. Some Pune is Navi. I. All Wide can be Long.
II. All Nashik can be Pune. II. All Long can be Road.
(a) If Only conclusion I follows. (a) If Only conclusion I follows.
(b) If Only conclusion II follows. (b) If Only conclusion II follows.
(c) If Either conclusion I or II follows. (c) If Either conclusion I or II follows.
(d) If Neither conclusion I nor II follows. (d) If Neither conclusion I nor II follows.
(e) If Both conclusions I and II follow. (e) If Both conclusions I and II follow.
32. Statements: 34. Statements:
No Road is Wide. Only a few Beers are Pale.
Some Road is Long. No Book is a Beers.
Only a few Light is Fuse. A few Book are Pale.
All Fuse are Long.
Conclusions:
I. All Pale being Book is a possibility
Page 528 of 722

Subscribe the Xpress Video Course & Mock Test Package for Bank & Insurance Exams
If there are any suggestions/ errors in our PDFs Feel Free to contact us via this email: admin@exampundit.in
IBPS RRB Clerk Prelims – Ultra Practice Bundle PDF
II. All Beers can never be Pale. (b) If Only conclusion II follows.
(a) If Only conclusion I follows. (c) If Either conclusion I or II follows.
(b) If Only conclusion II follows. (d) If Neither conclusion I nor II follows.
(c) If Either conclusion I or II follows. (e) If Both conclusions I and II follow.
(d) If Neither conclusion I nor II follows. 37. Statements:
(e) If Both conclusions I and II follow. Only a few Bat are Wicket.
35. Statements: No Bat is Out.
Only a few Bat are Wicket. Only a few LBW are Wicket.
No Bat is Out. Some Out is Catch.
Only a few LBW are Wicket. Conclusions:
Some Out is Catch. I. Some Bat are LBW.
Conclusions: II. All Catch being Out is possibility.
I. Some LBW can never be Out. (a) If Only conclusion I follows.
II. Some Catch is LBW. (b) If Only conclusion II follows.
(a) If Only conclusion I follows. (c) If Either conclusion I or II follows.
(b) If Only conclusion II follows. (d) If Neither conclusion I nor II follows.
(c) If Either conclusion I or II follows. (e) If Both conclusions I and II follow.
(d) If Neither conclusion I nor II follows. 38. Statements:
(e) If Both conclusions I and II follow. Only a few Uber are Ola.
36. Statements: Only a few Rapido are Ola.
Only a few Bat are Wicket. No Auto is Rapido.
No Bat is Out. Conclusions:
Only a few LBW are Wicket. I. All Ola can be Rapido.
Some Out is Catch. II. All Rapido can be Uber.
Conclusions: (a) If Only conclusion I follows.
I. Some Wicket being Out is Possibility. (b) If Only conclusion II follows.
II. Some Bat is being Catch is Possibility. (c) If Either conclusion I or II follows.
(a) If Only conclusion I follows. (d) If Neither conclusion I nor II follows.
Page 529 of 722

Subscribe the Xpress Video Course & Mock Test Package for Bank & Insurance Exams
If there are any suggestions/ errors in our PDFs Feel Free to contact us via this email: admin@exampundit.in
IBPS RRB Clerk Prelims – Ultra Practice Bundle PDF
(e) If Both conclusions I and II follow. Some Post are Mail.
39. Statements: Conclusions:
Only a few Uber are Ola. I. Some Post are definitely not project.
Only a few Rapido are Ola. II. All Post are project.
No Auto is Rapido. (a) If Only conclusion I follows.
Conclusions: (b) If Only conclusion II follows.
I. All Uber are Auto. (c) If Either conclusion I or II follows.
II. Some Rapido are Uber. (d) If Neither conclusion I nor II follows.
(a) If Only conclusion I follows. (e) If Both conclusions I and II follow.
(b) If Only conclusion II follows. 42. Statements:
(c) If Either conclusion I or II follows. No Mail is a Project.
(d) If Neither conclusion I nor II follows. Only a few Projects are Post.
(e) If Both conclusions I and II follow. Some Post are Mail.
40. Statements: Conclusions:
Only a few Uber are Ola. II. All Mail are projects.
Only a few Rapido are Ola. III. All Mail being Post is a possibility.
No Auto is Rapido. (a) If Only conclusion I follows.
Conclusions: (b) If Only conclusion II follows.
I. All Auto are Ola. (c) If Either conclusion I or II follows.
II. Some Uber are not Rapido. (d) If Neither conclusion I nor II follows.
(a) If Only conclusion I follows. (e) If Both conclusions I and II follow.
(b) If Only conclusion II follows. 43. Statements:
(c) If Either conclusion I or II follows. All Area are Square.
(d) If Neither conclusion I nor II follows. All Square are Triangle.
(e) If Both conclusions I and II follow. Some Triangle are Rectangle.
41. Statements: All Rectangle are Cube.
No Mail is a Project. Conclusions:
Only a few Projects are Post. I. All Cube are Square.
Page 530 of 722

Subscribe the Xpress Video Course & Mock Test Package for Bank & Insurance Exams
If there are any suggestions/ errors in our PDFs Feel Free to contact us via this email: admin@exampundit.in
IBPS RRB Clerk Prelims – Ultra Practice Bundle PDF
II. Some Triangle are Cube is possibility. (b) If Only conclusion II follows.
(a) If Only conclusion I follows. (c) If Either conclusion I or II follows.
(b) If Only conclusion II follows. (d) If Neither conclusion I nor II follows.
(c) If Either conclusion I or II follows. (e) If Both conclusions I and II follow.
(d) If Neither conclusion I nor II follows. 46. Statements:
(e) If Both conclusions I and II follow. All One is Three.
44. Statements: Only a few Three are Five.
All Area are Square. Some Five are Two.
All Square are Triangle. All Two are Seven.
Some Triangle are Rectangle. Conclusions:
All Rectangle are Cube. I. Some One can be Five.
Conclusions: II. All Two are Three.
I. All Square are Area. (a) If Only conclusion I follows.
II. All Area are Triangle. (b) If Only conclusion II follows.
(a) If Only conclusion I follows. (c) If Either conclusion I or II follows.
(b) If Only conclusion II follows. (d) If Neither conclusion I nor II follows.
(c) If Either conclusion I or II follows. (e) If Both conclusions I and II follow.
(d) If Neither conclusion I nor II follows. 47. Statements:
(e) If Both conclusions I and II follow. No One is Three.
45. Statements: Only a few Three are Five.
All Area are Square. Some Five are Two.
All Square are Triangle. All Two are Seven.
Some Triangle are Rectangle.
All Rectangle are Cube. Conclusions:
Conclusions: I. All Three are Seven.
I. All triangle can be square. II. Some Two are definitely Five.
II. All Square are Cube. (a) If Only conclusion I follows.
(a) If Only conclusion I follows. (b) If Only conclusion II follows.
Page 531 of 722

Subscribe the Xpress Video Course & Mock Test Package for Bank & Insurance Exams
If there are any suggestions/ errors in our PDFs Feel Free to contact us via this email: admin@exampundit.in
IBPS RRB Clerk Prelims – Ultra Practice Bundle PDF
(c) If Either conclusion I or II follows. Conclusions:
(d) If Neither conclusion I nor II follows. I. All Apple being Tasty is possibility.
(e) If Both conclusions I and II follow. II. Some Orange are Apple.
48. Statements: (a) If Only conclusion I follows.
No One is Three. (b) If Only conclusion II follows.
Only a few Three are Five. (c) If Either conclusion I or II follows.
Some Five are Two. (d) If Neither conclusion I nor II follows.
All Two are Seven. (e) If Both conclusions I and II follow.
Conclusions: 50. Statements:
I. All Two being Three is possibility. All Apple are Fruit.
II. All Five can be Seven. All Orange are fruit.
(a) If Only conclusion I follows. Some Fruit are Tasty.
(b) If Only conclusion II follows. Some Mango Are not Tasty.
(c) If Either conclusion I or II follows. Conclusions:
(d) If Neither conclusion I nor II follows. I. All Mango are being Orange is possibility.
(e) If Both conclusions I and II follow. II. Some Fruit are being Mango is possibility.
49. Statements: (a) If Only conclusion I follows.
All Apple are Fruit. (b) If Only conclusion II follows.
All Orange are fruit. (c) If Either conclusion I or II follows.
Some Fruit are Tasty. (d) If Neither conclusion I nor II follows.
Some Mango Are not Tasty. (e) If Both conclusions I and II follow.

Page 532 of 722

Subscribe the Xpress Video Course & Mock Test Package for Bank & Insurance Exams
If there are any suggestions/ errors in our PDFs Feel Free to contact us via this email: admin@exampundit.in
IBPS RRB Clerk Prelims – Ultra Practice Bundle PDF

24). Syllogism Questions - Solution and Detailed Explanation

Solution1:b) If Only conclusion II follows.

Solution 2: a) If Only conclusion I follow.

Solution5: d)Neither conclusion I nor II follows.

Solution 3:b) If Only conclusion II follow.

Solution4: e) If Both conclusions I and II follow.

Solution6: d) If Neither conclusion I nor II follows.


Page 533 of 722

Subscribe the Xpress Video Course & Mock Test Package for Bank & Insurance Exams
If there are any suggestions/ errors in our PDFs Feel Free to contact us via this email: admin@exampundit.in
IBPS RRB Clerk Prelims – Ultra Practice Bundle PDF

Solution7: b) If Only conclusion II follows. Solution10:b) If Only conclusion II follows.

Solution8:a) If Only conclusion I follow.


Solution11: e) If Both conclusions I and II follow.

Solution9:d) If Neither conclusion I nor II follows.

Solution12: b) If Only conclusion II follows.

Page 534 of 722

Subscribe the Xpress Video Course & Mock Test Package for Bank & Insurance Exams
If there are any suggestions/ errors in our PDFs Feel Free to contact us via this email: admin@exampundit.in
IBPS RRB Clerk Prelims – Ultra Practice Bundle PDF

Solution13:e) If Both conclusions I and II follow.

Solution16: If Both conclusions I and II follow.

Solution14: d) If Neither conclusions I and II follow.

Solution17: d) If Neither conclusion I nor II follows.

Solution15: a) If Only conclusion I follow.

Page 535 of 722

Subscribe the Xpress Video Course & Mock Test Package for Bank & Insurance Exams
If there are any suggestions/ errors in our PDFs Feel Free to contact us via this email: admin@exampundit.in
IBPS RRB Clerk Prelims – Ultra Practice Bundle PDF
Solution18:d) If Neither conclusion I nor II follows. Solution22: d)If Neither conclusion I nor II follows.

Solution19:b) If Only conclusion II follow.

Solution20: b) If Only conclusion II follows. Solution23:e) If Both conclusion I and II follows.

Solution21:e) If Both conclusions I and II follow.

Solution24: c)If Either conclusion I or II follows.

Page 536 of 722

Subscribe the Xpress Video Course & Mock Test Package for Bank & Insurance Exams
If there are any suggestions/ errors in our PDFs Feel Free to contact us via this email: admin@exampundit.in
IBPS RRB Clerk Prelims – Ultra Practice Bundle PDF

Solution27: b)If Only conclusion II follows.

Solution25: a)If Only conclusion I follow.

Solution28: b)If Only conclusion II follows.

Solution26:a) If Only conclusion I follow.

Solution29: d) If Neither conclusion I nor II follows.

Page 537 of 722

Subscribe the Xpress Video Course & Mock Test Package for Bank & Insurance Exams
If there are any suggestions/ errors in our PDFs Feel Free to contact us via this email: admin@exampundit.in
IBPS RRB Clerk Prelims – Ultra Practice Bundle PDF

Solution32: b) If Only conclusion II follows.


Solution30:d) If Neither conclusion I nor II follows.

Solution33: e)If Both conclusions I and II follow.

Solution31: d) If Neither conclusion I nor II follows

Page 538 of 722

Subscribe the Xpress Video Course & Mock Test Package for Bank & Insurance Exams
If there are any suggestions/ errors in our PDFs Feel Free to contact us via this email: admin@exampundit.in
IBPS RRB Clerk Prelims – Ultra Practice Bundle PDF

Solution34: b) If Only conclusion II follows.


Solution36: e) If Both conclusions I and II follow.

Solution35: d) If Neither conclusion I nor II follows.

Solution37: b) If Only conclusion II follows.

Page 539 of 722

Subscribe the Xpress Video Course & Mock Test Package for Bank & Insurance Exams
If there are any suggestions/ errors in our PDFs Feel Free to contact us via this email: admin@exampundit.in
IBPS RRB Clerk Prelims – Ultra Practice Bundle PDF

Solution41: a) If Only conclusion I follow.

Solution38: e) If Both conclusions I and II follow.

Solution42: b) If Only conclusion II follows.

Solution39: d) If Neither conclusion I nor II follows.

Solution40:d) If Neither conclusion I nor II follows.

Solution43: d) If Neither conclusion I nor II follows.

Page 540 of 722

Subscribe the Xpress Video Course & Mock Test Package for Bank & Insurance Exams
If there are any suggestions/ errors in our PDFs Feel Free to contact us via this email: admin@exampundit.in
IBPS RRB Clerk Prelims – Ultra Practice Bundle PDF
Solution47: b) If Only conclusion II follows.

Solution44: b) If Only conclusion II follows.

Solution48: e) If Both conclusions I and II follow.

Solution45: a) If Only conclusion I follows.


Solution49: a) If Only conclusion I follow.

Solution46: a) If Only conclusion I follows.


Solution50: e) If Both conclusions I and II follow.

Page 541 of 722

Subscribe the Xpress Video Course & Mock Test Package for Bank & Insurance Exams
If there are any suggestions/ errors in our PDFs Feel Free to contact us via this email: admin@exampundit.in
IBPS RRB Clerk Prelims – Ultra Practice Bundle PDF

25. Conditional Coding

Directions (1 – 5): Consider the following data: e) None of these


E J L O N 0 3 2 U 6 9 X 5 2. L695UO
} ^ # * ? & % $ ! > ~ | @ a) #>%@!@
b) #>!@!!
Each element is denoted by a symbol as shown above. c) #>!*!$
Further, consider the given conditions and answer d) #}!@~!
accordingly. e) None of these
1. If the third element is odd and sixth element is a vowel, 3. 6XOE2J
then both the elements should be coded as '!'. a) ||*}@|
2. If the second element is consonant and sixth element is b) >|*}$|
also a consonant, then both the elements should be coded c) @~*}$|
as the code of the second element. d) >^&}$|
3. If the fifth and sixth elements are odd, then the code for e) None of these
both the elements should be interchanged. 4. 2XU359
1. 0XUN2L a) $#!%%@
a) %!!?$| b) &?!%~@
b) &|!?$| c) #|!$~@
c) &|>~$| d) $|!%~@
d) &|!!$| e) None of these

Page 542 of 722

Subscribe the Xpress Video Course & Mock Test Package for Bank & Insurance Exams
If there are any suggestions/ errors in our PDFs Feel Free to contact us via this email: admin@exampundit.in
IBPS RRB Clerk Prelims – Ultra Practice Bundle PDF
5. L326JE a) NQHTFE
a) &%$>^> b) NQHTFN
b) #^$^^} c) EQHTFN
c) #%$>^} d) EQHTFE
d) @%$~^} e) None of these
e) None of these 7. 4$%3*6
Directions (6 – 10):In each question below is given a a) PAMHUI
group of letters followed by four combinations of b) PAMHUP
digits/symbols numbered (1), (2), (3) and (4). You have c) IAMHUP
to find out which of the combinations correctly d) IAMHUI
represents the group of letters based on the following e) None of these
coding system and the conditions that follow and mark 8. 9@$3%6
the number of that combination as your answer. If a) IWAHMI
none of combinations correctly represents the group of b) RWAHMI
letters, mark (5) i.e. 'None of these' as your answer. c) IWAHMR
Digit/S 9 @ 2 4 $ 3 7 % + 1 5 6 * 8 © d) RWHAMR
ymbol e) None of these
Letter R W F P A H B M E J Q I U N T 9. 728%9$
Conditions: a) AFNMRB
(i) If the first element is an even digit and the last element b) BFNMRB
is a symbol, both are to be coded as the code for the even c) AFNMRA
digit. d) BFNMRA
(ii) If the first element is a symbol and the last element is e) None of these
an odd digit, their codes are to be interchanged. 10. ©7+541
(iii) If the first element is an odd digit and the last element a) TBEQPT
is an even digit, both are to be coded as the code for the b) TBEQPJ
odd digit. c) JBEQPT
6. 853©2+ d) JBEQPJ
Page 543 of 722

Subscribe the Xpress Video Course & Mock Test Package for Bank & Insurance Exams
If there are any suggestions/ errors in our PDFs Feel Free to contact us via this email: admin@exampundit.in
IBPS RRB Clerk Prelims – Ultra Practice Bundle PDF
e) None of these b) GC*#KK
Directions (11 – 15): Consider the following data and c) GC*K#G
answer the questions based on the following d) GC*#KG
conditions. e) G*C#KG
Digit 1 0 5 2 6 3 9 7 4 8 13. 697845
Symbol @ A # C K $ % G * & a) K%G&*K
b) G%G&*K
Conditions: c) K%G*&K
1. If the first digit is even and the last digit is odd, then d) KG%&*K
both are to be coded as the code of the first digit. e) #%G&*#
2. If the first digit is odd and the last digit is even, then the 14. 987366
codes for the first and the last digits are to be interchanged. a) K&G$KK
3. If the first as well as the last digit is odd, then both are b) %&G$K%
to be coded as the code of the last digit. c) K&$GK%
4. If the first as well as the last digit is even, then both are d) K&G$%K
to be coded as the code for the highest digit present in that e) K&G$K%
particular group. 15. 928657
Each digit is denoted by a symbol as shown above. a) %C&K#%
Further, consider the given conditions and answer the b) GC&#KG
questions accordingly for a given number: c) GC&K#G
11. 243578 d) G&CK#G
a) &*$G# & e) GC&#K%
b) &*$#G&
c) C*$#G&
d) C*$#GC
e) &$*#G&
12. 124567 Directions (16 – 20): Consider the following data:
a) CC*#KC
Page 544 of 722

Subscribe the Xpress Video Course & Mock Test Package for Bank & Insurance Exams
If there are any suggestions/ errors in our PDFs Feel Free to contact us via this email: admin@exampundit.in
IBPS RRB Clerk Prelims – Ultra Practice Bundle PDF
Symb 1 A U 6 T 2 F G 4 I O 5 P a) !β$π¥!
ol b) !β$¥π@
Code @ & $ ! # % ^ * € β © π ¥ c) !β$¥π!
Each element in the 1st row is denoted by a symbol in its d) !β$%¥!
respective row below it. Further, consider the given e) None of these
conditions and answer accordingly. 19. 4PGIF2
1.If the first element is a consonant and the last element is a) €¥β*^%
vowel, then code for both the elements are to be b) ¥€*β^%
interchanged. c) €%%β^%
2. If the third element is consonant and the third element d) €¥*β^%
is preceded by an odd number then both are to be coded e) None of these
as %. 20. T1FU2G
3. If the first element is odd and the last element is even, a) #%%$%*
then code for the odd number must be replaced with the b) #@%$%*
code for the even number. c) #^@$%*
16. T2G5PO d) #@^$%*
a) #%*𥩠e) None of these
b) ©%*π¥# Directions (21 – 25): Study the following information
c) ©*%π¥# carefully and answer the questions given below it.
d) ©%π*¥# Digits in the numbers are to be coded as follows
e) None of these
17. U5G14F
a) $%%@^€
b) €$%%@€
Conditions:
c) $%%@€^
i) If the first digit is an odd number and the last digit is an
d) $π%@€^
even number, then their codes are to be interchanged.
e) None of these
18. 1IU5P6
Page 545 of 722

Subscribe the Xpress Video Course & Mock Test Package for Bank & Insurance Exams
If there are any suggestions/ errors in our PDFs Feel Free to contact us via this email: admin@exampundit.in
IBPS RRB Clerk Prelims – Ultra Practice Bundle PDF
ii) If the first digit is an even number and the last digit is d)#A$@#
an odd number then the third is to be the first digit. e) None of these
iii) If both the first and the last digits are an even number, 25. What is the code for “32506”?
then both are to be coded as the last digit. a) %@#$%
iv) If both the first and the last digits are an odd number, b) %$@#B
then both are to be coded as the first digit. c) B@#$B
21. What is the code for “27138”? d) %@#$B
a) @EABG e) None of these
b) GEAB@ Directions (26 – 30): In each of the questions given
c) GEABG below, a group of digits/letter is given followed by four
d) @EAB@ combinations of symbols numbered (a), (b), (c) and (d).
e) None of these You have to find out which of the four combinations
22. What is the code for “13054”? correctly represents the group of digits/letters based
a) DB$#D on the symbol codes and the conditions given below. If
b) AB$#A none of the four combinations represents the group of
c) AB$#D digits correctly, give (e) ‘None of these’ as the answer.
d) DB$#A
e) None of these
23. What is the code for “12345”? Conditions for coding-
a) AD@B# (1) if the first element is odd number and last element is
b) A@BDA vowel then both are coded to be as the code of first
c) #@BD# element.
d) #B@DA (2) if first as well as last element is odd number then both
e) None of these are coded as the code of last element.
24. What is the code for “41025”? (3) if first element is consonant and last element is vowel
a) DA$@# then code of first and last element are to be Interchanged.
b) DA$@D
c) DAD@#
Page 546 of 722

Subscribe the Xpress Video Course & Mock Test Package for Bank & Insurance Exams
If there are any suggestions/ errors in our PDFs Feel Free to contact us via this email: admin@exampundit.in
IBPS RRB Clerk Prelims – Ultra Practice Bundle PDF
(4) if first element is consonant and last element is b) >+#^©
divisible by three then both are coded to be as @. c) <^#+>
26. What is the code for 31OR9? d) ©*@!+
a) !&*<! e) None of these
b) !*&>! Directions (31 – 35): Numerals are coded as given
c) *©¥+! below and some conditions are given.
d) !@*<! Accordingly, you have to find out which coding is
e) None of these. correct.
27. What is the code for 3PQZU? 4 6 7 8 1 3 5 0 9 2
a) ¥>+#¥ # B M % E $ Z P & Y
b) ¥*<^*
c) ¥#>+¥ Conditions: -
d) #¥>+¥ a) If first digit as well as last digit is odd then both will be
e) None of these. coded as ‘A’
28. What is the code for Q9ZPU? b) If first digit as well as last digit is even then both will
a) @!+#> be coded as ‘0’
b) >!+#@ c) If first digit is even and last digit is odd, then the code
c) @!¥+> will be interchanged
d) ^@+!> d) If first digit is odd and last digit is even then both will
e) None of these be coded as the even number.
29. What is the code for ROPZ9? e) If the number starts or end with zero, then zero will be
a) @+#*@ coded as ‘*’
b) *><¥^ 31. 316726
c) @*#+@ a) BBEMY4
d) !*#+@ b) BEBM$4
e) None of these c) BEBMYB
30. What is the code for RXPZQ? d) BMBMY4
a) ^©#+> e) None of these
Page 547 of 722

Subscribe the Xpress Video Course & Mock Test Package for Bank & Insurance Exams
If there are any suggestions/ errors in our PDFs Feel Free to contact us via this email: admin@exampundit.in
IBPS RRB Clerk Prelims – Ultra Practice Bundle PDF
32. 276134 combinations correctly represents the word based on
a) 0M0BE$ the symbol codes and the conditions given below. If
b) 0MBE$0 none of the combinations matches, choose ‘None of
c) 0MBE#0 these’ as your answer.
d) MB0#$B Le Y F I P S E H R C U M WX A J G O
e) None of these tte
33. 097321 r
a) *&M$YE Co 7 @ 5 4 1 # 6 $ % 9 & 3 2 ? 0 8 !
b) *MYE&B de
c) EY$M&*
d) $EYM$* Condition 1:If first and last letter in a word are vowels
e) None of these then the code will be written in reversed order.
34. 156587 Condition 2:If there is no vowel in the word then code for
a) AMB0%A the first three letters of the word will be written in reverse
b) A$B%SA order.
c) A%BEZA Condition 3:If the letter on second and fourth position in
d) AZBZ%A the word is consonant then the codes of both them will be
e) None of these interchanged.
35. 825763 Note- If more than one condition satisfies the given word
a) #ZYNB% then condition 2 is to be applied only.
b) YZM%#B 36. What would be the code of the word ‘UXAMF’?
c) $YZMB% a) 9@0?3
d) #YMZ%B b) &9?25
e) None of these c) 9&?2@
Directions (36 – 40): In each of the following questions d) 93&5?
given below, a word is given followed by four e) None of these
combinations of symbols and digits labeled A, B, C and 37. What would be the code of the word ‘GIFEM’?
D. You have to find out which of the following four a) 8@5&#
Page 548 of 722

Subscribe the Xpress Video Course & Mock Test Package for Bank & Insurance Exams
If there are any suggestions/ errors in our PDFs Feel Free to contact us via this email: admin@exampundit.in
IBPS RRB Clerk Prelims – Ultra Practice Bundle PDF
b) #58&@
c) 8&#5@ Lette A D O U S P N E L T W K Z Q
d) 8@#&5 rs
e) None of these Num 5 % 6 1 $ 8 & 4 7 # @ 2 + 3
38. What would be the code of the word ‘WJSFC’? ber /
a) 781$@ Symb
b) 08%1@ ols
c) 03%1@ Code
d) 103@% s
e) None of these
39. What would be the code of the word ‘AHIUE’? Conditions:
a) #9?4$ (i) If the first element is a consonant and the last element
b) #956? is a vowel, then the consonant is to be coded as *
c) $954? (ii) If both the first and last elements are vowels, then the
d) #9?56 last vowel is to be coded as the code for the fourth
e) None of these element.
40. What would be the code of the word ‘HCYUX’? (iii) If the group of elements does not contain any vowel,
a) 6%792 third element is to be coded as the code for the last
b) #6%92 element.
c) 6#%&2 41. OTWDSA
d) 2%769 a) 6#@%$%
e) None of these b) %@#%5%
Directions (41 – 45): In this question is given a group c) 6#@%$5
of letters followed by five combinations of d) 6#@%$@
numbers/symbols codes. You have to find out which of e) 6#@5$5
the combinations correctly represents the group of 42. ENZQLA
letters based on the given coding system and the a) 4&+373
conditions and give that combination as your answer. b) 4&+575
Page 549 of 722

Subscribe the Xpress Video Course & Mock Test Package for Bank & Insurance Exams
If there are any suggestions/ errors in our PDFs Feel Free to contact us via this email: admin@exampundit.in
IBPS RRB Clerk Prelims – Ultra Practice Bundle PDF
c) 4*+375 Letters D N W A S E K T O L P U Z Q
d) 4&+375 Number/ 4 % $ 3 @ 8 & 5 1 + 2 # 6 7
e) *&+373 Symbols
43. KPELQU Codes
a) *84731 Conditions:
b) 284731 (Note: Counting of elements is done from left to right)
c) *84732 i. If the first element is a vowel and the last element is a
d) 28473* consonant, then all the vowels are to be coded as©
e) *8473* ii. If both the first and last elements are vowels, then the
44. WTDKNP first vowel is to be coded as the code for the third element
a) @#8&28 iii. If the group of elements does not contain any vowel,
b) @#%2&8 the codes for second and last element are to be
c) @#%2&% interchanged
d) @#%2&2 46. OZTENW
e) @#82&8 a) ©65©S%
45. KZLTDP b) ©658%$
a) 2+7#%7 c) 156©%$
b) 2+8#%8 d) ©65©%$
c) *+7#%8 e) 165©%S
d) #+82%8 47. AZKEUS
e) 2+7#%8 a) ©6&8©@
Directions (46 – 50): In this question is given a group b) ©6&©#@
of letters followed by five combinations of numbers / c) @6&©©3
symbols codes. You have to find out which of the d) ©6&©@©
combinations correctly represents the group of letters e) ©6&©©@
based on the following coding system and the 48. STKPDW
conditions and mark that combination as your answer. a) @$2&45
b) @$&452
Page 550 of 722

Subscribe the Xpress Video Course & Mock Test Package for Bank & Insurance Exams
If there are any suggestions/ errors in our PDFs Feel Free to contact us via this email: admin@exampundit.in
IBPS RRB Clerk Prelims – Ultra Practice Bundle PDF
c) @5&24$ e) 8$867#
d) @$&245 50. LPZDNS
e)&$@245 a) @246%+
49. EWDZQU b) +@6%42
a) 4$467# c) @264%+
b) 4S476# d) +@62%4
c) 8S476# e) +@64%2
d) 4$867#

25). Conditional Coding Decoding Questions - Answers with Explanation


1. Answer: B) the code should be: $|!%~@
By rule (2), If the second element is consonant and sixth 5. Answer: C)
element is consonant, then both the elements should be It follows none of the given rules, the code should be:
coded as the code of the second element. #%$>^}
the code should be: &|!?$| 6. Answer: B)
2. Answer: B) 8 5 3 © 2 +
By rule (1),If the third element is odd and sixth element is N Q H T F E
vowel, then both the elements should be coded as '!' Here condition (i) is applicable, hence the code is
the code should be: #>!@!! NQHTFN
3. Answer: B) 7. Answer: A)
By rule (2),If the second element is consonant and sixth 4 $ % 3 * 6
element is consonant, then both the elements should be P A M H U I
coded as the code of second element.
the code should be: >|*}$| 8. Answer: E)
4. Answer: D) 9 @ $ 3 % 6
By rule (3),If the fifth and sixth elements are odd, then the R W A H M I
code for both the elements should be interchanged.

Page 551 of 722

Subscribe the Xpress Video Course & Mock Test Package for Bank & Insurance Exams
If there are any suggestions/ errors in our PDFs Feel Free to contact us via this email: admin@exampundit.in
IBPS RRB Clerk Prelims – Ultra Practice Bundle PDF
Here condition (iii) is applicable, hence the code is: Code of 9 and 6 will be interchanged.
RWAHMR 8-&, 7-G, 3-$, 6-K
9. Answer: D) So, the code is K&G$K%.
7 2 8 % 9 $ 15. Answer: C)
B F N M R A By rule 3,
10. Answer: C) 9 and 7 will be coded as G
© 7 + 5 4 1 2-C, 8-&, 6-K, 5-#
T B E Q P J So, the code is GC&K#G
Here condition (ii) is applicable, hence the code is 16. Answer: B)
JBEQPT By rule no. 1

11. Answer: B) First element is a consonant and last element is vowel so


By rule 4, code for both the elements should be interchanged.
Highest number is 8 in that group so 2 and 8 will be coded T 2 G 5 P O
as &. © % * π ¥ #
4-*, 3-$, 5-#, 7-G
So, the code is &*$#G&. 17. Answer: C)
12. Answer: D) By rule no. 2
By rule 3, Third element is a consonant and the third element
1 and 7 will be code as G. preceded by an odd number then both should be coded as
2-C, 4-*, 5-#, 6-K %.
So, the code is GC*#KG. U 5 G 1 4 F
13. Answer: A) $ % % @ € ^
By rule 1,
6 and 5 will be coded as K 18. Answer: A)
9-%, 7-G, 8-&, 4-* By rule no. 3
So, the code is K%G&*K. First element is an odd number and last element is even,
14. Answer: E) then code for the odd number must be replaced with the
By rule 2, code for the even number.
Page 552 of 722

Subscribe the Xpress Video Course & Mock Test Package for Bank & Insurance Exams
If there are any suggestions/ errors in our PDFs Feel Free to contact us via this email: admin@exampundit.in
IBPS RRB Clerk Prelims – Ultra Practice Bundle PDF
1 I U 5 P 6 25. Answer: d
! β $ Π ¥ ! It follows condition (i). So, ‘32056’ can be coded as
19. Answer: D) ‘%@#$B’
No Rule follows. Code will remain unchanged. 26. Answer: A)
4 P G I F 2 It follows condition (ii). So ‘31OR9’ can be coded as
€ ¥ * Β ^ % ‘!&*<!’
27. Answer: C)
20. Answer: A) It follows condition (i). So, ‘3PQZU’ can be coded as
By rule no. 2 ‘¥#>+¥’
Third element is a consonant and the third element 28. Answer: A)
preceded by an odd number. so both should be coded as It follows condition (iii). So, ‘Q9ZPU’ can be coded as
%. ‘@!+#>’

T 1 F U 2 G 29. Answer: C)

# % % $ % * It follows condition (iv). So, ‘ROPZ9’ can be coded as


‘@*#+@’

21. Answer: c 30. Answer: C)

It follows condition (iii). So, ‘27138’ can be coded as No Rule follows. Code will remain unchanged.

‘GEABG’ So, ‘RXPZQ’ can be coded as ‘<^#+>’

22. Answer: d 31. Answer: C)

It follows condition (i). So, ‘13054’ can be coded as First number is odd and last number is even, so it will be

‘DB$#A’ coded as even number

23. Answer: b 32. Answer: B)

It follows condition (iv). So, ‘12345’ can be coded as condition b follow, so both will be coded as 0

‘A@BDA’ 33. Answer: A)

24. Answer: E condition e follow, the number starts with 0, so zero will

It follows condition (ii). So, ‘41025’ can be coded as be coded as *

‘$A$@#’ 34. Answer: D)

Page 553 of 722

Subscribe the Xpress Video Course & Mock Test Package for Bank & Insurance Exams
If there are any suggestions/ errors in our PDFs Feel Free to contact us via this email: admin@exampundit.in
IBPS RRB Clerk Prelims – Ultra Practice Bundle PDF
condition a follow, so both the first and last digit will be Then, the code of the word ‘AHIUE’ is ‘?659#’ after
coded as A reversing the code ‘#956?’.
35. Answer: C) 40. Answer: A)
condition c follows, so codes will be interchanged Here, we can see that there is no condition applicable in
36. Answer: C) this case. So, we can write the code of ‘HCYUX’ directly
Here, we can see that letters on second and fourth position from the given table:
in the word ‘UXAMF’ are consonants. Therefore, Then, the code of the word ‘HCYUX’ is ‘6%792’.
Condition 3 can be applied: 41. Answer: A)
Then, the code of the word ‘UXAMF’ is ‘92?&@’ after Condition (ii) is applicable
applying the condition ‘9&?2@’. Then the code of the word OTWDSA is 6#@%$%
37. Answer: E) 42. Answer: A)
Here, we can see that there is no condition applicable in Condition (ii) is applicable
this case. So, we can write the code of ‘GIFEM’ directly Then the code of the word ENZQLA is 4&+373
from the given table: 43. Answer: A)
Then, the code of the word ‘GIFEM’ is ‘85@#&’. Condition (i) is applicable
38. Answer: D) Then the code of the word KPELQU is *84731
Here, we can see that there is no vowel in the word 44. Answer: E)
‘WJSFC’. Therefore, Condition 2 can be applied. Condition (iii) is applicable
Also, the second and fourth letter is a consonant, thus 45. Answer: B)
Condition 4 is also applicable. Condition (iii) is applicable
But in such a case only condition 2 is to be applied. Then the code of the word KZLTDP is 2+8#%8
Thus, the code of the word ‘WJSFC’ is ‘301@%’ after 46. Answer: D)
reversing codes for the first three letters ‘103@%’. Condition (i) is applicable
39. Answer: B) Then the code of the word OZTENW is ©65©%$
Here, we can see that first and last letter in the word 47. Answer: E)
‘AHIUE’ are vowels. Therefore, Condition 1 can be Condition (i) is applicable
applied: Then the code of the word AZKEUS is ©6&©©@
48. Answer: D)
Page 554 of 722

Subscribe the Xpress Video Course & Mock Test Package for Bank & Insurance Exams
If there are any suggestions/ errors in our PDFs Feel Free to contact us via this email: admin@exampundit.in
IBPS RRB Clerk Prelims – Ultra Practice Bundle PDF
Condition (iii) is applicable Then the code of the word EWDZQU is 4$467#
Then the code of the word STKPDW is @$&245 50. Answer: E)
49. Answer: A) Condition (iii) is applicable
Condition (ii) is applicable Then the code of the word LPZDNS is +@64%2

IBPS RRB PO Prelims – Quantitative Aptitude

S.no Topic Page number


1. Approximation
Questions 560
Solutions 566
2. Simplification
Questions 571
Solutions 577
3. Missing Number
Questions 581
Solutions 586
4. Wrong Number
Questions 592
Solutions 597
5. Quadratic Equation
Questions 603
Solutions 610
6. DI – Table
Questions 624

Page 555 of 722

Subscribe the Xpress Video Course & Mock Test Package for Bank & Insurance Exams
If there are any suggestions/ errors in our PDFs Feel Free to contact us via this email: admin@exampundit.in
IBPS RRB Clerk Prelims – Ultra Practice Bundle PDF
Solutions 636
7. DI- Bar
Questions 642
Solutions 656
8. DI- Line
Questions 662
Solutions 675
9. DI- Pie
Questions 680
Solutions 699
10. Caselet DI
Questions 705
Solutions 715

1). Approximation Questions


Directions (1-50) What approximate value will come in place of the question mark (?) in the following question?
(Note: You are not expected to calculate the exact value.)
1. (15.252 ÷ 2.98) × 4.04 = ? × 14.69 b) 65
a) 20 c) 26
b) 80 d) 100
c) 150 e) 150
d) 320 3. 43.84% of 499.68 + 1499.87 = ? × 15.78
e) 240 a) 350
2. (449.89 ÷ 25.15) + 111.78 = ? × 4.99 b) 108
a) 5 c) 200

Page 556 of 722

Subscribe the Xpress Video Course & Mock Test Package for Bank & Insurance Exams
If there are any suggestions/ errors in our PDFs Feel Free to contact us via this email: admin@exampundit.in
IBPS RRB Clerk Prelims – Ultra Practice Bundle PDF
d) 180 e) √2
e) 240
8.
4. (224.98 + 325.29) ÷ 24.94 = ? × 1.87
a) 1
a) 158
b) 12
b) 94
c) 3
c) 72
d) 0
d) 46
e) 2
e) 11
9.
5.
a) 21
a) 13
b) 17
b) 7
c) 26
c) 11
d) 32
d) 10
e) 27
e) 21
10. 24.98% of 239.89 + 29.83% of 299.98 = ?
6. a) 120
a) 16 b) 80
b) 36 c) 170
c) 49 d) 180
d) 25 e) 150
e) 64 11. ?2 × 8.95 = 199.89 × 17.93
a) 16
b) 20
7.
c) 10
a) 4
d) 12
b) 2
e) 15
c) 8
12. 7.98 × 3.94 + 2.98 × 35.98 ÷ 5.96 = ?
d) 6
a) 40
Page 557 of 722

Subscribe the Xpress Video Course & Mock Test Package for Bank & Insurance Exams
If there are any suggestions/ errors in our PDFs Feel Free to contact us via this email: admin@exampundit.in
IBPS RRB Clerk Prelims – Ultra Practice Bundle PDF
b) 36 17. 102.91% of 640.02 – 3/7 % of 6300.102 + 12.001 =
c) 50 (?)
d) 46 a) 664
e) 56 b) 964
13. 29.98% of 249.98 ÷ 24.98 ÷ 2.98 = ? c) 864
a) 3 d) 644
b) 5 e) 744
c) 9 18. 1215.034 ÷ 26.98 ÷ √82 = ?
d) 11 a) 50
e) 1 b) 2
14. 19.89% of 149.98 + ∛1727.98 – 5.952 = ? c) 5
a) 12 d) 120
b) 6 e) 500
c) 16 19. 24.95% of 5690 + 15.09 × 1200 ÷ 3.96 = ?
d) 22 a) 5922.5
e) 2 b) 5912.5
15. (109.95 × 2.97) ÷ (3.14 × 4.95) = ? c) 5932.5
a) 12 d) 5902.5
b) 26 e) 5942.5
c) 36 20. 46.786 × 0.986 + 64.372 × 0.986 – 7.03 × 5.985 = ?
d) 22 a) 75
e) 48 b) 69
16. 13.005 × 7.02 + (84.99)2 × 10.99 = (?) c) 86
a) 79966 d) 102
b) 79866 e) 72
c) 79666 21. 11.95 × 17.98 ÷ 5.96 + 24.21 = ?
d) 79566 a) 40
e) 79366 b) 80
Page 558 of 722

Subscribe the Xpress Video Course & Mock Test Package for Bank & Insurance Exams
If there are any suggestions/ errors in our PDFs Feel Free to contact us via this email: admin@exampundit.in
IBPS RRB Clerk Prelims – Ultra Practice Bundle PDF
c) 70 a) 22
d) 60 b) 28
e) 50 c) 18
22. 27.98 - ? + 17.99 + (9.09)2 = 109.98 d) 12
a) 11 e) 16
b) 13 27. 4.962 × 6.932 = (?)2 + 135.98
c) 8 a) 23
d) 17 b) 31
e) 21 c) 33
23. 59.98% of 499.94 + 44.95% of 799.98 = ? d) 29
a) 660 e) 34
b) 620 28. ∛511.98 × √35.96 – ? × 15.98 = 31.98
c) 520 a) 3
d) 560 b) 6
e) 580 c) 9
24. (2/5 of 224.98) × 11.97 ÷ 4.98 = ? d) 1
a) 196 e) 11
b) 216 29. 53.92 × 35.96 ÷ 2.923 + 7.94 = ?
c) 208 a) 75
d) 202 b) 80
e) 192 c) 70
25. (√35.94 + 2.93) = 649.97 + ?
3
d) 72
a) 79 e) 88
b) 71 30. √1023.99 + √483.94 + 399.97 × 49.95% = ?
c) 85 a) 216
d) 91 b) 228
e) 89 c) 234
26. (23.89 ÷ 6.06 × 31.98) ÷ 7.94 = ? d) 246
Page 559 of 722

Subscribe the Xpress Video Course & Mock Test Package for Bank & Insurance Exams
If there are any suggestions/ errors in our PDFs Feel Free to contact us via this email: admin@exampundit.in
IBPS RRB Clerk Prelims – Ultra Practice Bundle PDF
e) 254 c) √13
31. 29.98% of (24.98 × 11.89) = ? + 39.99 d) 121
a) 45 e) 100
b) 50 36. 39.98% of 119.67 + (2.25 × 24.89) = ? – 39.88
c) 20 a) 300
d) 40 b) 180
e) 35 c) 140
32. 34.98% of 599.99 + 44.95% of 399.99 = ? d) 100
a) 260 e) 240
b) 410 37. (12.69 × 15.15) ÷ 4.95 + 121.21 = ?
c) 370 a) 160
d) 390 b) 120
e) 330 c) 80
33. √440.91 – √360.99 + √624.94 = ?3 d) 200
a) 2 e) 240
b) 4 38. (124.35 + 235.78) ÷ 17.89 = ? × 4.23
c) 7 a) 180
d) 5 b) 5
e) 3 c) 60
34. {64.98 – (55.96 – 47.98 ÷ 7.9 × 5.98)} = ? d) 240
a) 45 e) 120
b) 35 39. ?2 – 23.68% of 499.84 = √9999 + 68.76
c) 40 a) 32
d) 30 b) 17
e) 25 c) 2
35. 62.91 ÷ (2.93)2 × 3.98 – 16.97 = √? d) 52
a) √11 e) 74
b) 169 40. (14.782 + 364.89) ÷ 5 = ? – 122.22
Page 560 of 722

Subscribe the Xpress Video Course & Mock Test Package for Bank & Insurance Exams
If there are any suggestions/ errors in our PDFs Feel Free to contact us via this email: admin@exampundit.in
IBPS RRB Clerk Prelims – Ultra Practice Bundle PDF
a) 240 e) 14
b) 180 45. 14.94 × 77.95 ÷ 12.95 – 89.99 = ?
c) 320 a) 10
d) 90 b) -10
e) 280 c) 5
41. 17.89% of 249.87 + 194.95 = ? × 11.78 d) 0
a) 90 e) -5
b) 72 46. [(2.982 + 5.952) × 6.98)] ÷ 14.99 = ?
c) 2 a) 11
d) 45 b) 21
e) 20 c) 33
42. (419.78 ÷ 34.67) × 4.99 = ? – 139.98 d) 23
a) 450 e) 14
b) 320 47. 429.99 - 19.98 × 5.98 + 3.973 - 12.98 = ?2
c) 240 a) 22
d) 200 b) 26
e) 120 c) 32
43. 24.99% of 799.99 + 44.99% of 399.99 = ? d) 17
a) 280 e) 19
b) 320 48. 487.99 – 319.99 = 317.98 – 52 × ?
c) 420 a) 10
d) 380 b) 15
e) 480 c) 8
44. (23.98 + 35.96 × 3.96) ÷ 11.98 = ? d) 3
a) 22 e) 6
b) 18 49. 79.99% of 599.99 ÷ 11.92 – 39.99 + √483.94 = ?
c) 8 a) 22
d) 26 b) 32
Page 561 of 722

Subscribe the Xpress Video Course & Mock Test Package for Bank & Insurance Exams
If there are any suggestions/ errors in our PDFs Feel Free to contact us via this email: admin@exampundit.in
IBPS RRB Clerk Prelims – Ultra Practice Bundle PDF
c) 26 b) 6
d) 36 c) 2
e) 42 d) 8
50. 563.94 ÷ 5.96 + 25.99 = 239.99 ÷ ? e) 12
a) 4

1). Approximation Questions - Solution with Explanation

1. Answer: A) 550 ÷ 25 ~ ? × 2
(15.252 ÷ 2.98) × 4.04 = ? × 14.69 ? ~ 22 ÷ 2
(152 ÷ 3) × 4 ~ ? × 15 ? ~ 11
? ~ (225 ÷ 3) × (4/15) 5. Answer: C)
? ~ 20
2. Answer: C)
(449.89 ÷ 25.15) + 111.78 = ? × 4.99
(450 ÷ 25) + 112 ~ ? × 5 6. Answer: B)
? ~ 130 ÷ 5
? ~ 26
3. Answer: B)
43.84% of 499.68 + 1499.87 = ? × 15.78
44% of 500 + 1500 ~ ? × 16 7. Answer: E)
? ~ (0.44 × 500 + 1500) ÷ 16
? ~ 107.5
? ~ 108
4. Answer: E) 8. Answer: A)
(224.98 + 325.29) ÷ 24.94 = ? × 1.87
(225 + 325) ÷ 25 ~ ? × 2

Page 562 of 722

Subscribe the Xpress Video Course & Mock Test Package for Bank & Insurance Exams
If there are any suggestions/ errors in our PDFs Feel Free to contact us via this email: admin@exampundit.in
IBPS RRB Clerk Prelims – Ultra Practice Bundle PDF
13. Answer: E)
29.98% of 249.98 ÷ 24.98 ÷ 2.98 = ?
⇒ 30% of 250 ÷ 25 ÷ 3 = ?
⇒ 75 ÷ 25 ÷ 3 = ?
9. Answer: B)
⇒3÷3=?
⇒?=1
14. Answer: B)

10. Answer: E) 19.89% of 149.98 + ∛1727.98 – 5.952 = ?

24.98% of 239.89 + 29.83% of 299.98 = ? ⇒ 20% of 150 + ∛1728 – 62 = ?

⇒ 25% of 240 + 30% of 300 = ? ⇒ 30 + 12 – 36 = ?

⇒ 240 × (25/100) + 300 × (30/100) = ? ⇒ 42 – 36 = ?

⇒ 60 + 90 = ? ⇒6=?

⇒ 150 = ? 15. Answer: D)

11. Answer: B) (109.95 × 2.97) ÷ (3.14 × 4.95) = ?

?2 × 8.95 = 199.89 × 17.93 ⇒ (110 × 3) ÷ (3 × 5) = ?

⇒ ?2 × 9 = 200 × 18 ⇒ 330 ÷ 15 = ?

⇒ ?2 × 9 = 200 × 18 ⇒ 22 = ?

⇒ ?2 × 9 = 3600 16. Answer: D)

⇒ ?2 = 3600/9 ⇒ 13.005 × 7.02 + (84.99)2 × 10.99 = (?)

⇒ ?2 = 400 Approximating the value to the nearest integer:

⇒ ? = 20 ⇒ 13 × 7 + 852 × 11 = (?)

12. Answer: C) ⇒ 91 + 79475 = (?)

7.98 × 3.94 + 2.98 × 35.98 ÷ 5.96 = ? (?) = 79566

⇒ 8 × 4 + 3 × 36 ÷ 6 = ? 17. Answer: D)

⇒8×4+3×6=? ⇒ 102.91% of 640.02 – 3/7 % of 6300.102 + 12.001 = (?)

⇒ 32 + 18 = ? Approximating the value to the nearest integer:

⇒ ? = 50 ⇒ 103% of 640 – 3/7% of 6300 + 12 = (?)

Page 563 of 722

Subscribe the Xpress Video Course & Mock Test Package for Bank & Insurance Exams
If there are any suggestions/ errors in our PDFs Feel Free to contact us via this email: admin@exampundit.in
IBPS RRB Clerk Prelims – Ultra Practice Bundle PDF
(?) ≈ 659 – 27 + 12 = 644 22. Answer: D)
18. Answer: C) 27.98 - ? + 17.99 + (9.09)2 = 109.98
⇒ 1215.034 ÷ 26.98 ÷ √82 = ? ⇒ 28 - ? + 18 + (9)2 = 110
Approximating the values to the nearest integer: ⇒ 28 - ? + 18 + 81 = 110
⇒ 1215 ÷ 27 ÷ √81 = ? ⇒ 28 + 18 + 81 - 110 = ?
⇒ 1215 ÷ 27 ÷ 9 = ? ⇒ 127 - 110 = ?
⇒ 45/9 = ? ⇒ ? = 17
⇒?=5 23. Answer: A)
19. Answer: A) 59.98% of 499.94 + 44.95% of 799.98 = ?
24.95% of 5690 + 15.09 × 1200 ÷ 3.96 = ? ⇒ 60% of 500 + 45% of 800 = ?
Taking their approx. values ⇒ 300 + 360 = ?
⇒ 25% of 5690 + 15 × 1200 ÷ 4 = ? ∴ ? = 660
⇒ 1422.5 + 15 × 300 = ? 24. Answer: B)
⇒ ? = 1422.5 + 4500 (2/5 of 224.98) × 11.97 ÷ 4.98 = ?
∴ ? = 5922.5 ⇒ (2/5 of 225) × 12 ÷ 5 = ?
20. Answer: B)) ⇒ 90 × 12 ÷ 5 = ?
46.786 × 0.986 + 64.372 × 0.986 – 7.03 × 5.985 = ? ⇒ 216 = ?
⇒ 47 × 1 + 64 × 1 – 7 × 6 = ? 25. Answer: A)
⇒ 47 + 64 – 42 = ? (√35.94 + 2.93)3 = 649.97 + ?
⇒ 111 – 42 = ? ⇒ (√36 + 3)3 = 650 + ?
⇒ 69 = ? ⇒ (6 + 3)3 = 650 + ?
21. Answer: D) ⇒ (9)3 = 650 + ?
11.95 × 17.98 ÷ 5.96 + 24.21 = ? ⇒ 729 - 650 = ?
⇒ 12 × 18 ÷ 6 + 24 = ? ∴ ? = 79
⇒ 12 × 3 + 24 = ? 26. Answer: E)
⇒ 36 + 24 = ? (23.89 ÷ 6.06 × 31.98) ÷ 7.94 = ?
∴ ? = 60 ⇒ (24 ÷ 6 × 32) ÷ 8 = ?

Page 564 of 722

Subscribe the Xpress Video Course & Mock Test Package for Bank & Insurance Exams
If there are any suggestions/ errors in our PDFs Feel Free to contact us via this email: admin@exampundit.in
IBPS RRB Clerk Prelims – Ultra Practice Bundle PDF
⇒ (4 × 32) ÷ 8 = ? ⇒ √1023.99 + √483.94 + 399.97 × 49.95% = ?
⇒ 128 ÷ 8 = ? ⇒ √1024 + √484 + 400 × 50% = ?
∴ ? = 16 ⇒ 32 + 22 + 200 = ?
27. Answer: C) ∴ ? = 254
4.962 × 6.932 = (?)2 + 135.98 31. Answer: B)
⇒ 52 × 72 = (?)2 + 136 29.98% of (24.98 × 11.89) = ? + 39.99
⇒ 25 × 49 = (?)2 + 136 ⇒ 30% of (25 × 12) = ? + 40
⇒ 1225 = (?)2 + 136 ⇒ 30% of 300 = ? + 40
⇒ 1225 - 136 = (?)2 ⇒ 90 = ? + 40
⇒ 1225 - 136 = (?)2 ∴ ? = 50
⇒ 1089 = (?)2 32. Answer: D)
∴ ? = 33 34.98% of 599.99 + 44.95% of 399.99 = ?
28. Answer: D) ⇒ 35% of 600 + 45% of 400 = ?
∛511.98 × √35.96 – ? × 15.98 = 31.98 ⇒ 210 + 180 = ?
⇒ ∛512 × √36 – ? × 16 = 32 ∴ ? = 390
⇒ 8 × 6 – ? × 16 = 32 33. Answer: E)
⇒ 48 – ? × 16 = 32 √440.91 – √360.99 + √624.94 = ?3
⇒ 48 – 32 = ? × 16 ⇒ √441 – √361 + √625 = ?3
⇒ 16 = ? × 16 ⇒ 21 – 19 + 25 = ?3
∴?=1 ⇒ 46 – 19 = ?3
29. Answer: B) ⇒ 27 = ?3
53.92 × 35.96 ÷ 2.923 + 7.94 = ? ∴?=3
⇒ 54 × 36 ÷ 27 + 8 = ? 34. Answer: A)
⇒ 1944 ÷ 27 + 8 = ? {64.98 – (55.96 – 47.98 ÷ 7.9 × 5.98)} = ?
⇒ 72 + 8 = ? ⇒ {65 – (56 – 48 ÷ 8 × 6)} = ?
∴ ? = 80 ⇒ {65 – (56 – 6 × 6)} = ?
30. Answer: E) ⇒ {65 – (56 – 36)} = ?

Page 565 of 722

Subscribe the Xpress Video Course & Mock Test Package for Bank & Insurance Exams
If there are any suggestions/ errors in our PDFs Feel Free to contact us via this email: admin@exampundit.in
IBPS RRB Clerk Prelims – Ultra Practice Bundle PDF
⇒ {65 – 20} = ? ?2 – 24% of 500 ~ √10000 + 69
∴ ? = 45 ?2 ~ 100 + 69 + 120
35. Answer: D) ?2 ~ 289
62.91 ÷ (2.93)2 × 3.98 – 16.97 = √? ? ~ 17
⇒ 63 ÷ (3)2 × 4 – 17 = √? 40. Answer: A)
⇒ 63 ÷ 9 × 4 – 17 = √? (14.782 + 364.89) ÷ 5 = ? – 122.22
⇒ 7 × 4 – 17 = √? (152 + 365) ÷ 5 ~ ? – 122
⇒ 28 – 17 = √? ? ~ (118 + 122)
⇒ 11 = √? ? ~ 240
∴ ? = 121 41. Answer: E)

36. Answer: C) 17.89% of 249.87 + 194.95 = ? × 11.78


39.98% of 119.67 + (2.25 × 24.89) = ? – 39.88 18% of 250 + 195 ~ ? × 12
40% of 120 + (2 × 25) ~ ? – 40 ? ~ 240 ÷ 12
0.40 × 120 + 50 ~ ? – 40 ? ~ 20
? ~ 48 + 50 + 40 42. Answer: D)
? ~ 138 ~ 140 (419.78 ÷ 34.67) × 4.99 = ? – 139.98

37. Answer: A) (420 ÷ 35) × 5 ~ ? – 140

(12.69 × 15.15) ÷ 4.95 + 121.21 = ? 12 × 5 + 140 ~ ?


(13 × 15) ÷ 5 + 121 ~ ? ? ~ 200

? ~ 39 + 121 43. Answer: D)


? ~ 160 24.99% of 799.99 + 44.99% of 399.99 = ?

38. Answer: B) ⇒ 25% of 800 + 45% of 400 = ?


(124.35 + 235.78) ÷ 17.89 = ? × 4.23 ⇒ 200 + 180 = ?
(124 + 236) ÷ 18 ~ ? × 4 ∴ ? = 380
(360 ÷ 18) ÷ 4 ~ ? 44. Answer: E)
?~5 (23.98 + 35.96 × 3.96) ÷ 11.98 = ?
39. Answer: B) ⇒ (24 + 36 × 4) ÷ 12 = ?
?2 – 23.68% of 499.84 = √9999 + 68.76 ⇒ (24 + 144) ÷ 12 = ?
Page 566 of 722

Subscribe the Xpress Video Course & Mock Test Package for Bank & Insurance Exams
If there are any suggestions/ errors in our PDFs Feel Free to contact us via this email: admin@exampundit.in
IBPS RRB Clerk Prelims – Ultra Practice Bundle PDF
⇒ 168 ÷ 12 = ? ⇒ 361 = ?2
∴ ? = 14 ∴ ? = 19
45. Answer: D) 48. Answer: E)
14.94 × 77.95 ÷ 12.95 – 89.99 = ? 487.99 – 319.99 = 317.98 – 52 × ?
⇒ 15 × 78 ÷ 13 – 90 = ? ⇒ 488 – 320 = 318 – 25 × ?
⇒ 1170 ÷ 13 – 90 = ? ⇒ 168 = 318 – 25 × ?
⇒ 1170 ÷ 13 – 90 = ? ⇒ 25 × ? = 150
⇒ 90 – 90 = ? ⇒ ? = 150/25
∴?=0 ⇒?≈6
46. Answer: B) 49. Answer: A)
[(2.982 + 5.952) × 6.98)] ÷ 14.99 = ? 79.99% of 599.99 ÷ 11.92 – 39.99 + √483.94 = ?
⇒ [(32 + 62) × 7)] ÷ 15 = ? ⇒ 80% of 600 ÷ 12 – 40 + √484 = ?
⇒ [(9 + 36) × 7)] ÷ 15 = ? ⇒ 480 ÷ 12 – 40 + 22 = ?
⇒ [ 45 × 7)] ÷ 15 = ? ⇒ 40 – 40 + 22 = ?
⇒ 315 ÷ 15 = ? ∴ ? = 22
∴ ? = 21 50. Answer: C)
47. Answer: E) 563.94 ÷ 5.96 + 25.99 = 239.99 ÷ ?
429.99 - 19.98 × 5.98 + 3.973 - 12.98 = ?2 ⇒ 564 ÷ 6 + 26 = 240 ÷ ?
⇒ 430 - 20 × 6 + 43 - 13 = ?2 ⇒ 94 + 26 = 240 ÷ ?
⇒ 430 - 120 + 64 - 13 = ?2 ⇒ 120 = 240 ÷ ?
⇒ 494 -120 - 13 = ?2 ⇒ ? = 240/120
⇒ 494 - 133 = ?2 ⇒?≈2

2). Simplification Questions


Directions (1-50): What value should come in the place a) 108
of question mark (?) in the following questions? b) 107
1. 18/34 + 78/34 + 480/5 + 74/34 =? c) 121
Page 567 of 722

Subscribe the Xpress Video Course & Mock Test Package for Bank & Insurance Exams
If there are any suggestions/ errors in our PDFs Feel Free to contact us via this email: admin@exampundit.in
IBPS RRB Clerk Prelims – Ultra Practice Bundle PDF
d) 101 b) 47500
e) 116 c) 45900
2. 6/15 of 264/792 of? = 158 d) 76100
a) 1780 e) 63400
b) 1560 7. 4/5 of 48% of 700 – 15.4 =?
c) 1540 a) None
d) 1120 b) 152.4
e) 1185 c) 453.4
3. 3√10648 * 3√ 5832 * 3√13824 =? d) 253.4
a) 9504 e) 615.3
b) 1004 8. 125 – 43 + 522 =?
c) 9804 a) 2694
d) 8504 b) 2465
e) 7504 c) 2765
4. 50% of 25% of 4/5 of? = 640 d) 1786
a) 6540 e) 1456
b) 6600 4
9. 15% of 28 7 % of ? = 240
c) 6400
a) 5800
d) 5220
b) 6300
e) 4280
c) 6100
5. 440 + 4444 – 44444 + 44 – 40 + 44044=?
d) 5600
a) 4848
e) 5300
b) 4488
10. 1674 ÷ 27 × 9 + 18 = ?2
c) 5488
a) 26
d) 3418
b) 23
e) 5788
c) 24
6. (158+97) x (11 + 1.2 + 2.8) x (12) = ?
d) 35
a) 56100
Page 568 of 722

Subscribe the Xpress Video Course & Mock Test Package for Bank & Insurance Exams
If there are any suggestions/ errors in our PDFs Feel Free to contact us via this email: admin@exampundit.in
IBPS RRB Clerk Prelims – Ultra Practice Bundle PDF
e) 18 a) 81
11. 320% of 700 – 70% of 320 – 3.2% 0f 7000 = ? b) 9
a) 1792 c) 3
b) 224 d) 27
c) -1792 e) 729
d) -224
16.
e) 1972
a) 4060
12. (7294 – 3241 + 716) – (3267 + 2425 – 961) = ?
b) 4660
a) 35
c) 6040
b) 36
d) 4260
c) 37
e) 4160
d) 38
e) 43 17.
13. 32/35 ÷ 1/5 × 7/8 ÷ 2/35 = ? a) 1/50
a) 60 b) 1/25
b) 80 c) 1/30
c) 50 d) 1/40
d) 90 e) 1/60
e) 70
18.
14. a) 4
a) 400 b) 1
b) 441 c) 3
c) 361 d) 2
d) 484 e) 5
e) 289
19.
15. a) 0.8
b) 9
Page 569 of 722

Subscribe the Xpress Video Course & Mock Test Package for Bank & Insurance Exams
If there are any suggestions/ errors in our PDFs Feel Free to contact us via this email: admin@exampundit.in
IBPS RRB Clerk Prelims – Ultra Practice Bundle PDF
c) 0.9 d) 13/9
d) 1 e) 12/7
e) 8 24. 45 × 3/5 + ? ÷ 2 = 105
a) 172
20.
b) 127
a) 27
c) 156
b) 25
d) 165
c) 22
e) 128
d) 17
e) 12 25.
a) 6456
21.
b) 7456
a) 104
c) 5456
b) 96
d) 4440
c) 198
e) 5420
d) 98
26. 28% of 500 – 120 = ? ÷ 3
e) 100
a) 32
b) 60
22.
c) 54
a) 133
d) 48
b) 152
e) 64
c) 263
27. (122 × 5) ÷ 16 + 155 = ?
d) 126
a) 400
e) 283
b) 350
c) 540
23.
d) 200
a) 13/4
e) 250
b) 15/8
28. (3/8) × {(512 – 128)} = ? × 12
c) 3/4
Page 570 of 722

Subscribe the Xpress Video Course & Mock Test Package for Bank & Insurance Exams
If there are any suggestions/ errors in our PDFs Feel Free to contact us via this email: admin@exampundit.in
IBPS RRB Clerk Prelims – Ultra Practice Bundle PDF
a) 10 e) 300
b) 18 33. 24% of 500 + 302 = ? × 51
c) 12 a) 35
d) 14 b) 20
e) 16 c) 15
29. (183 × 5) ÷ 182= ? – (27 × 5) d) 25
a) 295 e) 30
b) 320 34. (3/8) × 560 + (2/5) × 350 – (3/4) × 200 = ?
c) 270 a) 100
d) 225 b) 180
e) 215 c) 150
30. {(450 ÷ 5) + 110} ÷ 5 = ? ÷ 3 d) 240
a) 100 e) 200
b) 250 35. 50% of 1200 – 320 = ? + 12% of 1500
c) 120 a) 100
d) 160 b) 120
e) 300 c) 150
31. (360 – 152) × 6 – 210 = ? × 12 d) 80
a) 100 e) 140
b) 50 5
36. √484 + ? = 4 6 of 102
c) 80
a) 453
d) 20
b) 463
e) 40
c) 471
32. (45 × 8) – 16% of 2000 = ? – 260
d) 477
a) 100
e) 501
b) 250
37. 27 x 27 - 9 = ? x 40
c) 400
a) 30
d) 450
Page 571 of 722

Subscribe the Xpress Video Course & Mock Test Package for Bank & Insurance Exams
If there are any suggestions/ errors in our PDFs Feel Free to contact us via this email: admin@exampundit.in
IBPS RRB Clerk Prelims – Ultra Practice Bundle PDF
b) 0 42. 45% of 52% of 600=?
c) -2 a) 144.20
d) 18 b) 145.10
e) 1 c) 111.10
38. 192 - 302 + 212 = ? d) 140.40
a) -108 e) 101.20
b) -68 1 1 1 1
43. 3 + 1 + 2 + 1 = ?
3 5 4 7
c) -98 389
a) 7 420
d) -168
289
e) -88 b) 7 420
298
39. 8 x 46 - 30 x 22 = ? c) 7 420
a) -314 189
d) 7 320
b) -277
489
c) -288 e) 7 520
1 7 5 1
d) -294 44. 1 3 ÷ 1 9 x 1 7 ÷ 7 2 = ?
e) -292 a) 6/35
40. 80% of 370 + 10% of 510 = ? b) 8/105
a) 347 c) 21/205
b) 447 d) 31/305
c) 267 e) 8/205
d) 377 45. 75157 - ? – 331 – 30 +530 = 69874
e) 367 a) 5452
2
41. [(185) ÷ 25 x 15] =? b) 6252
a) 25035 c) 4425
b) 25305 d) 4525
c) 20535 e) 4441
d) 24035
e) 26035 46.
Page 572 of 722

Subscribe the Xpress Video Course & Mock Test Package for Bank & Insurance Exams
If there are any suggestions/ errors in our PDFs Feel Free to contact us via this email: admin@exampundit.in
IBPS RRB Clerk Prelims – Ultra Practice Bundle PDF
a) 8.2 c) 17
b) 6.4 d) 14
c) 2.6 e) 19
d) 3.8 49. 616 + 472 – 811 + 317 = ? + 576
e) 2.8 a) 28
b) 16
47.
c) 24
a) 17
d) 18
b) 14.9
e) 14
c) 13.9
50. 12.5 × 80 + 37.5 × 16 – 6.25 × 112 = ?
d) 16
a) 1000
e) 14.7
b) 900
c) 2300
48.
d) 600
a) 13
e) 1300
b) 16

2). Simplification Questions - Solutions with Explanation

1. Answer: D) 3. Answer: A)
(18+78+74)/34+480/5 =? 22 x 18 x 24 = 9504
5+96 =? 4. Answer: C)
101 =? 50% of 25% of 4/5 of ? = 640
2. Answer: E) 640*2*4*5/(4) = ?
6/15 of 264/792 of ? = 158 6400 =?
? =158*15*792 / (6*264) 5. Answer: B)
? = 1185 440+4444+44+44044 – 44444-40 =?

Page 573 of 722

Subscribe the Xpress Video Course & Mock Test Package for Bank & Insurance Exams
If there are any suggestions/ errors in our PDFs Feel Free to contact us via this email: admin@exampundit.in
IBPS RRB Clerk Prelims – Ultra Practice Bundle PDF
48972 – 44484 =?
4488 = ?
6. Answer: C) 13. Answer: E)
255 x 15 x 12 =?
? = 45900
7. Answer: D)
14. Answer: B)
0.8 x 48 x 7 – 15.4 = ?
268.8 – 15.4 = ?
? = 253.4
8. Answer: C)
125 – 43 + 522 =? 15. Answer: C)
125-64+2704 =?
2765 =?
9. Answer: D)
16. Answer: A)

10. Answer: C)
17. Answer: C)

11. Answer: A)

18. Answer: D)

12. Answer: D)

Page 574 of 722

Subscribe the Xpress Video Course & Mock Test Package for Bank & Insurance Exams
If there are any suggestions/ errors in our PDFs Feel Free to contact us via this email: admin@exampundit.in
IBPS RRB Clerk Prelims – Ultra Practice Bundle PDF
19. Answer: B) ?/2 = 78
? = 156
25. Answer: A)



⇒ 6456
⇒9
26. Answer: B)
20. Answer: C)
28% of 500 – 120 = ? ÷ 3
0.28 × 500 – 120 = ? ÷ 3
? = 20 × 3 = 60

27. Answer: D)
⇒ 22 = ?
(122 × 5) ÷ 16 + 155 = ?
21. Answer: D)
(144 × 5) ÷ 16 + 155 = ?
? = 200
28. Answer: C)
⇒ 98 (3/8) × {(512 – 128)} = ? × 12
22. Answer: A) (3/8) × 384 = ? × 12
? = 144 ÷ 12
? = 12
29. Answer: D)
(183 × 5) ÷ 182= ? – (27 × 5)
? = (18 × 5) + 135
? = 225
? = 180 – 47 = 133
30. Answer: C)
23. Answer: B)
{(450 ÷ 5) + 110} ÷ 5 = ? ÷ 3
5/8 + 1/2 + 3/4 = (5 + 4 + 6)/8 = 15/8
(90 + 110) ÷ 5 = ? ÷ 3
24. Answer: C)
? = 40 × 3
45 × 3/5 + ? ÷ 2 = 105
? = 120
27 + ?/2 = 105
Page 575 of 722

Subscribe the Xpress Video Course & Mock Test Package for Bank & Insurance Exams
If there are any suggestions/ errors in our PDFs Feel Free to contact us via this email: admin@exampundit.in
IBPS RRB Clerk Prelims – Ultra Practice Bundle PDF
31. Answer: B) 37. Answer: D)
(360 – 152) × 6 – 210 = ? × 12 ? = (27 x 27 - 9)/40
(360 – 225) × 6 – 210 = ? × 12 = (729 - 9)/40
? = 600 ÷ 12 = 720/40 = 18
? = 50 38. Answer: C)
32. Answer: E) ? = 192 - 302 + 212
(45 × 8) – 16% of 2000 = ? – 260 = 361 - 900 + 441 = -98
360 – 0.16 × 2000 = ? – 260 39. Answer: E)
? = 360 – 320 + 260 ? = 8 x 46 - 30 x 22
? = 300 = 368 - 660
33. Answer: B) = -292
24% of 500 + 302= ? × 51 40. Answer: A)
0.24 × 500 + 900 = ? × 51 ? = (80 x 370/100) + (10 x 510/100)
? = (120 + 900) ÷ 51 = 20 = (29600/100) + (5100/100)
34. Answer: E) = 296 + 51 = 347
(3/8) × 560 + (2/5) × 350 – (3/4) × 200 = ? 41. Answer: C)
(3 × 70) + (2 × 70) – (3 × 50) = ? [1369 x 15] =?
? = 210 + 140 – 150 20535 =?
? = 200 42. Answer: D)
35. Answer: A) 45% of 52% of 600=?
50% of 1200 – 320 = ? + 12% of 1500 45/100*52/100*600 =?
0.50 × 1200 – 320 = ? + 0.12 × 1500 140.40 =?
? = 600 – 320 – 180 43. Answer: A)
? = 100 3+1+2+1(1/3+1/5+1/4+1/7) =?
36. Answer: C) 7(389/420) = ?
? = 102 x (29/6) - √484 44. Answer: A)
= 17 x 29 - 22 4/3 x 9/16 x 12/7 x 2/15=?
= 493 - 22 = 471 ? = 6/35
Page 576 of 722

Subscribe the Xpress Video Course & Mock Test Package for Bank & Insurance Exams
If there are any suggestions/ errors in our PDFs Feel Free to contact us via this email: admin@exampundit.in
IBPS RRB Clerk Prelims – Ultra Practice Bundle PDF
45. Answer: A)
75687 – 70235 =?
? = 5452
46. Answer: E) 49. Answer: D)
? = 1405 – 811 – 576
? = 18
50. Answer: B)
47. Answer: D)
?= 15.45 + 0.55
? = 16
48. Answer: D)

3). Missing Number series


Direction (1-50): What value should come in the place A.61
of (?) in the following number series? B.71
1. 10, 12, 15, 19, ?, 30 C.81
A.24 D.91
B.26 E.51
C.37 4. 5, 5, 10, ? , 320, 5120
D.28 A.50
E.25 B.60
2. 5, 12, 39, ?, 805 C.40
A.230 D.30
B.360 E.51
C.150 5. 10, 5, 14, ?, 18, -3
D.160 A.2
E.785 B.4
3. 11, 31, 21, 41, 31, ? C.1
Page 577 of 722

Subscribe the Xpress Video Course & Mock Test Package for Bank & Insurance Exams
If there are any suggestions/ errors in our PDFs Feel Free to contact us via this email: admin@exampundit.in
IBPS RRB Clerk Prelims – Ultra Practice Bundle PDF
D.8 B.210
E.- C.214
6. 15, 16, 24, 51, 115, ? D.220
A.186 E.228
B.198 11. 8, 9, 20, 63, 256, ?
C.214 A.1207
D.240 B.1287
E.245 C.1276
7. 12, 18, 36, 90, ?, 945 D.1285
A.280 E.1294
B.240 12. 4, 32, 77, 141, 226, ?
C.250 A.332
D.260 B.341
E.270 C.334
8. 4913, 2197, ?, 343, 125, 27 D.338
A.1197 E.330
B.1331 13. 7, 20, 37, 56, 79, ?
C.216 A.111
D.1728 B.108
E.512 C.118
9. 35, 46, 59, 76, 99, ? D.105
A.130 E.115
B.136 14. 26, 54, 181, 527, ?, 2596
C.142 A.1080
D.145 B.1120
E.148 C.1180
10. 19, 38, 67, 106, 155, ? D.1260
A.208 E.1220
Page 578 of 722

Subscribe the Xpress Video Course & Mock Test Package for Bank & Insurance Exams
If there are any suggestions/ errors in our PDFs Feel Free to contact us via this email: admin@exampundit.in
IBPS RRB Clerk Prelims – Ultra Practice Bundle PDF
15. 6, 30, 66, 114, 174, ? D.59
A.246 E.45
B.248 20. 10, 16, 28, ?, 78, 120
C.251 A.36
D.258 B.40
E.261 C.44
16. 18, 55, 167, 504, ?, 4553 D.48
A.1532 E.52
B.1516 21. 781, 556, 387, 266, 185, ?
C.1782 A.136
D.1592 B.98
E.2018 C.167
17. 960, 1023, 1088, 1155, 1224, ? D.197
A.1290 E.150
B.1285 22. 1716, 2183, 2728, 3357, 4076, ?
C.1295 A.4129
D.1225 B.4586
E.1320 C.4762
18. 32, 49, 75, 112, 162, ? D.4871
A.184 E.4891
B.217 23. 7, 45, 96, 148, 203, ?
C.227 A.243
D.235 B.245
E.248 C.251
19. 19, 19, 21, 27, 39, ? D.258
A.41 E.261
B.56 24. 5, 28, 47, 64, 77, ?
C.48 A.96
Page 579 of 722

Subscribe the Xpress Video Course & Mock Test Package for Bank & Insurance Exams
If there are any suggestions/ errors in our PDFs Feel Free to contact us via this email: admin@exampundit.in
IBPS RRB Clerk Prelims – Ultra Practice Bundle PDF
B.98 29. 1, 4, 20, 120,?, 6720
C.85 A.840
D.106 B.856
E.88 C.720
25. 6, 4, 5, 11, 39, ? D.732
A.160 E.680
B.193 30. 62, 122, 212, 338, 506, ?
C.189 A.722
D.183 B.738
E.171 C.714
26. 29, 15, 16, 34, 140, ? D.706
A.335 E.762
B.448 31. 20, 28, 37, 101, 126, ?
C.1128 A.325
D.347 B.342
E.455 C.337
27. 89, 88, 85, 78, 63, ? D.320
A.32 E.328
B.30 32. 23, 33, 63, 113, ?, 273
C.34 A.163
D.27 B.143
E.23 C.183
28. 190, 94, 46, 22, ?, 4 D.193
A.16 E.153
B.10 33. 1944, 968, 480, 236, 114, ?
C.9 A.53
D.13 B.60
E.8 C.65
Page 580 of 722

Subscribe the Xpress Video Course & Mock Test Package for Bank & Insurance Exams
If there are any suggestions/ errors in our PDFs Feel Free to contact us via this email: admin@exampundit.in
IBPS RRB Clerk Prelims – Ultra Practice Bundle PDF
D.70 B.486
E.49 C.502
34. 35, 49, 63, 77, 91, ? D.512
A.100 E.490
B.102 39. 678, 637, 594, 547, 494, ?
C.103 A.423
D.105 B.432
E.115 C.435
35. 88, 165, 220, 253, ? D.428
A.261 E.439
B.258 40. 24, 45, 80, 129, ?, 297
C.264 A.184
D.268 B.196
E.270 C.190
36. 85, 110, 94, 103, 99, ? D.206
A.20 E.212
B.60 41. 23, 138, 690, ?, 8280, 16560
C.80 A.1840
D.100 B.2240
E.110 C.2760
37. ?, 1235, 1252, 1295, 1384 D.3230
A.1235 E.3890
B.1230 42. 18, 19, 39, 118, 473, ?
C.1225 A.946
D.1220 B.1319
E.1233 C.2366
38. 12, 54, 124, 222, 348,? D.1893
A.498 E.2839
Page 581 of 722

Subscribe the Xpress Video Course & Mock Test Package for Bank & Insurance Exams
If there are any suggestions/ errors in our PDFs Feel Free to contact us via this email: admin@exampundit.in
IBPS RRB Clerk Prelims – Ultra Practice Bundle PDF
43. 13, 36, 65, 96, 133, ? 47. 1, 5/2, 5, ?, 13, 37/2, 25
A.156 A.21/2
B.168 B.19/2
C.148 C.17/2
D.174 D.23/2
E.181 E. None of these
44. 17, 21, ?, 46, 71, 107 48. 1290, 645, 430,? , 258, 215
A.15 A.317.5
B.30 B.322.5
C.25 C.215
D.20 D.332.5
E.35 E.324.25
45. 3, 15, 35, 63, ? 49. 1260, 210, 42, 10.5, 3.5,?
A.79 A.1.75
B.89 B.1.25
C.99 C.2.25
D.68 D.1.60
E.72 E.2.75
46. 1, 9, 21, ?, 63, 93 50. 28, 41, 56, 73, 92, ?
A.39 A.120
B.42 B.113
C.37 C.131
D.41 D.135
E. None of these E.130

3). Missing Number series - Solutions and Explanations


1. Answer: A 10 +2 = 12
Page 582 of 722

Subscribe the Xpress Video Course & Mock Test Package for Bank & Insurance Exams
If there are any suggestions/ errors in our PDFs Feel Free to contact us via this email: admin@exampundit.in
IBPS RRB Clerk Prelims – Ultra Practice Bundle PDF
12 + 3 = 15 6. Answer: D
15 + 4 = 19 15 + 13 = 16
19 +5 = 24 16 + 23 = 24
24 + 6 = 30 24 + 33 = 51
2. Answer: D 51 + 43 = 115
5*2 + 2 = 12 115 + 53 = 240
12*3 + 3 =39 7. Answer: E
39*4 + 4 = 160 12 * 1.5 = 18
160*5 + 5 = 805 18 * 2 = 36
3. Answer: E 36 * 2.5 = 90
11 + 20 = 31 90 * 3 = 270
31 – 10 = 21 270 * 3.5 = 945
21 + 20 =41 8. Answer: B
41 - 10 = 31 Cube of prime numbers
31+ 20 = 51 173 = 4913
4. Answer: C 133 = 2197
5*1=5 113 = 1331
5 * 2 = 10 73 = 343
10* 4 = 40 53 = 125
40* 8 = 320 33 = 27
320*16 = 5120 9. Answer: A
5. Answer: C 35 46 59 76 99 130
Alternate number’s series: 11 13 17 23 31
10 + 4 =14 2 4 6 8
14 + 4= 18 10. Answer: C
And 19 + 19 = 38
5 -4 = 1 38 + 29 = 67
1-4 = -3 67 + 39 = 106
Page 583 of 722

Subscribe the Xpress Video Course & Mock Test Package for Bank & Insurance Exams
If there are any suggestions/ errors in our PDFs Feel Free to contact us via this email: admin@exampundit.in
IBPS RRB Clerk Prelims – Ultra Practice Bundle PDF
106 + 49 = 155 30 + (12 * 3) = 66
155 + 59 = 214 66 + (12 * 4) = 114
11. Answer: D 114 + (12 * 5) = 174
8 *1 + 1 = 9 174 + (12 * 6) = 246
9 * 2 + 2 = 20 16. Answer: B
20 * 3 + 3 = 63 18 * 3 + 1 = 55
63 * 4 + 4 = 256 55 * 3 + 2 = 167
256 * 5 + 5 = 1285 167 * 3 + 3 = 504
12. Answer: C 504 * 3 + 4 = 1516
4 + (2 * 14) = 32 1516 * 3 + 5 = 4553
32 + (3 * 15) = 77 17. Answer: C
77 + (4 * 16) = 141 312 – 1 = 960
141 + (5 * 17) = 226 322 – 1 = 1023
226 + (6 * 18) = 334 332 – 1 = 1088
13. Answer: B 342 – 1 = 1155
7+ 13 =20 352 – 1 = 1224
20 + 17 = 37 362 – 1 = 1295
37 + 19 = 56 18. Answer: C
56 + 23 = 79 32 + 42 + 1 = 49
79 + 29 = 108 49 + 52 + 1 = 75
14. Answer: D 75 + 62 + 1 = 112
26 +33+ 1= 54 112 + 72 + 1 = 162
54 +53+ 2 = 181 162 + 82 + 1 = 227
181 +73 + 3 = 527 19. Answer: D
527 +93 + 4 = 1260 19 + 12 – 1 = 19
1260 +113 + 5 = 2596 19 + 22 – 2 = 21
15. Answer: A 21 + 32 – 3 = 27
6 + (12 * 2) = 30 27 + 42 – 4 = 39
Page 584 of 722

Subscribe the Xpress Video Course & Mock Test Package for Bank & Insurance Exams
If there are any suggestions/ errors in our PDFs Feel Free to contact us via this email: admin@exampundit.in
IBPS RRB Clerk Prelims – Ultra Practice Bundle PDF
39 + 52 – 5 = 59 The answer is, 88
20. Answer: D 25. Answer: C
10 + (2 * 3) = 16 The pattern is, *1–2, *2–3, *3–4, *4–5, *5–6
16 + (3 * 4) = 28 The answer is, 189
28 + (4 * 5) = 48 26. Answer: C
48 + (5 * 6) = 78 The pattern is, *0.5 + 0.5, *1 + 1, *2, +2, *4 + 4, *8 + 8
78 + (6 * 7) = 120 The answer is, 1128
21. Answer: A 27. Answer: A
781 – 152 = 556
556 – 132 = 387
387 – 112 = 266
The answer is, 32
266 – 92 = 185
28. Answer: B
185 – 72 = 136
190 – 2 = 188/2 = 94
22. Answer: E
94 – 2 = 92, 92/2 = 46
123 – 12 = 1716
46 – 2 = 44, 44/2 = 22
133 – 14 = 2183
22 – 2 = 20, 20/2 = 10
143 – 16 = 2728
29. Answer: A
153 – 18 = 3357
1*4=4
163 – 20 = 4076
4 * 5 = 20
173 – 22 = 4891
20 * 6 = 120
23. Answer: B
120 * 7 = 840
7 + (19 * 2) = 45
840 * 8 = 6720
45 + (17 * 3) = 96
30. Answer: A
96 + (13 * 4) = 148
43 – 2 = 62
148 + (11 * 5) = 203
53 – 3 = 122
203 + (7 * 6) = 245
63 – 4 = 212
24. Answer: E
73 – 5 = 338
The pattern is, +23, +19, +17, +13, +11
Page 585 of 722

Subscribe the Xpress Video Course & Mock Test Package for Bank & Insurance Exams
If there are any suggestions/ errors in our PDFs Feel Free to contact us via this email: admin@exampundit.in
IBPS RRB Clerk Prelims – Ultra Practice Bundle PDF
83 – 6 = 506 165 + 55 = 220
93 – 7 = 722 220 + 33 = 253
31. Answer: B 253 + 11 = 264
20 + 23 = 28 36. Answer: D
28 + 32 = 37 85 + 52 = 110
37 + 43 = 101 110 – 42 = 94
101 + 52 = 126 94 + 32 = 103
126 + 63 = 342 103 – 22 = 99
32. Answer: C 99 + 12 = 100
23 + 10 = 33 37. Answer: B
33 + 30 = 63
63 + 50 = 113
113 + 70 = 183
183 + 90 = 273
33. Answer: A
1944/2 – 4 = 968 38. Answer: C

968/2 – 4 = 480 12 + (14 * 3) = 54

480/2 – 4 = 236 54 + (14 * 5) = 124

236/2 – 4 = 114 124 + (14 * 7) = 222

114/2 – 4 = 53 222 + (14 * 9) = 348

34. Answer: D 348 + (14 * 11) = 502


39. Answer: C
678 – 41 = 637
637 – 43 = 594
594 – 47 = 547
547 – 53 = 494
35. Answer: C
494 – 59 = 435
88 + 77 = 165
40. Answer: D
Page 586 of 722

Subscribe the Xpress Video Course & Mock Test Package for Bank & Insurance Exams
If there are any suggestions/ errors in our PDFs Feel Free to contact us via this email: admin@exampundit.in
IBPS RRB Clerk Prelims – Ultra Practice Bundle PDF
24 + (3 * 7) = 45
45 + (5 * 7) = 80
80 + (7 * 7) = 129
129 + (11 * 7) = 206
206 + (13 * 7) = 297 45. Answer: C
41. Answer: C 22 – 1 = 3
23 * 6 = 138 42 – 1 = 15
138 * 5 = 690 62 – 1 = 35
690 * 4 = 2760 82 – 1 = 63
2760 * 3 = 8280 102 – 1 = 99
8280 * 2 = 16560 46. Answer: A
42. Answer: C
18 * 1 + 1 = 19
19 * 2 + 1 = 39
39 * 3 + 1 = 118
118 * 4 + 1 = 473 Here, 3 + 5 = 8 (sum of prime numbers)
473 * 5 + 1 = 2366 5 + 7 = 12
43. Answer: D 7 + 11 = 18
13 + 23 = 36 (addition of prime numbers from 23 and so 11 + 13 = 24
on) 13 + 17 = 30
36 + 29 = 65 Therefore, 21 + 18 = 39
65 + 31 = 96 47. Answer: C
96 + 37 = 133 1 + 3/2 = 5/2
133 + 41 = 174 5/2 + 5/2 = 5
44. Answer: B 5 + 7/2 = 17/2
17/2 + 9/2 = 13
13 + 11/2 = 37/2
37/2 + 13/2 = 25
Page 587 of 722

Subscribe the Xpress Video Course & Mock Test Package for Bank & Insurance Exams
If there are any suggestions/ errors in our PDFs Feel Free to contact us via this email: admin@exampundit.in
IBPS RRB Clerk Prelims – Ultra Practice Bundle PDF
48. Answer: B 10.5 ÷ 3 = 3.5
1290 x ½ = 645 3.5 ÷ 2 = 1.75
645 x 2/3 = 430 50. Answer: B
430 x ¾ = 322.5 9 * 3 + 1= 28
322.5 x 4/5 = 258 10 * 4 + 1 = 41
258 x 5/6 = 215 11 * 5+ 1 = 56
49. Answer: A 12* 6+ 1 = 73
1260 ÷ 6 = 210 13 * 7 + 1 = 92
210 ÷ 5 = 42 14 * 8+ 1 = 113
42 ÷ 4 = 10.5

4). Wrong Number series


Direction (1-50). Find out the wrong number in the E.5395
following number series. 3. 643, 768, 805, 1147, 1211, 1940
1. 2.5, 5, 25, 200, 2201, 30800 A.805
A.2.5 B.643
C.768
B.200 D.1211
C.2201 E.1940
D.25 4. 8, 5, 6, 11, 21, 53.5
E.5 A.53.5
2. 70, 68, 135, 395, 1575, 7869 B.11
A.1575 C.21
B.68 D.5
C.135 E.6
D.70 5. 339, 341, 345, 353, 369, 400

Page 588 of 722

Subscribe the Xpress Video Course & Mock Test Package for Bank & Insurance Exams
If there are any suggestions/ errors in our PDFs Feel Free to contact us via this email: admin@exampundit.in
IBPS RRB Clerk Prelims – Ultra Practice Bundle PDF
A.400 E.2
B.341 10. 0.5, 2, 4, 7.5, 9.5, 13
C.345 A.3
D.353 B.14
E.339 C.10.5
6. 5, 20, 50, 112, 230, 470 D.5
A.112 E.7.5
B.20 11. 513, 520, 530, 544, 561, 580
C.50 A.513
D.230 B.530
E.470 C.520
7. 20, 25, 21, 27, 24, 29 D.544
A.20 E.561
B.21 12. 305, 336, 373, 422, 494, 616
C.27 A.305
D.24 B.422
E.29 C.373
8. 3,4,10,17,33,58 D.494
A.58 E.616
B.4 13. 15, 14, 25, 75, 296, 1475
C.10 A.14
D.17 B.1475
E.33 C.25
9. 2, 22, 47, 77, 115 D.296
A.115 E.15
B.77 14. 4, 13, 41, 125, 389, 1183
C.47 A.13
D.22 B.389
Page 589 of 722

Subscribe the Xpress Video Course & Mock Test Package for Bank & Insurance Exams
If there are any suggestions/ errors in our PDFs Feel Free to contact us via this email: admin@exampundit.in
IBPS RRB Clerk Prelims – Ultra Practice Bundle PDF
C.41 A.164
D.1183 B.78
E.125 C.220
15. 105, 116, 131, 149, 167, 190 D.117
A.131 E.47
B.190 20. 13, 26, 78, 312, 1560, 9380
C.149 A.13
D.167 B.26
E.105 C.312
16. 32, 52, 92, 172, 332, 650 D.9380
A.332 E.1560
B.92 21. 8, 19, 35, 56, 82, 111
C.650 A.111
D.172 B.56
E.33 C.19
17. 256, 240, 218, 179, 130, 66 D.82
A.218 E.35
B.66 22. 67, 132, 183, 222, 251, 275
C.179 A.251
D.240 B.222
E.130 C.275
18. 32, 39, 56, 63, 80, 99 D.183
A.39 E.132
B.56 23. 19, 21, 33, 73, 188, 473
C.99 A.21
D.80 B.188
E.63 C.33
19. 24, 47, 78, 117, 164, 220 D.473
Page 590 of 722

Subscribe the Xpress Video Course & Mock Test Package for Bank & Insurance Exams
If there are any suggestions/ errors in our PDFs Feel Free to contact us via this email: admin@exampundit.in
IBPS RRB Clerk Prelims – Ultra Practice Bundle PDF
E.73 C.530
24. 13, 23, 41, 65, 101, 143 D.425
A.65 E.200
B.101 29. 13, 14, 29, 88, 353, 1768
C.23 A.88
D.143 B.353
E.41 C.14
25. 1.2, 12, 108, 972, 6048, 36288 D.29
A.6048 E.1768
B.12 30. 343, 279, 247, 231, 223, 214
C.108 A.214
D.972 B.231
E.36288 C.247
26. 96, 420, 131, 385, 162, 358 D.279
A.385 E.223
B.162 31. 0, 6, 22, 60, 120
C.420 A.6
D.358 B.22
E.131 C.60
27. 1, 6, 21, 46, 80, 126 D.120
A.46 E.0
B.126 32. 512, 24, 343, 25, 216
C.80 A.24
D.21 B.343
E.6 C.25
28. 145, 200, 265, 340, 425, 530 D.216
A.340 E.512
B.265 33. 12, 20, 30, 44, 56
Page 591 of 722

Subscribe the Xpress Video Course & Mock Test Package for Bank & Insurance Exams
If there are any suggestions/ errors in our PDFs Feel Free to contact us via this email: admin@exampundit.in
IBPS RRB Clerk Prelims – Ultra Practice Bundle PDF
A.12 E.94
B.20 38. 36, 72, 180, 540, 1890, 7650
C.30 A.7650
D.44 B.180
E.56 C.54
34. 10, 24, 68, 100, 222 D.540
A.10 E.1890
B.24 39. 7, 42, 210, 840, 2820, 5040
C.68 A.42
D.100 B.210
E.222 C.2820
35. 3, 7, 13, 20, 31 D.840
A.3 E.5040
B.7 40. 49, 144, 289, 441, 729, 1024
C.13 A.289
D.20 B.144
E.21 C.1024
36. 4, 6, 15, 31, 56, 92 D.441
A.4 E.729
B.15 41. 5, 7, 13, 25, 45, 70
C.6 A.7
D.46 B.13
E.21 C.25
37. 7, 14, 23, 36, 55, 94, 163 D.45
A.7 E.70
B.25 42. 10, 19, 37, 73, 140, 289
C.14 A.19
D.55 B.73
Page 592 of 722

Subscribe the Xpress Video Course & Mock Test Package for Bank & Insurance Exams
If there are any suggestions/ errors in our PDFs Feel Free to contact us via this email: admin@exampundit.in
IBPS RRB Clerk Prelims – Ultra Practice Bundle PDF
C.140 E. None of these
D.289 47. 8, 14, 20, 50, 98, 194
E. None of these A.20
43. 298, 294, 433, 850, 2080, 6215 B.50
A.6215 C.98
B.2080 D.194
C.850 E. None of these
D.433 48. 12, 88, 162, 234, 308, 372
E. None of these A.308
44. 15, 18, 26, 41, 65, 120 B.372
A.120 C.88
B.65 D.234
C.41 E.162
D.18 49. 97, 110, 136, 175, 227, 290
E. None of these A. 290
45. 20, 25, 50, 170, 800, 3925 B.175
A.25 C.110
B.50 D.227
C.170 E.136
D.800 50. 243, 152, 108, 72, 48, 32
E.3925 A.152
46. 8, 20, 50, 122, 312.5, 781.25 B.108
A.20 C.32
B.50 D.48
C.122 E.72
D.312.5

Page 593 of 722

Subscribe the Xpress Video Course & Mock Test Package for Bank & Insurance Exams
If there are any suggestions/ errors in our PDFs Feel Free to contact us via this email: admin@exampundit.in
IBPS RRB Clerk Prelims – Ultra Practice Bundle PDF

4). Wrong Number series - Solutions and Explanations

1. Answer: C 5. Answer: A
2.5 x 2 = 5 339 + 21 = 341
5 x 5 = 25 341 + 22 = 345
25 x 8 = 200 345 + 23 = 353
200 x 11 = 2200 353 + 24 = 369
2200 x 14 = 30800 369 + 25 = 401
2. Answer: C 6. Answer: A
70 x 1 - 2 =68 5*2 + 10 = 20
68 x 2 – 3 = 133 20*2 +10 =50
133 x 3 – 4 = 395 50*2 +10 = 110
395 x 4 – 5 = 1575 110*2+10 = 230
1575 x 5 -6 = 7869 230*2+10 = 470
3. Answer: A 7. Answer: B
643 + 53 = 768 20+5=25
768 + 62 = 804 25-3 = 22
804 + 73 = 1147 22+5 = 27
1147 + 82 = 1211 27 – 3 = 24
1211 + 93 = 1940 24 + 5 = 29
4. Answer: B 8. Answer: C
8 x 0.5 +1 = 5 3 + 12=4
5x1+1=6 4+22 = 8
6 x 1.5 + 1 = 10 8 +32 = 17
10 x 2 + 1 = 21 17 + 42 = 33
21 x 2.5 +1 = 53.5 33 +52 = 58

Page 594 of 722

Subscribe the Xpress Video Course & Mock Test Package for Bank & Insurance Exams
If there are any suggestions/ errors in our PDFs Feel Free to contact us via this email: admin@exampundit.in
IBPS RRB Clerk Prelims – Ultra Practice Bundle PDF
9. Answer: A 41 x 3 +4 = 127
2 22 47 77 112 127 x 3 + 8 = 389
20 25 30 35 389 x 3 + 16 = 1183
10. Answer: E 15. Answer: C
0.5 + 1.5 = 2 105 + 11 = 116
2 +2 = 4 116 +15 = 131
4+ 2.5 =6.5 131 + 17 = 148
6.5 + 3 = 9.5 148 + 19 = 167
9.5 + 3.5 = 13 169 + 21 = 190
11. Answer: B 16. Answer: C
513 + 7 = 520 32 + 20 = 52
520 + 11 = 531 52 + 40 = 92
531 + 13= 544 92 + 80 = 172
544 + 17= 561 172 + 160 = 332
561 + 19= 580 332 + 320 = 652
12. Answer: D 17. Answer: A
305 336 373 422 495 616 256 – 42 = 240
31 37 49 73 121 240 – 52 = 215
6 12 24 48 215 – 62 = 179
13. Answer: C 179 – 72 = 130
15 x 1-1 = 14 130 – 82 = 66
14 x 2 – 2 = 26 18. Answer: B
26 x 3 – 3 = 75 32 + 7 = 39
75 x 4 - 4 = 296 39 + 11 = 50
296 x 5 -5 = 1475 50 + 13 = 63
14. Answer: E 63 + 17 = 80
4 x 3 + 1 = 13 80 + 19 = 99
13 x 3 +2 = 41 19. Answer: C
Page 595 of 722

Subscribe the Xpress Video Course & Mock Test Package for Bank & Insurance Exams
If there are any suggestions/ errors in our PDFs Feel Free to contact us via this email: admin@exampundit.in
IBPS RRB Clerk Prelims – Ultra Practice Bundle PDF
52 – 1 = 24 73 + (7 * 16) = 185
72 – 2 = 47 185 + (9 * 32) = 473
92 – 3 = 78 24. Answer: A
112 – 4 = 117 13 + 10 since, 10 = ( 5 x 2 ) = 23
132 – 5 = 164 23 + 18 since, 18 = (9 x 2 ) = 41
152 – 6 = 219 41 + 26 since, 26 = (13 x 2 ) = 67
20. Answer: D 67 + 34 since, 34 = (17 x 2) = 101
13 * 2 = 26 101 + 42 since, 42 = (21 x 2 ) = 143
26 * 3 = 78 25. Answer: D
78 * 4 = 312 1.2 * 10 = 12
312 * 5 = 1560 12 * 9 = 108
1560 * 6 = 9360 108 * 8 = 864
21. Answer: A 864 * 7 = 6048
8 + (11 + 0) = 19 6048 * 6 = 36288
19 + (11 + 5) = 35 26. Answer: A
35 + (11 + 10) = 56 96 + 182 = 420
56 + (11 + 15) = 82 420 – 172 = 131
82 + (11 + 20) = 113 131 + 162 = 387
22. Answer: C 387 – 152 = 162
67 + 82 + 1 = 132 162 + 142 = 358
132 + 72 + 2 = 183 27. Answer: C
183 + 62 + 3 = 222 1 + (5 * 1) = 6
222 + 52 + 4 = 251 6 + (5 * 3) = 21
251 + 42 + 5 = 272 21 + (5 * 5) = 46
23. Answer: B 46 + (5 * 7) = 81
19 + (1 * 2) = 21 81 + (5 * 9) = 126
21 + (3 * 4) = 33 28. Answer: C
33 + (5 * 8) = 73 122 + 12 = 145
Page 596 of 722

Subscribe the Xpress Video Course & Mock Test Package for Bank & Insurance Exams
If there are any suggestions/ errors in our PDFs Feel Free to contact us via this email: admin@exampundit.in
IBPS RRB Clerk Prelims – Ultra Practice Bundle PDF
142 + 22 = 200 63 = 216
162 + 32 = 265 33. Answer: D
182 + 42 = 340 32 + 3 = 12
202 + 52 = 425 42 + 4 = 20
222 + 62 = 520 52 + 5 = 30
29. Answer: E 62 + 6 = 42
13 * 1 + 1 = 14 72 + 7 = 56
14 * 2 + 1 = 29 34. Answer: D
29 * 3 + 1 = 88 23 + 2 = 10
88 * 4 + 1 = 353 33 – 3 = 24
353 * 5 + 1 = 1766 43 + 4 = 68
30. Answer: A 53 – 5 = 120 (not 100)
343 – 64 = 279 63 + 6 = 222
279 – 32 = 247 35. Answer: D
247 – 16 = 231 12 + 2 = 3
231 – 8 = 223 22 + 3 = 7
223 – 4 = 219 32 + 4 = 13
31. Answer: B 42 + 5 = 21 (not 20)
13 – 1 = 0 52 + 6 = 31
23 – 2 = 6 36. Answer: A
33 – 3 = 24 1 + 12 = 2 (not 4)
43 – 4 = 60 2 + 22 = 6
53 – 5 = 120 6 + 32 = 15
32. Answer: A 15 + 42 = 31
83 = 512 31 + 52 = 56
42 = 16 56 + 62 = 92
73 = 343 37. Answer: D
2
5 = 25 10 x 1/2 + 21 = 7
Page 597 of 722

Subscribe the Xpress Video Course & Mock Test Package for Bank & Insurance Exams
If there are any suggestions/ errors in our PDFs Feel Free to contact us via this email: admin@exampundit.in
IBPS RRB Clerk Prelims – Ultra Practice Bundle PDF
20 x 1/2 + 22 = 14 13 + 9 + 3 = 25
30 x 1/2 + 23 = 23 25 + 16 + 4 = 45
40 x 1/2 + 24 = 36 45 + 25 + 5 = 75 (not 70)
50 x 1/2 + 25 = 57 (not 55) 42. Answer: C
60 x 1/2 + 26 = 94 10 * 2 – 1 = 19
70 x 1/2 + 27 = 163 19 * 2 – 1 = 37
38. Answer: A 37 * 2 – 1 = 73
36 * 2 = 72 73 * 2 – 1 = 145 (not 140)
72 * 2.5 = 180 145 * 2 – 1 = 289
180 * 3 = 540 43. Answer: B
540 * 3.5 = 1890 298 * 1 – 4 = 294
1890 * 4 = 7560 (not 7650) 294 * 1.5 – 8 = 433
39. Answer: C 433 * 2 – 16 = 850
7 * 6 = 42 850 * 2.5 – 32 = 2093 (not 2080)
42 * 5 = 210 2093 * 3 – 64 = 6215
210 * 4 = 840 44. Answer: A
840 * 3 = 2520 (not 2820) 15 + 1 * 3 = 18
2520 * 2 = 5040 18 + 2 * 4 = 26
40. Answer: D 26 + 3 * 5 = 41
72 = 49 41 + 4 * 6 = 65
122 = 144 65 + 5 * 7 = 100 (not 120)
172 = 289 45. Answer: C
222 = 484 (not 441) 20 + 5 = 25
272 = 729 25 + 52 = 50
322 = 1024 50 + 53 = 175 (not 170)
41. Answer: E 175 + 54 = 800
5+1+1=7 800 + 55 = 3925
7 + 4 + 2 = 13 46. Answer: C
Page 598 of 722

Subscribe the Xpress Video Course & Mock Test Package for Bank & Insurance Exams
If there are any suggestions/ errors in our PDFs Feel Free to contact us via this email: admin@exampundit.in
IBPS RRB Clerk Prelims – Ultra Practice Bundle PDF
8 * 2.5 = 20 234 + 70 = 304 (Not 308)
20 * 2.5 = 50 304 + 68 = 372
50 * 2.5 = 125 (not 122) 49. Answer: A
125 * 2.5 = 312.5 97 + (13 * 1) = 110
312.5 * 2.5 = 781.25 110 + (13 * 2) = 136
47. Answer: A 136 + (13 * 3) = 175
8 * 2 – 2 = 14 175 + (13 * 4) = 227
14 * 2 – 2 = 26 (not 20) 227 + (13 * 5) = 292 (not 290)
26 * 2 – 2 = 50 50. Answer: A
50 * 2 – 2 = 98 243 / 1.5 = 162
98 * 2 – 2 = 194 162 / 1.5 = 108
48. Answer: A 108 / 1.5 = 72
12 + 76 = 88 72 / 1.5 = 48
88 + 74 = 162 48 / 1.5 = 32
162 + 72 = 234

5). Quadratic Equations


Directions: In the given question, two equations numbered l and II are given. You have to solve both the
equations and mark the appropriate answer
1. I) x2 + 5x + 6 = 0 2. I) x2 = 784
II) y2 +7y + 12 = 0 II) y = √784
a) x > y a) if x > y
b) x ≥ y b) if x ≥ y
c) x < y c) if x < y
d) x ≤ y d) x ≤ y
e) x = y or the relationship cannot be established e) x = y or the relationship cannot be established
Page 599 of 722

Subscribe the Xpress Video Course & Mock Test Package for Bank & Insurance Exams
If there are any suggestions/ errors in our PDFs Feel Free to contact us via this email: admin@exampundit.in
IBPS RRB Clerk Prelims – Ultra Practice Bundle PDF
3. I) 5x + 3y = 16 7. I. x2 – 7x – 98 = 0
II) 3x + 2y = 34 II. y2 + 16y – 57 = 0
a) x > y a) x < y
b) x ≥ y b) x > y
c) x < y c) x = y OR the relationship cannot be determined
d) x ≤ y d) x ≥ y
e) x = y or the relationship can not be established e) x ≤ y
4. I. x2 + 3x - 18 = 0 8. I. x2 -3x -10 = 0
II. y2 – 16 = 0 II. y2 -37y +330 = 0
a) x > y a) x < y
b) x < y b) x > y
c) x ≤ y c) x ≤ y
d) x ≥ y d) x ≥ y
e) x = y or relationship cannot be determined e) x = y or relationship cannot be established
5. I) 6x + 3y = 18 9. I. x2 -5x +4 = 0
II) 3x + 4y = 15 II. y2 +23y -24 = 0
a) if x < y a) x < y
b) if x > y b) x > y
c) if x ≤ y c) x ≤ y
d) if x ≥ y d) x ≥ y
e) x = y or if the relationship cannot be established e) x = y or relationship cannot be established
6. I. x2 + 15x – 76 = 0 10. I. x2 – 27x + 180 = 0
II. y2 + 20y – 96 = 0 II. y2 – 5y – 84 = 0
a) x < y a) x > y
b) x > y b) x < y
c) x = y OR the relationship cannot be determined c) x = y or the relationship cannot be established
d) x ≥ y d) x ≥ y
e) x ≤ y e) x ≤ y
Page 600 of 722

Subscribe the Xpress Video Course & Mock Test Package for Bank & Insurance Exams
If there are any suggestions/ errors in our PDFs Feel Free to contact us via this email: admin@exampundit.in
IBPS RRB Clerk Prelims – Ultra Practice Bundle PDF
11. I. x2 – 21x + 108 = 0 15. I. x2 +6x +8 = 0
II. y2 – 24y + 128 = 0 II. y2 -3y -10 = 0
a) x > y a) x < y
b) x < y b) x > y
c) x = y or the relationship cannot be established c) x ≤ y
d) x ≥ y d) x ≥ y
e) x ≤ y e) x = y or relationship cannot be established
12. I. x2 – 16x + 63 = 0 16. I. x2 -19x -66 = 0
II. y2 – 2y – 24 = 0 II. y2 +20y +64 = 0
a) x > y a) x < y
b) x < y b) x > y
c) x = y or the relationship cannot be established c) x ≤ y
d) x ≥ y d) x ≥ y
e) x ≤ y e) x = y or relationship cannot be established
13. I. x2 + 17x + 60 = 0 17. I. x2 -22x +72 = 0
II. y2 – 3y – 40 = 0 II. y2 -32y +247 = 0
a) x > y a) x < y
b) x < y b) x > y
c) x = y or the relationship cannot be established c) x ≤ y
d) x ≥ y d) x ≥ y
e) x ≤ y e) x = y or relationship cannot be established
14. I. x2 – 7x – 18 = 0 18. I. x2 +6x -352 = 0
II. y2 – 20y + 99 = 0 II. y2 +25y +156 = 0
a) x < y a) x < y
b) x > y b) x > y
c) x = y OR the relationship cannot be determined c) x ≤ y
d) x ≥ y d) x ≥ y
e) x ≤ y e) x = y or relationship cannot be established
Page 601 of 722

Subscribe the Xpress Video Course & Mock Test Package for Bank & Insurance Exams
If there are any suggestions/ errors in our PDFs Feel Free to contact us via this email: admin@exampundit.in
IBPS RRB Clerk Prelims – Ultra Practice Bundle PDF
19. I. x2 – 24x + 128 = 0 23. I. x2 + 2x - 15 = 0
II. y2 + 7y – 120 = 0 II. y2 - 15y + 44 = 0
a) x > y a) x > y
b) x < y b) x < y
c) x = y or the relationship cannot be established c) x = y or the relationship cannot be established ×
d) x ≥ y d) x ≥ y
e) x ≤ y e) x ≤ y
20. I. x2 + 28x + 195 = 0 24. I. x2 - 12x + 32 = 0
II. y2 + 26y + 168 = 0 II. y2 - y - 12 = 0
a) x > y a) x > y
b) x < y b) x < y
c) x = y or the relationship cannot be established c) x = y or the relationship cannot be established
d) x ≥ y d) x ≥ y
e) x ≤ y e) x ≤ y
21. I. x2 – 6x – 160 = 0 25. I. x2 - 12x + 27 = 0
II. y2 – 30y + 224 = 0 II. y2 - 3y - 18 = 0
a) x > y a) x > y
b) x < y b) x < y
c) x = y or the relationship cannot be established c) x = y or the relationship cannot be established
d) x ≥ y d) x ≥ y
e) x ≤ y e) x ≤ y
22. I. x2 – 26x + 165 = 0 26. I. x2 - 16x + 55 = 0
II. y2 – 24y + 143 = 0 II. y2 - 15y + 56 = 0
a) x > y a) x > y
b) x < y b) x < y
c) x = y or the relationship cannot be established c) x = y or the relationship cannot be established
d) x ≥ y d) x ≥ y
e) x ≤ y e) x ≤ y
Page 602 of 722

Subscribe the Xpress Video Course & Mock Test Package for Bank & Insurance Exams
If there are any suggestions/ errors in our PDFs Feel Free to contact us via this email: admin@exampundit.in
IBPS RRB Clerk Prelims – Ultra Practice Bundle PDF
27. I. x2 – 20x + 96 = 0 31. I. x2 – 8x + 12 = 0
II. y2 + y – 42 = 0 II. y2 + 6y + 5 = 0
a) x > y a) x ≥ y
b) x < y b) y ≥ x
c) x = y or the relationship cannot be established c) Relationships cannot be established between x & y
d) x ≥ y d) x > y
e) x ≤ y e) x < y
28. I. x2 + 13x – 198 = 0 32. x2 – 2x + 1 = 0
II. y2 – 11y – 102 = 0 y2 – 6y + 9 = 0
a) x < y a) x < y
b) x > y b) x > y
c) x = y OR the relationship cannot be determined c) x ≤ y
d) x ≥ y d) x ≥ y
e) x ≤ y e) x = y or relationship between x and y cannot be
29. I. x2 + 7x – 8 = 0 established
II. y2 – 21y + 108 = 0 33. I x2 + 6x + 9 = 0
a) x < y II y2 + 7y + 12 = 0
b) x > y a) x > y
c) x = y OR the relationship cannot be determined b) x ≥ y
d) x ≥ y c) x < y
e) x ≤ y d) x ≤ y
30. I. x2 – 19x + 84 = 0 e) x = y or the relationship can’t be established.
II. y2 + 14y + 45 = 0 34. I x2 – 8x + 15 = 0
a) x < y II y2 + 14y + 24 = 0
b) x > y a) x > y
c) x = y OR the relationship cannot be determined b) x ≥ y
d) x ≥ y c) x < y
e) x ≤ y d) x ≤ y
Page 603 of 722

Subscribe the Xpress Video Course & Mock Test Package for Bank & Insurance Exams
If there are any suggestions/ errors in our PDFs Feel Free to contact us via this email: admin@exampundit.in
IBPS RRB Clerk Prelims – Ultra Practice Bundle PDF
e) x = y or relationship between x and y cannot be c) a ≤ b
established d) a ≥ b
35. I. x2 – 9x – 136 = 0 e) a = b or the relationship cannot be determined
II. y2 + 18y – 63 = 0 39. I. a = √1225
a) x < y II. b2 = 1225
b) x > y a) a > b
c) x = y OR the relationship cannot be determined b) a < b
d) x ≥ y c) a ≥ b
e) x ≤ y d) a ≤ b
36. x2 – x – 6 = 0 e) a = b or relation cannot be established
y2 + 2y – 3 = 0 40. I. a2 - 64 = 0
a) x < y II. 9b2 - 729 = 0
b) x > y a) a < b
c) x ≤ y b) a > b
d) x ≥ y c) a ≤ b
e) x = y or relationship between x and y cannot be d) a ≥ b
established e) a = b or the relationship cannot be determined
37. I. x2 +17x +70 = 0 41. I. x2 -22x +120 = 0
II. y2 -7y -30 = 0 II. y2 -3y -130 = 0
a) x < y a) x < y
b) x > y b) x > y
c) x ≤ y c) x ≤ y
d) x ≥ y d) x ≥ y
e) x = y or relationship cannot be established e) x = y or relationship cannot be established
38. I. a2 - 784 = 0 42. I) x2 – 5x + 6 = 0
II. b2 - 24b = 0 II) y4 – 5y2 + 6 = 0
a) a < b a) x > y
b) a > b b) x ≥ y
Page 604 of 722

Subscribe the Xpress Video Course & Mock Test Package for Bank & Insurance Exams
If there are any suggestions/ errors in our PDFs Feel Free to contact us via this email: admin@exampundit.in
IBPS RRB Clerk Prelims – Ultra Practice Bundle PDF
c) x < y b) x < y
d) x = y or Relationship between x and y cannot be c) x = y or the relationship cannot be established
determined d) x ≥ y
e) x ≤ y e) x ≤ y
43. I. 4x2 – 9x + 2 = 0 47. I. x2 – 23x + 132 = 0
II. y2 + 29y – 62 = 0 II. y2 – 20y + 100 = 0
a) x < y a) x > y
b) x > y b) x < y
c) x = y OR the relationship cannot be determined c) x = y or the relationship cannot be established
d) x ≥ y d) x ≥ y
e) x ≤ y e) x ≤ y
44. I. x2 – 17x + 60 = 0 48. I. x2 + 3x – 88 =0
II. y2 – 23y + 120 = 0 II. y2 – 33y + 242 = 0
a) x > y a) x > y
b) x < y b) x < y
c) x = y or the relationship cannot be established c) x = y or the relationship cannot be established
d) x ≥ y d) x ≥ y
e) x ≤ y e) x ≤ y
45. I. x2 – x – 56 = 0 49. I. x2 +5x +6 = 0
II. y2 – 15y + 56 = 0 II. y2 -19y -42 = 0
a) x > y a) x < y
b) x < y b) x > y
c) x = y or the relationship cannot be established c) x ≤ y
d) x ≥ y d) x ≥ y
e) x ≤ y e) x = y or relationship cannot be established
46. I. x2 – 88 = 273 50. I. x2 +18x +80 = 0
II. y = √361 II. y2 +29y +190 = 0
a) x > y a) x < y
Page 605 of 722

Subscribe the Xpress Video Course & Mock Test Package for Bank & Insurance Exams
If there are any suggestions/ errors in our PDFs Feel Free to contact us via this email: admin@exampundit.in
IBPS RRB Clerk Prelims – Ultra Practice Bundle PDF
b) x > y d) x ≥ y
c) x ≤ y e) x = y or relationship cannot be established

5). Quadratic Equations - Solutions with Explanation


1. Answer: B) ∴ From the above, we can say x ≤ y
Solving equation I 3. Answer: C)
x + 5x + 6 = 0 As per the given data,
x + 2x + 3x + 6 = 0 From I)
⇒ x(x + 2) + 3(x + 2) = 0 ⇒ 5x + 3y = 16
⇒ (x + 3)(x + 2) =0 Multiply 3 on both sides, we get
⇒ x = – 3 or – 2 ⇒ 15x + 9y = 48 ….equation (1)
Solving equation II From II)
y + 7y + 12 = 0 ⇒ 3x + 2y = 34
y + 4y + 3y + 12 = 0 Multiply 5 on both sides, we get
⇒ y(y + 4) + 3 (y + 4) = 0 ⇒ 15x + 10y = 170 …equation (2)
⇒ (y + 3) (y + 4) = 0 Subtract equation (1) from equation (2), we get
⇒ y = – 3 or – 4 ⇒ -y = -122
∴x≥y ⇒ y = 122
2. Answer: D) Substitute y = 122 in equation (1)
As per the given data, ⇒ 15x + 9 × (122) = 48
From I) ⇒ 15x = 48 - 1098
⇒ x2 = 784 ⇒ x = -1050/15 = -70
⇒ x = ± 28 ∴x<y
From II) 4. Answer: E)
⇒ y = √(784) = 28 As per the given data,
Page 606 of 722

Subscribe the Xpress Video Course & Mock Test Package for Bank & Insurance Exams
If there are any suggestions/ errors in our PDFs Feel Free to contact us via this email: admin@exampundit.in
IBPS RRB Clerk Prelims – Ultra Practice Bundle PDF
By solving x2 + 3x - 18 = 0, we get ⇒ x = 1.8
⇒ x2 + 3x - 18 = 0 ∴x<y
⇒ x2 + 6x - 3x - 18 = 0 6. Answer: C)
⇒ x(x + 6) - 3(x + 6) = 0 I. x2 + 15x – 76 = 0
⇒ (x - 3)(x + 6) = 0 ⇒ x2 + 19x – 4x – 76 = 0
⇒ x = 3 and x = - 6 ⇒ x(x + 19) – 4(x + 19) = 0
Also by solving y2 - 16 = 0 ⇒ (x + 19)(x – 4) = 0
⇒ y2 - 42 = 0 Then, x = - 19 or x = + 4
⇒y=±4 II. y2 + 20y – 96 = 0
When x = 3 and y = 4, then x < y ⇒ y2 + 24y – 4y – 96 = 0
When x = 3 and y = - 4, then x > y ⇒ y(y + 24) – 4(Y + 24) = 0
When x = - 6 and y = 4, then x < y ⇒ (y + 24)(y – 4) = 0
When x = - 6 and y = - 4, then x < y Then, y = - 24 or y = + 4
∴ Relationship cannot be determined So, when x = + 4, x > y for y = - 24 and x = y for y = + 4
5. Answer: A) And when x = - 19, x > y for y = - 24 and x < y for y = +
From the given data, 4
⇒ 6x + 3y = 18 - - - - equation (1) ∴ So, we can observe that no clear relationship cannot be
Also given that 3x + 4y = 15 determined between x and y.
Multiply 2 on both sides, we get 7. Answer: C)
⇒ 2 (3x + 4y = 15 ) I. x2 – 7x – 98 = 0
⇒ 6x + 8y = 30 - - - - equation (2) ⇒ x2 – 14x + 7x – 98 = 0
Subtract equation (1) from equation (2), we get ⇒ x(x – 14) + 7(x – 14) = 0
⇒ 5y = 12 ⇒ (x – 14)(x + 7) = 0
⇒ y = 2.4 Then, x = +14 or x = -7
Substitute y = 2.4 in equation (1) II. y2 + 16y – 57 = 0
⇒ 6x + 3 (2.4) = 18 ⇒ y2 + 19y – 3y – 57 = 0
⇒ 6x = 10.8 ⇒ y(y + 19) – 3(y + 19) = 0

Page 607 of 722

Subscribe the Xpress Video Course & Mock Test Package for Bank & Insurance Exams
If there are any suggestions/ errors in our PDFs Feel Free to contact us via this email: admin@exampundit.in
IBPS RRB Clerk Prelims – Ultra Practice Bundle PDF
⇒ (y + 19)(y – 3) = 0 So, x ≥ y
Then, y = -19 or y = +3 10. Answer: D)
So, when x = +14, x > y for y = -19 and x > y for y = +3 From I:
And when x = - 7, x > y for y = -19 and x < y for y = +3 x2 – 27x + 180 = 0
∴ So, we can observe that no clear relationship cannot be x2 – 12x – 15x + 180 = 0
determined between x and y. x(x – 12) – 15(x – 12) = 0
8. Answer: A) (x – 12)(x – 15) = 0
From equation I: x = 12, 15
x2 -3x -10 = (x + 2)(x -5)= 0 From II:
=> x = -2, 5 y2 – 5y – 84 = 0
From equation II: y2 – 12y + 7y – 84 = 0
y2 -37y +330 = (y -22)(y -15) = 0 y(y – 12) + 7(y – 12)= 0
=> y = 22, 15 (y – 12)(y + 7) = 0
X = -2 X = 5 y = 12, -7
Y = 22 x < y x<y X Relation y
Y = 15 x < y x<y 12 = 12
So, x < y 15 > 12
9. Answer: D) 12 > -7
From equation I: 15 > -7
x2 -5x +4 = (x -4)(x -1)= 0 Therefore, x ≥ y
=> x = 4, 1 11. Answer: C)
From equation II: From I:
y2 +23y -24 = (y + 24)(y -1) = 0 x2 – 21x + 108 = 0
=> y = -24, 1 x2 – 12x – 9x + 108 = 0
X=4 X=1 x(x – 12) – 9(x – 12) = 0
Y = -24 x > y x>y (x – 12)(x – 9) = 0
Y=1 x>y x=y x = 9, 12
From II:
Page 608 of 722

Subscribe the Xpress Video Course & Mock Test Package for Bank & Insurance Exams
If there are any suggestions/ errors in our PDFs Feel Free to contact us via this email: admin@exampundit.in
IBPS RRB Clerk Prelims – Ultra Practice Bundle PDF
y2 – 24y + 128 = 0 9 > -4
y2 – 16y – 8y + 128 = 0 7 > 6
y(y – 16) – 8(y – 16)= 0 7 > -4
(y – 16)(y – 8) = 0 Therefore, x > y
y = 8, 16 13. Answer: E)
X Relation y From I:
9 > 8 x2 + 17x + 60 = 0
9 < 16 x2 + 5x + 12x + 60 = 0
12 > 8 x(x + 5) + 12(x + 5) = 0
12 < 16 (x + 5)(x + 12) = 0
Therefore, no relation can be established between x and x = -5, -12
y From II:
12. Answer: A) y2 – 3y – 40 = 0
From I: y2 – 8y + 5y – 40 = 0
x2 – 16x + 63 = 0 y(y – 8) + 5(y – 8)= 0
x2 – 7x – 9x + 63 = 0 (y – 8)(y + 5) = 0
x(x – 7) – 9(x – 7) = 0 y = 8, -5
(x – 7)(x – 9) = 0 X Relation y
x = 7, 9 -5 < 8
From II: -5 = -5
y2 – 2y – 24 = 0 -12 < 8
y2 – 6y + 4y – 24 = 0 -12 < -5
y(y – 6) + 4(y – 6)= 0 Therefore, x ≤ y
(y – 6)(y + 4) = 0 14. Answer: E)
y = 6, -4 I. x2 – 7x – 18 = 0
X Relation y ⇒ x2 – 9x + 2x – 18 = 0
9 > 6 ⇒ x(x – 9) + 2(x – 9) = 0

Page 609 of 722

Subscribe the Xpress Video Course & Mock Test Package for Bank & Insurance Exams
If there are any suggestions/ errors in our PDFs Feel Free to contact us via this email: admin@exampundit.in
IBPS RRB Clerk Prelims – Ultra Practice Bundle PDF
⇒ (x – 9)(x + 2) = 0 y2 +20y +64 = (y + 4)(y + 16) = 0
Then, x = + 9 or x = - 2 => y = -4, -16
II. y2 – 20y + 99 = 0 X = 22 X = -3
⇒ y2 – 11y – 9y + 99 = 0 Y = -4 x>y x>y
⇒ y(y – 11) – 9(y – 11) = 0 Y = -16 x > y x>y
⇒ (y – 11)(y – 9)= 0 So, x > y
Then, y = + 11 or y = + 9 17. Answer: E)
So, when x = + 9, x < y for y = + 11 and x = y for y = + 9 From equation I:
And when x = - 2, x < y for y = + 11 and x < y for y = + x2 -22x +72 = (x -18)(x -4)= 0
9 => x = 18, 4
∴ So, we can clearly observe that x ≤ y. From equation II:
15. Answer: C) y2 -32y +247 = (y -19)(y -13) = 0
From equation I: => y = 19, 13
x2 +6x +8 = (x + 2)(x + 4)= 0 X = 18 X = 4
=> x = -2, -4 Y = 19 x < y x<y
From equation II: Y = 13 x > y x<y
y2 -3y -10 = (y -5)(y + 2) = 0 So, relationship cannot be established between x and y
=> y = 5, -2 18. Answer: E)
X = -2 X = -4 From equation I:
Y=5 x<y x<y x2 +6x -352 = (x + 22)(x -16)= 0
Y = -2 x = y x<y => x = -22, 16
So, x ≤ y From equation II:
16. Answer: B) y2 +25y +156 = (y + 13)(y + 12) = 0
From equation I: => y = -13, -12
x2 -19x -66 = (x -22)(x + 3)= 0 X = -22 X = 16
=> x = 22, -3 Y = -13 x < y x>y
From equation II: Y = -12 x < y x>y

Page 610 of 722

Subscribe the Xpress Video Course & Mock Test Package for Bank & Insurance Exams
If there are any suggestions/ errors in our PDFs Feel Free to contact us via this email: admin@exampundit.in
IBPS RRB Clerk Prelims – Ultra Practice Bundle PDF
So, relationship cannot be established between x and y y2 + 26y + 168 = 0
19. Answer: D) y2 + 12y + 14y + 168 = 0
From I: y(y + 12) + 14(y + 12) = 0
x2 – 24x + 128 = 0 (y + 12)(y + 14) = 0
x2 - 8x – 16x + 128 = 0 y = -12, -14
x(x – 8) – 16(x – 8) = 0 X Relation Y
(x – 8)(x – 16) = 0 -13 < -12
x = 8, 16 -13 > -14
From II: -15 < -12
y + 7y – 120 = 0
2
-15 < -14
y2 + 15y – 8y – 120 = 0 So, no relation can be established between x and y.
y(y + 15) – 8(y + 15) = 0 21. Answer: C)
(y + 15)(y – 8) = 0 From I:
y = -15, 8 x2 – 6x – 160 = 0
X Relation Y x2 + 10x – 16x – 160 = 0
8 = 8 x(x + 10) – 16(x + 10) = 0
8 > -15 (x + 10)(x – 16) = 0
16 > 8 x = -10, 16
16 > -15 From II:
So, x ≥ y y2 – 30y + 224 = 0
20. Answer: C) y2 – 14y – 16y + 224 = 0
From I: y(y – 14) – 16(y – 14) = 0
x2 + 28x + 195 = 0 (y – 14)(y – 16) = 0
x2 + 13x + 15x + 195 = 0 y = 14, 16
x(x + 13) + 15(x + 13) = 0 X Relation y
(x + 13)(x + 15) = 0 -10 < 14
x = -13, -15 -10 < 16
From II:
Page 611 of 722

Subscribe the Xpress Video Course & Mock Test Package for Bank & Insurance Exams
If there are any suggestions/ errors in our PDFs Feel Free to contact us via this email: admin@exampundit.in
IBPS RRB Clerk Prelims – Ultra Practice Bundle PDF
16 > 14 (x – 3)(x + 5) = 0
16 = 16 x = 3, -5
So, no relation can be established between x and y. From II:
22. Answer: C) y2 – 15y + 44 = 0
From I: y2 – 4y – 11y + 44 = 0
x2 – 26x + 165 = 0 y(y – 4) – 11(y – 4) = 0
x2 – 11x – 15x + 165 = 0 (y – 4)(y – 11) = 0
x(x – 11) – 15(x – 11) = 0 y = 4, 11
(x – 11)(x – 15) = 0 X Relation y
x = 11, 15 3 < 4
From II: 3 < 11
y2 – 24y + 143 = 0 -5 < 4
y2 – 13y – 11y + 143 = 0 -5 < 11
y(y – 13) – 11(y – 13) = 0 So, x < y
(y – 13)(y – 11) = 0 24. Answer: D)
y = 11, 13 From I:
X Relation y x2 – 12x + 32 = 0
11 = 11 x2 – 8x – 4x + 32 = 0
11 < 13 x(x – 8) – 4(x – 8) = 0
15 > 11 (x – 8)(x – 4) = 0
15 > 13 x = 8, 4
So, no relation can be established between x and y. From II:
23. Answer: B) y2 - y – 12 = 0
From I: y2 + 3y – 4y – 12 = 0
x2 + 2x – 15 = 0 y(y + 3) – 4(y + 3) = 0
x2 – 3x + 5x – 15 = 0 (y + 3)(y – 4) = 0
x(x – 3) + 5(x – 3) = 0 y = -3, 4

Page 612 of 722

Subscribe the Xpress Video Course & Mock Test Package for Bank & Insurance Exams
If there are any suggestions/ errors in our PDFs Feel Free to contact us via this email: admin@exampundit.in
IBPS RRB Clerk Prelims – Ultra Practice Bundle PDF
X Relation y x2 - 16x + 55 = 0
8 > -3 x2 - 11x - 5x + 55 = 0
8 > 4 x(x - 11) - 5(x - 11) = 0
4 > -3 (x - 5)(x - 11) = 0
4 = 4 x = 5, 11
So, x ≥ y From II:

25. Answer: C) y2 - 15y + 56 = 0


From I: y2 - 7y - 8y + 56 = 0
x2 – 12x + 27 = 0 y(y - 7) - 8(y - 7) = 0
x2 – 3x – 9x + 27 = 0 (y - 7)(y - 8) = 0
x(x – 3) – 9(x – 3) = 0 y = 7, 8
(x – 3)(x – 9) = 0 X Relation y
x = 3, 9 5 < 7
From II: 5 < 8
y2 - 3y – 18 = 0 11 > 7
y2 + 3y – 6y – 18 = 0 11 > 8
y(y + 3) – 6(y + 3) = 0 So, no relationship can be established between x and y.
(y + 3)(y – 6) = 0 27. Answer: A)
y = -3, 6 From I:
X Relation y x2 – 20x + 96 = 0
3 > -3 x2 – 8x – 12x + 96 = 0
3 < 6 x(x – 8) – 12(x – 8) = 0
9 > -3 (x – 8)(x – 12) = 0
9 > 6 x = 8, 12
So, no relationship can be established between x and y. From II:

26. Answer: C) y2 + y – 42 = 0
From I: y2 + 7y – 6y – 42 = 0
y(y + 7) – 6(y + 7) = 0
Page 613 of 722

Subscribe the Xpress Video Course & Mock Test Package for Bank & Insurance Exams
If there are any suggestions/ errors in our PDFs Feel Free to contact us via this email: admin@exampundit.in
IBPS RRB Clerk Prelims – Ultra Practice Bundle PDF
(y + 7)(y – 6) = 0 ⇒ (x + 8)(x – 1) = 0
y = -7, 6 Then, x = - 8 or x = + 1
X Relation Y II. y2 – 21y + 108 = 0
8 > -7 ⇒ y2 – 12y – 9y + 108 = 0
12 > -7 ⇒ y(y – 12) – 9(y – 12) = 0
8 > 6 ⇒ (y – 12)(y – 9) = 0
12 > 6 Then, y = + 12 or y = + 9
So, x > y So, when x = - 8, x < y for y = + 12 and x < y for y = + 9
28. Answer: C) And when x = + 1, x < y for y = + 12 and x < y for y = +
I. x2 + 13x – 198 = 0 9
⇒ x2 + 22y – 9y – 198 = 0 So, we can observe that x < y.
⇒ x(x + 22) – 9(y + 22) = 0 30. Answer: B)
⇒ (x + 22)(x – 9) = 0 I. x2 – 19x + 84 = 0
Then, x = -22 or x = +9 ⇒ x2 – 12x – 7x + 84 = 0
II. y2 – 11y – 102 = 0 ⇒ x(x – 12) – 7(x – 12) = 0
⇒ y2 – 17y + 6y – 102 = 0 ⇒ (x – 12)(x – 7) = 0
⇒ y(y – 17) + 6(y – 17) = 0 Then, x = + 12 or x = + 7
⇒ (y – 17)(y + 6) = 0 II. y2 + 14y + 45 = 0
Then, y = + 17 or y = -6 ⇒ y2 + 9y + 5y + 45 = 0
So, when x = -22, x < y for y = + 17 and x < y for y = -6 ⇒ y(y + 9) + 5(y + 9) = 0
And when x = +9, x < y for y = + 17 and x > y for y = -6 ⇒ (y + 9)(y + 5) = 0
∴ So, we can observe that no clear relationship cannot be Then, y = - 9 or y = - 5
determined between x and y. So, when x = + 12, x > y for y = - 9 and x > y for y = - 5
29. Answer: A) And when x = + 7, x > y for y = - 9 and x > y for y = - 5
I. x2 + 7x – 8 = 0 ∴ So, we can observe that x > y.
⇒ x2 + 8x – x – 8 = 0 31. Answer: D)
⇒ x(x + 8) – 1(x + 8) = 0 I. x2 – 8x + 12 = 0

Page 614 of 722

Subscribe the Xpress Video Course & Mock Test Package for Bank & Insurance Exams
If there are any suggestions/ errors in our PDFs Feel Free to contact us via this email: admin@exampundit.in
IBPS RRB Clerk Prelims – Ultra Practice Bundle PDF
⇒ x2 – 6x – 2x + 12 = 0 ⇒ x2 + 6x + 9 = 0
⇒ x(x – 6) – 2(x – 6) = 0 ⇒ x2 + 3x + 3x + 9 = 0
⇒ (x – 6)(x – 2) = 0 ⇒ x (x + 3) + 3 (x + 3 ) = 0
Then, x = 6 or x = 2 ⇒ (x + 3) (x + 3) = 0
II. y2 + 6y + 5 = 0 ∴x=-3
⇒ y2 + 5y + y + 5 = 0 Also given that y2 + 7y + 12 = 0
⇒ y(y + 5) + 1(y + 5) = 0 ⇒ y2 + 3y + 4y + 12 = 0
⇒ (y + 5)(y + 1) = 0 ⇒ y (y + 3) + 4 (y + 3) = 0
Then, y = - 5 or y = - 1 ⇒ (y + 3) (y + 4) = 0
∴ As, we can observe that both of the values of x are ∴ y = - 3 and y = - 4
positive and both of the values of y are negative then we When x = - 3 and y = - 3, then x = y
can clearly say that x > y. x = - 3 and y = - 4, then x > y
32. Answer: A) ∴x≥y
For eq 1: 34. Answer: A)
x2 – 2x + 1 = 0 From the given data,
⇒ x2 – x – x + 1 = 0 ⇒ x2 – 8x + 15 = 0
⇒ (x - 1)2 = 0 ⇒ x2 - 3x - 5x + 15 = 0
∴x=1 ⇒ x(x - 3) - 5(x - 3) = 0
For eq 2: ⇒ (x - 3)(x - 5) = 0
y2 – 6y + 9 = 0 ∴ x = 3 and x = 5
⇒ y – 3y – 3y + 9 = 0
2
Also given that y2 + 14y + 24 = 0
⇒ (y – 3)2 = 0 ⇒ y2 + 12y + 2y + 24 = 0
∴y=3 ⇒ y(y + 12) + 2(y + 12) = 0
When x = 1, y = 3 ⇒ (y + 2)(y + 12) = 0
∴ x and y are related as x < y ⇒ y = - 2 and y = - 12
33. Answer: B) ∴ When x = 3 and y = - 2, then x > y
From the given data, When x = 3 and y = - 12, then x > y

Page 615 of 722

Subscribe the Xpress Video Course & Mock Test Package for Bank & Insurance Exams
If there are any suggestions/ errors in our PDFs Feel Free to contact us via this email: admin@exampundit.in
IBPS RRB Clerk Prelims – Ultra Practice Bundle PDF
When x = 5 and y = - 2, then x > y ⇒ y2 + 3y− y − 3 = 0
When x = 5 and y = - 12, then x > y ⇒ (y + 3)(y – 1) = 0
∴x>y ∴ y = −3 or y = 1
35. Answer: C) When x = −2, y = -3, then x > y
I. x2 – 9x – 136 = 0 When x = −2, y = 1, then x < y
⇒ x2 – 17x + 8x – 136 = 0 When x = 3, y = -3, then x > y
⇒ x(x – 17) + 8(x – 17) = 0 When x = 3, y = 1, then x > y
⇒ (x – 17)(x + 8) = 0 ∴ No relation is obtained between x and y
Then, x = + 17 or x = - 8 37. Answer: A)
II. y2 + 18y – 63 = 0 From equation I:
⇒ y2 + 21y – 3y – 63 = 0 x2 +17x +70 = (x + 10)(x + 7)= 0
⇒ y(y + 21) – 3(y + 21) = 0 => x = -10, -7
⇒ (y + 21)(y – 3) = 0 From equation II:
Then, y = - 21 or y = + 3 y2 -7y -30 = (y -10)(y + 3) = 0
So, when x = + 17, x > y for y = - 21 and x > y for y = + => y = 10, -3
3 X = -10 X = -7
And when x = - 8, x > y for y = - 21 and x < y for y = + 3 Y = 10 x < y x<y
∴ So, we can observe that no clear relationship cannot be Y = -3 x<y x<y
determined between x and y. So, x < y
36. Answer: E) 38. Answer: E)
For eq 1: I. a2 - 784 = 0
x2 – x – 6 = 0 ⇒ a2 - 282 = 0
⇒ x2 + 2x – 3x – 6 = 0 ⇒ Use: (a2 - b2) = (a - b)(a + b)
⇒ (x + 2)(x – 3) = 0 ⇒ (a + 28)(a - 28) = 0
∴ x = −2 or x = 3 Then, a = (-28) or a = (28)
For eq 2: II. b2 - 24b = 0
y2 + 2y – 3 = 0 ⇒ b(b - 24) = 0

Page 616 of 722

Subscribe the Xpress Video Course & Mock Test Package for Bank & Insurance Exams
If there are any suggestions/ errors in our PDFs Feel Free to contact us via this email: admin@exampundit.in
IBPS RRB Clerk Prelims – Ultra Practice Bundle PDF
Then, b = (0) or a = (24) And when a = 8, a > b for b = -9 and a < b for b = 9
So, when a = (-28), a < b for b = (0) and a < b for b = ∴ So, the relationship cannot be determined.
(24) 41. Answer: E)
And when a = (28), a > b for b = (0) and a > b for b = From equation I:
(24) x2 -22x +120 = (x -10)(x -12)= 0
∴ So, the relationship cannot be determined. => x = 10, 12
39. Answer: C) From equation II:
I. a = √1225 y2 -3y -130 = (y -13)(y + 10) = 0
⇒ a = √(35 × 35) => y = 13, -10
⇒ a = +35 X = 10 X = 12
II. b2 = 1225 Y = 13 x<y x<y
⇒ b2 = (35)2 Y = -10 x > y x>y
⇒ b = ± 35 So, relationship cannot be established between x and y
So, when a = +35, a = b for a = +35 and a > b for a = -35 42. Answer: A)
∴ We can observe that a ≥ b. I) x2 – 5x + 6 = 0
40. Answer: E) ⇒ (x – 2) (x – 3) = 0
I. a2 - 64 = 0 ⇒ x = 2, 3
Use: (a2 - b2) = (a - b)(a + b) II) y4 – 5y2 + 6 = 0
⇒ (a + 8)(a - 8) = 0 Let y2 = t
Then, a = 8 or a = -8 ⇒ y = √t
II. 9b2 - 729 = 0 ⇒ t2 – 5t + 6 = 0
⇒ 9(b2 - 81) = 0 ⇒ (t – 2) (t – 3) = 0
2 2
Use: (a - b ) = (a - b)(a + b) ⇒ t = 2, 3
⇒ (b2 - 81) = 0 ⇒ y = ±√2, ±√3
⇒ (b + 9)(b - 9) = 0 ⇒ For x = 2 and y = ±√2
Then, b = 9 or b = -9 ⇒x>y
So, when a = -8, a > b for b = -9 and a < b for b = 9 ⇒ For x = 3 and y = ±√2

Page 617 of 722

Subscribe the Xpress Video Course & Mock Test Package for Bank & Insurance Exams
If there are any suggestions/ errors in our PDFs Feel Free to contact us via this email: admin@exampundit.in
IBPS RRB Clerk Prelims – Ultra Practice Bundle PDF
⇒x>y (x – 12) (x - 5) = 0
⇒ For x = 2 and y = ±√3 x = 12, 5
⇒x>y From II:
⇒ For x = 2 and y = ±√3 y2 – 23y + 120 = 0
⇒x>y y2 – 15y – 8y + 120 = 0
∴x>y y(y – 15) – 8(y – 15) = 0
43. Answer: C) (y – 15)(y – 8) = 0
I. 4x2 – 9x + 2 = 0 y = 8, 15
⇒ 4x2 – 8x – x + 2 = 0 X Relation Y
⇒ 4x(x – 2) – 1(x – 2) = 0 12 > 8
⇒ (x – 2)(4x – 1) = 0 12 < 15
Then, x = + 2 or x = + ¼ 5 < 8
II. y2 + 29y – 62 = 0 5 < 15
⇒ y2 + 31y – 2y – 62 = 0 So, the relationship cannot be established between x and
⇒ y(y + 31) – 2(y + 31) = 0 y.
⇒ (y + 31)(y – 2) = 0 45. Answer: C)
Then, y = - 31 or y = + 2 From I:
So, when x = + 2, x > y for y = - 31 and x = y for y = + 2 x2 – x – 56 = 0
And when x = + 1/4, x > y for y = - 31 and x < y for y = x2 + 7x – 8x – 56 = 0
+2 x(x + 7) - 8 (x + 7) = 0
∴ So, we can observe that no clear relationship cannot be (x + 7) (x – 8) = 0
determined between x and y. x = -7, 8
44. Answer: C) From II:
From I: y2 – 15y + 56 = 0
x2 – 17x + 60 = 0 y2 – 7y – 8y + 56 = 0
x2 – 12x - 5x + 60 = 0 y(y – 7) – 8(y – 7) = 0
x(x – 12) - 5 (x – 12) = 0 (y – 8)(y – 7) = 0
y = 8, 7
Page 618 of 722

Subscribe the Xpress Video Course & Mock Test Package for Bank & Insurance Exams
If there are any suggestions/ errors in our PDFs Feel Free to contact us via this email: admin@exampundit.in
IBPS RRB Clerk Prelims – Ultra Practice Bundle PDF
X Relation Y y2 – 20y + 100 = 0
-7 < 8 y2 – 10y – 10y + 100 = 0
-7 < 7 y(y – 10) – 10(y – 10) = 0
8 = 8 (y – 10)(y – 10) = 0
8 > 7 y = 10, 10
So, the relationship cannot be established between x and X Relation Y
y. 12 > 10
46. Answer: E) 11 > 10
From I: So, x > y
x2 – 88 = 273 48. Answer: B)
x2 = 361 From I:
x = 19, -19 x2 + 3x – 88 = 0
From II: x2 + 11x – 8x – 88 = 0
y = √361 x(x + 11) – 8(x + 11) = 0
y = 19 (x + 11)(x - 8) = 0
X Relation Y x = -11, 8
19 = 19 From II:
-19 < 19 y2 – 33y + 242 = 0
So, x ≤ y y2 – 22y – 11y + 242 = 0

47. Answer: A) y(y – 22) – 11(y – 22) = 0


From I: (y – 11)(y – 22) = 0
x2 – 23x + 132 = 0 y = 11, 22
x2 – 12x - 11x + 132 = 0 X Relation y
x(x – 12) - 11(x – 12) = 0 -11 < 22
(x – 12) (x - 11) = 0 -11 < 11
x = 12, 11 8 < 22
From II: 8 < 11

Page 619 of 722

Subscribe the Xpress Video Course & Mock Test Package for Bank & Insurance Exams
If there are any suggestions/ errors in our PDFs Feel Free to contact us via this email: admin@exampundit.in
IBPS RRB Clerk Prelims – Ultra Practice Bundle PDF
So, x < y 50. Answer: D)
49. Answer: C) From equation I:
From equation I: x2 +18x +80 = (x + 8)(x + 10)= 0
x2 +5x +6 = (x + 3)(x + 2)= 0 => x = -8, -10
=> x = -3, -2 From equation II:
From equation II: y2 +29y +190 = (y + 10)(y + 19) = 0
y2 -19y -42 = (y -21)(y + 2) = 0 => y = -10, -19
=> y = 21, -2 X = -8 X = -10
X = -3 X = -2 Y = -10 x > y x=y
Y = 21 x < y x<y Y = -19 x > y x>y
Y = -2 x<y x=y So, x ≥ y
So, x ≤ y

6). DI Based on Table


Directions (01-05): Study the following information carefully and answer the questions given below.
The given table shows the number of persons using different networks in different cities.

1) What is the ratio of the number of persons using all C.2:1


the three networks in Kolkata to Delhi? D.2:3
A.3:2 E.1:2
B.4:5
Page 620 of 722

Subscribe the Xpress Video Course & Mock Test Package for Bank & Insurance Exams
If there are any suggestions/ errors in our PDFs Feel Free to contact us via this email: admin@exampundit.in
IBPS RRB Clerk Prelims – Ultra Practice Bundle PDF
2) What is the difference between the number of E.110%
persons using Airtel and Jio in Chennai and the 4) What is the difference between the average
number of person using Vodafone and Airtel in number of Jio and Airtel users in all the cities
Mumbai? together?
A.1200 A.680
B.1400 B.660
C.1600 C.640
D.1800 D.650
E.None of these E.620
3) The number of persons using all the networks in 5) The total number of users in Mumbai is
Bangalore is approximately what percent of the total approximately what percent of the total number of
number of persons using all the networks in Vodafone users in all the cities together?
Chennai? A.52%
A.102% B.55%
B.106% C.59%
C.104% D.61%
D.108% E.64%

Directions (6-10): Study the following information carefully and answer the given questions?
The following table shows the total number of people of different age groups who get benefit of Mudra Yojana in 5
different states of a country in a certain year.

Page 621 of 722

Subscribe the Xpress Video Course & Mock Test Package for Bank & Insurance Exams
If there are any suggestions/ errors in our PDFs Feel Free to contact us via this email: admin@exampundit.in
IBPS RRB Clerk Prelims – Ultra Practice Bundle PDF

6) Find the difference between the total number of 8) Find the average number of people who get benefit
people who get benefit of Mudra Yojana in the age of Mudra Yojana in the age group of above 40 years
group of 31 – 40 and above 50 years in the state of in all the given states together?
Andhra Pradesh and that of above 40 years in the A.49600
state of Kerala? B.48500
A.28000 C.47200
B.24000 D.50400
C.20000 E.None of these
D.30000 9) Total number of people who get benefit of Mudra
E.None of these Yojana from Tamilnadu is approximately what
7) Find the ratio between the total number of people percentage of total number of people who get benefit
who get benefit of Mudra Yojana in the age group of of Mudra Yojana from Karnataka of all the given age
31 – 40 years and that of 21 – 30 years in all the given groups?
states together? A.136 %
A.123: 92 B.145 %
B.137: 105 C.162 %
C.88: 65 D.113 %
D.77: 53 E.125 %
E.None of these 10) Find the difference between the average number
of people who get benefit of Mudra Yojana in the age
Page 622 of 722

Subscribe the Xpress Video Course & Mock Test Package for Bank & Insurance Exams
If there are any suggestions/ errors in our PDFs Feel Free to contact us via this email: admin@exampundit.in
IBPS RRB Clerk Prelims – Ultra Practice Bundle PDF
group of 21 – 30 years to that of above 50 years in all C.18600
the given states together? D.17800
A.19200 E.None of these
B.20400
Directions (11-15): The given table shows the five different school students like four different subjects.

11) If the average number of students in school E is C.1040


225 and the number of students like English in E is D.1010
80% of the number students like chemistry in B. E. None of these
What is the average number of students like 13) What is the ratio of the number of students like
chemistry in all the schools together? English in A, B and D together to the number of
A.218 students like Maths in A and E together?
B.228 A.11:17
C.238 B. 2:3
D.248 C.13:19
E.None of these D.7:10
12) The number of students like physics in A is two- E.None of these
three of the number of students like Maths in C. 14) The number of students like Maths in B is equal
What is the total number of students in A? to the number of students like Chemistry in E and the
A.1020 number of the students like physics in C is 80% of the
B.1030 number of students like Maths in E. If the ratio of the

Page 623 of 722

Subscribe the Xpress Video Course & Mock Test Package for Bank & Insurance Exams
If there are any suggestions/ errors in our PDFs Feel Free to contact us via this email: admin@exampundit.in
IBPS RRB Clerk Prelims – Ultra Practice Bundle PDF
number of students like physics in C to the number of 15) The number of students like Physics in D is
students like Maths in D is 8:9, then what is the total approximately what percent of the number of
number of students like Maths in all the schools students like English in B?
together? A.73%
A.1280 B. 75%
B.1680 C.77%
C.1530 D.79%
D.1490 E.71%0
E.None of these

Directions (16-20): The given table shows the number of employees and ratio of the male to female employees
in three different companies from five different cities.

16) What is the average number of male employees in 17) The number of male employees in TCS from
Wipro from all the cities together? Mumbai is approximately what percent of the
A.2190 number of female employees in Infosys from
B.2100 Bangalore?
C.2160 A.60%
D.2180 B.70%
E.None of these C.80%

Page 624 of 722

Subscribe the Xpress Video Course & Mock Test Package for Bank & Insurance Exams
If there are any suggestions/ errors in our PDFs Feel Free to contact us via this email: admin@exampundit.in
IBPS RRB Clerk Prelims – Ultra Practice Bundle PDF
D.90% A. 48:25
E.None of these B. 51:29
18) What is a total number female employee in Delhi C. 53:31
from all the compaines together? D.49:30
A.3200 E.None of these
B.3400 20) What is the average number of female employees
C.3100 in Infosys from all the cities together?
D.3500 A.720
E.3600 B.740
19) What is the ratio of the number of male C.760
employees from Kolkata, Mumbai and Bangalore D.750
together in TCS to the number of female employees E.None of these
from Chennai, Mumbai and Delhi together in Wipro?
Directions (21-25): The given table shows the number of employees in four different companies and the
percentage of male, ratio of the male to female employees in IT department. The companies have two different
departments IT and HR.

21) What is the average number of male employees in A.1200


all the companies together? B.1400

Page 625 of 722

Subscribe the Xpress Video Course & Mock Test Package for Bank & Insurance Exams
If there are any suggestions/ errors in our PDFs Feel Free to contact us via this email: admin@exampundit.in
IBPS RRB Clerk Prelims – Ultra Practice Bundle PDF
C.1500 E.63%
D.1300 24) What is the difference between the male to female
E.1100 employees in HR department in all the companies
22) What is the ratio of the total number of male to together?
female in HR department employees in D? A.1460
A.4:3 B.1680
B.6:1 C.1720
C.7:2 D.Cannot be determine
D.8:3 E.None of these
E.None of these 25) What is the average number of female employees
23) The number of male employees in IT department in IT department in all the companies together?
in C is approximately what percent of the total A.440
number of employees in HR department in D? B.470
A.43% C.460
B.50% D.480
C.53% E.None of these
D.77%

Directions (26-30): The given table shows the marks scored by 5 students in 5 different subjects.

Page 626 of 722

Subscribe the Xpress Video Course & Mock Test Package for Bank & Insurance Exams
If there are any suggestions/ errors in our PDFs Feel Free to contact us via this email: admin@exampundit.in
IBPS RRB Clerk Prelims – Ultra Practice Bundle PDF
26) What is the difference between the total marks C.16:15
scored by Shon and Esai in all the subjects together? D.18:13
A.60 E.None of these
B.80 29) Total marks scored in Chemistry by all the
C.70 students together is approximately what percent
D.90 more than that of the total marks scored by Arul in
E.None of these all the subjects together?
27) What is the sum of the average of the marks A.25%
scored in English by all the students together and the B.27%
average of the marks scored in Hindi by all the C.29%
students together? D.31%
A.110 E.None of these
B.115 30) What is the difference between the average marks
C.120 scored by Divya and Bharathi in all the subjects
D.125 together?
E.None of these A.4
28) What is the ratio of the total marks scored in B.10
Maths by all the students together to the total marks C.8
scored in Physics by all the students together? D.2
A.15:13 E.None of these
B.17:11
Directions (31-35): Study the following information carefully and answer the given questions:
The given table shows the number of employee works in five different companies and the ratio of the employees of
two different departments-IT and HR.

Page 627 of 722

Subscribe the Xpress Video Course & Mock Test Package for Bank & Insurance Exams
If there are any suggestions/ errors in our PDFs Feel Free to contact us via this email: admin@exampundit.in
IBPS RRB Clerk Prelims – Ultra Practice Bundle PDF

31) 60% of employees in IT department are female in what is the difference between the total number of
A and 40% of employees in HR departmentare male male and female employees in all the companies
in A. Find the total number of female employees in together?
company A. A.2400
A.1800 B.2500
B.1500 C.2600
C.2000 D.2700
D.2400 E.None of these
E.None of these 34) The number of employees in IT department in
32) What is the difference between the total number company C and D together is approximately what
of employees in IT department in all the companies percent of the number of employees in HR
together and the total number of employees in HR department in company B and E together?
departments in all the companies together? A.224%
A.400 B.113%
B.500 C.87%
C.600 D.90%
D.300 E. None of these
E.None of these 35) The total number of IT department employees in
33) If the ratio of the number of male to female all the companies together is approximately what
employees in all the companies together is 3:2, then

Page 628 of 722

Subscribe the Xpress Video Course & Mock Test Package for Bank & Insurance Exams
If there are any suggestions/ errors in our PDFs Feel Free to contact us via this email: admin@exampundit.in
IBPS RRB Clerk Prelims – Ultra Practice Bundle PDF
percent of the total employees of all the companies C. 44%
together? D. 56%
A. 52% E. None of these
B. 48%

Directions (36-40): Study the following information carefully and answer the given questions:
The following table shows the total number of crops (In quintal) produced by 5 different farmers in a certain year.

36) The total crops produced by Rakesh is E.None of these


approximately what percentage of total crops 38) Find the difference between the average of Rye
produced by Abinav? produced to that of Oats produced by all the given
A.85 % farmers together?
B.72 % A.234
C.97 % B.156
D.58 % C.118
E.115 % D.192
37) Find the average crops produced by Ravi? E.None of these
A.574 39) Find the ratio between total mustard produced to
B.612 that of total wheat produced by all the farmers?
C.656 A.185 : 113
D.738 B.397 : 284
Page 629 of 722

Subscribe the Xpress Video Course & Mock Test Package for Bank & Insurance Exams
If there are any suggestions/ errors in our PDFs Feel Free to contact us via this email: admin@exampundit.in
IBPS RRB Clerk Prelims – Ultra Practice Bundle PDF
C.229 : 175 A.Ravi
D.52 : 17 B.Manoj
E.None of these C.Rakesh
40) Which farmer produced third lowest number of D.Abinav
crops? E.Naveen

Directions (41-45): The following table shows the number (lakhs) of TV sets manufactured by six companies
over the years…

41) What is the average number of TV sets A.20%


manufactured by company C for all the given years ? B.23%
A.49,00,000 C.25%
B.48,50,000 D.16%
C.49,50,000 E.18%
D.48,00,000 43) What is the ratio between the total number TV
E.None of these sets manufactured by all the companies together in
42) The number of TV sets manufactured by 2007 and that in 2006 respectively ?
company E in 2004 is approximately what percent of A.56:57
the total number of TV sets manufactured by all the B.57:56
companies together in 2004 ? C.29:28
Page 630 of 722

Subscribe the Xpress Video Course & Mock Test Package for Bank & Insurance Exams
If there are any suggestions/ errors in our PDFs Feel Free to contact us via this email: admin@exampundit.in
IBPS RRB Clerk Prelims – Ultra Practice Bundle PDF
D.28:29 E.97%
E.None of these 45) The number of TVs manufactured by company A
44) Total number of TV’s manufactured by company is approx. what percent of the total number of TVs
A is approx. what percent of the total TVs manufactured by company B In all the years together
manufactured by company F for all the given years ?
together ? A.200%
A.85% B.186%
B.87% C.134.6%
C.90% D.158%
D.92% E.122%

Directions (46-50): Study the following graph carefully and answer the given questions
The following table shows the number of students participated in Volley ball, cricket and other games in five
different schools

46) What is the difference between the total number B.8


of students participated in volley ball from school A C.6
and D and the total number of students participated D.4
in cricket from school B and C? E.10
A.2

Page 631 of 722

Subscribe the Xpress Video Course & Mock Test Package for Bank & Insurance Exams
If there are any suggestions/ errors in our PDFs Feel Free to contact us via this email: admin@exampundit.in
IBPS RRB Clerk Prelims – Ultra Practice Bundle PDF
47) Find the total number of students participated in 49) What is the difference between the total number
other games from all the schools of students participated in volley ball and the total
A.1244 number of students participated in Cricket?
B.1324 A.75
C.1388 B.120
D.1364 C.50
E.1266 D.100
48) If a total student in school F is 20% more than the E.80
total number of students participated in cricket from 50) In which school has maximum students
schools E and C together, and then find the total participated in other games?
students in school F A.A
A.432 B.D
B.456 C.E
C.424 D.C
D.464 E.B
E.564

6). DI Based on Table - Solutions and Detailed Explanations


1) Answer: B = 10800
Required ratio = (3000 + 3600 + 5800):(4200 + 4800 + Number of person using Vodafone and Airtel in Mumbai
6500) = 3200 + 6000 = 9200
= 12400:15500 Difference = 10800 – 9200 = 1600
= 124:155 3) Answer: D
= 4:5 Required percentage = (5000 + 6400 + 5200)/(4800 +
2) Answer: C 6000 + 4500) * 100
Number of person using Airtel and Jio in Chennai = (4800 = 108%
+ 6000) 4) Answer: A
Page 632 of 722

Subscribe the Xpress Video Course & Mock Test Package for Bank & Insurance Exams
If there are any suggestions/ errors in our PDFs Feel Free to contact us via this email: admin@exampundit.in
IBPS RRB Clerk Prelims – Ultra Practice Bundle PDF
Average users in Jio = 6000 + 6400 + 4800 + 5600 + The total number of people who get benefit of Mudra
3600/5 Yojana in the age group of 21 – 30 years in all the given
= 5280 states together
Average users in Airtel = 4800 + 5000 + 4200 + 6000 + = >48000 + 40000 + 36000 + 32000 + 54000 = 210000
3000/5 Required ratio = 274000: 210000 = 137: 105
= 4600 8) Answer: A
Difference = 5280 – 4600 = 680 The average number of people who get benefit of Mudra
5) Answer: C Yojana in the age group of above 40 years in all the given
Required percentage = (6000 + 5600 + 3200)/(4500 + states together
5200 + 6500 + 3200 + 5800) * 100 = > (28000 + 16000 + 30000 + 18000 + 32000 + 24000 +
= 59% 26000 + 20000 + 24000 + 30000) / 5
6) Answer: C = > 248000 / 5 = 49600
The total number of people who get benefit of Mudra 9) Answer: D
Yojana in the age group of 31 – 40 and above 50 years in Total number of people who get benefit of Mudra Yojana
the state of Andhra Pradesh from Tamilnadu of all the given age groups
= >56000 + 18000 = 74000 = >48000 + 62000 + 28000 + 16000 = 154000
The total number of people who get benefit of Mudra Total number of people who get benefit of Mudra Yojana
Yojana in the age group of above 40 years in the state of from Karnataka of all the given age groups
Kerala = >36000 + 44000 + 32000 + 24000 = 136000
= >24000 + 30000 = 54000 Required % = (154000 / 136000) * 100 = 113 %
Required difference = 74000 – 54000 = 20000 10) Answer: B
7) Answer: B The average number of people who get benefit of Mudra
The total number of people who get benefit of Mudra Yojana in the age group of 21 – 30 years in all the given
Yojana in the age group of 31 – 40 years in all the given states together
states together = > (48000 + 40000 + 36000 + 32000 + 54000) / 5
= > 62000 + 56000 + 44000 + 52000 + 60000 = 274000 = > 42000

Page 633 of 722

Subscribe the Xpress Video Course & Mock Test Package for Bank & Insurance Exams
If there are any suggestions/ errors in our PDFs Feel Free to contact us via this email: admin@exampundit.in
IBPS RRB Clerk Prelims – Ultra Practice Bundle PDF
The average number of people who get benefit of Mudra Number of students like Maths in D=9/8 * 320=360
Yojana in the age group of above 50 years in all the given Total number of students like Maths=360 + 200 + 420 +
states together 360 + 400
= > (16000 + 18000 + 24000 + 20000 + 30000) / 5 =1740
= > 21600 15) Answer: A
Required difference = 42000 – 21600 = 20400 Required percentage=160/220 * 100=73%
11) Answer: C Directions (16-20):
Total number of students in school E=225 * 4=900
Number of students like English in E=900 – (200 + 180 +
400)=120
Number of students like chemistry in B=100/80 *
120=150
Average of the number of students like chemistry in all the
schools together=(240 + 320 + 280 + 200 + 150)/5
=238
16) Answer: D
12) Answer: A
Average number of male in Wipro = (1500 + 2400 + 2400
Number of students like physics in A=2/3 * 420
+ 1600 + 3000)/5
=280
= 2180
Total number of students in school A=240 + 280 + 360 +
17) Answer: C
140
Required percentage=400/500 * 100=80%
=1020
18) Answer: D
13) Answer: C
Required total=900 + 1000 + 1600=3500
Required ratio=(140 + 220 + 160):(360 + 400)
19) Answer: B
=520:760
Required ratio=(1500 + 400 + 3200):(900 + 400 + 1600)
=13:19
=5100:2900
14) Answer: E
=51:29
Number of students like Maths in B=200
20) Answer: C
Number of students like Physics in C=80/100 * 400=320
Page 634 of 722

Subscribe the Xpress Video Course & Mock Test Package for Bank & Insurance Exams
If there are any suggestions/ errors in our PDFs Feel Free to contact us via this email: admin@exampundit.in
IBPS RRB Clerk Prelims – Ultra Practice Bundle PDF
Required average = (1600 + 500 + 1000 + 200 + 500) / 5 Difference=2800 – 1120=1680
= 760 25) Answer: B
Directions (21-25): Average = (600 + 300 + 480 + 500)/4=470
26) Answer: B
Total marks scored by Shon=(70 + 50 + 60 + 40 + 80)=300
Total marks scored by Esai=(60 + 20 + 30 + 20 + 90)=220
Required Difference=300 – 220=80
27) Answer: C
Average in English=(70 + 90 + 40 + 60 + 20)/5=56
21) Answer: C Average in Hindi=(20 + 70 + 80 + 60 + 90)/5=64
Average=(1200 + 1800 + 900 + 2100)/4 Required Sum=56 + 64=120
=1500
28) Answer: B
22) Answer: B Total Marks in Maths=90 + 80 + 70 + 40 + 60=340
HR male in D=2100 – 900=1200 Physics=0 + 40 + 60 + 90 + 30=220
HR female in D=700 – 500=200 Required ratio=340:220
Required ratio=1200: 200 =17:11
= 6:1
29) Answer: C
23) Answer: A Total marks in Chemistry=25 + 80 + 50 + 90 + 20=265
Required percentage=600/1400 * 100=42.85% Total marks scored by Arul=90 + 25 + 0 + 70 + 20=2050
24) Answer: B Required percentage= [(265 – 205)/205] * 100=29%
Total male employees=1200 + 1800 + 900 + 2100=6000
30) Answer: A
Total female employees=800 + 600 + 900 + 700=3000 Total marks scored by Divya=(40 + 90 + 90 + 60 +
IT department male employees=800 + 900 + 600 + 60)/5=68
900=3200 Total marks scored by Bharathi=(80 + 80 + 40 + 90 +
IT department female employees=600 + 300 + 480 + 70)/5=72
500=1880 Required Difference=72 – 68=4
HR department male employees=6000 – 3200=2800
HR department female employees=3000 – 1880=1120
Page 635 of 722

Subscribe the Xpress Video Course & Mock Test Package for Bank & Insurance Exams
If there are any suggestions/ errors in our PDFs Feel Free to contact us via this email: admin@exampundit.in
IBPS RRB Clerk Prelims – Ultra Practice Bundle PDF
Directions (31-35): Male employee in E=2800 * 3/5=1680
Female employees in E=2800 * 2/5=1120
Total number of male employees =1800 + 1500 + 1920 +
1200 + 1680=8100
Total number of female employees =1200 + 1000 + 1280
+ 800 + 1120=5400
Required Difference=8100 – 5400=2700
34) Answer: B
Required percentage=(2000 + 500)/(1000 + 1200) * 100
31) Answer: A
=114%
Female employees in IT department=60/100 * 1400=840
35) Answer: A
Female employees in HR department=1600 * 60/100=960
Total employees in IT department=1400 + 1500 + 2000 +
Total number of female employees in A=840 + 960=1800
500 + 1600=7000
32) Answer: B
Total number of employees=3000 + 2500 + 3200 + 2000
Total employees in IT department=1400 + 1500 + 2000 +
+ 2800=13500
500 + 1600=7000
Required percentage= [(7000/13500)] * 100=52%
Total employees in HR department=1600 + 1000 + 1200
36) Answer: C
+ 1500 + 1200=6500
The total crops produced by Rakesh
Required Difference=7000 – 6500=500
= > 640 + 920 + 340 + 900 + 300 = 3100
33) Answer: D
The total crops produced by Abinav
Male employee in A=3000 * 3/5=1800
= > 200 + 1050 + 280 + 840 + 820 = 3190
Female employees in A=3000 * 2/5=1200
Required % = (3100/3190)*100 = 97 %
Male employee in B=2500 * 3/5=1500
37) Answer: A
Female employees in B=2500 * 2/5=1000
The average crops produced by Ravi
Male employee in C=3200 * 3/5=1920
= > (750 + 660 + 250 + 560 + 650)/5
Female employees in C=3200 * 2/5=1280
= > 2870/5 = 574
Male employee in D=2000 * 3/5=1200
38) Answer: D
Female employees in D=2000 * 2/5=800

Page 636 of 722

Subscribe the Xpress Video Course & Mock Test Package for Bank & Insurance Exams
If there are any suggestions/ errors in our PDFs Feel Free to contact us via this email: admin@exampundit.in
IBPS RRB Clerk Prelims – Ultra Practice Bundle PDF
The average of Rye produced by all the given farmers 41) Answer: C
together Avg no. by C = (48+52+50+45+55+47)/6 = 495 in lakhs
= > (640 + 480 + 750 + 200 + 260)/5 42) Answer: A
= > 2330/5 No.TVs manufactured by E in 2004 = 56(in lakhs)
The average of Oats produced by all the given farmers By all in 2004 = 281(lakhs)
together % = 56/281*100 = 20%
= > (340 + 180 + 250 + 280 + 320)/5 43) Answer: A
= > 1370/5 in 2006 =46+30+55+50+54+50 = 285
Required difference = (2330/5) – (1370/5) In 2007= 52+38+47+40+51+52 = 280
= > 960/5 = 192 ratio = 280:285 = 56:57
39) Answer: B 44) Answer: E
Total mustard produced by all the farmers By A = 45+40+48+49+46+52=280
= > 920 + 580 + 660 + 1050 + 760 = 3970 By F = 49+45+48+44+50+52 =288
Total wheat produced by all the farmers %=280/288 *100 = 97%
= > 300 + 440 + 650 + 820 + 630 = 2840 45) Answer: C
Required ratio = 3970 : 2840 = 397 : 284 By A= 280
40) Answer: A B = 208
Total number of crops produced by Rakesh Ratio = (280/208)*100 = 134.6%
= > 640 + 920 + 340 + 900 + 300 = 3100 Directions (46-50):
Total number of crops produced by Manoj
= > 480 + 580 + 180 + 600 + 440 = 2280
Total number of crops produced by Ravi
= > 750 + 660 + 250 + 560 + 650 = 2870
Total number of crops produced by Abinav
= > 200 + 1050 + 280 + 840 + 820 = 3190
Total number of crops produced by Naveen 46) Answer: D
= > 260 + 760 + 320 + 720 + 630 = 2690 Required difference = (140+288)-(224+200)=428-424=4
Ravi has produced third lowest number of crops.
Page 637 of 722

Subscribe the Xpress Video Course & Mock Test Package for Bank & Insurance Exams
If there are any suggestions/ errors in our PDFs Feel Free to contact us via this email: admin@exampundit.in
IBPS RRB Clerk Prelims – Ultra Practice Bundle PDF
47) Answer: C Required difference = (140+128+280+288+180)-
Required total = 252+288+320+288+240=1388 (168+224+200+144+180)
48) Answer: B =>1016-916=100
Total students in school F = (180+200)*120/100=456 50) Answer: D
49) Answer: D

7). Bar Graph DI

Directions (1 – 5): Study the following information and answer the following questions:
The following Bar Graph Shows the Exports (in billion rupees) made by India to different countries A, B, C, D
and E for the year 2005 and 2006.

1. What is percentage growth rate in exports for the country C from year 2005 to the year 2006?
a) 25%
b) 33.33%

Page 638 of 722

Subscribe the Xpress Video Course & Mock Test Package for Bank & Insurance Exams
If there are any suggestions/ errors in our PDFs Feel Free to contact us via this email: admin@exampundit.in
IBPS RRB Clerk Prelims – Ultra Practice Bundle PDF
c) 50%
d) 66.67%
e) None of the above
2. What is the difference between the total export of countries A and E together for the year 2005 and total
exports of the countries B and D for the year 2006?
a) 5 billion rupees
b) 1 billion rupees
c) 4 billion rupees
d) 6 billion rupees
e) None of the above
3. What is the ratio of total exports made in year 2005 to total exports made in year 2006?
a) 125 : 127
b) 107 : 115
c) 123 : 105
d) 107 : 123
e) None of the above
4. If the exports for the country D in the next year 2007 increased up to the value of the exports in the year 2005.
What is the percentage growth rate?
a) 33.34%
b) 41.23%
c) 47.54%
d) 66.67%
e) Can’t be determined
5. Which of the countries have shown highest percentage growth rate over the year 2005 to 2006?
a) C
b) A
c) B
d) D
Page 639 of 722

Subscribe the Xpress Video Course & Mock Test Package for Bank & Insurance Exams
If there are any suggestions/ errors in our PDFs Feel Free to contact us via this email: admin@exampundit.in
IBPS RRB Clerk Prelims – Ultra Practice Bundle PDF
e) None of these
Directions (6 – 10): Answer the questions based on the information given below.
Given Bar graph shows the percentage distribution out of total smartphones market for the year 2002 and 2012.
Total number of smartphones sold in 2002: 24000
Total number of smartphones sold in 2012: 270000

70%
60%
60%

50%

40%
2002
30% 30%
30% 25% 2012
20%
20% 15%
10%
10% 6%
2% 2%
0%
Oppo vivo Nokia Apple Samsung

6. What is the total of number of smartphones sold by was 4000 Rs. Then calculate percentage increase or
Apple in the year 2002 and 2012 together? decrease in the revenue obtained in 2012 than 2002?
a) 42940 a) 225% decrease
b) 41940 b) 325% increase
c) 43338 c) 275% increase
d) 42169 d) 250% decrease
e) None of the above e) None of the above
7. If the price of average Nokia smartphones in 2002 8. Which of the following company has highest growth
was 2000 Rs. and the average price in the year 2012 rate in number of smartphones sold from the year 2002
to the year 2012?
Page 640 of 722

Subscribe the Xpress Video Course & Mock Test Package for Bank & Insurance Exams
If there are any suggestions/ errors in our PDFs Feel Free to contact us via this email: admin@exampundit.in
IBPS RRB Clerk Prelims – Ultra Practice Bundle PDF
a) Samsung c) 14.19
b) Apple d) 11.25
c) Oppo e) None of these
d) Vivo 10. What is the average number of smartphones sold
e) None of the above of company Samsung in the year 2002 and 2012?
9. How many times the total number of smartphones a) 51500
sold in the year 2012 has increased over the total b) 37350
number of smartphones sold in the year 2002? c) 17270
a) 13.38 d) 68370
b) 7.12 e) None of the above

Directions (11 – 15): Answer the questions based on the information.


Bar graph given below shows the goals scored and goals accepted by top 5 teams in a football league for a season.
35
30
30

25 24

20 20
20
16 Goals Scored
15
15 14
12 12 Goals Accepted

10
5
5

0
United FC City FC Athletic FC Golden FC Sporting Club

11. What is the ratio of total number of goals scored by a) 21 : 22


Athletic FC and city FC together to the total number b) 11 : 12
of goals scored by Sporting Club and united FC c) 6 : 7
together? d) 7 : 6
Page 641 of 722

Subscribe the Xpress Video Course & Mock Test Package for Bank & Insurance Exams
If there are any suggestions/ errors in our PDFs Feel Free to contact us via this email: admin@exampundit.in
IBPS RRB Clerk Prelims – Ultra Practice Bundle PDF
e) None of the above 14. What is the average number of goals accepted by
12.If a team gets 2 points for scoring a goal and 1 point these 5 teams together in the tournament?
is subtracted if the team accepts the goal. Which of the a) 22
following teams will have second highest number of b) 13.6
points? c) 7.8
a) United FC d) 12.8
b) City FC e) None of these
c) Athletic FC 15. Out of the total goals scored for team City FC, 25
d) Sporting Club goals were scored by forward players. Then what is the
e) None of these percentage contribution of forward players in the total
13. Which of the following teams have accepted goals scored?
minimum number of goals in the whole tournament? a) 33.34%
a) Athletic FC b) 42.33%
b) City FC c) 83.33%
c) Sporting Club d) None of these
d) Golden FC. e) Can’t be determined
e) None of these

Directions (16-20): Answer the questions based on the information given below.
Bar graph given below shows the average of 5 batsmen throughout their career. Average was calculated as
follows = (Total number of runs scored / Number of matches played) and everyone has played different number
of matches.

Page 642 of 722

Subscribe the Xpress Video Course & Mock Test Package for Bank & Insurance Exams
If there are any suggestions/ errors in our PDFs Feel Free to contact us via this email: admin@exampundit.in
IBPS RRB Clerk Prelims – Ultra Practice Bundle PDF

Batting Average
54.2

34.1

20 24.5
12.8

Rahul
Rohit
Rajesh
Ramesh
Roshan

16. If Ramesh has played a total 280 matches in his b) 166.67%


career then what is the total number of runs scored by c) 221.22%
him? d) 133.33%
a) 4600 e) Can’t be determined
b) 5680 19. If there in another batsman with total runs 7800
c) 5820 scored in 390 matches in his career. How many of
d)5600 above-mentioned batsmen have batting average more
e) None of the above than him?
17. What are the total runs scored by all the five a) 3
batsmen together in their career? b) 2
a) 195 c) 4
b) 150 d) 1
c) 155 e) Cannot be determined.
d) Cannot be determined. 20. If everyone has played equal number of matches in
e) None of these his career then what will be the overall batting average
18. The batting average of Rohit is what percent of for all five of them?
Roshan? a) 28.72
a) 177% b) 29.12
Page 643 of 722

Subscribe the Xpress Video Course & Mock Test Package for Bank & Insurance Exams
If there are any suggestions/ errors in our PDFs Feel Free to contact us via this email: admin@exampundit.in
IBPS RRB Clerk Prelims – Ultra Practice Bundle PDF
c) 17.34 e) None of the above
d) 30.24
Directions (21 – 25): Study the information carefully and answer the question asked below.
Bar graph given below shows the sale of different car models in the Indian market for Q1 of 2020. Figure is
shown in thousands.

Number of Cars Sold in Q1 2020


(in thousands)

120
108
100
84 75
80
60
40 21 25
20
0
Sedan Coupe SUV hatchback MUV

21. If in Q2 of 2020 the sale of sedan was reported to c) 15.45%


be 147 thousands units. What is the percentage growth d) 19.52%
rate? e) None of the above
a) 75% 23. If the ratio of the sale of number of units of Sedan
b) 18% to the sale of number of units for Coupe?
c) 80% a) 2 : 3
d) 10% b) 3 : 2
e) 120% c) 1 : 4
22. By what percent the sale of number of units of d) 4 : 1
Coupe and SUV together is more or less than Sale of e) None of
number of units of MUV and Sedan in Q1? these
a) 15.35%
b) 18.34%
Page 644 of 722

Subscribe the Xpress Video Course & Mock Test Package for Bank & Insurance Exams
If there are any suggestions/ errors in our PDFs Feel Free to contact us via this email: admin@exampundit.in
IBPS RRB Clerk Prelims – Ultra Practice Bundle PDF
24. How many types of cars have above average sale in 25. Type Sedan, MUV and Coupe gave how much
the Q1? percent of sale out of total sale made in Q1?
a) 1 a) 41.53%
b) 2 b) 61.64%
c) 3 c) 23.33%
d) 4 d) 66.67%
e) None of these e) 33.34%

Directions (26 – 30): Read the following information carefully and answer the following questions carefully.
Bar graph given below shows both theoretical and practical reading obtained while conducting a scientific
experiment. Total 5 rounds of reading were taken. Study the information carefully and answer the questions.
134
132.4
132
132
130.12
129.86
130

128
127 127.2
126.5 Theorotical
126 Practical
125 125
124.5
124

122

120
Reading 1 Reading 2 Reading 3 Reading 4 Reading 5

26. What is the average value of reading obtained b) 125.50


when readings were taken practically? c) 118.50
a) 128.09 d) 117.14

Page 645 of 722

Subscribe the Xpress Video Course & Mock Test Package for Bank & Insurance Exams
If there are any suggestions/ errors in our PDFs Feel Free to contact us via this email: admin@exampundit.in
IBPS RRB Clerk Prelims – Ultra Practice Bundle PDF
e) 126.58 29. If the practical value obtained for Reading 3 was
27. Which of the following reading gave most mistakenly entered as 126.5 instead of 124.5. What is
accuracy? Most accurate reading means least now the difference between theoretical and practical
difference between theoretically calculated values to value?
the practically obtained value. a) 2.50
a) Reading 1 b) 0.5
b) Reading 4 c) 0.70
c) Reading 3 d) 0.34
d) Reading 5 e) None of the above.
e) None of these 30. What is the ratio of difference obtained in the
28. What is the difference between practical readings reading 1 and 2 when calculated theoretically to the
taken for last two readings? difference obtained in Reading 1 and 3 when readings
a) 3.12 were taken practically?
b) 2.54 a) 2 : 1
c) 1.17 b) 2 : 5
d) 4.54 c) 1 : 1
e) None of the above d) Cannot be determined
e) None of these

Directions (31 – 35): Read the following information carefully and answer the following questions given below.
Following Bar graph shows the average weight of watermelons produced in 5 different farms P, Q, R, S, T and
number of watermelons produced (in thousands).

Page 646 of 722

Subscribe the Xpress Video Course & Mock Test Package for Bank & Insurance Exams
If there are any suggestions/ errors in our PDFs Feel Free to contact us via this email: admin@exampundit.in
IBPS RRB Clerk Prelims – Ultra Practice Bundle PDF

9 8.4
8 7.5
7.2
7

6
5 5
5 4.5 Average Weight of
Watermelon Produced
4
3 3 Number of Watermelons
3 produced (in thousands)
2.1
2
1.2
1

0
P Q R S T

31. What is the difference between the total weight of a) 312400


watermelons produced in farm P and Q? b) 216000
a) 12700 c) 123000
b) 30000 d) 126000
c) 15200 e) None of these
d) 13500 34. If the selling price of watermelons is same for every
e) None of these farm 8 Rs per unit. Then which farm was most
32. By what percentage the production (number of profitable in terms of revenue for the farmers?
units produced) of watermelons in farm R is less than a) P
production in farm P? b) Q
a) 66.67% c) R
b) 25% d) S
c) 33.34% e) None of these
d) 14.33% 35. If the average weight of watermelons is improved
e) None of these to 3.6 Kg in farm T for the next season, what is the
33. What is total revenue obtained by farm S if the percentage increase in the revenue obtained from farm
watermelons are sold 14 Rs. per kg. from farm S? T?
Page 647 of 722

Subscribe the Xpress Video Course & Mock Test Package for Bank & Insurance Exams
If there are any suggestions/ errors in our PDFs Feel Free to contact us via this email: admin@exampundit.in
IBPS RRB Clerk Prelims – Ultra Practice Bundle PDF
a) 11% d) 16.67%
b) 15% e) Cannot be determined.
c) 13%

Directions (36 – 40): Read the data given below and answer the following questions.
Below given Bar graph shows the percentage composition of units sold by 4 different companies in 3 segments of
consumer electronics.
Total number of AC sold: 6000
Total number of Fan sold: 22000
Total number of Cooler sold: 18000
100%

90%

80% 40%

70% 55%

60% 75%
Godrej

50% Samsung
30% Hitachi
40%
25% Voltas
30%

20% 20%
5%
10% 25%
15%
10%
0%
AC Fan Cooler

36. What is the number of units sold by Hitachi in the d) 900


air cooler and fan segment together? e) None of these
a) 500 37. By what percent number of units sold by Godrej is
b) 600 more than number of units sold by Samsung in all
c) 700 types of consumer electronics?

Page 648 of 722

Subscribe the Xpress Video Course & Mock Test Package for Bank & Insurance Exams
If there are any suggestions/ errors in our PDFs Feel Free to contact us via this email: admin@exampundit.in
IBPS RRB Clerk Prelims – Ultra Practice Bundle PDF
a) 42.90% a) 3400
b) 90.42% b) 2200
c) 92.45% c) 2500
d) 88.87% d) Cannot be determined
e) None of these e) None of these
38. The average sale of fans is what percent of the 40. What is the difference between average sales of
average sale of coolers for all companies together? Samsung in all 3 products and average sales of Voltas
a) 250% in all these 3 products?
b) 312% a) 2183.34
c) 233.34% b) 2283.34
d) 244.45% c) 2383.34
e) None of the above d) 1283.34
39. If the total number of units of fans produced for e) None of these
the next year is increased by 25%, then what is the sale
in number of units for the Samsung Company?

Direction (41-45): Read the data given below and answer the following questions.
The Bar below shows the marks scored by 6 different students in an exam. The exam was conducted out of 100
marks.

Page 649 of 722

Subscribe the Xpress Video Course & Mock Test Package for Bank & Insurance Exams
If there are any suggestions/ errors in our PDFs Feel Free to contact us via this email: admin@exampundit.in
IBPS RRB Clerk Prelims – Ultra Practice Bundle PDF

Marks Obtained by Students


96
100 90
90
72 75
80 70
70
64
60
50
40
30
20
10
0
Smita Rhea Sanjana Radhika Sneha Balika

41. What is the average score obtained by students in 43. If the marks of Radhika were entered 50
the exams? mistakenly in the report card then by what percent her
a) 78.18 score was reduced?
b) 77.84 a) 31.33%
c) 72.54 b) 22.5%
d) Cannot be determined c) 33.34%
e) None of these d) 66.67%
42. Who among the above-mentioned students got e) 25%
highest percentage of marks in the exam? 44. If there is another student Ritu whose score is 40 is
a) Sanjana considered then what is the difference is observed
b) Rhea between average marks obtained?
c) Radhika a) 2.50
d) Balika b) 5.41
e) None of these c) 3.54
d) cannot be determined

Page 650 of 722

Subscribe the Xpress Video Course & Mock Test Package for Bank & Insurance Exams
If there are any suggestions/ errors in our PDFs Feel Free to contact us via this email: admin@exampundit.in
IBPS RRB Clerk Prelims – Ultra Practice Bundle PDF
e) None of these b) 9 : 8
45. What is the ratio of marks obtained by Smita to the c) 5 : 3
marks obtained by Balika? d) 8 : 9
a) 6 : 7 e) None of these

Directions (46 – 50): Read the data given below and answer the following questions.
The Bar Graph shows that Population (in thousands) of 5 villages in 2018 and Percentage growth rate (given in
%) of the year 2019.

35 32 32
30 30
30
25 25
25
20 20
Population in thousands
20
15 15
15 Percentage growth rate
(given in %)
10

0
M N O P Q

46. Which of the following villages have shown highest a) 1


growth rate in the population for the year 2019? b) 2
a) M c) 3
b) N d) 4
c) O e) 5
d) P 48. What is the ratio of population of village P in 2019
e) Q to the population of the village M in the year 2018?
47. How many of the following villages will have a) 1 : 2
population more than 38,000 in the year 2019? b) 3 : 1
Page 651 of 722

Subscribe the Xpress Video Course & Mock Test Package for Bank & Insurance Exams
If there are any suggestions/ errors in our PDFs Feel Free to contact us via this email: admin@exampundit.in
IBPS RRB Clerk Prelims – Ultra Practice Bundle PDF
c) 2 : 1 d) O
d) 1 : 3 e) Q
e) None of these 50. What is the average population of villages M, O
49. Which of the following villages have shown highest and Q together in 2019?
growth in terms of number of people from year 2018 a) 72000
to 2019? b) 74000
a) P c) 80000
b) M d) 90000
c) N e) None of these

7). Bar Graph DI - Solutions with Explanation


Solution(1-5): Required ratio = 107 : 123
1. Answer: C) 4. Answer: A)
Exports for the country C from year 2005 = 34 billion Exports for the country D in the next year 2007 = 24
Exports for the country C from year 2006 = 51 billion billions
Percentage growth rate = (51 – 34) / 34 * 100 = 50% What is the percentage growth rate = (24-18) / 18 * 100 =
2. Answer: B) 33.34%
Total export of countries A and E together for the year 5. Answer: A)
2005 = 12 + 20 = 32 billion Percentage growth rate for country A = (15-12) / 12 * 100
Total exports of the countries B and D for the year 2006 = = 25%
15 + 18 = 33 billion Percentage growth rate for country B= (17-15)/ 15*100 =
Required difference = 33 -32 = 1 billion 13.33%
3. Answer: D) Percentage growth rate for country C = (51-34) /34 * 100
Total exports made in year 2005 = 12 + 17 + 34 + 24 + 20 = 50%
= 107 billions Percentage growth rate for country D = (24-18) /18 *100
Total exports made in year 2006 = 15 + 15 + 51 + 18 + 24 = 33.33 %
= 123 billions

Page 652 of 722

Subscribe the Xpress Video Course & Mock Test Package for Bank & Insurance Exams
If there are any suggestions/ errors in our PDFs Feel Free to contact us via this email: admin@exampundit.in
IBPS RRB Clerk Prelims – Ultra Practice Bundle PDF
Percentage growth rate for country E = (24-20) / 20 * 100 We have to calculate highest growth rate over 2002.
= 20% Only Oppo, Vivo and apple has shown growth.
Highest growth rate = C - 50% Highest is clearly from the graph for Vivo 2% of 24000 to
Solution(6-10). 30% of 270000.
6. Answer: B) 9. Answer: D)
Total of number of smartphones sold by Apple in the year Total number of smartphones sold in the year 2012 =
2002 = 6 % of 24000 270000
= 6/100 * 24000 = 1440 The total number of smartphones sold in the year 2002 =
Total of number of smartphones sold by Apple in the year 24000
2012 = 15% of 270000 = 11.25
15/100 * 270000 = 40500 10. Answer: B)
Total of number of smartphones sold by Apple in the year Average number of smartphones sold of company
2002 and 2012 together = 41940 Samsung in the year 2002 and 2012 = (30% of 24000 +
7. Answer: C) 25% of 270000) / 2
Price of average Nokia smartphones in 2002 was 2000 Rs. = (7200 + 67500) / 2 = 37350
Revenue obtained in 2002 = number of units sold *
average price in 2002 Solution(11-15).
= 2000 * 60 % of 24000 11. Answer: A)
= 288 Lacs Total number of goals scored by Athletic FC and city FC
Average price in the year 2012 was 4000 Rs. together = 12 +30 = 42
Revenue obtained in 2012 = number of units sold * Total number of goals scored by Sporting Club and united
average price in 2012 FC together = 20+ 24 = 44
= 4000 * 10% of 270000 Required ratio = 42 : 44 = 21 : 22
= 1080 Lacs 12. Answer: D)
Percentage increase or decrease in the revenue obtained = Team gets 2 points for scoring a goal and 1 point is
((1080 – 288 ) /288) * 100 subtracted if the team accepts the goal.
= 275% Points for united FC = 2 x 24 – 20 = 28
8. Answer: D) Points for City FC = 2 x 30 – 15 = 45
Page 653 of 722

Subscribe the Xpress Video Course & Mock Test Package for Bank & Insurance Exams
If there are any suggestions/ errors in our PDFs Feel Free to contact us via this email: admin@exampundit.in
IBPS RRB Clerk Prelims – Ultra Practice Bundle PDF
Points for Athletic FC = 2 x 12 – 12 = 12 Average of new batsman = total runs scored by him /
Points for Golden FC = 2 x 14 – 16 = 12 number of matches played by him
Points for Sporting FC = 2 x 20 -5 = 35 = 7800 / 390 = 20
Second highest number of points = Sporting FC Batsmen having average more than him = 3
13. Answer: C) 20. Answer: B)
Clearly Sporting FC have accepted least number of goals If everyone has played equal number of matches in their
in the tournament = 5 career then there is no role of number of matches played
14. Answer: B) in the calculation of average.
Average number of goals accepted by these 5 teams = (34.1 + 54.2 + 12.8 + 20 + 24.5) / 5
together in the tournament = (20+15+12+16+5) / 5 = 13.6 = 29.12
15. Answer: C)
Total goals scored for team City FC = 30 Solution(21-25).
Percentage contribution of forward players in the total 21. Answer: A)
goals scored = 25 / 30 * 100 = 83.33% Q2 of 2020 the sale of sedan was reported to be 147000.
Solution(16-20). Q1 sale = 84000
16. Answer: D) Percentage growth rate = (147000 – 84000) / 84000 * 100
Ramesh has played a total 280 matches. =
Total runs scored = average x number of matches played = 75%
= 280 x 20 = 5600 22. Answer: B)
17. Answer: D) Sale of number of units of Coupe and SUV together = 21
This answer we cannot determine since we don’t know the + 108 = 129 thousands
number of matches played by the individual batsman. Sale of number of units of MUV and Sedan = 84 + 25 =
18. Answer: C) 109 thousands
Batting average of Rohit = 54.2 Percentage more = (129- 109) / 109 * 100 = 18.34%
Batting average of Roshan = 24.5 23. Answer: D)
Percentage = 54.2 / 24.5 * 100 = 221.22% Sale of number of units of Sedan = 84 thousands
19. Answer: A) Sale of number of units for Coupe = 21
Ratio = 84 : 21 = 4 : 1
Page 654 of 722

Subscribe the Xpress Video Course & Mock Test Package for Bank & Insurance Exams
If there are any suggestions/ errors in our PDFs Feel Free to contact us via this email: admin@exampundit.in
IBPS RRB Clerk Prelims – Ultra Practice Bundle PDF
24. Answer: C) 29. Answer: B)
Average sale in the Q1 = ( 84+ 21 +108 + 75 + 25 ) / 5 = Practical value obtained for Reading 3 was mistakenly
62.6 entered as 126.5 instead of 124.5
Number of car types having average sale more than 62.6 New Difference between theoretical and practical value =
thousands = 3 125 – 124.5 = 0.5
25. Answer: A) 30. Answer: C)
Sedan, MUV and Coupe sale together = 84 +21 +25 = 130 Difference obtained in the reading 1 and 2 when
thousands calculated theoretically = 127 – 125 = 2
Total Sale = 84 + 25 + 21 + 108 + 75 = 313 thousands Difference obtained in Reading 1 and 3 when readings
Required percentage = 130 / 313 * 100 = 41.53% were taken practically = 126.5 – 124.5 = 2
Required ratio = 2 : 2 = 1 : 1
Solution(26-30).
26. Answer: A) Solution(31-35).
Total of the reading when taken practically = 124.5 + 31. Answer: D)
127.2 + 126.5 + 129.86 + 132.4 = 640.46 Total weight of watermelons produced in farm P =
Average = 640.46 / 5 = 128.09 average weight x number of units produced from P
27. Answer: E) = 4.5 x 5000 = 22500 kg.
Let’s calculate, Difference between theoretically Total weight of watermelons produced in farm Q =
calculated values to the practically obtained value. average weight x number of units produced from Q
Reading 1 = 125 – 124.5 = 0.5 = 5 x 7200 = 36000
Reading 2 = 127.2 – 127 = 0.2 Required difference = 36000 – 22500 = 13500
Reading 3 = 126.5 – 125 = 1.5 32. Answer: E)
Reading 4 = 130.12 – 129.86 = 0.26 Production (number of units produced) of watermelons in
Reading 5 = 132.4 – 132 = 0.4 farm R = 1200
Most accurate reading = Reading 2 Production (number of units produced) of watermelons in
28. Answer: B) farm P = 5000
Difference between practical readings taken for last two Percentage less = (5000-1200) / 5000 * 100 = 76%
readings = 132.4 – 129.86 = 2.54 33. Answer: D)
Page 655 of 722

Subscribe the Xpress Video Course & Mock Test Package for Bank & Insurance Exams
If there are any suggestions/ errors in our PDFs Feel Free to contact us via this email: admin@exampundit.in
IBPS RRB Clerk Prelims – Ultra Practice Bundle PDF
Revenue obtained by farm S if the watermelons are sold = 6000 * 40/100 + 22000 * 75/100 + 18000 * 55/100
14 Rs. per kg. = 28800
= average weight x number of units produced x rate Units sold by Samsung = Number of AC sold + Number
= 3 * 3000 * 14 = 126000 of Fan Sold + Number of Cooler sold
34. Answer: E) = 6000 *30/100 + 22000 * 25/100 + 18000 * 25/100
Selling price of watermelons is same for every farm 8 Rs = 11800
per unit. Percentage more = (28800 – 11800) / 11800 * 100 =
Since the rate is same revenue will depend on the number 144.07%
of units produced. 38. Answer: D)
The farm producing most number of units of watermelons Average sale of fans = Total sale / number of companies
will produce most revenue and will be most profitable. sold Fans
That is farm T = 84000 units = 22000 / 2 = 11000
35. Answer: E) Average sale of coolers =companies sold Coolers sold
Average weight of watermelons is improved to 3.6 Kg in Coolers
farm T. = 18000 / 4 = 4500
Percentage increase in the revenue obtained cannot be Required percentage = 11000 / 4500 *100 =
obtained since we don’t know the number of units 244.45%
produced in next season for farm T.
39. Answer: D)
Solution(36-40). Total number of units of fans produced for the next year
36. Answer: D) is increased by 25%
Number of units sold by Hitachi in the cooler = 18000 * 5 Sale in number of units for the Samsung Company cannot
/ 100 = 900 be determined since we don’t know the percentage
Number of units sold by Hitachi in the fan segment = 0 composition in the total sale.
Total sale = 900 + 0 = 900 40. Answer: B)
37. Answer: E) Average sales of Samsung in all 3 products = Total sale of
Units sold by Godrej = Number of AC sold + Number of Samsung / Number of segments
Fan Sold + Number of Cooler sold = (6000 * 30/100 + 22000 * 25/100 + 18000 *25/100) / 3
Page 656 of 722

Subscribe the Xpress Video Course & Mock Test Package for Bank & Insurance Exams
If there are any suggestions/ errors in our PDFs Feel Free to contact us via this email: admin@exampundit.in
IBPS RRB Clerk Prelims – Ultra Practice Bundle PDF
= 11800 / 3 = 3933.34 Difference is observed between average marks obtained =
Average sales of Voltas in all these 3 products = Total 77.84 – 72.42 = 5.41
sale of Voltas / number of segments 45. Answer: B)
= (6000 * 10/100 + 18000 *15/100) / 2 Marks obtained by Smita = 72
= 3300 / 2 = 1650 Marks obtained by Balika = 64
Difference = 3933.34 – 1650 = 2283.34 Ratio = 72 : 64 = 9 : 8
Solution(41-45).
41. Answer: B) Solution(46-50). Common Explanation
Average score obtained by students in the exams = (72 + Population of villages in 2019
90 + 96+ 75 + 70 + 64) / 6 M = Population in 2018 * (100 + growth rate) / 100
= 467 / 6 = 77.84 = 20000*115/100 = 23000
42. Answer: A) N = 30000*130/100 = 39000
Highest percentage of marks in the exam = Students O = 25000*132/100 = 33000
scoring highest marks out of 100 P = 32000*125/100 = 40000
Who is Sanjana with 96 marks. Q = 15000*120/100 = 18000
43. Answer: C) 46. Answer: C)
Marks of Radhika were entered 50 mistakenly in the Village O has shown highest growth rate in the population
report card. for the year 2019 that is 32%.
Actual marks = 75 47. Answer: B)
Percentage reduction = (75-50) / 75 * 100 = 33.34% Population of villages in 2019
44. Answer: B) M = Population in 2018 * (100 + growth rate) / 100
Average score obtained by students in the exams = (72 + = 20000*115/100 = 23000
90 + 96+ 75 + 70 + 64) / 6 N = 30000*130/100 = 39000
= 467 / 6 = 77.84 O = 25000*132/100 = 33000
Ritu whose score is 40 is considered P = 32000*125/100 = 40000
New average score obtained by students in the exams = Q = 15000*120/100 = 18000
(72 + 90 + 96+ 75 + 70 + 64 + 40) / 7 Only 2 villages will have population more than 38000.
= 507 / 7 = 72.42 48. Answer: C)
Page 657 of 722

Subscribe the Xpress Video Course & Mock Test Package for Bank & Insurance Exams
If there are any suggestions/ errors in our PDFs Feel Free to contact us via this email: admin@exampundit.in
IBPS RRB Clerk Prelims – Ultra Practice Bundle PDF
Population of village P in 2019 = 40000 Growth in Village O = 33000 – 25000 = 8000
Population of the village M in the year 2018 = 20000 Growth in Village P = 40000 – 32000 = 8000
Ratio = 40000: 20000 = 2 : 1 Growth in Village Q = 18000 – 15000 = 3000
49. Answer: C) Highest growth is observed for village N.
Growth for Villages M (number of people) 50. Answer: B)
Population in 2019 – Population in 2018 Population of villages M, O and Q together in 2019 =
= 23000 – 20000 = 3000 23000 + 33000 + 18000 = 74000
Growth in Village N = 39000 – 30000 = 9000

8). DI Based on Line Graph

Directions (1-05): Study the following information carefully and answer the questions given below.
The given line graph shows the expenditure of the Anu, Banu, Saran and Divya in three different years.

1) What is the difference between the expenditure of B.Rs.600


Divya in 2016 and the expenditure of Banu in 2017? C.Rs.1000
A.Rs.800 D.Rs.900
Page 658 of 722

Subscribe the Xpress Video Course & Mock Test Package for Bank & Insurance Exams
If there are any suggestions/ errors in our PDFs Feel Free to contact us via this email: admin@exampundit.in
IBPS RRB Clerk Prelims – Ultra Practice Bundle PDF
E.Rs.1100 4) What is the ratio of the total expenditure of Banu to
2) What is the total expenditure in 2015 of all the Saran in all the years together?
persons together? A.38:31
A.Rs.8800 B.39:32
B.Rs.9200 C.40:33
C.Rs.9600 D.37:30
D.Rs.9800 E.None of these
E.None of these 5) The total expenditure of Divya in all the years
3) If the total income of Anu in 2016 is Rs.4000 and together is approximately what percent of the total
then what is the savings of Anu in 2016? expenditure of Anu in all the years together?
A.Rs.1800 A.81%
B.Rs.2000 B.83%
C.Rs.2400 C.85%
D.Rs.2600 D.87%
E.None of these E. None of these

Directions (6-10): The given line graph shows the expenditure and savings of the company in six different years
(in lakhs).

Page 659 of 722

Subscribe the Xpress Video Course & Mock Test Package for Bank & Insurance Exams
If there are any suggestions/ errors in our PDFs Feel Free to contact us via this email: admin@exampundit.in
IBPS RRB Clerk Prelims – Ultra Practice Bundle PDF
6) What is the difference between the company income D.89000
in 2014 and 2016? E.78000
A.80000 9) In which of the following years the income of the
B.90000 company is same?
C.120000 A.2014 and 2015
D. 100000 B.2014, 2016 and 2017
E.None of these C. None
7) What is ratio of the incomes in 2015 to the sum of D.2015, 2016 and 2017
the savings in 2013 and 2017 together? E.2016 and 2014
A.2:1 10) If the income of the company in 2019 is 150% of
B.5:2 the income in 2013 and the savings of the company in
C.7:4 2019 is 80% of the savings in 2018, then what is the
D.4:3 expenditure of the company in 2019?
E.None of these A.2.27 lakh
8) What is approximate average of the savings in all B. 3.25 lakh
the years together? C.2.55 lakh
A.80000 D.3.35 lakh
B.83000 E.None of these
C.86000
Directions (11-15): The given line graph shows the number of apple and the number of apple sold in five different
shops.

Page 660 of 722

Subscribe the Xpress Video Course & Mock Test Package for Bank & Insurance Exams
If there are any suggestions/ errors in our PDFs Feel Free to contact us via this email: admin@exampundit.in
IBPS RRB Clerk Prelims – Ultra Practice Bundle PDF

11) What is the ratio of the number selling apples from 13) What is the average number of unsold apples in all
A and B together to the number of selling apples from the shops together?
D and E together? A.30
A.21:23 B.40
B.26:27 C.25
C.29:30 D.45
D.25:29 E.None of these
E.None of these 14) What is the ratio of the number of apples from
12) The number of sold apples from C is shop E to number of apples sold from B?
approximately what percent of the number of apples A. 2:1
from D? B.3:2
A.20% C.4:3
B.30% D.5:6
C.40% E.None of these
D.50% 15) What is the total number of apples from all the
E.None of these shops together?
Page 661 of 722

Subscribe the Xpress Video Course & Mock Test Package for Bank & Insurance Exams
If there are any suggestions/ errors in our PDFs Feel Free to contact us via this email: admin@exampundit.in
IBPS RRB Clerk Prelims – Ultra Practice Bundle PDF
A.1200 D.1800
B.1400 E.None of these
C.1500

Directions (16-20): Ratio of Exports to Imports (in terms of money in Rs. crores) of Two Companies Over the
Years

16) In how many of the given years were the exports B.1.30
more than the imports for Company B? C.0.71
A.5 D.2.20
B.4 E.1.15
C.2 18) If the exports of Company A in 1998 were Rs. 237
D.3 crores, what was the amount of imports in that year
E.1 (crores)?
17) If the imports of Company B in 1999 were A.243
increased by 40 percent, what would be the ratio of B.189
exports to the increased imports? C.281
A.1.25 D.316

Page 662 of 722

Subscribe the Xpress Video Course & Mock Test Package for Bank & Insurance Exams
If there are any suggestions/ errors in our PDFs Feel Free to contact us via this email: admin@exampundit.in
IBPS RRB Clerk Prelims – Ultra Practice Bundle PDF
E.320 E.880
19) In 1995, the export of Company A was double that 20) In which year(s) was the difference between
of Company B. If the imports of Company B during imports and exports of Company B the maximum?
the year was Rs. 180 crores, what was the approximate A.2000
amount of imports of Company A during that year? B.1996
A.840 C.1997
B.690 D.Cannot be determined
C.820 E. None of these
D.730

Directions (21-25): Study the following graph carefully and answer the given questions:
The line graph shows the number of cars manufactured in six different months in two different companies

21) What is the ratio of 120% of the number of cars D.153:86


manufactured by company A on January and April E.27:89
together to the 80% of the number of cars sold by 22) Number of cars manufactured by company B on
company B in march and June together? March and April together is what percentage
A.154:83 more/less than the number of cars manufactured by
B.81:127 company A on January and June together?
C.23:49 A.6.25%
Page 663 of 722

Subscribe the Xpress Video Course & Mock Test Package for Bank & Insurance Exams
If there are any suggestions/ errors in our PDFs Feel Free to contact us via this email: admin@exampundit.in
IBPS RRB Clerk Prelims – Ultra Practice Bundle PDF
B.7.5% 24) What is the difference between the total number of
C.10.5% cars manufactured by company A and the total
D.5% number of cars manufactured by company B?
E.12.5% A.590
23) If the number of cars manufactured by company B B.640
on July is 120% of the average number of cars C.630
manufactured by company A for all the months D.710
together (Exclude February and march ), and then E.None of these
find the number of cars manufactured by B on July? 25) In which of the following month the difference
A.724 between the number of cars manufactured by
B.684 company A and B is maximum?
C.586 A.January
D.498 B.February
E.386 C.March
D.April
E.June

Directions (26-30): Read the following Graph and answer the questions carefully. The following diagram shows
different ores exported by various companies in 2017.

Page 664 of 722

Subscribe the Xpress Video Course & Mock Test Package for Bank & Insurance Exams
If there are any suggestions/ errors in our PDFs Feel Free to contact us via this email: admin@exampundit.in
IBPS RRB Clerk Prelims – Ultra Practice Bundle PDF

26) Avg tones of total gold exported by all companies 28) Which of the following companies are exported the
is what percentage of all ores exported by Company C second maximum tones of ores in the year 2017?
in 2017? A.C
A.25% B.D
B.33.33% C.A & B
C.16.67% D.B & C
D.37.5% E.None of these
E.None of these 29) In 2018, the quantity of gold ore exported is 50%
27) What percentage of increase or decrease in total more than the total quantity of gold ore exported by
Quantity of Gold ores exported than to the total all companies in 2017. Similarly silver exported in 2018
quantity of coal ores exported? is 20% greater than the total quantity of silver ore
A.10.2% increase exported by all companies in 2017. Find the ratio of
B.9.4% increase gold ore exported to the silver ore?
C.9.4% decrease A.5:6
D.10.2 % decrease B.6:5
E.None of these C.7:6
D.6:7
Page 665 of 722

Subscribe the Xpress Video Course & Mock Test Package for Bank & Insurance Exams
If there are any suggestions/ errors in our PDFs Feel Free to contact us via this email: admin@exampundit.in
IBPS RRB Clerk Prelims – Ultra Practice Bundle PDF
E.None of these B.75%
30) Total Quantity of ores exported by company C is C.90%
what percentage of total ores exported by company D? D.70%
A.80% E.None of these

Direction (31-35): Study The Following Graph Carefully And Answer The Questions Given Below

31) For company A, What is the percent decrease in B.115%


Production from 2004 to 2005? C.159%
A.65% D.125%
B.25% E.None of these
C.75% 33) For company A in Which year is the percentage
D.45% increase/decrease in the production from the previous
E.None of these year the highest?
32) In 2011, the production of company B is A.2006
approximately what % of that in 2010? B.2005
A.157% C.2007
Page 666 of 722

Subscribe the Xpress Video Course & Mock Test Package for Bank & Insurance Exams
If there are any suggestions/ errors in our PDFs Feel Free to contact us via this email: admin@exampundit.in
IBPS RRB Clerk Prelims – Ultra Practice Bundle PDF
D.2010 E. None of these
E.None of these 35) Which of the following is the closest average
34) What is the difference in the total production of the production of company B for the given years?
two companies for the given years? A.2.87t
A.2700000 B.3.87
B.1500000 C.4.58
C.2000000 D.5.55
D.3000000 E.None of these

Directions (36-40): Study the following graph carefully to answer the questions given below

36) By what % approximately is the total investment D.20%


of Y in schemes B,D and E together less than that E.23%
made by X in the same schemes? 37) What is the respective ratio between the total
A.30% investment of Y in schemes C and D together and that
B.10% of X in the schemes E and F together?
C.15% A.21:28
Page 667 of 722

Subscribe the Xpress Video Course & Mock Test Package for Bank & Insurance Exams
If there are any suggestions/ errors in our PDFs Feel Free to contact us via this email: admin@exampundit.in
IBPS RRB Clerk Prelims – Ultra Practice Bundle PDF
B.10:13 A.Rs.25000
C.11:17 B. Rs.55000
D.10:11 C.Rs.44000
E.9:13 D.Rs.51000
38) The investment by X in scheme F in comparison to E.Rs.60000
scheme B is more by what %? 40) By what % approximately is the amount
A.100% invested by X in scheme A and C together more than
B.110% that invested by Y in the schemes B and E together?
C.90% A.55%
D.88% B.62%
E.75% C.70%
39) What was the average investment of Y in the D.58%
schemes taken into consideration? E.50%

Directions (41-45): Study the following graph carefully and answer the questions that follow.
Number of vehicles manufactured (in thousand) by two companies during six years

Page 668 of 722

Subscribe the Xpress Video Course & Mock Test Package for Bank & Insurance Exams
If there are any suggestions/ errors in our PDFs Feel Free to contact us via this email: admin@exampundit.in
IBPS RRB Clerk Prelims – Ultra Practice Bundle PDF

41) What is the difference between the total number of D.60


vehicles manufactured by company P in 2010, 2011 E.130
and 2013 together and company Q in 2011, 2012 and 43) What is the percentage decrease in the same
2013 together? (in thousand) number of vehicles manufactured by company Q from
A.120 2010 to 2011?
B.210 A.40
C.180 B.35
D.270 C.20
E.190 D.25
42) What is the average of the number of vehicles E.33
manufactured by company Q over six years? (in 44) Out of the number of vehicles manufactured by
thousand) company P in 2012, 15000 pieces were found to be
A.170 defective and out of those manufactured by company
B.150 Q in 2013, 10000 pieces were found to bedefective.
C.90 What is the ratio of non-defective vehicles

Page 669 of 722

Subscribe the Xpress Video Course & Mock Test Package for Bank & Insurance Exams
If there are any suggestions/ errors in our PDFs Feel Free to contact us via this email: admin@exampundit.in
IBPS RRB Clerk Prelims – Ultra Practice Bundle PDF
manufactured by company P in 2012 to that by Q in to those manufactured by the same company in the
2013? 2009.what is the total number of vehicles
A.9:8 manufactured by the same company in 2014?
B.11:4 A.247 thousand
C.3:8 B.297 thousand
D.5:8 C.211 thousand
E.7:4 D.310 thousand
45) In 2014 there was an increase of 30% in the number E.283 thousand
of vehicles manufactured by company P as compared

Directions (46-50): Study the following information carefully and answer the given questions
The following line graph shows the number of TVs sold by two companies P and Q in 6 different days of a certain
week.

46) Find the average number of TVs sold by company B.360


P on all the given days together? C.400
A.380 D.420

Page 670 of 722

Subscribe the Xpress Video Course & Mock Test Package for Bank & Insurance Exams
If there are any suggestions/ errors in our PDFs Feel Free to contact us via this email: admin@exampundit.in
IBPS RRB Clerk Prelims – Ultra Practice Bundle PDF
E.None of these E.None of these
47) Find the ratio between the total number of TVs 49) Find the difference between the total number of
sold by both the companies together on Monday to that TVs sold by company P to that of company Q on all the
of Saturday? given days together?
A.25 : 27 A.120
B.32 : 35 B.80
C.13 : 21 C.100
D.15 : 19 D.40
E.None of these E.None of these
48) If the total number of TVs sold by company P on 50) Total number of TVs sold by both the companies
Sunday is 20 % more than that of Friday on same together on Wednesday is what percentage more than
company, then find the sum of the total number of TVs the total number of TVs sold by both the companies
sold by company P on Monday, Wednesday and together on Friday?
Sunday together? A.33.33 % more
A.1256 B.12 % more
B.1112 C.45.28 % more
C.1064 D.12 % less
D.948 E.33.33 % less

8). DI Based on Line Graph – Solution with Explanation


1) Answer: C 4) Answer: A
Difference = 3200 – 2200 = Rs.1000 Required ratio = (2400 + 3000 + 2200):(1800 + 1600 +
2) Answer: D 2800)
Required total = 3600 + 2400 + 1800 + 2000 = Rs.9800 = 7600:6200
3) Answer: E = 38:31
Savings = 4000 – 1500 = Rs.2500 5) Answer: B

Page 671 of 722

Subscribe the Xpress Video Course & Mock Test Package for Bank & Insurance Exams
If there are any suggestions/ errors in our PDFs Feel Free to contact us via this email: admin@exampundit.in
IBPS RRB Clerk Prelims – Ultra Practice Bundle PDF
Required percentage = (2000 + 3200 + 1500)/(3600 + 11) Answer: C
1500 + 3000) * 100 Required ratio = (180 + 400): (250 + 350)
= 83% = 580: 600
6) Answer: D = 29: 30
Income in 2014=2.5 + 1=3.5 lakhs 12) Answer: C
Income in 2016=3 + 1.5=4.5 lakh Required percentage = 120/300 * 100
Difference=4.5 – 3.5=1 lakh = 40%
7) Answer: E 13) Answer: B
Income in 2015=4 + 0.5=4.5 lakh Unsold apple in A = 200 – 180 = 20
Savings in 2013 and 2017=0.5 + 1=1.5 lakh Unsold apple in B = 450 – 400 = 50
Required ratio=4.5:1.5 Unsold apple in C = 150 – 120 = 30
=3:1 Unsold apple in D = 300 – 250 = 50
8) Answer: B Unsold apple in E = 400 – 350 = 50
Average=(0.5 + 1 + 0.5 + 1.5 + 1 + 0.5)/6 Required average = (20 + 50 + 30 + 50 + 50)/5
=500000/6 = 40
=83000 14) Answer: E
9) Answer: D Required ratio = 400: 400
Income in 2013=2 + 0.5=2.5 lakh = 1: 1
Income in 2014=2.5 + 1=3.5 lakh 15) Answer: C
Income in 2015=4 + 0.5=4.5 lakh Required total=200 + 450 + 150 + 300 + 400
Income in 2016=3 + 1.5=4.5 lakh =1500
Income in 2017=3.5 + 1=4.5 lakh 16) Answer: A
Income in 2018=1.5 + 0.5=2 lakh In graph given exports to imports ratio. If exports more
10) Answer: D than the imports the ratio more than 1.
Income in 2019=(2 + 0.5)* 150/100 From graph for company B the ratios of 1995, 1996,
=3.75 lakh 1997, 1998 and 2000 have ratio more than 1.
Savings in 2019=80/100 * 0.5=0.4 lakh 17) Answer: C
Expenditure in 2019=3.75 – 0.4=3.35 lakh Actual ratio of exports to the imports in 1999 = 1
Page 672 of 722

Subscribe the Xpress Video Course & Mock Test Package for Bank & Insurance Exams
If there are any suggestions/ errors in our PDFs Feel Free to contact us via this email: admin@exampundit.in
IBPS RRB Clerk Prelims – Ultra Practice Bundle PDF
Let the import and export both be 100 24) Answer: E
Imports increased by 40 % = 140 % of imports Required difference = (420+640+400+600+720 +540)-
be the ratio of exports to the increased imports = 100/140 (560+480+540+360+450+320)
= 0.71 =>3320-2710
18) Answer: D =>610
Exports of Company A in 1998 were Rs. 237 crores 25) Answer: D
Exports to imports ratio = 0.75 January = 560-420=140
Imports = 237/0.75 = 316 February =640-480=160
19) Answer: A March = 540-400=140
Imports of Company B in 1995 = 180 April = 600-360=240
Exports of Company B in 1995 = 1.75*180 = 315 June= 540-320=220
The export of Company A = 2* 315 =630 26) Answer: B
The import of Company A = 630/0.75 = 840 Avg tones of total gold = (15+10+12+11)/4 = 48/4 = 12
20) Answer: E tones
In 1995 & 1997 the ratio is 1.75. so it has maximum All ores exported by Company C = 16+12+8 = 36 tones
difference percentage = (12/36)*100 = 33.33 %
21) Answer: D 27) Answer: C
Required ratio = 120/100 *(420+600):80/100*(540+320) total Quantity of gold ores = 48 tones
=>6*(1020): 4*(860) total Quantity of coal ores = 53 tones
=>153:86 this information shows that percentage of decrease will
22) Answer: A be our result
Required percentage ={(540+360)- decrease percentage = (5/53) *100 = 9.4% decrease.
(420+540)}/(420+540)*100 28) Answer: A
=>(60/960) *100=6.25% total ores
23) Answer: B A =35, B=35, C =36, D= 45
Required number of cars = So, C exported Second maximum tones of ores.
120/100*(420+600+720+540)/4 29) Answer: B
=>6/5*2280/4=6/5*570=684 total Quantity of gold ore = 48
Page 673 of 722

Subscribe the Xpress Video Course & Mock Test Package for Bank & Insurance Exams
If there are any suggestions/ errors in our PDFs Feel Free to contact us via this email: admin@exampundit.in
IBPS RRB Clerk Prelims – Ultra Practice Bundle PDF
In 2018 50% increase = (50/100) *48 = 24 Company A ( in lakh ) –> ( 4+3+8+8+8+7+8+12) =
total Quantity will be exported = 72. 5800000
total Quantity of silver ore = 50 Company B ( in lakh ) –> ( 1+1+1+2+3+5+7+11) =
In 2018 20% increase = (20/100)* 50 =10 3100000
total Quantity will be exported = 60. Difference = (5800000 – 3100000) = 2700000
Ratio= 72:60 = 6:5. 35) Answer: B
30) Answer: A Company B –>( 1+1+1+2+3+5+7+11) = 31
Percentage = (36/45) *100 Average -> 31/8 = 3.87
= 80% 36) Answer: E
31) Answer: B Investment in schemes B,D and E by Y = 560+580+380
2004 => 4 =100% = Rs.152000
2005 => 3 =? % Investment in schemes B,D and E by X = 440+720+820
Percentage = (1/4)*100 = 25% = Rs.198000
32) Answer: A Required % = (198000-152000)/198000 = 23%(approx.)
2011 company B’s production = 11 37) Answer: C
2010 company B’s production = 7 Required ratio = (520+580):(820+880) = 11:17
Percentage = (11/7)*100 = 157.14 38) Answer: A
33) Answer: A Required % = [(880-440)/440]*100 = 100%
2005 = ((4-3)/4)*100 = 25% decrease 39) Answer: D
2006 = ((8-3)/8)*100 = 62.5% increase Required average = (400+560+520+580+380+620)/6 =
2007 = ((8-8)/8)*100 = 0 Rs.51000
2008 = ((8-8)/8)*100 = 0 40) Answer: B
2009 = ((8-7)/8)*100 = 12.5% decrease Investment by X in schemes A and C = Rs.(740+780)
2010 = ((8-7)/7)*100 = 14.28% increase =Rs. 152000
2011 = ((12-8)/8)*100= 50% increase Investment by Y in schemes B and E = Rs.(560+380) =
Ans = 2006 Rs.94000
34) Answer: A Required % = [(152000-94000)/94000 ]*100 = 62%
41) Answer: C
Page 674 of 722

Subscribe the Xpress Video Course & Mock Test Package for Bank & Insurance Exams
If there are any suggestions/ errors in our PDFs Feel Free to contact us via this email: admin@exampundit.in
IBPS RRB Clerk Prelims – Ultra Practice Bundle PDF
Total number of vehicles manufactured by P in 2010, Total number vehicles manufactured by Company P in
2011 and 2013 together = 220 + 160 + 310 = 690 2014 = (190×130)/ 100
thousand = 247 thousand
Total number of vehicles manufactured by Q in 2010, 46) Answer: A
2011 and 2013 together = 120 + 1-80 + 210 = 510 The average number of TVs sold by company P on all
thousand the given days together
Required difference = 690 – 510 = 180 thousand = > (280 + 320 + 400 + 440 + 360 + 480) / 6
42) Answer: B = > 2280 / 6
Required average of Company Q = = > 380
(130+100+160+120+180+210) / 6 = 900 / 6 47) Answer: C
= 150 thousand The total number of TVs sold by both the companies
43) Answer: D together on Monday
Required percentage = [(40) / 160] × 100 = 25% = > 280 + 240 = 520
44) Answer: A The total number of TVs sold by both the companies
Total number of vehicles manufactured by company P in together on Saturday
2012 = 240 thousand 15000 vehicles are defective in that = > 480 + 360 = 840
year. Required ratio = 520 : 840 = 13 : 21
Non-defective vehicles manufactured in 2012 = 240 — 48) Answer: B
15 = 225 thousand The sum of the total number of TVs sold by company P
Again, total number of vehicles manufactured by on Monday, Wednesday and Sunday together
company Q in 2013 = 210 thousand = > 280 + 400 + 360 * (120/100)
10000 vehicles are defective. = > 280 + 400 + 432 = 1112
Non-defective vehicles manufactured by Company Q in 49) Answer: D
2013 = 210 — 10= 200 thousand The total number of TVs sold by company P on all the
Required ratio = 225 : 200 = 9 : 8. given days together
45) Answer: A = > 280 + 320 + 400 + 440 + 360 + 480 = 2280
Total number of vehicles manufactured by Company P in The total number of TVs sold by company Q on all the
2009 = 190 thousand given days together
Page 675 of 722

Subscribe the Xpress Video Course & Mock Test Package for Bank & Insurance Exams
If there are any suggestions/ errors in our PDFs Feel Free to contact us via this email: admin@exampundit.in
IBPS RRB Clerk Prelims – Ultra Practice Bundle PDF
= > 240 + 380 + 480 + 560 + 300 + 360 = 2320 = > 480 + 400 = 880
Required difference = 2320 – 2280 = 40 Total number of TVs sold by both the companies
50) Answer: A together on Friday
Total number of TVs sold by both the companies = > 360 + 300 = 660
together on Wednesday Required % = [(880 – 660) / 660] * 100 = 33.33 % more

9). DI Based on Pie Chart

Directions (1 – 5): Study the following information and answer the following questions:
The following Pie Chart Shows distribution of Branches of a bank named Bank of Jaipur in 6 different states in
India. Bank of Jaipur does not have branches other than below mentioned states.

1. If West Bengal has total 144 branches of Bank of Jaipur then the number of branches in state Kerala and
Uttar Pradesh together is?
a) 180
b) 240
c) 120
d) 480
Page 676 of 722

Subscribe the Xpress Video Course & Mock Test Package for Bank & Insurance Exams
If there are any suggestions/ errors in our PDFs Feel Free to contact us via this email: admin@exampundit.in
IBPS RRB Clerk Prelims – Ultra Practice Bundle PDF
e) None of the above
2. West Bengal and Rajasthan together constitute how much percent of branches of Bank of Jaipur?
a) 54%
b) 78%
c) 44%
d) 56%
e) None of the above
3. What is the ratio of number of branches of state Maharashtra has to the number of branches state J and K
has?
a) 5 : 7
b) 7 : 5
c) 12 : 5
d) 5 : 12
e) None of the above
4. If Bank of Jaipur has total 1220 branches operational all over India. By what percent they have to increase
the number of branches in Kerala to have 90 operational branches in Kerala?
a) 32.33%
b) 41.23%
c) 47.54%
d) 60%
e) Can’t be determined
5. Bank of Jaipur branches in Rajasthan are how much percent more or less than number of branches in Uttar
Pradesh state?
a) 100%
b) 211.33%
c) 150%
d) 255.77%
e) None of these
Page 677 of 722

Subscribe the Xpress Video Course & Mock Test Package for Bank & Insurance Exams
If there are any suggestions/ errors in our PDFs Feel Free to contact us via this email: admin@exampundit.in
IBPS RRB Clerk Prelims – Ultra Practice Bundle PDF
Directions (6 – 10): Answer the questions based on the information given below.
Given Pie chart shows the distribution of exports of the state TamilNadu in the year 2019 to different countries.
The given figures are in Rs. (in Lacs).

Export of TN 2019 (in Lacs)

170 Malaysia
289
Singapore
Australia
187
51 34 Sri Lanka
Other

6. Export to Malaysia constitutes how much percent out of total exports of state TamilNadu?
a) 25.23%
b) 23.25%
c) 26.25%
d) 25.26%
e) None of the above
7. If half the exports out of exports to other countries went to Europe, calculate the difference between exports
went to Europe and Malaysia?
a) 31.5 Lacs
b) 11 Lacs
c) 25.5 Lacs
d) 22 Lacs
e) None of the above
8. What is the ratio of total exports went to Singapore and Australia together to the total exports went to Sri
Lanka and Malaysia together?

Page 678 of 722

Subscribe the Xpress Video Course & Mock Test Package for Bank & Insurance Exams
If there are any suggestions/ errors in our PDFs Feel Free to contact us via this email: admin@exampundit.in
IBPS RRB Clerk Prelims – Ultra Practice Bundle PDF
a) 1 : 1
b) 2 : 1
c) 1 : 2
d) Cannot be determined.
e) None of the above
9. If the ratio of Import to export for TN state is 19 : 17. It is also known that one-third of Imports were from
Malaysia. Then the imports from Malaysia are how much percent more than the exports of Malaysia in 2019.
a) 30.38%
b) 37.57%
c) 54.19%
d) 60.19%
e) None of these
10. Find the exports made to Sri Lanka in 2020 if the exports made to Sri Lanka are decreased by one-third in
2020 as compared to 2019.
a) 51
b) 34
c) 17
d) 68
e) None of the above

Directions (11 – 15): Answer the questions based on the information.


Pie chart given below shows the runs scored by 5 batsmen of Indian cricket team in a tournament. Total 5
matches were played in tournament and each player has played different number of matches.

Page 679 of 722

Subscribe the Xpress Video Course & Mock Test Package for Bank & Insurance Exams
If there are any suggestions/ errors in our PDFs Feel Free to contact us via this email: admin@exampundit.in
IBPS RRB Clerk Prelims – Ultra Practice Bundle PDF

Runs Scored by 5 Batsmen

150

380
Sharma
190
Dhawan
Kohli
Dhoni
Kartik
320
320

11. If Sharma has played all 5 matches of the series, then calculate the batting average in the tournament?
a) 95
b) 67
c) 60
d) 76
e) None of the above
12. What is ratio of total runs made by openers in the tournament to the total runs made by other 3 batsmen?
Note: Sharma and Dhawan used to open every match in tournament?
a) 35 : 33
b) 72 : 52
c) 21 : 19
d) 11 : 19
e) None of these
13. How many runs will Dhawan have to score in next match if it is played to match Sharma’s average if Sharma
is not playing next match and both have not missed any match till date in this tournament?

Page 680 of 722

Subscribe the Xpress Video Course & Mock Test Package for Bank & Insurance Exams
If there are any suggestions/ errors in our PDFs Feel Free to contact us via this email: admin@exampundit.in
IBPS RRB Clerk Prelims – Ultra Practice Bundle PDF
a) 122
b) 125
c) 136
d) Cannot be determined.
e) None of these
14. Runs scored by Kartik are how much percent more / less by total runs scored by Dhoni in the tournament?
a) 22.02%
b) 21.05%
c) 27.5%
d) 75.75%
e) None of these
15. Among all batsmen, everyone played all 5 matches then who among them has got 2nd best batting average in
the tournament?
a) Sharma
b) Dhawan
c) Kohli
d) Both Kohli and Dhawan
e) Can’t be determined
Directions (16-20): Answer the questions based on the information given below.
Pie chart given below shows the % of marks obtained in 5 different subjects by a student in the board exams of
the year 2020.
Note: The Total marks for English, History, Geography, polity and Maths were 100, 50, 50, 100 and 150
respectively.

Page 681 of 722

Subscribe the Xpress Video Course & Mock Test Package for Bank & Insurance Exams
If there are any suggestions/ errors in our PDFs Feel Free to contact us via this email: admin@exampundit.in
IBPS RRB Clerk Prelims – Ultra Practice Bundle PDF

Percentage of marks obtained in each subject

83.33% 90%
English
History
Geography

60% Polity
60%
Maths

80%

16. What is the overall percentage obtained by the student in the board exams?
a) 46.67%
b) 56.67%
c) 66.67%
d) 76.67%
e) None of the above
17. What are the total marks obtained by the student in Maths and History?
a) 195
b) 150
c) 155
d) 160
e) None of these
18. Marks obtained in the subject geography are how much percent more / less than the marks obtained in the
subject History?
a) 77%

Page 682 of 722

Subscribe the Xpress Video Course & Mock Test Package for Bank & Insurance Exams
If there are any suggestions/ errors in our PDFs Feel Free to contact us via this email: admin@exampundit.in
IBPS RRB Clerk Prelims – Ultra Practice Bundle PDF
b) 66.67%
c) 50%
d) 33.33%
e) Can’t be determined
19. If there is another student who has same total percentage as total percentage of above mentioned student,
What are the marks obtained by another student in the Maths subject?
a) 37.50%
b) 22.22%
c) 33.33%
d) 44.44%
e) Cannot be determined.
20. If the student gets 20 marks more in history and polity subject, then which subject will have the best
percentage obtained?
a) History
b) Polity
c) Science
d) Geography
e) None of the above

Directions (21 – 25): Study the information carefully and answer the question asked below.
Pie chart given below shows the different types A, B, C, D and E of grains (in Kg.) sold by a shopkeeper in the
month of July.

Page 683 of 722

Subscribe the Xpress Video Course & Mock Test Package for Bank & Insurance Exams
If there are any suggestions/ errors in our PDFs Feel Free to contact us via this email: admin@exampundit.in
IBPS RRB Clerk Prelims – Ultra Practice Bundle PDF

Type of Grains Sold (in Kg)

150 120

80 B
175 C
D

125 E

21. In the month of August if the shopkeeper sells 25% of grains of Type A only. What is the sell in Kg. of type
A in month of August if his total sell has increased by 20% than July?
a) 195
b) 180
c) 8
d) 105
e) 120
22. By what percent the sell of grains of type E and D together is more or less than sell of type C and B together?
a) 35.35%
b) 23.33%
c) 45.45%
d) 49.52%
e) None of the above

Page 684 of 722

Subscribe the Xpress Video Course & Mock Test Package for Bank & Insurance Exams
If there are any suggestions/ errors in our PDFs Feel Free to contact us via this email: admin@exampundit.in
IBPS RRB Clerk Prelims – Ultra Practice Bundle PDF
23. If the ratio of the sale of gains in the month of June and July is 12 : 15. Then what is total sale of grains in
Kg. in the month of June?
a) 252
b) 562
c) 550
d) 520
e) None of these
24. How many types of grains have above average sale in the month of August?
a) 1
b) 3
c) 2
d) 4
e) None of these
25. Type A, B and C gave how much percent of sale out of total sale made by the shopkeeper?
a) 64.61%
b) 61.64%
c) 23.33%
d) 66.67%
e) 33.34%

Directions (26 – 30): Read the following information carefully and answer the following questions carefully.
Pie chart given below shows average marks obtained by students from 5 different schools P, Q, R, S, T in
National level exam. The exam has maximum marks 200.

Page 685 of 722

Subscribe the Xpress Video Course & Mock Test Package for Bank & Insurance Exams
If there are any suggestions/ errors in our PDFs Feel Free to contact us via this email: admin@exampundit.in
IBPS RRB Clerk Prelims – Ultra Practice Bundle PDF

Average marks obtained by 5 schools

125 125

P
160 150 Q
R
S
170 T

26. The total marks obtained by School P are how much more / less than the total marks obtained by School Q
if both schools have 50 students each?
a) 1250
b) 1550
c) 1850
d) 1750
e) 1650
27. What is average of marks obtained by all schools together if it is known that all schools have 70 students each
participating in the exam?
a) 185
b) 146
c) 164
d) 175
e) None of these
28. What is the ratio of average marks of school Q to the average marks of school R as per the given pie chart?

Page 686 of 722

Subscribe the Xpress Video Course & Mock Test Package for Bank & Insurance Exams
If there are any suggestions/ errors in our PDFs Feel Free to contact us via this email: admin@exampundit.in
IBPS RRB Clerk Prelims – Ultra Practice Bundle PDF
a) 17 : 15
b) 7 : 8
c) 15 : 17
d) 8 : 7
e) None of the above
29. If the average marks obtained by school P and Q together is 140. It is known that 50 students from school P
took participation in the competition, then find the number of students from school Q who took participation?
a) 25
b) 50
c) Cannot be determined
d) 75
e) None of the above.
30. By what percent average marks obtained by school R and school S together are more or less than the total
marks obtained by school Q and T together?
a) 20%
b) 25%
c) 72%
d) 23%
e) 69%

Directions (31 – 35): Read the following information carefully and answer the following questions given below.
Following Pie chart shows the number of subscribers of different YouTube channels have at the end of July
2020.

Page 687 of 722

Subscribe the Xpress Video Course & Mock Test Package for Bank & Insurance Exams
If there are any suggestions/ errors in our PDFs Feel Free to contact us via this email: admin@exampundit.in
IBPS RRB Clerk Prelims – Ultra Practice Bundle PDF

Number of subscribers (in millions)

4.5
12
6 TrekTech
Technical Teacher

3 ZooGo
TechnoKing
RaptorOnline
7 LostParadise
18

31. By what percent the number of subscribers of Technical Teacher are more than the number of subscribers
of TrekTech?
a) 20%
b) 30%
c) 50%
d) 70%
e) None of these
32. What is the average number of subscribers (in millions) for the above mentioned 6 YouTubers together?
a) 6.21
b) 8.21
c) 7.29
d) 8.33
e) None of these
33. If the number of subscribers of TrekTech are increased to 15 million at the end of August month then what
is the growth in percent compared to the end of july?

Page 688 of 722

Subscribe the Xpress Video Course & Mock Test Package for Bank & Insurance Exams
If there are any suggestions/ errors in our PDFs Feel Free to contact us via this email: admin@exampundit.in
IBPS RRB Clerk Prelims – Ultra Practice Bundle PDF
a) 40%
b) 30%
c) 20 %
d) 25%
e) None of these
34. Average view time of a subscriber of ZooGo is 7 minutes 15 seconds and average view time of Raptoronline
is 8 minutes 45 seconds per subscriber. What is the total view time for ZooGo and Raptoronline in the month of
july?
a) 102.25 million minutes
b) 218. 78 million minutes
c) 103.25 million minutes
d) 35.7 million minutes
e) None of these
35. What is the ratio of total number of subscribers for ZooGo and TechnoKing together to the number of
subscribers of TrekTech?
a) 11 : 12
b) 5 : 6
c) 1 : 3
d) 6 : 5
e) None of the above

Directions (36 – 40): Read the data given below and answer the following questions.
Top Leading companies sold following numbers (in thousands) of different models of Smartphones in 2019.
Answer the following questions based on the given data. Data is given for models A, B, C, D and E.

Page 689 of 722

Subscribe the Xpress Video Course & Mock Test Package for Bank & Insurance Exams
If there are any suggestions/ errors in our PDFs Feel Free to contact us via this email: admin@exampundit.in
IBPS RRB Clerk Prelims – Ultra Practice Bundle PDF

number of smartphones sold (in thousands)

60
120
A
B
135
C
75
D
45
E

36. What is the ratio of number of smartphones sold of model C and D together to the number of smartphones
sold for the models A and E together?
a) 1 : 1
b) 5 : 6
c) 2 : 3
d) 1 : 2
e) None of these
37. Find the difference in thousands between the numbers of highest number of smartphones sold to the lowest
number of smartphones sold for any model?
a) 100
b) 90
c) 200
d) 95
e) 125
38. How many types of the models have its sale above average for the year 2019?
a) 3
b) 1
c) 5
Page 690 of 722

Subscribe the Xpress Video Course & Mock Test Package for Bank & Insurance Exams
If there are any suggestions/ errors in our PDFs Feel Free to contact us via this email: admin@exampundit.in
IBPS RRB Clerk Prelims – Ultra Practice Bundle PDF
d) 2
e) None of the above
39. If the sale of total number of smartphones increases by 17% in 2020. What is the number of units in
thousands sold of model C in 2020?
a) 340
b) 220
c) 250
d) Cannot be determined
e) None of these
40. By how much percent the sale of model B is more than model E?
a) 23%
b) 25%
c) 45%
d) 50%
e) 37%
Direction (41-45): Read the data given below and answer the following questions.
The Pie below shows the distribution of the percentage of tigers in wildlife sanctuaries in India. Data is given for
5 different wildlife sanctuaries in India of the year 2019. Total number of tigers in these 5 sanctuaries is 1500.

Percentage of Tiger population (1500)

15%
30%
BandhavGadh
15%
Kabini
5% Chandrapur
Melghat
35%
Jharkhand

Page 691 of 722

Subscribe the Xpress Video Course & Mock Test Package for Bank & Insurance Exams
If there are any suggestions/ errors in our PDFs Feel Free to contact us via this email: admin@exampundit.in
IBPS RRB Clerk Prelims – Ultra Practice Bundle PDF
41. Find the % increase or decrease in the total number of tigers in sanctuary E in 2020 as compare to the
previous year?
a) 1.1%
b) 2.25%
c) 1.28%
d) Cannot be determined
e) None of these
42. The average of the number of tigers in all the sanctuaries together is for the year 2019?
a) 445
b) 500
c) 300
d) 405
e) None of these
43. What is the difference between number of tigers in Kabini and Melghat for the year 2019?
a) 300
b) 225
c) 160
d) 120
e) 390
44. What is the ratio of the number of tigers from Jharkhand to the difference between number of tigers from
Melghat and Chandrapur?
a) 2 : 5
b) 3 : 2
c) 3 : 5
d) 4 : 5
e) None of these
45. If the number of tigers in BandhavGadh increased by 10% and Melghat increased by 32% in the year 2020.
Then what is the respective ratio of number of tiger since the year 2020?
Page 692 of 722

Subscribe the Xpress Video Course & Mock Test Package for Bank & Insurance Exams
If there are any suggestions/ errors in our PDFs Feel Free to contact us via this email: admin@exampundit.in
IBPS RRB Clerk Prelims – Ultra Practice Bundle PDF
a) 6 : 7
b) 1 : 7
c) 5 :3
d) 1 : 4
e) None of these

Directions (46 – 50): Read the data given below and answer the following questions.
The Pie Chart shows that number of male athletes from each country for the Diamond league sports
championship 2019.

Number of male athletes selected

200
315
India
225 UAE

170 USA
Brazil
Russia

450 Japan
375

46. What is the ratio of male athletes selected to the female athletes selected from UAE if 275 female athletes
were selected from UAE?
a) 9 : 11
b) 15 : 9
c) 11 : 9
d) 8 : 7

Page 693 of 722

Subscribe the Xpress Video Course & Mock Test Package for Bank & Insurance Exams
If there are any suggestions/ errors in our PDFs Feel Free to contact us via this email: admin@exampundit.in
IBPS RRB Clerk Prelims – Ultra Practice Bundle PDF
e) 31 : 19
47. Find the average number of male athletes selected from India, USA and Brazil together?
a) 257.5
b) 341.67
c) 111.75
d) 433.07
e) 107.12
48. If the ratio of male athletes to female athletes from USA is 3 : 2. And the same ratio for India is 1 : 1. What
is the difference between number of female athletes from India and USA?
a) 38
b) 25
c) 100
d) 50
e) 80
49. By what percent male athletes from Japan are less than the male athletes from Brazil?
a) 16%
b) 26%
c) 46%
d) 15%
e) 25%
50. What is the percentage of male athletes selected for country Brazil out of total male athletes selected?
a) 14.42%
b) 21.61%
c) 27.42%
d) 19.66%
e) 21.15%

Page 694 of 722

Subscribe the Xpress Video Course & Mock Test Package for Bank & Insurance Exams
If there are any suggestions/ errors in our PDFs Feel Free to contact us via this email: admin@exampundit.in
IBPS RRB Clerk Prelims – Ultra Practice Bundle PDF

9). DI Based on Pie Chart - Solutions and Detailed Explanations

Solution(1-5) Kerala and UP has combined 15% + 5% = 20% branches.


1. Answer: B) Thus, 144 / x = 12 / 20
West Bengal has total 144 branches which is 12% of total X = 240 branches Kerala and UP together.
branches.
2. Answer: D) Solution(6-10).
West Bengal and RJ together constitute 12% + 44% = 6. Answer: B)
56% of branches. It is clearly visible from the pie chart. Total exports for the state of TN = 170+187+34+51+289
3. Answer: A) = 731 Las
Let total number of branches be 100x. Exports to Malaysia = 170 Lacs
Branches in MH = 10% of 100x = 10x Percentage = 170 / 731 * 100 = 23.25%
Branches in J & K = 14% of 100x = 14x 7. Answer: C)
Ratio = 10x : 14x = 5 :7 Export to other countries = 289 Lacs
4. Answer: C) Half went to Europe = 289 / 2
Currently there are 1220 total branches in India. Difference between exports to Europe and Malaysia = 170
Branches in Kerala = 5% of 1220 = 61 – 289/2 =25.5 Lacs
Now to make number of branches 90 8. Answer: A)
Increase in number of branches = 90-61 = 29 Exports went to Australia and Singapore together = 187 +
Percentage increase = 29 / 61 * 100 = 47.54% 34 = 221 Lacs
5. Answer: E) total exports went to Sri Lanka and Malaysia together =
Let total number of branches be 100x. 51 + 170 = 221 Lacs
Branches in RJ = 44% of 100x = 44x Required ratio = 221 : 221 = 1 : 1
Branches in UP = 15% of 100x = 15x 9. Answer: D)
Percentage = 44x – 15x = 29x ratio of Import to export for TN state is 19 : 17.
= 29x / 15x * 100 = 193.33% more Total exports to TN = 731 Lacs
Imports for TN = 19 / 17 = x / 731
Page 695 of 722

Subscribe the Xpress Video Course & Mock Test Package for Bank & Insurance Exams
If there are any suggestions/ errors in our PDFs Feel Free to contact us via this email: admin@exampundit.in
IBPS RRB Clerk Prelims – Ultra Practice Bundle PDF
X = 817 Lacs Runs scored by other 3 = 320 + 190 + 150 = 660
One third imports came from Malaysia = 1 / 3 * 817 = Ratio = 700 : 660 = 35 : 33
272.33 13. Answer: C)
Percentage more = 272.33 – 170 / 170 * 100 = 60.19% Sharma’s average = 380 / 5 = 76
10. Answer: B) Dhawan’s current average = 320 / 5 = 64
Exports made to Sri Lanka are decreased by one-third in Runs to be scored in next match to equal the average =
2020 as compared to 2019. (64*5 + x ) / 6 = 76
New exports to Sri Lanka = 2/3 of exports made in 2019 X = 136
= 2/ 3 * 51 = 34 Lacs 14. Answer: B)
Runs scored by Kartik are how much percent more / less
Solution(11-15). by total runs scored by Dhoni
11. Answer: D) = (190-150) / 190 = 21.05% less
Sharma has played all 5 matches in the series. 15. Answer: D)
Average = Total runs / number of matches = 380/5 = 76 If everybody plays 5 matches, then 2nd best average
12. Answer: A) That figure will 2nd best total number of runs since number
ratio of total runs made by openers in the tournament to of matches played will be same for everyone.
the total runs made by other 3 batsmen Dhawan and Kohli will be the answer as both have 320
Openers = Sharma and Dhawan runs.
Runs scored by them = 380+ 320 = 700
Solution(16-20). overall percentage obtained by the student in the board
Common Explanation exams =
Marks obtained in English = 100 * 90% = 90 Total marks obtained / Total marks * 100 =
Marks obtained in history = 50 * 60% = 30 = (90 + 30+ 40 + 60 + 125) / (100 + 50 + 50 +100 + 150)
Marks obtained in geography = 50 * 80% = 40 * 100
Marks obtained in polity = 100 * 60% = 60 = 345 / 450 = 76.67%
Marks obtained in Maths = 150 * 83.33% = 125 17. Answer: C)
16. Answer: D) the total marks obtained by the student in Maths and
History = 125+ 30 = 155
Page 696 of 722

Subscribe the Xpress Video Course & Mock Test Package for Bank & Insurance Exams
If there are any suggestions/ errors in our PDFs Feel Free to contact us via this email: admin@exampundit.in
IBPS RRB Clerk Prelims – Ultra Practice Bundle PDF
18. Answer: D) Percentage less = 300 – 230 / 300 * 100 = 23.33%
Marks obtained in the subject geography = 40 23. Answer: D)
marks obtained in the subject History = 30 ratio of the sale of gains in the month of June and July is
Percentage more = (40 -30) / 30 *100 = 33.33% 12 : 15
19. Answer: E) Sale of grains in the month of June = 12 /15 = x / 650 =
This figure cannot be determined since we only know the 520
overall percentage which is not enough to calculate marks 24. Answer: C)
obtained in Maths. Average sale in the month of July = 650 / 5 = 130
20. Answer: A) Types of grains having above average sale = 2 (Type B
If students get 20 marks more in history = 30 + 20 = 50 and E)
Percentage = 50/50 * 100 = 100% 25. Answer: A)
If students gets 20 more marks in polity = 60 +20 = 80 Total Sale by Type A,B,C = 120 +175 +125 = 420 kg
Percentage = 80/ 100 *100 = 80% Total sale = 650 kg
History will have best percentage obtained. Percentage = 420 / 650 * 100 = 64.61%

Solution(21-25). Solution(26-30).
Common Explanation 26. Answer: A)
Total sale in July = 120+175+125+80+150 = 650 total marks obtained by School P =
21. Answer: A) = number of students * average marks
In august total sell has increased by 20% than July. = 125 * 50 = 6250
Total sale in July = 650 total marks obtained by School Q
Sale in august = 120 /100 *650 = 780 = number of students * average marks
In the month of August if the shopkeeper sells 25% of = 150 * 50 = 7500
grains of Type A only. Difference of Total marks= 7500-6250= 1250
= 780 * 25 /100 = 195 kg. 27. Answer: B)
22. Answer: B) Average of marks obtained by all schools together if it is
the sell of grains of type E and D = 150 + 80 = 230 kg known that all schools have 70 students each.
sell of type C and B together = 175 + 125 = 300 kg = Total marks obtained / total number of students
Page 697 of 722

Subscribe the Xpress Video Course & Mock Test Package for Bank & Insurance Exams
If there are any suggestions/ errors in our PDFs Feel Free to contact us via this email: admin@exampundit.in
IBPS RRB Clerk Prelims – Ultra Practice Bundle PDF
= (125+150+170+160+125) * 70 / 350 Percentage = (18 -12) / 12 * 100 = 50%
= 146 32. Answer: E)
28. Answer: C) Average number of subscribers = 12+18+7+3+6+4.5 / 6 =
Average marks of school Q to the average marks of school 50.5 / 6 = 8.41 millions
R 33. Answer: D)
= 150 : 170 subscribers of TrekTech are increased to 15 million.
= 15 : 17 July subscribers = 12 millions
29. Answer: D) Percentage increase = 15 – 12 / 12 *100 = 25%
Average marks obtained by school P and Q together is 34. Answer: C)
140. Average view time of subscribers of ZooGo is 7 minutes
50 students from school P took participation in the 15 seconds = 7 * 60 + 15 = 435 sec
competition. average view time of Raptoronline is 8 minutes 45
Using the formula for average = Total marks by p and Q / seconds = 8 *60 + 45 = 480 + 45 = 525 sec
total students of P and Q = Average Total view time for ZooGo = 435 * 7 million = 3045
Let x students took participation from Q million sec
(125 * 50 + 150 * x) / (50 + x) = 140 = 3045 /60 = 50.75 million minutes
X = 75 Total view time for RaptorOnline = 525 * 6 million = 3150
30. Answer: A) million sec
average marks obtained by school R and school S together = 3150 /60 = 52.5 million minutes
= 170 +160 = 330 Total view time = 103.25 million min
total marks obtained by school Q and T together = 35. Answer: B)
150+125 = 275 Total number of subscribers for ZooGo and TechnoKing
Percentage = 330 – 275 / 275 * 100 = 20% together = 7 + 3 = 10 million
Number of subscribers of TrekTech = 12 million
Solution(31-35). Ratio = 10 : 12 = 5 : 6
31. Answer: C) Solution(36-40).
the number subscribers of Technical Teacher = 18 mil 36. Answer: A)
number of subscribers of TrekTech = 12 mil
Page 698 of 722

Subscribe the Xpress Video Course & Mock Test Package for Bank & Insurance Exams
If there are any suggestions/ errors in our PDFs Feel Free to contact us via this email: admin@exampundit.in
IBPS RRB Clerk Prelims – Ultra Practice Bundle PDF
Number of smartphones sold of model C and D together sale of model E = 60000
= 45 + 135 = 180 thousands Percentage more = (75 – 60) / 60* 100 = 25%
Number of smartphones sold for the models A and E
together = 180 thousands Solution(41-45).
Ratio = 180 : 180 = 1 : 1 Common Explanation:
37. Answer: B) Number of tigers in BandhavGadh = 1500 * 30 / 100 =
Numbers of highest number of smartphones sold = 135 450
thousands for Model D Number of tigers in Kabini = 1500 * 35 /100 = 525
Lowest number of smartphones sold for any model = 45 Number of tigers in Chandrapur = 1500 * 5/100 = 75
thousands for Model C Number of tigers in Melghat = 1500 * 15 /100 = 225
Required difference = 135 – 45 = 90 thousands Number of tigers in Jharkhand = 1500* 15 /100 = 225
38. Answer: D) 41. Answer: D)
Average sale for the year 2019 = 120+75+45+135+60 = This cannot be determined since we don’t have data from
435 / 5 = 87 year 2020.
Number of models having above average sale = Type A , 42. Answer: C)
Type D = Average of the number of tigers in all the sanctuaries
2 together is for the year 2019
= Total number of tiger / number of sanctuaries
39. Answer: D) = 1500 / 5 = 300
Sale of smartphones in 2019 = 435 43. Answer: A)
sale of total number of smartphones increases by 17% in Number of tigers in Kabini = 525
2020 = 435 * 117 /100 = 508.95 Number of tigers in Melghat = 225
The number of units in thousands sold of model C in 2020 Required difference = 300
cannot be determined because we don’t know the 44. Answer: B)
percentage of type C sold out of total number of Number of tigers from Jharkhand = 225
smartphones sold. Difference between number of tigers from Melghat and
40. Answer: B) Chandrapur = 225 – 75 = 150
sale of model B = 75000 Required ratio = 225 : 150 = 3 : 2
Page 699 of 722

Subscribe the Xpress Video Course & Mock Test Package for Bank & Insurance Exams
If there are any suggestions/ errors in our PDFs Feel Free to contact us via this email: admin@exampundit.in
IBPS RRB Clerk Prelims – Ultra Practice Bundle PDF
45. Answer: C) Required ratio = 225 : 275 = 9 : 11
Number of tigers in BandhavGadh increased by 10% 47. Answer: B)
= 450 * 110 / 100 = 495 Average number of male athletes selected from India,
Number of tigers in Melghat increased by 32% USA and Brazil together
= 225 * 132 / 100 = 297 = (200+450+375) / 3 = 1025 / 3 = 341.67
Required ratio = 495 : 297 = 5 : 3 48. Answer: C)
Solution(46-50). Ratio of male athletes to female athletes from USA is 3 :
Common Explanation: 2
46. Answer: A) 3 / 2 = 450 / x
Male athletes from UAE = 225 x (say) = 300
Female athletes from UAE = 275 Ratio for India is 1 : 1
Female athletes from India = 200 Required percentage = (375-315) / 375 * 100 = 16% less
Difference between number of female athletes from India 50. Answer: B)
and USA Percentage of male athletes selected for country Brazil out
= 300 -200 = 100 of total male athletes selected
49. Answer: A) = 375 / (200+225+450+375+170+315) * 100
Male athletes from Japan = 315 = 375 / 1735 * 100 = 21.61%
Male athletes from Brazil = 375

10. Caselet DI Questions

Directions (1 – 5): Study the following information of Male student in Class VII is 1200. The number of
and answer the following questions: Male student in Class VII and VI together is 2100. The
There are three class i.e. V, VI and VII. In each of the number of male student in class VI is 20% more than
given class male student and female student are there. the number of female student in class V. The ratio of
Total number of student in class V is 1800. The number number of Male student to Female student in class VII

Page 700 of 722

Subscribe the Xpress Video Course & Mock Test Package for Bank & Insurance Exams
If there are any suggestions/ errors in our PDFs Feel Free to contact us via this email: admin@exampundit.in
IBPS RRB Clerk Prelims – Ultra Practice Bundle PDF
is 8:7, respectively. The total number of female student e) None of the above
in all classes together is 2400. 5. The number of male student of class VI is what
1. Find the number of female in class VII? percentage of the number of male student of class
a) 950 VII?
b) 1050 a) 75%
c) 1550 b) 88%
d) 850 c) 62.5%
e) None of the above d) 55.75%
2. What is the ratio of number of female to number e) None of these
of male from class VI? Directions (6 – 10): Answer the questions based on
a) 1:3 the information given below.
b) 3:5 The data given below show the number of male and
c) 2:3 female who visited Taj Mahal from 4 different countries
d) 2:1 i.e. USA, Russia, Germany and Japan. There are 1710
e) None of these people who visited the Taj Mahal form the USA. Total
3. Find the average number of male student in each number of male from USA is 290 less than the number
of the following class? of female from same country who visited the Taj Mahal.
a) 1060 Total number of female who visited Taj Mahal from
b) 1590 Russia is half the number of male form same country.
c) 1525 Ratio between total number of male who visited Taj
d) 1530 Mahal from Germany and Japan are in the ratio of 1:2.
e) None of these Number of Female from Japan is 1200 which is 200 less
4. Find the difference between the number of female than the number of female form Germany who visited
in class VI and the number of males in class V? Taj Mahal. Total number of Female and male form all
a) 330 the countries who visited Taj Mahal is 5000 and 5670
b) 325 respectively.
c) 320 6. Find the total number of male and female form
d) 450 Germany who visited Taj Mahal?
Page 701 of 722

Subscribe the Xpress Video Course & Mock Test Package for Bank & Insurance Exams
If there are any suggestions/ errors in our PDFs Feel Free to contact us via this email: admin@exampundit.in
IBPS RRB Clerk Prelims – Ultra Practice Bundle PDF
a) 2065 c) 131:50
b) d) 17:143
2215 e) None of these
8. Find the average number of Female form USA,
c) 2120 Japan and Germany together?
d) 1550 a) 1450
e) None of these b): 1253
7. Find the ratio between total number of male from c) 1250
USA and Japan who visited the Taj Mahal? d) 1200
a) 137:50 e) None of these
b) 141:50
9. Number of male who visited Taj Mahal from Russia is what % of total number of people from Japan who
visited Taj Mahal?
a) 106.06%
b) 105.5%
c) 108.06%
d) 116.06%
e) None of these
10. Find the difference between total number of population from Japan and USA who visited Taj Mahal?
a) 930
b) 690
c) 530
d) 550
e) None of these
Directions (11 – 15): Answer the questions based on the information given below.
Out of 200 people who attended a marriage party 100 had GolGape, 120 had Chicken Tica and 40 had none of these
two. 100 people had exactly one of two items.
11. How many people like had both of the given items?
Page 702 of 722

Subscribe the Xpress Video Course & Mock Test Package for Bank & Insurance Exams
If there are any suggestions/ errors in our PDFs Feel Free to contact us via this email: admin@exampundit.in
IBPS RRB Clerk Prelims – Ultra Practice Bundle PDF
a) 35
b) 50
c) 55
d) 60
e) 45
12. How many person does not like Chiken Tica?
a) 55
b) 80
c) 35
d) 50
e) None of these
13. Person who like only Golgape is what % more or less than the person who like only chicken Tica?
a) 38.5%
b) 25.6%
c) 15.2%
d) 33.33%
e) 32.5%
14. What is the maximum possible number of people who had only Chicken Tica?
a) 45
b) 50
c) 55
d) 60
e) None of these
15. What is the ratio between numbers of people who like only Golgape to Person who like Chicken Tica?
a) 1:1
b) 2:3
c) 1:5
d) 1:3
Page 703 of 722

Subscribe the Xpress Video Course & Mock Test Package for Bank & Insurance Exams
If there are any suggestions/ errors in our PDFs Feel Free to contact us via this email: admin@exampundit.in
IBPS RRB Clerk Prelims – Ultra Practice Bundle PDF
e) None of these
Directions (16-20): Answer the questions based on c)
the information given below. 155
The data given below is about students studying in 3
different class i.e. 8th, 9th and 10th. d) 160
There are 125 Male student in class 8th and 9th together. e) None of these
Total number of female studying in class 10th is 40 18. What is the ratio between total numbers of male
more than the number of male studying in class 8th. studying in class 10 to total number of female
Total number of students studying in class 10th is 360. studying in the same class?
Number of female studying in class 8th and class 9th is a) 5:4
10 and 20 less respectively than the number of male b) 4:5
studying in same class. Total number of student c) 2:1
studying in class 8th is 150. d) 1:2
16. Find the number of male studying in class 8th is e) Can’t be determined
what % of numbers of female studying in same 19. Number of girls studying in class 9 and 10 is what
class? percentage more or lesser than the total number of
a) 114.28% male studying in same class.?
b) 134.28% a) 28.75%
c) 104.28% b) 27.75%
d) 124.28% c) 37.5%
e) None of the above d) 50%
17. Find the difference between total number of e) None of these
Male and total number is Female taken from all the 20. If 25 new male student join the class 8 and 9th in
classes together? the ratio of 2:3. Then find the difference between
a) 125 number of male student from class 8th and class 9th.
b) 150 After joining of new students
a) 32
b) 25
Page 704 of 722

Subscribe the Xpress Video Course & Mock Test Package for Bank & Insurance Exams
If there are any suggestions/ errors in our PDFs Feel Free to contact us via this email: admin@exampundit.in
IBPS RRB Clerk Prelims – Ultra Practice Bundle PDF
c) 29 is 240 more than twice the number of people who travel
d) 31 on previous day. Number of people who travel on
e) None of these Wednesday is the average number of people who travel
on Monday and Tuesday together, while number of
Directions (21 – 25): Answer the questions based on people who travel on Thursday is 350 less than the
the information given below. number of people who travels on Friday. It's is also
The data below is about no of passengers traveling by known that total number of passengers from Monday to
using Mumbai local train. Friday is 5 times the number of travelers on Wednesday.
There are 17500 people traveling on Monday. Number
of people who travel by Mumbai local train on Tuesday
21. Find the difference between numbers of travelers 23. Find the ratio between numbers of travelers on
on Monday to Wednesday together to number of Wednesday to Friday?
travelers from Thursday and Friday together? a) 5274:5309
a) 26370 b) 5574:5309
b) 24570 c) 5674:5309
c) 25.50% d) 5574:5309
d) e) 6274:5309
24675 24. What is the average number of travelers from all
5 day taken together?
e) None of these a) 14480
22. Total number of travelers traveling on Monday b) 15280
is what % of total number of travelers traveling on c) 26260
last two days? d) 26370
a) 15.50% e) None of these
b) 33.18% 25. On which of the following day total number of
c) 25.20% travelers traveling by Mumbai local train is 2nd
d) 25.50% highest?
e) None of these a) Monday
Page 705 of 722

Subscribe the Xpress Video Course & Mock Test Package for Bank & Insurance Exams
If there are any suggestions/ errors in our PDFs Feel Free to contact us via this email: admin@exampundit.in
IBPS RRB Clerk Prelims – Ultra Practice Bundle PDF
b) Tuesday c) 883.5
c) Friday d) 375.5
d) Thursday e) 225.8
e) None of these 28. Total number of ticket sold by B is what % of
Direction (26-30) Direction : Answer the question total number of ticket sold by A and C together?
based on the information given below a) 12%
There are 3 cinema house where two type of ticket is b) 84%
sold i.e. Silver and Gold. Number of Silver ticekt sold c) 31%
by C is 120 more than the numbr of Gold ticket sold by d) 52%
A. Ratio between number of gold ticket and silver ticket e) 35%
sold by B is 27:35. Totla number of ticket sold by A is 29. Which of the following cinema house sold
1000 which is equals to total numbe of ticket sold by C. maximum number of Silver ticket?
It is also given that total number of silver ticket sold by a) Both A and C
all the cinema house is 1470. It is also know that Total b) C
number of Gold ticket sold by A and B is 700. c) B
26. Find the ratio between gold tickets sold by C to d) A
Silver ticket sold by B cinema House? e) None of these
a) 9:7 30. Silver ticket sold by C is how much less than the
b) 7:9 same ticket sold by A?
c) 1:2 a) 8
d) 3:2 b) 9
e) None of these c) 4
27. Find the average number of ticket sold by C? d) 5
a) 387.5 e) None of these
b) 500
Directions (31 – 35): Answer the question based on the following information
There are 4 type of bank Private, Public, RRB and Cooperative bank and data given below show the number of male
and female employees working in the given bank. There are 12500 male employees working in Private bank which is
Page 706 of 722

Subscribe the Xpress Video Course & Mock Test Package for Bank & Insurance Exams
If there are any suggestions/ errors in our PDFs Feel Free to contact us via this email: admin@exampundit.in
IBPS RRB Clerk Prelims – Ultra Practice Bundle PDF
2700 more than the number of female employees working in RRB bank. Total number of female working in Public and
RRB bank is 22500. Number of male employees working in Public sector bank and RRB bank is 1800 more and 2700
less than the number of female employees working in same bank. Number of female working is cooperative bank is
2250 less than the number of female employees working in public bank. It is given that total number of female working
in all the bank is 55000.
31. Find the ratio between total numbers of female working in public bank to private bank?
a) 254:441
b) 280:445
c) 320:441
d) 350:413
e) None of these
32. Find the average number of male employees working in all the bank together?
a) Can’t be determined
b) 22550
c) 17750
d) 21250
e) 31250
33. What is the difference between total number of male employees working in public and private bank to
number of female employees working in RRB and Cooperative bank together?
a) 6750
b) 7220
c) 5120
d) 5525
e) None of these
34. Number of male employees working in cooperative bank is what % of total number of female employees
working in RRB and Public bank together?
a) 7:3
b) Can’t be determined
Page 707 of 722

Subscribe the Xpress Video Course & Mock Test Package for Bank & Insurance Exams
If there are any suggestions/ errors in our PDFs Feel Free to contact us via this email: admin@exampundit.in
IBPS RRB Clerk Prelims – Ultra Practice Bundle PDF
c) 8:3
d) 9:2
e) 11:3
35. Find the total number of employees from all the banks together, if total number of employees working in
Cooperative bank is 35000?
a) 117550
b) 113650
c) 112550
d) 113580
e) None of the above

Directions (36 – 40): Answer the question based on


the information given below
Five friend Pritam, Ujwal, Yogesh, Akash and Manu a) 115
have Rupees 2100 among themselves. The amount with b) 105
Pritam is as much less than that with Akash as it more c) 100
than that with Manu. The amount with Yogesh is 300 d) 125
less than that with Pritam, whereas Ujwal has twice that e) None of these
of Pritam. It is also known that difference between 38. The ratio of the amount with Yogesh and Ujjwal
amount with Akahsh and amount with manu is 50. is?
36. The amount with how many of them is more than a) 1:9
the average amount with them? b) 2:7
a) 2 c) 1:5
b) 3 d) 1:8
c) 4 e) None of theses
d) Can’t be determined 39. The amount with Akash is?
e) None of these a) 425
37. The amount with Yogesh is? b) 225
Page 708 of 722

Subscribe the Xpress Video Course & Mock Test Package for Bank & Insurance Exams
If there are any suggestions/ errors in our PDFs Feel Free to contact us via this email: admin@exampundit.in
IBPS RRB Clerk Prelims – Ultra Practice Bundle PDF
c) 195 42. The amount that A has is how much more than
d) 145 what B and C together have?
e) None of these a) 100
40. Amount with Manu is what % more or less than b) 200
the amount with pritam? c) 300
a) 12.50% d) 150
b) 5.50% e) Can’t be determined
c) 7.75% 43. If the shares of A and C are interchanged then B
d) 8.5% has how much more than what A has?
e) 6.25% a) 100
b) 200
Direction (41-45) – Answer the question based on c) 400
information given below. d) 250
A, B and C sold their three cycles manufactured in e) Can’t be determined
different year to Mr. D. Mr. D gave total or Rupees 1700 44. The ratio in which the amount share among A, B
to the three and said that A should get rupees 50 more and C is?
than half of the total amount as his cycle was used less. a) 3:5:6
B cycle being used more than the A’s Cycle he should b) 9:6:2
th
get about 6/17 the total amount and the last one gets c) 6:2:9
the remaining amount. Each individual gets his amount d) 3:2:1
only in denomination of rupees 100. e) 3:6:6
41. What is the difference between the amount 45. The difference of the amount that C and A have
received by A and C? is what % of the amount A have?
a) 700 a) 37%
b) 200 b) 71%
c) 250 c) 72%
d) 350 d) 35%
e) None of these e) None of these
Page 709 of 722

Subscribe the Xpress Video Course & Mock Test Package for Bank & Insurance Exams
If there are any suggestions/ errors in our PDFs Feel Free to contact us via this email: admin@exampundit.in
IBPS RRB Clerk Prelims – Ultra Practice Bundle PDF
Directions (46 – 50): Study the following graph and answer the following questions.
In a City level cricket match 6 team A, B, C, D, E and F participated. Each of the team score different score in their
individual match. Team A score 256 run which is 124 less than the twice of run scored by team C. Total Run Scored
by Team B and D is 550, which is in the ratio of 2:3. Team F scored run equal to the average run scored by A, B, C
and D, while run scored by E is 3/5 of the combine run scored by B and D.
46. Find the ratio between run scored by C and F?
a) 190:249
b) 125:123
c) 129:111
d) 129:321
e) None of these
47. Run scored by A and B together is what % of run scored by E and F together?
a) 25.25%
b) 85.20%
c) 56.20%
d) 82.21%
e) Can’t be determined
48. Run scored by A and D is how much more or less than the run scored by C and F?
a) 335
b) 255
c) 335
d) 147
e) 455
49. Find the average run scored by each of the given team?
a) 156.52
b) 262.5
c) 165.56
d) 156.56
Page 710 of 722

Subscribe the Xpress Video Course & Mock Test Package for Bank & Insurance Exams
If there are any suggestions/ errors in our PDFs Feel Free to contact us via this email: admin@exampundit.in
IBPS RRB Clerk Prelims – Ultra Practice Bundle PDF
e) None of these
50. Run Scored by Team D is how much % more or less than the Average run scored by each of the given
team?
a) 13.50%
b) 25.71%
c) 23.50%
d) 13.45%
e) None of these

10. Caselet DI Questions - Answers with Explanation


Solution(1-5).
Common Explanation:
ATQ,
Male student is class VI = 120% of Female student in Class V
900 = 120/100 * Female student in Class V
Female student in Class V = 900*100/120 = 750
The ratio of number of Male student to Female student in class VII is 8:7, respectively
Female student in class VII = 1200/8 * 7 = 1050
Class Male student Female student Total student
V 1800-750 = 1050 750 1800
VI 2100-1200 = 900 2400-750-1050 = 600 1500
VII 1200 1050 2250

1. Answer: B) Female in Class VII = 1050


2. Answer: C)
Required ratio = 600:900 = 2:3
3. Answer: E)
Page 711 of 722

Subscribe the Xpress Video Course & Mock Test Package for Bank & Insurance Exams
If there are any suggestions/ errors in our PDFs Feel Free to contact us via this email: admin@exampundit.in
IBPS RRB Clerk Prelims – Ultra Practice Bundle PDF
Average number of male student = 1050+900+1200/3 = 1050
4. Answer: D)
Required difference = 1050 – 600 = 450
5. Answer: A)
Required % = 900/1200 * 100 = 75%
Solution(6 – 10):
Common Explanation:
ATQ,
Total number of male from USA is 290 less than the number of female from same country who visited the Taj Mahal
Let total number of Female from USA = X and number of Male form USA = X-290
X + X-290 = 1710 accordingly X = 1000
ATQ,

Countries Male Female Total


USA 1000-290 = 710 1000 1710
Russia Y Y/2
Germany Z 1200+200 = 1400
Japan 2Z 1200
710+Y+Z +2Z = 5670 ---------------- (1)
1000+Y/2+1400 + 1200 = 5000 ----------------- (2)
From ----- (2) Y = 2800
By putting Value of Y in the equation no 1 = Z = 5670-710-2800 = 2180/3 = 720
Countries Male Female Total
USA 1000-290 = 710 1000 1710
Russia Y = 2800 Y/2 = 1400 4200
Germany Z = 720 1200+200 = 1400 2120
Japan 2Z = 1440 1200 2640

Page 712 of 722

Subscribe the Xpress Video Course & Mock Test Package for Bank & Insurance Exams
If there are any suggestions/ errors in our PDFs Feel Free to contact us via this email: admin@exampundit.in
IBPS RRB Clerk Prelims – Ultra Practice Bundle PDF
6. Answer: C)
Number of male and female form Germany who visited Taj Mahal = 720+1400 = 2120
7. Answer: E)
Required ratio = 710:1440 = 71:144
8. Answer: D)
Average number of Female from USA, Japan and Germany = (1000+1200+1400)/3 = 1200
9. Answer: A)
Required % = 2800/2640 * 100 = 106.06%
10. Answer: A)
Required difference = 2640 – 1710 = 930
Solution(11-15)
Common Explanation

Person who like Golgape = 100 = A+B


Peroson who like Chicket Tica = 120 = B+C
Person who like none of these = 40
Person who like exactly one of two item = 100 = A+C
ATQ,
Page 713 of 722

Subscribe the Xpress Video Course & Mock Test Package for Bank & Insurance Exams
If there are any suggestions/ errors in our PDFs Feel Free to contact us via this email: admin@exampundit.in
IBPS RRB Clerk Prelims – Ultra Practice Bundle PDF
2(A+B+C) = 100+120+100 = 320
A+B+C = 160
A = 160 – 120 = 40
B = 160 – 100 = 60
C = 160 – 100 = 60
D = Person who do not like any of two = 40
11. Answer: D)
People had like both the items = B = 60
12. Answer: B)
Person who does not like Chicken Tica = Person who like only GolGape + person who like none of them = A = 40 +
40 = 80
13. Answer: D)
Person who like Only GolGape A = 40
Person who like Only Chicken Tica = B = 60
Required %= 40-60/60 *100 = 33.33%
14. Answer: D)
Required answer = C = 60
15. Answer: D)
Required ratio = 40:(60+60) = 40:120 = 1:3
Solution(16-20).
Common Explanation:
Let Male student in class 8th = X
Accordingly, Male student in class 9th = 125-X
Girls studying in class 10th = X+40
Class Male Female Total
8th X X-10
9th 125-X 125-X-20

Page 714 of 722

Subscribe the Xpress Video Course & Mock Test Package for Bank & Insurance Exams
If there are any suggestions/ errors in our PDFs Feel Free to contact us via this email: admin@exampundit.in
IBPS RRB Clerk Prelims – Ultra Practice Bundle PDF
10th 360-X+40 X+40 360
ATQ,
X + X – 10 = 150
X = 80
Class Male Female Total
8th 80 70 150
9th 45 25 60
10th 240 120 360

16. Answer: A)
Required % = 80/70 * 100 = 114.28%
17. Answer: B)
Required difference = (80+45+240) – (70+25+120) = 150
18. Answer: C)
Required ratio = 240:120 = 2:1
19. Answer: E)
Number of girls studying in class 9 and 10th = 25+120 = 145
Number of male studying in class 9 and 10th = 45+240 = 285
Required % = 285-145/285 * 100 = 49.122%
20. Answer: E)
ATQ,
Number of male student in class 8th now = 80+ 25*2/5 = 90
Male student of class 9th = 45+ 25*3/5 = 60
Required difference = 90-60 = 30
Solution(21-25).
Common Explanation:
Day No of travelers

Page 715 of 722

Subscribe the Xpress Video Course & Mock Test Package for Bank & Insurance Exams
If there are any suggestions/ errors in our PDFs Feel Free to contact us via this email: admin@exampundit.in
IBPS RRB Clerk Prelims – Ultra Practice Bundle PDF
Monday 17500
Tuesday 17500*2 +240 = 35240
Wednesday 17500+35240/2 = 26370
Thursday X-350
Friday X
ATQ,
Total number of travelers from Monday to Friday = 26370 * 5 = 131850
Accordingly,
17500 + 35240+ 26370+X-350 + X = 131850
X = 26545
Day No of travelers
Monday 17500
Tuesday 35240
Wednesday 26370
Thursday 26195
Friday 26545
21. Answer: A)
Required difference = (17500+35240+26370) – (26195+26545) = 26370
22. Answer: B)
Required % = 17500/(26195+26545) *100 = 33.18%
23. Answer: A)
Required ratio = 26370:26545 = 5274:5309
24. Answer: D)
Required average = (17500+35240+26370+26195+26545)/5 = 26370
25. Answer: C)
Required answer is = Friday (2nd highest)
Solution(26-30).
Common Explanation
Page 716 of 722

Subscribe the Xpress Video Course & Mock Test Package for Bank & Insurance Exams
If there are any suggestions/ errors in our PDFs Feel Free to contact us via this email: admin@exampundit.in
IBPS RRB Clerk Prelims – Ultra Practice Bundle PDF
ATQ,
Cinema House Silver Gold
A 1000-X X
B 35Y 27Y
C X+120 1000-X-120

Accordingly,
1000-X+35Y+X+120 = 1470 ----------- (1) = 35Y = 1470-1000-120 = 350 and Y = 10
X + 27Y = 700
Bank Male employees Female
employees

Private 12500 55000-12700-


9800-10450 =
22050

Public 12700+1800 = 22500-9800=


14500 12700

RRB 9800-2700 = 12500-2700=


7100 9800

Cooperative Data is not 12700-2250 =


given 10450

X = 430
Cinema House Silver Gold

Page 717 of 722

Subscribe the Xpress Video Course & Mock Test Package for Bank & Insurance Exams
If there are any suggestions/ errors in our PDFs Feel Free to contact us via this email: admin@exampundit.in
IBPS RRB Clerk Prelims – Ultra Practice Bundle PDF
A 100-430 = 570 430

B 350 270

C 430+120 = 550 1000-430-120


= 450

26. Answer: A)
Required ratio = 450:350 = 9:7
27. Answer: B)
Required average = 550+450/2 = 500
28. Answer: C)
Required % = (350+270)/(570+430+550+450) * 100 = 620/2000 *100 = 31%
29. Answer: D)
Required answer = A (sold maximum silver ticket)
30. Answer: E)
Required answer = 570 – 550 = 20
Solution(31-35). Common Explanation
31. Answer: A)
Required ratio = 12700:22050 = 254:441
32. Answer: A)
Average amount with them = 2100 / 5 = 420

Bank Male employees Female


employees

Page 718 of 722

Subscribe the Xpress Video Course & Mock Test Package for Bank & Insurance Exams
If there are any suggestions/ errors in our PDFs Feel Free to contact us via this email: admin@exampundit.in
IBPS RRB Clerk Prelims – Ultra Practice Bundle PDF
Private 12500 55000-12700-
9800-10450 =
22050

Public 12700+1800 = 22500-9800=


14500 12700

RRB 9800-2700 = 12500-2700=


7100 9800

Cooperative Data is not 12700-2250 =


given 10450

Answer can’t be determined (as the data of male student of cooperative bank is not given)
33. Answer: A)
Required difference = (14500+12500) – (9800+10450) = 6750
34. Answer: B)
Answer can’t be determined (as the data of male student of cooperative bank is not given)
35. Answer: B)
Required number = 12500+22050+14500+12700+7100+9800+35000 = 113650
Solution(36-40). Common Explanation
Common explanation
Amount with Pritam = Akash – X or Amount with Akash = Amount with Pritam + X
Amount with Pritam = Manu + X or Amount with Manu = Amount with Pritam - X
Amount with Yogesh = Pritam - 30
Amount with ujjwal = 2* Pritam
Accordingly,
Pritam + Pritam + X + Pritam - X + Pritam - 200 + 2*Pritam = 2200

Page 719 of 722

Subscribe the Xpress Video Course & Mock Test Package for Bank & Insurance Exams
If there are any suggestions/ errors in our PDFs Feel Free to contact us via this email: admin@exampundit.in
IBPS RRB Clerk Prelims – Ultra Practice Bundle PDF
6 Pritam = 2400
Amount with Pritam = 400
Accordingly,
Amount with Yogesh = 400-300 = 100
Amount with Ujjawal = 2*400 = 800
ATQ,
400 + X – ( 400 –X ) = 50
400 + X -400 + X = 50
Or X = 25
Now,
Amount with Akash = 400 +25 = 425
Amount with Manu = 400-25 = 375
Person Amount
Pritam 400
Ujwal 800
Yogesh 100
Akash 425
Manu 375

37. Answer: C)
The amount with yogesh = 100
38. Answer: D)
Required ratio = 100:800 = 1:8
39. Answer: A)
The amount with akash = 425
40. Answer: E)
Required % = 400-375/400 * 100 = 6.25%
Solution(41-45).
Page 720 of 722

Subscribe the Xpress Video Course & Mock Test Package for Bank & Insurance Exams
If there are any suggestions/ errors in our PDFs Feel Free to contact us via this email: admin@exampundit.in
IBPS RRB Clerk Prelims – Ultra Practice Bundle PDF
Common Explanation
A get = 50 + 1/2 of 1700 = 900
B get = 6/17 * 1700 = 600
C get = 1700 – 900 – 600 = 200
41. Answer: A)
Required difference = 900 – 200 = 700
42. Answer: A)
Required answer = 900 – (600+200) = 100.
43. Answer: C)
Now A will gate = 200
And required difference = 600 – 200 = 400
44. Answer: B)
Required ratio = 900: 600: 200 = 9:6:2
45. Answer: E)
Required % = (900-200) / 900 * 100 = 77.77%
Solution(46-50).
Common Explanation
Run scored by A = 256
2*Run scored by C -124= 256 Run scored by C = (256+124) / 2 = 190
Run Score by B = 550*2/5 = 220
Run Score by D = 550*3/5 = 330
Run Scored by F = (256+220+190+330)/4 = 249
Run Scored by E = 3/5 * (550) = 330
46. Answer: A)
Required ratio = 190:249
47. Answer: D)
Run Scored BY A and B together = 256+220 = 476
Run scored by E and F together = 330+249 = 579
Page 721 of 722

Subscribe the Xpress Video Course & Mock Test Package for Bank & Insurance Exams
If there are any suggestions/ errors in our PDFs Feel Free to contact us via this email: admin@exampundit.in
IBPS RRB Clerk Prelims – Ultra Practice Bundle PDF
Required % = 476/579 * 100 = 82.21%
48. Answer: D)
Required answer = (256+330) – (190+249) = 147
49. Answer: B)
Required average = (256+190+220+330+249+330)/6 = 262.5
50. Answer: B)
Average run scored by each team = (256+190+220+330+249+330)/6 = 262.5

IBPS RRB Free Practice Model Question Paper PDF – Download Now

THE COMPLETE Static GK Capsule for Upcoming Exams

The COMPLETE Static Banking Awareness PDF

Join Our What’s App Group & Get Instant Notification on Study

Materials & PDFs

Click Here to Join Our Official Telegram Channel

Page 722 of 722

Subscribe the Xpress Video Course & Mock Test Package for Bank & Insurance Exams
If there are any suggestions/ errors in our PDFs Feel Free to contact us via this email: admin@exampundit.in

You might also like